You are on page 1of 300

ORGANIC CHEMISTRY

ASSIGNMENTS

ASSIGNMENT-1

Q.1

Match the following:


Column I
(Wrong IUPAC name)
(A)

(B)

CH 3 CH 2 CH CH 2 CH 3
|
CN
Pentane 3 cyanide
Br Cl
| |
CH 2 C CH CH 3
2 chloro 3 bromo 3 butene

[4]
Column II
(Reason)
(P)

Numbering of selected carbon chain is wrong

(Q)

Selection of carbon chain is wrong

(R)

Selection of principal F.G. is wrong

(S)

Alphabetic order is not considered for writting

OH
(C)

Bicyclo [4,2,0] octan-2-ol


CH3
CH 3

(D)
3,5-dimethyl octa-1,3-diene

the name

(T)

Suffix of functional group is wrong


[Ans. (A) Q,T (B) P, S (C) P (D) P]

Q.2

Match the following:


Column I

Column II (DOU)

(A)

Azulene

(P)

(B)

Napthalene

(Q)

(C)

Tetralin

(R)

(D)

1,2,3-tri cyclo propyl cyclo propane

(S)

(T)

[4]

[Ans. (A) S, (B) S, (C) Q, (D) P]

Q.3

Q.4

Question No. 3 to 4 ( 2 questions)


Questions below consist of an Assertion in column I and Reason in column II. Use the following
key to choose the appropriate answer
(A) If both Assertion and Reason are correct and Reason is the correct explanation of the Assertion.
(B) If Assertion and Reason are correct, but Reason is not the correct explanation of the Assertion.
(C) If Assertion is correct, but Reason is incorrect.
(D) If Assertion is incorrect, but Reason is correct.
Assertion : Effective molecular mass of acetic acid is 120.
[3]
Reason :
It form dimer due to intramolecular H-bonding.
[Ans. C]
Assertion :
Reason :

Q.5

Cyclohexanol is less soluble in water than 1-hexanol.


1-hexanol can form intermolecular H-bond with H2O.

and

[3]
[Ans.D]

are

(A) Chain isomers

[3]

(B*) Metamers

(C) Positional isomers (D) Conformers

Q.6

The boiling points of isomeric alcohols follows the order


(A*) primary > secondary > tertiary
(B) tertiary > secondary > primary
(C) secondary > tertiary > primary
(D) does not follow any order

Q.7

Explain why 2-methylpyrrolidine boils at a higher temperature than its isomers N- methylpyrrolidine.
[3]

Q.8

Explain why p-dichlorobenzene is more soluble in n-propyl alcohol than methyl alcohol, while
o-dichlorobenzene is less soluble in n-propyl alcohol than methanol.
[3]

Q.9(a) How many metamers (aromatic) are possible for the structural formula of
(b)

[3]

O Et .

Write bond line structural formula for all the possible isomers having the molecular formula C4H6.[4]

[Ans. (a) 4 , (b) 9]


Q.10 Fill up the blank space showing minimum number of C-atoms required to exhibit the given isomerism.
[24]
Chain Position Functional Metamerism
(a )
( b)
(c )
(d )
(e )
(f )

[Ans.

(a )
( b)
(c )
(d )
(e )
(f )

Alkane
Alkene
Alkyne
Alcohol
Ether
Aldehyde

Alkane
Alkene
Alkyne
Alcohol
Ether
Aldehyde

Chain
4
4
5
4
5
4

Position
6
4
4
3
4
5

Functional Metamerism

]
2
4
3

Well begin is half done

ASSIGNMENT-2

Q.1

Match the column:


Column I

[4]
Column II
Cl

Br
Cl

(A)
(B)

A pair of metamers
A pair of functional isomer

(P)

&

(Q)

Br
Acetaldoxime & acetamide

CH2OH

OH
(C)

A apir of position isomer

&

(R)

CH3
(D)

A pair of chain isomer

(S)

CH 3 CH 2 CH 2 & CH 3 CH CH 3
|
|
CN
CN

Cl
(T)

Cl

COC
O
Cl

&

O
Cl

COC
O

[Ans. (A) T, (B) Q,R (C) P (D) S]

Q.2

Match the column :


Column I
(Compound)

CH3
(A)

[4]
Column II
(Double bond equivalent)

H
CN

O
O

(P)

(Q)

(R)

(S)

19

(T)

N
COOH

(penicillin)

(B)

(Hexahelicene)

(C)

(Prismane)
O
C
C

(D)

C
O

OH
OH

(Ninhydrin)

[Ans. (A) Q; (B) S; (C) P; (D) R]


Q.3

Statement-1 : Homologoues are having similar physical properties & different chemical properties.
Statement-2 : They are having different molecular masses & having same F.G.
[3]
(A) Statement-1 is true, statement-2 is true and statement-2 is correct explanation for statement-1.
(B) Statement-1 is true, statement-2 is true and statement-2 is NOT the correct explanation for statement-1.
(C*) Statement-1 is false, statement-2 is true.
(D) Statement-1 is true, statement-2 is false.

Q.4

Statement-1 :
IUPAC name of H2C = CHCH2CCH is pent-4-en-1-yne.
[3]
Statement-2 :
Alkene is preferred over alkyne when they are placed at same position.
(A) Statement-1 is true, statement-2 is true and statement-2 is correct explanation for statement-1.
(B) Statement-1 is true, statement-2 is true and statement-2 is NOT the correct explanation for statement-1.
(C) Statement-1 is true, statement-2 is false.
(D*) Statement-1 is false, statement-2 is true.

Q.5

Which of the following is/are a bicyclo compound(s):

[3]

(I)

(II)

(III)

(IV)

(A) III only

(B*) III and IV only

(C) II, III and IV

(D) I, II, III and IV

Q.6

Compare water solubility and boiling point of iso-pentyl alcohol and iso pentyl amine.
[Ans. (boiling point i)
(H2O solubility ii)]

Q.7

Boiling point of ethanol (78C) is more than that of ethanamine (16.6C), but boiling point of ethanoic
acid (117C) is less than that of ethanamide (221C). Explain with reasons.
[3]

Q.8

Compare boiling point:

(I)

CH 3
|
(a) CH 3CH 2 CH CH 2 NH 2

[4]

CH 3
|
(b) CH 3 CH 2 CH NH CH 3

CH 3
|
(c) CH 3 CH 2 N CH 2 CH 3
(II)

(III)

(CH2OH)2
A

(CH2OMe)2
B

[Ans. a > b > c]


CH2OH CH2OMe
C
[Ans. A > C > B]

CH 3
|
CH 3 C CH 3
|
CH 3
neopantane

n-hexane

2,3-dimethylbutane

1-pentanol 2-methyl-2-butanol
[Ans.neopantane < 2,3-dimethylbutane < n-hexane < 2-methyl-2-butanol < 1-pentanol ]
(IV)

[3]

3-methylbutanol, n-Pentane, 2, 2-dimethylpropanol and n-pentanol

[Ans. 4 > 1 > 3 > 2]

OH

H
OO

NH2

Q.9

NH2

[9]

NH
(a)
(b)
(c)
(d)
(e)
(f)
(g)
(h)
(i)

O
Find out the total number of 1 hydrogen.
Find out the total number of 2 hydrogen.
Find out the total number of 3 hydrogen.
Find out the total number of 1 carbon.
Find out the total number of 2 carbon.
Find out the total number of 3 carbon.
Find out the total number of 4 carbon.
Find out the total number of functional groups.
Find out the number of degree of unsaturation.
[Ans. a-9, b-11, c-3, d-5, e-9, f-3, g-0, h-6, i-5]

Q.10 What isomerism is exhibited by the following pairs of compound.


(a)
Me4 C
&
Me2CHCH2CH3
CH 3 C CH2
(b)
CH3 COCH2OH
&
OH O
(c)
o-xylene
&
p-xylene
(d)
Butanal
&
2-buten-2-ol
(e)
Bicyclo [1,1,0]
&
2-methyl cyclopropene
(f)
Acrylic acid
&
Vinyl formate

(g)

&
O

(h)

CCNCCCC
|
H

&

C C NH C C C
|
C

(i)

Et C NH Me
||
O O

&

Me C NH Et
||
O O

(j)

CO

&

CO

[Ans. (a) chain (b) func.(c) posi. (d) func. (e) func. (f) func. (g) posi. (h) chain (i) meta (j) meta ]

A diamond is merely a lump of coal that did well under pressure.

[10]

ASSIGNMENT-3

Q.1

Match the column:


Column-I and column-II contains four entries each. Entry of column-I are to be uniquely matched with
only one entry of column-II.
[4]
Column I
Column II
(Compound)
(Boiling point in C)
(A)

H3CCH2CH2 CH2CH2OH

(P)

290C

(B)

H 3C CH CH 3
|
OH

(Q)

138C

(C)

H 2C CH CH 2
|
|
|
OH OH OH

(R)

102C

(S)

82.4C

(D)

CH 3
|
H 3C C CH 2 CH 3
|
OH

[Ans. (A) Q; (B) S; (C) P; (D) R]


Q.2

Match the Column :


Column I
(A)
C6 H14

Column II
(P)
Tautomerism

(B)

C4 H10 O

(Q)

Positional isomerism

(C)

C4 H8 O

(R)

Functional isomerism

(D)

C2 H5 N

(S)

Geometrical isomerism

[4]

[Ans. (A) Q (B) Q,R (C) P,Q,R,S (D) P,R,S]


H5C2
Q.3

C=C

Statement 1 :
CH3

H5C2

Cl
Br

and

Cl
C=C

Br

are structural isomers.

[3]

CH3

Statement 2 : The above mentioned compounds can show geometrical isomerism.


(A) Statement-1 is true, statement-2 is true and statement-2 is correct explanation for statement-1.
(B*) Statement-1 is true, statement-2 is true and statement-2 is NOT the correct explanation for
statement-1.
(C) Statement-1 is false, statement-2 is true.
(D) Statement-1 is true, statement-2 is false.

O
Q.4

Statement 1 : Resonating structures of

NH

can show geometrical isomerism at room temperature.

Statement 2 : Lone pair of nitrogen will participate in resonance but only E form is stable in its resonating
structure.
[3]
(A) Statement-1 is true, statement-2 is true and statement-2 is correct explanation for statement-1.
(B) Statement-1 is true, statement-2 is true and statement-2 is NOT the correct explanation for statement-1.
(C*) Statement-1 is false, statement-2 is true.
(D) Statement-1 is true, statement-2 is false.
Q.5

Q.6

Q.7

Conformational changes in a ethane leads to change in


(A*) torsional angle
(B) bond angle
(C) bond length

(D) all of the above

Which of the following isomeric structure have lowest energy?

(A)

(B)

(C)

(D*)

[3]

The terpene ocimene has the IUPAC name (3E)-3,7-dimethyl-1,3,6-octatriene, what is the structural
formula of this compound?
[3]

(A)

Q.8

[3]

(B)

(C)

(D*)

NC
NC
|
|
CH3 CH 2 CH 2 CH 2 and CH3 C CH3 are
|
CH3
(A*) chain isomers

(B) positional isomers (C) both

[3]

(D) none

Q.9
(a)
(b)
(c)

(d)
(e)
(f)

Circle the member of each pair that is more soluble in water.


CH3CH2OCH2CH3 or CH3CH2CH2CH2CH3
CH3CH2NHCH3 or CH3CH2CH2CH3
CH3CH2OH or CH3CH2CH2CH2OH

[6]

or
C3H8, C2H5OH, (CH3)2O and CH2OHCH2OH
3-methylbutanol, n-Pentane, 2, 2-dimethylpropanol and n-pentanol

[Ans. (a)CH3CH2OCH2CH3, (b)CH3CH2NHCH3, (c) CH3CH2OH , (d)


(e) 4>2>3>1, (f) 3>1>4>2

Q.10 For each pair of compounds, circle the compound you expect to have the higher boiling point. Explain
your reasoning.
[12]
(a)
(CH3)3CC(CH3)3 and (CH3)2CHCH2CH2CH(CH3)2
(b)
CH3(CH2)6CH3 and CH3(CH2)5CH2OH
(c)
HOCH2 (CH2)4CH2OH and (CH3)3CCH(OH)CH3
(d)
(CH3CH2CH2)2NH and (CH3CH2)3N
(e)

and

(f)

and
N-methylpyrrolidine

(g)
(h)
(i)
(j)
(k)
(l)

piperidine
and

tetrahydropyran
cyclopentanol
CH3CH2OCH3 or CH3CH(OH)CH3
CH3CH2CH2CH3 or CH3CH2CH2CH2CH3
CH3CH2CH2CH2CH3 or (CH3)2CHCH2CH3
CH3CH2CH2CH2CH3 or CH3CH2CH2CH2CH2Cl
CH3CH2OCH2CH3 and CH3CH2CH2CH2OH
[Ans. (a) (CH3)2CHCH2CH2CH(CH3)2, (b) CH3(CH2)5CH2OH, (c) HOCH2 (CH2)4CH2OH,
(d)(CH3CH2CH2)2NH, (e)

, (f)

piperidine, (g)

CH 3 CH(OH)CH 3 ,
(i)
CH 3 CH 2 CH 2 CH 2 CH 3
,
(k)CH3CH2CH2CH2CH2Cl, (l) CH3CH2CH2CH2OH ]

(j)

cyclopentanol, (h)
CH 3 CH 2 CH 2 CH 2 CH 3 ,

Discipline is the bridge between goal and accomplishment.

ASSIGNMENT-4

Q.1

Match the column :


[4]
Column-I and column-II contains four entries each. Entry of column-I are to be uniquely matched with
only one entry of column-II.
Column I
Column II
(Compound)
(Solubility in g / 100 g H2O)
(A)
CH3CH2OH
(P)
0.05
(B)
(C6H5)2CHOH
(Q)
12.5
(C)
CH3(CH2)5CH2OH
(R)

(D)

H 3C CH CH 2 CH 3
|
OH

(S)

0.2

[Ans. (A) R; (B) P; (C) S; (D) Q]


Q.2

Match the column:


Column I

[4]
Column II
OH

(A)

(P)

Conformation of maximum torsional strain

(B)

(Q)

Conformation with strongest intramolecular hydrogen bond

(C)

(R)

Highest boiling point out of ABCD

(D)

(S)

Conformation of minimum Vander Waal strain

(T)

Maximum heat of combustion out of ABCD

OH

[Ans. (A) R,T (B) Q, (C) P, (D) S ]

Question No. 3 to 4 ( 2 questions)


Questions below consist of an Assertion in column I and Reason in column II. Use the following
key to choose the appropriate answer
(A) If both Assertion and Reason are correct and Reason is the correct explanation of the Assertion.
(B) If Assertion and Reason are correct, but Reason is not the correct explanation of the Assertion.
(C) If Assertion is correct, but Reason is incorrect.
(D) If Assertion is incorrect, but Reason is correct.
Q.3

Statement-1 :
Statement-2 :

Boat form is the least stable conformation of cyclohexane.


Boat form is eclipsed form.

[3]
[Ans. D]

Q.4

Statement-1 :
Statement-2 :

Gauche form of butane is less stable than anti-form.


Anti-form is more symmetrical the gauche.

[3]
[Ans. B]

Q.5

Potential energy barrier for the direct interconversion of the two gauche forms of butane is 3.6 kcal/mol.
What should be the expected potential energy barrier for direct interconversion of the two gauche form
of 1,2-dichloroethane Vander Waals radius of a covalently bond chlorine atom is about the same as that
of a methyl group.
[3]
(A) 2.8 kcal/mol

Q.6

(B) 3.6 kcal/mol

(D*) 9.3 kcal/mol

Select correct statement(s) about (a) & (b).

(A*) a and b have equal torsional strain


(C*) a is having higher dipole moment
Q.7

(C) 3.8 kcal/mol

[3]

(B*) a is more stable


(D) b is the least stable form of this compound.

Find out the correct option(s) ?

[3]

NH
CH3
C=C

(A*)

NH

Orientation is E

D
C=C

(B*) H

Orientation is Z

CH3
(C)

CH3

Orientation is Z

CH3
(D*)

CH3

geometrical isomers are not possible

Q.8

Which compound(s) will show the geometrical isomerism?


CH3

Cl

NH

(A*)

(B)

Cl
C=C=C

H
N
(C*)

[3]

CH 3

(D)

H3C

N
H

Q.9(a) Arrange the groups or atoms accroding to their priority order using CIP sequence rules ?

(1)

, CH2CH3 , CH=CH2 , CCH

(2)

COOH, COCl, CONH2 ,CHO, COCH3

(3)

CH2NH2, CH2Cl, CH2Br,CH2CN, CH2CH2Cl, CH2OH

(4)

CN, OH, Cl, D, H, SO3H, SH, F, Br, I, NH2

O
(5)
[Ans. (1)
(3)
(5)
(b)

[5]

O,

O
,

2 >5 > 1 > 4 > 3


3>2>6>1>5>4
2>4>1>3 ]

(2)
(4)

2>1>3>5>4
10 > 9 > 3 > 6 > 7 > 8 > 2 > 11 > 1 > 4 > 5

Which of the following compounds should have the larger energy barrier to rotation about the indicated
bond?
[3]
(a)

Me3C CMe 3

(c)

CH2 = CH CH = CH2

(b)

Me 3Si SiMe 3

[Ans. c > a > b ]


Q.1084goc-1
(a)

Find total number of Geometrical isomerism of following compounds.


CH3CH=CHCH=NOH

[6]

(b)

(c)

(d)

CH3(CH=CH)3 Ph

(e)

(f)

CH3CCCH=CHCH3

[Ans. (a) 4, (b) 2, (c) 2, (d) 8, (e) 2, (f) 2]


Ill run till I can, Ill walk till I can, when I can only crawl, Ill crawl but by the grace of god,
Ill always be moving forward.

ASSIGNMENT-5

Q.1

Match the following:


Column I
(Compound)

[4]
Column II
(Double bond equivalent)

(A)

(P)

12

(Q)

13

(R)

14

(S)

16

N
H
H

(B)

Porphyrin (C20H14N4)

(C)
Ph

(D)

[Ans. (A) Q, (B) S, (C) R, (D) P]

Q.2

Match the following structural formulae with their possible geometrical isomers?
Column I
Column II
(Structural formula)
(Total geometrical isomers)

(A) CH3CH = CH

CH2CH3

(P)

(Q)

(R)

(S)

[4]

CH=CHCH3

(B) CH3CH = C
CH=CHCH3

(C) CH3CH=CH

CH=CHCH3

(D) Cl CH = CH CH = CH CH = CH CH3

[Ans. (A) S, (B) R (C) Q (D) P ]


Q.3

Statement-1 :
Van-der-walls forces are present only in non polar compounds
[3]
Statement-2 :
In polar compounds stronger dipolar forces are present.
(A) Statement-1 is true, statement-2 is true and statement-2 is correct explanation for statement-1.
(B) Statement-1 is true, statement-2 is true and statement-2 is NOT the correct explanation for statement-1.
(C) Statement-1 is true, statement-2 is false.
(D*) Statement-1 is false, statement-2 is true.
O

Q.4

Statement-1 :

tautomer of this compound can show geometrical isomer..

[3]

OH
Statement-2 :

across the double bond both the terminals having different groups.

(A) Statement-1 is true, statement-2 is true and statement-2 is correct explanation for statement-1.
(B) Statement-1 is true, statement-2 is true and statement-2 is NOT the correct explanation for statement-1.
(C) Statement-1 is true, statement-2 is false.
(D*) Statement-1 is false, statement-2 is true.
Me
CH3 CD2

CHCH3
C=C

Q.5

14

CH3 CH2

CF3
C=C

CH2Cl
H
Configuration of both the double bond in this compound respectively are
(A) 2E 4E
(B*) 2E 4Z
(C) 2Z 4E
(D) 2Z 4Z

[3]

Q.6

What is the correct stereochemical assignment for each of the following alkenes?

F
(I)

F
(II)

(III)

(A*) I = E; II = Z; III = E; IV = Z
(C) I = E; II = E; III = Z; IV = Z
Q.7

[3]

(IV)

(B) I = Z; II = E; III = E; IV = E
(D) I = E; II = E; III = Z; IV = E

Correct statement about given Ketoxime is


OH

C=N

CH3

[3]

..

(A)
(A*) A is named as syn-p-tolylphenyl ketoxime
(B*) A is named as anti-phenyl-p-tolyl ketoxime
(C*) Compound (A) is Z isomer
(D) Compound (A) is E isomer
Q.8

What is correct about (B) & C)

[3]

CH3
180
(C)
front
carbon

+180
(B)
rear
carbon

CH3

(A*) Both are achiral molecules


(C*) Both are meso

(B*) Both contains chiral carbon


(D*) Both having same bond length.

Q.9

How many chiral carbon atoms are present in the following compounds?

(i)

(ii)

(iii)

(iv)

[4]

[Ans. (i) 2, (ii) 3, (iii) 4, (iv) 10]


Q.10 Assign priority number to the following groups as per Cahn, Ingold, Prelog sequence rule
(a) CH2OH,
CH3,
CH2CH2OH,
H
(b) Cl,
Br,
CH=CH2,
CH3
O
||
(c) C H
OH,
CH3,
CH2OH
(d) CH(CH3)2,
CH2CH2Br,
Cl,
CH2CH2CH2Br
(e) CH=CH2,

(f) CH=CH2,

(g) CH2CH2CH2I
[Sol.

CH2CH3

CCH

CH CH CH 3
|
Br
(a) 1 > 3 > 2 > 4
(b) 2 > 1 > 3 > 4
(e) 3 > 1 > 2 > 4
(f) 4 > 2 > 3 > 1
(see the Cahn, Ingold, Prelog rule)
]

CH CH 2
|
|
CH 3 CH 3
CH CH 2 CH 3
|
Cl
(c) 2 > 1 > 4 > 3
(g) 4 > 3 > 2 > 1

[21]

CH3
CH 3
|
C
|
CH 3

CH 3
|
CH
|
CH 3

(d) 3 > 1 > 2 > 4

Dreams ....Are not the ones you see when you get in to the sleep, But are those which will
never let you sleep...Till they are achieved.....

ASSIGNMENT-6

Q.1

Indicate whether each of the following structure has the R configuration or the S-configuration.[21]

(a)

(b)

(e)

(f)

(g)

(h)

(i)

(j)

(k)

(l)

(m)

(n)

(o)

(q)

(c)

(r)

(d)

(p)

CH3
(t) Br

(s)

CH2CH3
H
CH3
CH2CH3

(u)
[Ans.. (a) R (b) R (c) S
(m) S (n) R (o) S
(u) 2R 3S
]
Q.2

(d) S
(p) S

(e) R (f) S
(q) 2R 3R

(g) R (h) S
(r) 2R 3R

(i) S (j) S
(s) 2S 3R

(k) S (l) R
(t) 3S 4R

Indicate whether each of the following structure has the R configuration or the S-configuration. [26]

(a)

(e)

(i)

(b)

(f)

(j)

(c)

(d)

(g)

(h)

(k)

(l)

(m)

(n)

(o)

(p)

(q)

(r)

(s)

(t)

(u)

(v)

(y)

(z)

(w)

(x)

[Ans. (a) R, (b) R, (c) S, (d) S, (e) R, (f) S, (g) R, (h) S, (i) S, (j) S, (k) S, (l) R, (m) S, (n) R, (o) S, (p) S
(q) 2R 3R, (r) 3S 4R, (s) 2S 3R, (t) 3S 4R, (u) 1R 2S, (v) 5R 6S, (w) 3S 4R, (x) 4S 5R,
(y) 2R3R4R, (z) R
]

In time of trouble, don't say "God I have a big problem." say "Hey Problem. I have a big God.

ASSIGNMENT-12

Q.1

Match the column :


Column I
(Property)

[4]
Column II
(Compound)
H

(A)

Plane of symmetry

Me
C

(P)

C
H

Me

Me

(B)

Centre of symmetry

Me

(Q)

Me
H
(C)

Meso Compound

(R)

C
Me

(D)

Chiral atom is / are present

(S)

(T)

Ph
H
O
Ph
CO
H
H

Cl
H

C=C=C=C

COOH
H

H
Cl

[Ans. (A) P,S,T (B) P,T (C) S (D) Q,R,S]

Q.2

Column I
Cl
(A)

Column II

[4]

Me
Me

(P)

Total number of stereo isomers are odd.

(Q)

Total number of stereocenter are even

(R)

Compounds having plane of symmetry

Cl

Br
(B)

Br
H

Cl
C

(C)

Cl

(D)

C
H

or axis of symmetry

COOH
H
OH
HO
H
COOH

(S)

Compounds have zero dipole in given form.

(T)

Compounds having centre of symmetry.

[Ans. (A) P,Q,R (B) Q,R,S,T (C) Q,R,S,T (D) P,Q,R]

F
Q.3

Statement-1 :

[3]

F
(I)

F
(II)

(I) and (II) are optically inactive molecules.


Statement-2 :

Molecules containing plane of symmetry or centre of symmetry are optically


inactive.
(A*) Statement-1 is true, statement-2 is true and statement-2 is correct explanation for statement-1.
(B) Statement-1 is true, statement-2 is true and statement-2 is NOT the correct explanation for statement-1.
(C) Statement-1 is true, statement-2 is false.
(D) Statement-1 is false, statement-2 is true.

Q.4

Select the correct statement:

[5]

(A*) H

Et is optically inactive while


CH3

is optically active

CH 3 CH 2 CH 3
( II)

(B*) CH 3 CH 2 CH 2 CH 3
( I)

About C() C() bonds; (I) has more energy barrier for rotation as compared to (II).
(C*) The boat form of cyclohexane is less stable than chair form due to torsional strain as well as flag
pole interaction.
(D*) In t-butyl substituted cyclohexane, most stable chair conformer has CMe3 group at equitorial
position.
Q.5

Most stable form of meso-2,3- diflouro-2,3-butandiol is:

(A)

Q.6

(B)

(C*)

Cl

Cl

Cl

Cl

[3]

(B) It is Z isomer
(D) It has Centre of Symmetry

Consider the following statements regarding the given projection and select the correct statement(s)?
[3]
(W)

(X)

(A*) W & Y are diastereoisomers


(C*) W, X, Y and Z are optically active
Q.8

(D)

True statement(s) about this compound is/are:

(A) It is E isomer
(C*) It is optically active
Q.7

[3]

(Y)

(Z)

(B*) Z is the newmann projection of X


(D) Y & Z are meso.

30.9 g of (+) 2-butanol was taken in a 309 mL solution and was mixed with 15.45 g of () 2-butanol
placed in 154.5 mL and the final solution is passed through 444 mm tube. What will be observed rotation
( in degree) for the final solution if specific rotation of 2-butanol is 13.5.
[5]
[Ans. 0002.00 or 0001.99]

Q.9

From the following set of compounds, select:


(a) enantiomer pairs
(b) distereomer pairs in which both are optically active
(c) label them as D or L sugar

(I)

(IV)

(II)

[5]

(III)

(V)

[Ans.

Also II D-sugar
V L-sugar
Sugar I, III and IV are all L-sugar; Any pair of two structure other than II and V are diastereomer pair ]
Q.10 How many stereoisomers are possible for following compounds:
(a)

(b)

CH3 CH = CH CH = CH CH = C = CH2

(c)
(d)
(e)
(f)
(g)

OH OH
|
|
HOOC CH CH COOH
1,2-dichlorocyclopropane
1,3- dimethylcyclobutane
2-bromo-3- chlorobutane
1,3-dimethyl cyclohexane
[Ans. (a) 8 (b) 4 (c) 3 (d) 3 (e) 2 (f) 4 (g) 3 ]
Tough Times never last, tough people do.

[7]

ASSIGNMENT-13
Q.1

Column I

Column II

(A)

(P)

Chiral

(B)

(Q)

Plane of symmetry

(C)

(R)

(D)

(S)

E (Entegen)

(T)

Even number of stereoisomer possible

[4]

[Ans. (A) P,R,S,T (B) P, R,T (C) P, S, (D) P,T]


Q.2

Match the column :


Column I
Compound
Cl

[4]
Column II
Number of Geometrical isomerism

(A)

(P)

(B*)

(Q)

(C)

(R)

(D)

(S)

(T)

Br

Q.3

Statement-1 :

O
||
Resonance energy of CH 2 CH C NH 2 is very high as compaired to

O
||
CH 3CH 2 C NH 2 .
Statement-2 :
Another double bond is introduced in conjugation to C = O.
(A) Statement-1 is true, statement-2 is true and statement-2 is correct explanation for statement-1.
(B) Statement-1 is true, statement-2 is true and statement-2 is NOT the correct explanation for statement-1.
(C) Statement-1 is true, statement-2 is false.
(D*) Statement-1 is false, statement-2 is true.
[3]
Q.4

The configuration of chiral carbon in the molecule.

[3]

CHO
CH2OH

HO
H

(A*) D, R

(B) D, S

(C) L, R

(D) L, S

Paragraph for question nos. 5 to 7


Observe following structures and answer questions given below:
Me

HO

H
Me
(I)

Q.5

Q.6

Q.7

Me

HO

HO

H
H

OH
Me

(II)

What is the relationship between II & III.


(A) Enantiomers
(B) Diastereomers

Me
H
HO

OH

OH
Me
H
(III)

(C*) Identical

Which of the following is pair of diastereomers


(A*) I & III
(B) II & III
(C) I & IV

[9]

OH

(IV)
(D) Positional isomers

(D) None

Compound IV is a vicinal diol, total how many vicinal diol's are possible with the same molecular formula
as IV.
(A) 3
(B) 4
(C) 5
(D*) 6

Q.8

Select correct statement:


CH3

CH2 OH

(A*)

[5]

and

are functional isomers


OH

(B*)

has eight stereoisomers

Cl
(C) Me CH CH CH Me has four optically active stereoisomers
|
|
|
Cl Cl Cl
Me
H

CH3
CH3 is meso compound

(D) H
Me

Q.9

Find relationship between following pairs.


CH2OH
CHO

HO

(a)

[6]
Cl

OH

CHO
&

CH3

Cl

CH2OH

Me

CH3
H

Et

(b)

Et

CH3

Me

Et
Me
Me

&
Et

CH3

& CH 3 O CH CH 3
|
CH 3

(c) CH3 O CH2 CH2CH3

Me

Br
OH

(d)

OH
&

Br

Me

OH
OH
(e)

Cl
&

Br Br D

Br

D H Br

&

(f)
[Sol.

Cl

O
O
(a) Diastereoisomers (b) Identical
(e) Geometrical isomer (f) Identical

(c) Positional isomer

(d) Enantiomer

Q.10 Among following pairs which will require lower heat of activation for cis to trans interconversion: [8]
(a)
MeCH=CHMe
&
Me3CCH=CHCMe3
(b)
MeCH=CHMe
&
MeCH=CHOMe
(c)

CH=CH

&

CH=CH

NO2

(d)

CH=CH

&

CH=CH

OMe

(e)

O2 N

(f)

Me O C CH CH C O Me
||
||
O
O

(g)
[Sol.

CH=CH

&

NO2

&

O2N

&

Me O C CH CH O C Me
||
||
O
O

(h)

CH=CH

&

a to h in all case right one]

Confidence begets confidence. The man who wins is the man who thinks he can

OMe

ASSIGNMENT-14

Q.1

Match the column:


Group
(A)
NO
(B)
(C)
(D)

[4]

NH 3
NC
NH2

(P)

Effect
+I

(Q)
(R)
(S)

I
+M
M
[Ans. (A) Q,R,S ; (B) Q ; (C) Q ; (D) Q, R]

Q.2

Match the column-I with column-II. Note that column-I may have more than one matching options in
column-II.
[4]
ColumnI (stability)
Column-II (Reason)

(A)

> CH 3

(P)

Inductive effect

(B)

< CH 3

(Q)

Resonance

(C)

> H3C CH 2

(R)

Hyperconjugation

(S)

steric hindrance

(D)

>

[Ans. (A) P,Q,R; (B) P; (C) P, R; (D) Q]


Q.3

Statement-1 :
Correct order of acidic strength is CH3CH3 > CH2=CH2 > HCCH [3]
Statement-2 :
CH bond energy in ethene is more than ethane but less than ethyne.
(A) Statement-1 is true, statement-2 is true and statement-2 is correct explanation for statement-1.
(B) Statement-1 is true, statement-2 is true and statement-2 is NOT the correct explanation for statement-1.
(C) Statement-1 is true, statement-2 is false.
(D*) Statement-1 is false, statement-2 is true.

Page No.# 1

Q.4

Among the following pair of compounds geometrical isomers are?


O

OCMe

OCMe

(A)

&

OCMe

COMe

Me
N

(B)
H

Me

Me

Me
&

C=C=C

H
(D*) None of these
Q.5

Me
&

Me

Me
(C)

[3]

H
C=C=C
Me

Identify total number of 2 amines in given compound?


Me
N

[3]

O
N
NH
N

(A) 1
Q.6

(C) 3

(D) 4

Identify total number of geometrical isomers in given compound?


H
Me

(A) 2
Q.7

(B*) 2

H2NN
(B*) 4

[3]

NNH2
(C) 6

(D) 8

Which order regarding basicity is correct:

(A*)

(C*)

>

>

[4]

(B*)

(D*)

<

>

Page No.# 2

Q.8

Q.9

Which of the following carboxylic acids could be resolved by reaction with an enantiomerically pure
chiral amine?
[3]

(A)

(B)

(C*)

(D)

Identify relationship between following pairs :

[4]

(a)

(b)

(c)

(d)

If they are enantiomer answer will be 1, if they are diastereomers answer will be 2, if they are constitutional
isomers answer will be 3 and if they are identical present 4 as the answer. Present sum of answer of each
part a + b + c + d in OMR sheet.
(for example if a, b, c, d are enantiomer then final answer is a + b + c + d = 4).
[Ans. 13]
Q.10 S(+) Mono Sodium Glutamate (MSG) is a flavour enhancer used in many foods. Fast foods often
contain substantial amount of MSG and is widely used in Chinese food. If one mole of above MSG was
placed in 845 ml solution and passed through 200 mm tube, the observed rotation was found to be +
9.6.
[9]
+
COONa

OOCCH2CH2C

NH3

MSG
(a)

Find out the specific rotation of () MSG:


(A) + 24
(B) + 56.8

(C) 48

(D*) None of these

(b)

Find out the approximate percentage composition of () MSG in a mixture containing (+) MSG and()
MSG whose specific optical rotation is 20.
(A) 83.3 %
(B) 16.7 %
(C*) 91.6 %
(D) 74 %

(c)

If 33.8 g of (+) MSG was put in 338 ml solution and was mixed with 16.9 g of () MSG put in 169 ml
solution and the final solution was passed through 400 mm tube. Find out observed rotation of the final
solution
(A) + 1.6
(B) + 4.8
(C*) + 3.2
(D) None of these
Page No.# 3

Q.11

[6]

(a)

is dimethyl derivative of a compound 'A' and is fairly stable but most of the molecules of 'A'

(b)

gets converted into another compound 'B' on keeping. What are the structures of 'A' and 'B'. Explain the
reason of conversion A into B.
Write increasing order of CN bond length in following compounds.

(c)

(i)
(ii)
(iii)
Write increasing order of heat of hydrogenation

(iv)

CH3
CH3

(i)

(ii)

A is

[Sol.(a)

(iii)

and it tautomerise into B as it is more stable because it is aromatic compound

B is
(b)
(c)

(i) < (ii) < (iii) < (iv)


(ii) > (i) > (iii)

Challenges of the life should be accepted strongly, Dont say Why me?, instead tell God Try
me!!!
Page No.# 4

ASSIGNMENT-15

Q.1

(a)

Column-I (Carbanions)

Column -II (Half lives)

(A)

(P)

4.3 108 s

(B)

(Q)

2.5 105 s

(C)

(R)

8.7 107 s

(D)

(S)

1.4 104 s

[4]

[Ans. (A) R, (B) S, (C) Q, (D) P ]


(b)

Match the column :


Column I
(Compounds)

[4]
Column II
(Magnitude of heat of hydrogenation)

(A)

(P)

52.5 Kcal / mole

(B)

(Q)

45.9 Kcal / mole

(C)

(R)

26.6 Kcal / mole

(D)

(S)

30.7 Kcal / mole


[Ans. (A) S (B) R (C) Q (D) P ]

Page No.# 5

Q.2

Column I

CH3

Cl
H
H

(A)

Column II

H
OH
Cl

CH3

Cl
HO
H

&

CH3
Me
H

Et
Me

&

OH

(Q)

Diastereomers

(R)

Conformer

(S)

Positional

SH
H

Et

OH

H2C

Et
C

H2C

CH

Et

H
Cl

Cl
H

H
&

H
Cl

CH

Me
Cl

Me

&

(D)

Identical

H
H

(C)

(P)

CH3

HS

(B)

H
H
Cl

[4]

H
[Ans. (A) P, (B) S, (C) R, (D) Q ]

Q.3

Questions below consist of an Assertion in column I and Reason in column II. Use the following
key to choose the appropriate answer
[3]
(A) If both Assertion and Reason are correct and Reason is the correct explanation of the Assertion.
(B) If Assertion and Reason are correct, but Reason is not the correct explanation of the Assertion.
(C) If Assertion is correct, but Reason is incorrect.
(D) If Assertion is incorrect, but Reason is correct.
Assertion : n-propyl carbocation is more stable than ethyl carbocation.
Reason : n-propyl carbocation rearrange to give 2 carbocation.
[Ans. D]

Q.4

Which of following pair is Diastereomers:

H
(A)

CO2H
OH

HO

CH3 H

CO2H
H

OH

OH

(B*)

CH3

C=C

C
Br Br
H

C=C

H
CH3

CO2H

CH3

Et

Cl

Cl

OH

HO

SH

HS

Et

CH3

H
CO2H

(C)

[3]

Cl

Cl

Cl

Cl

(D)

CH3
Page No.# 6

Q.5

What is the correct order of basicity values of the following compounds?


CH2 NH2
CH2 NH2
(A) |
> |
> CH3NH2 > NH3
+SMe
Br
2
(B) NH3 > CH3NH2 > CH2=CH-NH2 > NH2-CH2-CH2-N+R3

(C)

>

>

>

[3]

>

(D*) None
Q.6

Consider the following compound

(A*)

(B*)

[4]

O
||
(C*)
CH 3CCOOH

(D*)

Which of the above compounds reacts with NaHCO3 giving CO2


Q.7

Out of following the most basic isomers of the formula C4H7N is


(A*) CCCCNH2 (B) CCCCNH2 (C) CCCCNH2 (D)

Q.8

[3]
N
|
H

Select incorrect statement from the following.

[4]
O

(A*)

(B)

COOH
CH3

is having very high resonance energy than

COOH
is more acidic than

(C*)

CH2 is more stable than OHr.

(D*)

O
||
O is more stable than Me C O

Page No.# 7

Q.9

Following figure give Potential energy of different types of isomers of unsaturated compound CnH2n2.

Potential Energy

There are three isomers shown is the following figure with there H of (standard heat of formation),
which involves two alkynes and one alkadiene, with 5 carbon atoms. Usually alkynes are less stable than
isomeric alkadienes as sp-hybridisation state is relatively less stable than the sp2-hybridisation state.
Based on it answer the following questions.
[9]

(a)

Which of the following will react with NaNH2 most readily.


(A*) A
(B) B
(C) C

(D) All of these

(b)

Which is most stable alkyne?


(A) A
(B*) B
(C) C
(D) Additional data os required to decide relative stability

(c)

Which are of the following is not the isomer of the compounds shown is figure:
(A)

(B*)

(C)

(D)

Q.10 Observed the following compound and answer the following questions.
Me
NH2 c
b
N
H a O

(a)

(b)
(c)

[9]

O
d

Me

OH
e

Arrange the H-atom in decreasing order of their acidic strength.


(A) a > b > e
(B*) a > e > b
(C) b > a > c

(D) e > a > b

Arrange the N-atom in decreasing order of their basicity


(A*) b > d > c
(B) c > b > d
(C) d > c > b

(D) c > d > b

What will be the degree of unsaturation


(A) 5
(B) 6

(D) 8

(C*) 7

Success is waiting for you. Difficulties are blocking the path. Take the weapon of confidence and
reach your success every day!.....
Page No.# 8

ASSIGNMENT-16

Q. 1

Match the following :

[4]

Column I

Column II
(Number of planes of symmetry)

Me

(A)
Me

CCl4
H

(Q)

(R)

(S)

Me

(B)

(C)

(P)

CH3 CH3

(D)
H3C

[Ans. (A) S (B) R (C) P (D) Q]

Q.2

Q.3

Question No. 2 to 3 ( 2 questions)


Questions below consist of an Assertion in column I and Reason in column II. Use the following
key to choose the appropriate answer
(A) If both Assertion and Reason are correct and Reason is the correct explanation of the Assertion.
(B) If Assertion and Reason are correct, but Reason is not the correct explanation of the Assertion.
(C) If Assertion is correct, but Reason is incorrect.
(D) If Assertion is incorrect, but Reason is correct.
Assertion
:
Heat of hydrogenation of trans-2-butene is more than cis-2-butene.
[3]
Reason
:
Trans-2-butene is more stable cis-2-butene.
[Ans. D]
Assertion :
Reason :

Cyclohexene shows geometrical isomerism.


Its two sp2 H-atoms are cis to each other.

[And. D] [3]

Page No.# 9

Q.4

Which of the following is most stable carbocation.

[3]

OEt

OEt

(A)

(B)

OEt

(C*)

OEt

(D)

Question No. 5 to 8 (4 questions)

Cortisone

Q.5

Q.6

Q.7
Q.8

Q.9

Cortisone contains which functional groups?


(A) Ether, alkene, alcohol
(C*) Alcohol, ketone, alkene
Correct order of acidic strength in Cortisone
(A) a > b > c > d
(B*) b > a > c > d
Total number of chiral center in Cortisone :
(A) 4
(B) 5

[3]

(C) c > b > a > d

(D) b > c > a > d


[3]

(C*) 6

(D) 7

Total stereoisomer of the compound Cortisone is


(A) 32
(B*) 64
(C) 66

[3]

(D) 128

Arrange the following in increasing order of their heat of combustion:

I.
Q.10

[3]

(B) Alcohol, ketone, amine


(D) Ether, amine, ketone

II.

III.

[5]

[Ans. I < II < III ]

For the following compounds, arrange the labelled proton in increasing order of their case of deprotonation:

C CH3
(a)

(b)

CH2
H1

(c)

[5]

H2
[Ans. (a) 2 < 1 < 3

(b) 3 < 1 < 2

(c) 3 < 1 < 2 ]

There isn't any problem which is without a gift for you in its hand.
Page No.# 10

ASSIGNMENT-17

Q.1

Four compounds are given in column I, match these with their pK a1 values given in column II. [4]
Column I

Column II

(Compound)

( pK a1 )

(A)

CH3COOH

(P)

4.87

(B)

COOH
|
COOH

(Q)

4.76

(C)

ClCH2COOH

(R)

2.86

(D)

CH3 CH2COOH

(S)

1.2
[Ans. (A) Q, (B) S, (C) R, (D) P]

Q.2

Statement-I : A tertiary carbocation is more stable than a secondary carbocation which is more
stable than a primary carbocation.
Statement-II : The inductive effect operates through -bonds and decrease rapidly with increase
in distance.
[3]
(A) Statement-1 is true, statement-2 is true and statement-2 is correct explanation for statement-1.
(B*) Statement-1 is true, statement-2 is true and statement-2 is NOT the correct explanation for statement-1.
(C) Statement-1 is false, statement-2 is true.
(D) Statement-1 is true, statement-2 is false.

Q.3

Statement-1: p-amino benzoic acid do not exists as zwittor ion


[3]
Statement-2: Acidic and basic groups in this compound are too far from each other to react, so they
can't form salt.
(A) Statement-1 is true, statement-2 is true and statement-2 is correct explanation for statement-1.
(B) Statement-1 is true, statement-2 is true and statement-2 is NOT the correct explanation for statement-1.
(C) Statement-1 is false, statement-2 is true.
(D*)Statement-1 is true, statement-2 is false.

Q.4

Arrange the following in increasing order of acid strength.


(f)

(a)

OH

[3]

(b)

COOH

O
(e)

HO3S
(A) c < f < d < a < b < e
(C) f < c < d < b < e < a

(d)

(c)

HCH2 CH2H
(B) f < d < c < b < e < a
(D*) c < d < f < a < b < e

Page No.# 11

Q.5

Select true statement(s) :


[3]
(A*) Resonance effects bond length
(B*) cis-1-bromo-1,2-difluoro ethene and Z-1-bromo-1,2-difluoro ethene are geometrical isomers

O
COH
(C*) In CH2

most acidic H is connected directly to oxygen not on carbon.

COH
O

OH
OH

(D*) Boiling point of

Q.6

OH
is less than

Which of the following is the correct order of acidic nature.

[3]

(I)

(III)
H

(B) I > II > III

(C) II > I > III

(D) III > II > I

Which of the following is the correct order of basic nature.

(P)

(Q)

N
H

(A) Q > R > P


Q.8

(II)

H
H
(A*) III > I > II

Q.7

[3]

(R)

N
N
(C) R > P > Q

(B*) R > Q > P

(D) Q > P > R

Correct increasing order of acidic strength of following compounds is / are ?


OH

O
(A) HN

,
OH

(B*)

H
N

OH

OH

N
H

OH

Cl
F

OH

(C*)

(D*) CH3C CH

OH

H2O

OH

Cl

OH
F

Br

,
Br

Cl

Br

[3]

HF

Page No.# 12

Q.9

Among the following compounds select those


(i)
Which will readily dimerise.
(ii)
Which are Quasi aromatic

(1)

(2)

[6]

C 4 H 24

(3)

tBu

(4)

tBu

(5)

(6)
tBu

[Ans. (1)Quasi
(4)Quasi
(6) Quasi

Azulene

(2) Quasi
(3) Quasi
(5) It is antiaromatic but does not undergos dimerisedue to bulky t-Bu groups.
]

Q.10 Resonance energy of

is much lower than that of

but resonance energies of

are identical. Explain.

Q.11

[5]

Arrange the following in increasing order of pKa values


(a) HNO3
H2 O
HCO2H
(b) CH4
PhCH3
PhC CH
(c)PhCH3

(d) PhCH3

(e) NH3

(f)

[6]
CH3CO2 H

CF3 CO2 H

HCN

and

HCO2H

PhOH

NH3

CH3OH

PhNH2

CH2=CHOH

[Ans. (a) 1 < 3 < 4 < 5 < 2 (b) 4 < 3 < 2 < 1
(e) 2 < 4 < 3 < 1
(f) 4 < 1 < 3 < 2 ]

HC COH
(c) 3 < 2 < 4 < 1

(d) 3 < 2 < 4 < 1

Don't repeat what you have already done, unless you can do it in a better way
Page No.# 13

ASSIGNMENT-18

Q.1

Match the pKa values:

[4]

Column I

Column II

(A)

H3O+

(P)

(B)

CH3CO2 H

(Q)

(C)

NH 4

(R)

4.76

(D)

H2SO4

(S)

9.4

[Ans. (A) Q; (B) R; (C) S ; (D) P ]


Assertion & Reason. (Q.2 & Q.3)
[3 + 3]
(A) Statement-1 is true, statement-2 is true and statement-2 is correct explanation for statement-1.
(B) Statement-1 is true, statement-2 is true and statement-2 is NOT the correct explanation for statement-1.
(C) Statement-1 is true, statement-2 is false.
(D) Statement-1 is false, statement-2 is true.
O
Q.2

Statement-1 :

Statement-2 :

S
is more stable than

[Ans. D]

Overlapping between orbitals of C and O is better than orbitals of C and S

O
O
Q.3

Statement-1 :

having more resonance energy than


O

Statement-2 :

Extended conjugation is more effective the cross conjugation.

Q.4

[Ans. D]

[3]
Correct order of basicity is
(A) 3>1>2>4
(B) 3<1<2<4

(C) 3<4<1<2

(D*) 3>4>1>2

Page No.# 14

Q.5

Q.6

Q.7

Which one of following represents different molecules?

(A)

and

(C)

and

[3]

(B*)

and

(D)

and

Arrange the following carbocations in the increasing order of their stability.


(I)

(II)

(III)

(A*) I > II > III

(B) I > II = III

(C) I > III > II

[3]

(D) III > I > II

The most stable resonating structure of following compound is

(A)

(B)

(C)

(D*)

[3]

Q.8

[5]

(i)

Rank the following in order of decreasing acidity.

(ii)

[Ans. 1 > 4 > 3 > 2]


Consider the following compound. Rank the labeled proton in increasing order of acidity

(a)
[Ans. (a)
(b)
Q.9
(a)
(b)

(b)

Write the stepwise ionization of acid

Carboxy H is more acidic than hydroxy or thioxy hydroxy, between OH and SH, latter H
is more acidic thus the order of acid strength is: c < b < a
x>y>z
]

In which solution F

NaF is dissolved in (CH3)2SO and CH3OH.


Explain why iso octane is less viscous than n-octane.

[5]
will be more reactive and why?

Page No.# 15

Q.10
(a)
Write correct order of acidity of a to e marked in structure of vitamin B6.

[6]

[Ans. b > e > d > a > c]

(b)

Write correct order of acidic strength as a to d.

[Ans. a > c > b > d ]

(c)

Write the correct order of pKa of marked position CH bond in following compound.

[Ans. R > T > P > Q > S ]

Q.11
(a)

[6]
Ordinarily the barrier to rotation about a carbon-carbon double bond is quite high but compound A
have a rotational barrier of only about 20 K cal / mole
What is the reason for this ?
(A)

(b)

Which is more basic & why ?

(c)

Among

&

[Ans. A < B]

which is more acidic & why? Explain through canonical

forms.
You can't get different fruit, if you keep doing the same things.
Page No.# 16

ASSIGNMENT-19

Q.1

Match the names of carboxylic acids in column I with pka value in column II.
Column I
Column II
(A)

Benzoic acid

(P)

4.17

(B)

Ethanoic acid

(Q)

4.14

(C)

o-methyl benzoic acid

(R)

4.74

(D)

p-flourobenzoic acid

(S)

3.91

[4]

[Ans. (A) P, (B) R, (C) S, (D) Q]


I
O2N
Q.2

Br

NO2
[3]

z
NO2

Correct order of bond length is


(A) z > y > x
(B*) x > y > z

Q.3

(C) y > x > z

(D) x > z > y

Ph N 2 is more stable than R N 2 because

[3]

(A) in Ph N 2 , +ve charge undergoing delocalisation in the ring.


(B) Ph having I effect whereas R having +I effect.
(C) in Ph group carbon is more electron defficient w.r.t. carbon in R group.
(D*) all are incorrect
Q.4

Q.5

Arrange the following in the increasing order of their basic strength:

[3]

I.

II.

III.

IV.

(A) I < II < III < IV

(B) IV < III < II < I

(C) IV < I < II < III

(D*) III < II < IV < I

Energies of three resonating structures of a compound are E1, E2 & E3 respectively and energy of real
molecule is E0 . If E1 > E2 > E3 then the resonance energy will be:
[3]
(A) (E + E2 + E3) E0
(B) E1 E0
(C*) E3 E0

(D)

E1 E 2 E 3
3

Page No.# 17

Q.6

Which order of acid strength is wrong

(A)

(C*)

Q.7

>

>

[3]

(B)

>

(D)

>

In which of the following the 2nd structure having more resonance energy.
(A*)

(B*)

[4]

vs

vs

(C*) CH2=CHCH=CH CH 2 vs

(D*)

Q.8
(a)

vs

[3+3]
In which of the following pairs, indicated bond is of greater strength.

O
||
(i) CH 3 C Cl & CH3 CH 2 Cl , (ii)

(b)

&

(iii)

&

Arrange the following compound, in order of CN bond length.


(A)

(B)

(C)

(D)

[Ans. D > C > A > B ]

Page No.# 18

Q.9

Arrange in order of CH bond energy

[4]

[Ans. d < f < b < c < a < e


On the basis of stability of free radical formed after removal of H

Q.10 Which of the following pairs is stronger acid. Suggest a suitable explanation in each case.

(i)

(ii)

and

and

Q.11

(a)

[4]

[8]

Arrange these compounds in basic strength order:


[Ans.
(b)

I > II > III

Following compound contains three C=C, arrange these in stability order:

[Ans. b > a > c ]


(c)

(i) Write the correct order of case of deprotonation of labelled H-atoms.

[Ans. 1 > 2 > 3 > 4 ]

Page No.# 19

(ii)

[Ans. II > I > III ]

Opportunities are better recognised not when they are coming but when they are going.
Page No.# 20

ASSIGNMENT-16
Q. 1

Match the following :


Column I

[4]
Column II
(Number of planes of symmetry)

Me

(A)
Me

CCl4
H

CH3 CH3

(D)

Q.2

Q.3

Q.4

(Q)

(R)

(S)

Me

(B)

(C)

(P)

H
H3C
Question No. 2 to 3 ( 2 questions)
Questions below consist of an Assertion in column I and Reason in column II. Use the following
key to choose the appropriate answer
(A) If both Assertion and Reason are correct and Reason is the correct explanation of the Assertion.
(B) If Assertion and Reason are correct, but Reason is not the correct explanation of the Assertion.
(C) If Assertion is correct, but Reason is incorrect.
(D) If Assertion is incorrect, but Reason is correct.
Assertion
:
Heat of hydrogenation of trans-2-butene is more than cis-2-butene.
[3]
Reason
:
Trans-2-butene is more stable cis-2-butene.

Assertion :
Reason :

Cyclohexene shows geometrical isomerism.


Its two sp2 H-atoms are cis to each other.

[3]

Which of the following is most stable carbocation.


OEt

[3]
OEt

(A)

OEt

(C)

(B)

OEt

(D)
Page No.# 1

Question No. 5 to 8 (4 questions)

Cortisone

Q.5

Q.6

Q.7
Q.8

Q.9

Cortisone contains which functional groups?


(A) Ether, alkene, alcohol
(C) Alcohol, ketone, alkene
Correct order of acidic strength in Cortisone
(A) a > b > c > d
(B) b > a > c > d
Total number of chiral center in Cortisone :
(A) 4
(B) 5

[3]

(C) c > b > a > d

(D) b > c > a > d


[3]

(C) 6

(D) 7

Total stereoisomer of the compound Cortisone is


(A) 32
(B) 64
(C) 66

[3]

(D) 128

Arrange the following in increasing order of their heat of combustion:

I.
Q.10

[3]

(B) Alcohol, ketone, amine


(D) Ether, amine, ketone

II.

[5]

III.

For the following compounds, arrange the labelled proton in increasing order of their case of deprotonation:

C CH3
(a)

(b)

CH2

(c)

[5]

H2

H1

There isn't any problem which is without a gift for you in its hand.
Page No.# 2

CLASS : XIII (VXYZ)


Q.1

MARKS:42

DATE : 02-03/09/2009

TIME :40 MIN. DPP. NO.-17

Four compounds are given in column I, match these with their pK a1 values given in column II. [4]
Column I

Column II

(Compound)

( pK a1 )

(A)

CH3COOH

(P)

4.87

(B)

COOH
|
COOH

(Q)

4.76

(C)

ClCH2COOH

(R)

2.86

(D)

CH3 CH2COOH

(S)

1.2

Q.2

Statement-I : A tertiary carbocation is more stable than a secondary carbocation which is more
stable than a primary carbocation.
Statement-II : The inductive effect operates through -bonds and decrease rapidly with increase
in distance.
[3]
(A) Statement-1 is true, statement-2 is true and statement-2 is correct explanation for statement-1.
(B) Statement-1 is true, statement-2 is true and statement-2 is NOT the correct explanation for statement-1.
(C) Statement-1 is false, statement-2 is true.
(D) Statement-1 is true, statement-2 is false.

Q.3

Statement-1: p-amino benzoic acid do not exists as zwittor ion


[3]
Statement-2: Acidic and basic groups in this compound are too far from each other to react, so they
can't form salt.
(A) Statement-1 is true, statement-2 is true and statement-2 is correct explanation for statement-1.
(B) Statement-1 is true, statement-2 is true and statement-2 is NOT the correct explanation for statement-1.
(C) Statement-1 is false, statement-2 is true.
(D)Statement-1 is true, statement-2 is false.

Q.4

Arrange the following in increasing order of acid strength.


(f)

(a)

OH

[3]

(b)

COOH

O
(e)

HO3S
(A) c < f < d < a < b < e
(C) f < c < d < b < e < a

(d)

(c)

HCH2 CH2H
(B) f < d < c < b < e < a
(D) c < d < f < a < b < e

Page No.# 3

Q.5

Select true statement(s) :


[3]
(A) Resonance effects bond length
(B) cis-1-bromo-1,2-difluoro ethene and Z-1-bromo-1,2-difluoro ethene are geometrical isomers

O
COH
(C) In CH2

most acidic H is connected directly to oxygen not on carbon.

COH
O

OH
OH

(D) Boiling point of

Q.6

OH
is less than

Which of the following is the correct order of acidic nature.

[3]

(III)
H

(B) I > II > III

(C) II > I > III

(D) III > II > I

Which of the following is the correct order of basic nature.

(P)

(Q)

N
H

(A) Q > R > P


Q.8

(II)

H
H
(A) III > I > II

Q.7

(I)

[3]

(R)

N
N
(C) R > P > Q

(B) R > Q > P

(D) Q > P > R

Correct increasing order of acidic strength of following compounds is / are ?


OH

O
(A) HN

,
OH

(B)

H
N

OH

OH

N
H

OH

Cl
F

OH

(C)

(D) CH3C CH

OH

H2O

OH

Cl

OH
F

Br

,
Br

Cl

Br

[3]

HF

Page No.# 4

Q.9

Among the following compounds select those


(i)
Which will readily dimerise.
(ii)
Which are Quasi aromatic

(1)

(2)

[6]

C 4 H 24

(3)

tBu

(4)

tBu

(5)

(6)
tBu

Q.10 Resonance energy of

Azulene

is much lower than that of

but resonance energies of

are identical. Explain.

Q.11

[5]

Arrange the following in increasing order of pKa values


(a) HNO3
H2 O
HCO2H
(b) CH4
PhCH3
PhC CH
(c)PhCH3

(d) PhCH3

CF3 CO2 H

HCN

(e) NH3

(f)

and

HCO2H

[6]
CH3CO2 H

PhOH

NH3

CH3OH

PhNH2

CH2=CHOH

HC COH

Don't repeat what you have already done, unless you can do it in a better way
Page No.# 5

ASSIGNMENT-18

Q.1

Match the pKa values:

[4]

Column I

Column II

(A)

H3O+

(P)

(B)

CH3CO2 H

(Q)

(C)

NH 4

(R)

4.76

(D)

H2SO4

(S)

9.4

Assertion & Reason. (Q.2 & Q.3)


[3 + 3]
(A) Statement-1 is true, statement-2 is true and statement-2 is correct explanation for statement-1.
(B) Statement-1 is true, statement-2 is true and statement-2 is NOT the correct explanation for statement-1.
(C) Statement-1 is true, statement-2 is false.
(D) Statement-1 is false, statement-2 is true.
O
Q.2

Statement-1 :

Statement-2 :

S
is more stable than

Overlapping between orbitals of C and O is better than orbitals of C and S

O
O
Q.3

Statement-1 :

having more resonance energy than


O

Statement-2 :

Extended conjugation is more effective the cross conjugation.

Q.4

[3]
Correct order of basicity is
(A) 3>1>2>4
(B) 3<1<2<4

(C) 3<4<1<2

(D) 3>4>1>2

Page No.# 6

Q.5

Q.6

Q.7

Which one of following represents different molecules?

(A)

and

(B)

(C)

and

(D)

[3]

and

and

Arrange the following carbocations in the increasing order of their stability.


(I)

(II)

(III)

(A) I > II > III

(B) I > II = III

(C) I > III > II

[3]

(D) III > I > II

The most stable resonating structure of following compound is

(A)

(B)

(C)

(D)

[3]

Q.8

[5]

(i)

Rank the following in order of decreasing acidity.

(ii)

Consider the following compound. Rank the labeled proton in increasing order of acidity

(a)

Q.9
(a)
(b)

(b)

Write the stepwise ionization of acid

[5]
NaF is dissolved in (CH3)2SO and CH3OH. In which solution F will be more reactive and why?
Explain why iso octane is less viscous than n-octane.

Page No.# 7

Q.10
(a)
Write correct order of acidity of a to e marked in structure of vitamin B6.

[6]

(b)

Write correct order of acidic strength as a to d.

(c)

Write the correct order of pKa of marked position CH bond in following compound.

Q.11
(a)

[6]
Ordinarily the barrier to rotation about a carbon-carbon double bond is quite high but compound A
have a rotational barrier of only about 20 K cal / mole
What is the reason for this ?
(A)

(b)

Which is more basic & why ?

(c)

Among

&

which is more acidic & why? Explain through canonical

forms.
You can't get different fruit, if you keep doing the same things.
Page No.# 8

CLASS : XIII (VXYZ)


Q.1

MARKS:46

DATE : 07-08/08/2009

TIME :40 MIN. DPP. NO.-19

M atch the names of carboxylic acids in column I with pka value in column II.

Column I

[4]

Column II

(A)

Benzoic acid

(P)

4.17

(B)

Ethanoic acid

(Q)

4.14

(C)

o-methyl benzoic acid

(R)

4.74

(D)

p-flourobenzoic acid

(S)

3.91

I
O2N
Q.2

Br

NO2
[3]

z
NO2

Correct order of bond length is


(A) z > y > x
(B) x > y > z

Q.3

(C) y > x > z

(D) x > z > y

Ph N 2 is more stable than R N 2 because

[3]

(A) in Ph N 2 , +ve charge undergoing delocalisation in the ring.


(B) Ph having I effect whereas R having +I effect.
(C) in Ph group carbon is more electron defficient w.r.t. carbon in R group.
(D) all are incorrect
Q.4

Q.5

Arrange the following in the increasing order of their basic strength:

[3]

I.

II.

III.

IV.

(A) I < II < III < IV

(B) IV < III < II < I

(C) IV < I < II < III

(D) III < II < IV < I

Energies of three resonating structures of a compound are E1, E2 & E3 respectively and energy of real
molecule is E0 . If E1 > E2 > E3 then the resonance energy will be:
[3]
(A) (E + E2 + E3) E0
(B) E1 E0
(C) E3 E0

(D)

E1 E 2 E 3
3

Page No.# 9

Q.6

Q.7

Which order of acid strength is wrong

[3]

(A)

>

(B)

>

(C)

>

(D)

>

In which of the following the 2nd structure having more resonance energy.
(A)

(B)

[4]

vs

vs

(C) CH2=CHCH=CH CH 2 vs

(D)

Q.8
(a)

vs

[3+3]
In which of the following pairs, indicated bond is of greater strength.

O
||
(i) CH 3 C Cl & CH3 CH 2 Cl , (ii)

(b)

&

(iii)

&

Arrange the following compound, in order of CN bond length.


(A)

(B)

(C)

(D)

Page No.# 10

Q.9

Arrange in order of CH bond energy

[4]

Q.10 Which of the following pairs is stronger acid. Suggest a suitable explanation in each case.

(i)

(ii)

and

and

Q.11

(a)

[4]

[8]

Arrange these compounds in basic strength order:


(b)

Following compound contains three C=C, arrange these in stability order:

(c)

(i) Write the correct order of case of deprotonation of labelled H-atoms.

(ii)

Opportunities are better recognised not when they are coming but when they are going.
Page No.# 11

ASSIGNMENT-20

Q.1

Column I

Column II

[4]

Cl

(A)

Cl

Cl

Cl

Cl

(P)

Compound can show optical isomerism

(Q)

Compound can show geometrical isomerism

(R)

Compound can show positional isomerism

(S)

Structure may have centre of symmetry in one


of its form.
[Ans. (A) P,Q,R,S (B) R,S(C) P,Q,R]

Cl
Cl

(B)
Cl

(C)

Q.2

Assertion & Reason.


[3]
(A) Statement-1 is true, statement-2 is true and statement-2 is correct explanation for statement-1.
(B) Statement-1 is true, statement-2 is true and statement-2 is NOT the correct explanation for statement-1.
(C) Statement-1 is true, statement-2 is false.
(D) Statement-1 is false, statement-2 is true.
Statement-1 :
LDA is a very strong nucleophile but it is not a very good base.
Statement-2 :
Nucleophilicity is a kinetic property but basicity is thermodynamic property.
[Ans. D]

Q.3

Select the incorrect statement.


(A) Cyclopropane is more acidic than propane
Br
F

(B*)

is a meso compound with (2S,3S) configuration


F Br
(C) N-methylaniline is a non-resolvable compound
COOH

(D)

COOH

is more acidic than


NO2

NO2

[3]

Q.4

Give number of the products including stereoisomers formed in the following reaction.

[3]

CH2 NBS
h

(A) 5
Q.5

(B*) 6

(C) 7

Percentage of hydrate

[3]

OH
|
+ H2O l H C OH
|
H

OH
|
+ H2O l CH 3 C OH
|
CH 3

X & Y respectively is
(A) X = 0.2 %, Y = 99.95
(C) X = 0.2%, Y = 0.2%
Q.6

(B*) X = 99.9%, Y = 0.2%


(D) X = 99.9%, Y = 99.9%

Which is the most likely site of protonation.

(A) a

Q.7

(D) 4

(B) b

[3]

(C) c

(D) d

O
||

Me C O CH 2 CH 2 N H 3 NaOH
Q

[3]

Q is:

Q.8

O
||
(A) Me C O CH 2 CH 2 NH 2

O
||
(B*) Me C NH CH 2 CH 2 OH

(C)

(D) MeCOONa + HOCH2CH2NH2

Which form of given compound is more stable. MeCH CH Me


|
|
(a) meso
Et
Et
(b) optically active

[3]

Q.9

o-nitrobenzoic acid (Pka = 2.21) is stronger acid than 3,5-dinitrobenzoic acid (Pka = 2.80) in water
whereas in ethanolic solution the opposite is true (their respective Pka in ethanol are 8.82 and 8.09).
Suggest a possible reason for this.
[5]

Q.10 Give product(s) in each of the following reactions


(a)

(b)
(c)

(d)

/ hv
CH3 CH CH2 CH2 CH3 Br
2
(A)
|
CH 3

( C H CO ) O

+ NBS 6 5 2 (B)
hv
CH3 CH2 CH = CH2 + Me3COCl
(C) + (D)
CH 3
|
CH 3 C O Cl /
|
CH 3

C6H5 CH2 CH2 CH3 (E)

Remember you are born to live not living because you are born.

[5]

ASSIGNMENT-21

Q.1

Column I
(Molecular formula)

Column II
(Theoritically possible isomers)

(A)

C2 H5 N

(P)

Tautomers

(B)

C3 H6 O

(Q)

Functional isomers

(C)

C2 H4 O2

(R)

Geometrical isomers

[4]

(S)
Optical isomers
[Ans. (A) P,Q,R (B) P,Q,R,S (C) P,Q,R,S ]

Q.2

Statement-1 : Yield of R Br follows the order 1 < 2 < 3 in Hundsdicher reaction


[3]
Statement-2 : Hundsdicher reaction follows free redical mechanism.
(A) Statement-1 is true, statement-2 is true and statement-2 is correct explanation for statement-1.
(B) Statement-1 is true, statement-2 is true and statement-2 is NOT the correct explanation for statement-1.
(C) Statement-1 is true, statement-2 is false.
(D*) Statement-1 is false, statement-2 is true.

Q.3

Which of the following is/are the chain termination step during photochlorination of ethane?
(1)

[3]

CH3 CH 2 Cl CH3CH2Cl

(2) CH 3 CH 2 CH 3CH 2 CH3CH2CH2CH3


(3)

Cl Cl Cl2

(4)
Cl H HCl
(A) 1 and 2
(B) 1 and 3

Q.4

(C*) 1, 2 and 3

(D) 1, 2, 3 and 4

Pick the correct statement for monochlorination of R-secbutyl chloride.


Cl

2
300C

(A) There are four possible products ; three are optically active one is optically inactive
(B) There are five possible products ; three are optically inactive & two are optically active
(C*) There are five possible products ; two are optically inactive & three are optically active
(D) There are four possible products ; two are optically active & two are optically inactive

[3]

OH

Q.5

2RCOONa

2RCOOH

(B) forward direction


(D) can not be predicted without pKa

Select stable molecule (s):

(A*)

(B*) CCl3CH(OH)2

[3]

ONa

above reaction is favoured is which direction.


(A*) backward direction
(C) equal in both direction
Q.6

ONa

OH

[3]

(C)

(D)

Q.7

When an aqueous solution containing MeCOOK and Me3CCOOK is electrolysed, possible product(s)
is/are:
[5]
(A*) Me Me
(B*) Me4C
(C*) Me3CCOOCMe3
(D) CH2 = CH2

Q.8

CH3NO2 when dissolved in NaOH it forms salt. When the Na salt is acidified at low temperature there
is not always an immediate separation of oily drops (of nitro form). On standing however the acidified
solution slowly deposits oily drops. How you explain the time factor in this observation.
[3]

Q.9

Hydrate of following compound are stable explain.

(a)

O
||
(b) CCl 3 C H

O
||
(d) CF3 C CF3

O O O
|| || ||
(e) Ph C C C Ph

Q.10 Identify unknown:

(ii ) H 2SO 4 ,

Na

(ii)

dry ether
14

* C)
(C
I

(iii)

(c)

[6]
i ) Mg ( Hg )
(

(i)

[5]

Cu

Q.11

Each of the following carbocations has the potential to rearrange to a more stable one. Write the structure
of the rearranged carbocation.
[8]
(a)

CH3CH2CH2+

(c)

(CH 3 )3 C CHCH 3

(b)

(CH 3 ) 2 CH CHCH 3

(d)

(CH3CH2)3CCH2+

(e)

(f)

(g) C H 2 CH 2 CH 2
|
CH 3 CH 2 O

(h)

Every bad situation will have something positive. even a stopped clock
shows correct time twice a day... so be positive in life.

ASSIGNMENT-28

The reaction of alkene or Alkyne with ozone (O3) followed by hydrolysis is known as
ozonolysis.
It is two types : (I)
Reductive ozonolysis In presence of reducing agent
(II)
Oxidative ozonolysis In presence of oxidising agent
Reducing agents :
Zn, H2O or Zn, CH3COOH or (CH3)2S or (Ph)3P etc.

OZONOLYSIS :

Oxidising agents :

Example 1 :

step I
70C

O3

C=C
R

H2O2 or R C O O H or Ag2O etc.


||
O
R H O/Zn
3

+ RCH

C=O
Reductive
ozonolysis R

[SCT- Cut the double bond


and paste two oxgen atoms
and vice versa]

Oxidative
ozonolysis H2O2

RCR + RCOH
O

Mechanism :

R
R

R
C

C=O
H

H
R

R
R

R C

RCO

O
R

O=C
H

O
R

RCR + RCH
O

H
Note : In case of oxidative ozonolysis aldehyde (not ketone) further undergoes oxidation which
gives acid as product.
Q.1

Give the product of the following reaction.

(i)

i ) O3
H2C = CH2 (

(ii ) Zn / H 2O

H
[Sol.

H
O + O=C

[7 2 =14]

(ii)

i ) O3
CH3CH = CH2 (

(ii ) Zn / H 2O

[Sol.

(iii)

O
H
||
CH 3 C H + H

C=O ]

CH 3
|
i ) O3
CH 3 C CH 2 (

(ii ) Zn / H 2O

CH3
[Sol.

CH3

H
+

C=O ]

i ) O3
(

(iv)

(ii ) Zn / H 2O

O
[Sol.

O
H]

H
i ) O3
(

(v)

(ii ) Zn / H 2O

H
[Sol.

i ) O3
(

(vi)

(ii ) Zn / H 2O

O
[Sol.

(vii)

+ H

i ) O3
H2C=CHCH2CH=CHCH3 (

(ii ) Zn / H 2O

H
[Sol.
Q.2
(i)

C=O + O C CH 2 C O + O C CH 3 ]
|
|
|
H
H
H
Find out the structure of reactant.
H

i ) O3
X (
CH 3 CH 2 C H
(ii ) Zn / H 2O
||
O

[Sol.

[11 2 =22]

H
(ii)

(i ) O3

O +

X
(ii ) Zn / H 2O

H
[Sol.

]
H
H
O+

(iii)

(i ) O3

(ii ) Zn / H 2O

H
[Sol.

]
O

(iv)

(i ) O3

(ii ) Zn / H 2O

H
H ]

[Sol.

(v)

i ) O3
X (

(ii ) Zn / H 2O

[Sol.

O
(vi)

i ) O3
X (

(ii ) Zn / H 2O

[Sol.

CH3

C=O +

H3C

O
(vii)

i ) O3
X (

(ii ) Zn / H 2O

[Sol.

C=O

(viii)

i ) O3
(

X
(ii ) Zn / H 2O
C10 H12

[Sol.

O + HCHO
O

O
(ix)

[Sol.

(x)

(i ) O
X 3
C12H18 (ii ) Zn / H 2O

+ O=C=O + O

+ HCHO

CH3
C=CC=CH2
CH3 ]

i ) O3
C 6 H 4 (
C3H2O3
(
ii
)
Zn
/ H 2O
( X)

[Sol.

(xi)

i ) O3
(

X
(ii ) Zn / H 2O
C12 H18

+ HCHO
O

[Sol.

Q.3

Give the ozonolysis product of the following.


H

(i)

O3 ( )

X 3 O
Zn

H
O

[Sol.

[3 2 =6]

O3

(ii)

Zn / H 2 O

How many species.


H
[Sol.

Only one

]
O
3/
O

(iii)

Zn / H 2 O

How many species are found.


CH3
[Sol.
CH3

CH3

C
+

C
O

O
C

C
H

O
C

C
H

When it rains all birds occupy shelter, but eagles, the only bird, avoids rain by flying above the
cloud....Problem is same but ATTITUDE makes a DIFFERENCE.

ASSIGNMENT-29

OXYMERCURATION-DEMERCURATION (OMDM)
OMDM is a hydration process of alkene according to Markawnikoffs rule with no rearrangement of
cyclic mercuinium ion. In oxymercuration, the alkene is treated with mercuric acetate in aqueous
tetrahydrofuran (THF). When reaction with that reagent is complete, sodium borohydride and hydroxide
ion are added to the reaction mixture.
(i ) Hg ( OAC ) 2 , H 2O,THF
R CH CH 3
RCH = CH2
(ii ) NaBH4 ,HO
|
OH

(i)Hg(OAc)2, MeOH, THF


(ii) NaBH 4, HO

R CH CH 3
|
OMe

AcO = CH3COO
Mechanism for oxymercuration:
OAc
OAc
CH3CH = CH2

+ Hg OAc

Hg

CH3CH CH2

H2 O

CH3CHCH2HgOAc
+

OH

+ AcO

H
OAc
CH3CHCH2HgOAc
OH

+ AcOH

Sodium borohydride (NaBH4) converts the carbon-mercury bond into a carbon-hydrogen bond. Because
the reaction results in the loss of mercury, it is called demercuration.

4 CH 3CHCH 3 + Hg + AcO
CH 3CHCH 2 HgOAc NaBH
HO
|
|
OH
OH

OCH3
(i ) Hg ( OCOCH 3 ) CH 3OH

Q.1

(ii ) NaBH 4 ,OH

[Ans.

[2]

OH
i ) Hg ( OAc ) 2 , H 2O
(

Q.2

[Ans.

[2]

(ii ) NaBH 4 ,OH

CH3

Q.3

H3C

i ) Hg (CF3COO )2 ,CH 3OH


(

( ii ) NaBH 4 , HO

[Ans.

OCH3

] [2]

OCH3
i ) Hg (OAc ) 2 ,CH3 O H
(

Q.4

[Ans.

[2]

(ii ) NaBH4 ,OH

OH
Q.5

i ) Hg ( OAc ) 2 , H 2O
(

[Ans.

[2]

(ii ) NaBH 4 ,OH

CCH

COCH3
i ) Hg ( OAc ) 2 , H 2O
(

Q.6

[Ans.

[2]

(ii ) NaBH 4 ,OH

Q.7

How could each of the following compounds be synthesized from an alkene by OMDM?
OH
(i)

OCH2CH3
(ii)

CH 3
|
(iii) CH 3CCH 2CH 3
|
OH

CH 3
|
(iv) CH 3CCH 2CH 3
|
OCH 3

OH

[Sol.

(i)

i ) Hg ( OAc ) 2 , H 2O
(

(ii ) NaBH 4 ,OH

OCH2CH3
(ii)

(iii)

CH 3
|
i ) Hg ( OAc ) 2 , H 2O
(

CH 3CCH 2CH 3
|
(ii ) NaBH 4 ,OH
OH

(iv)

CH 3
|
i ) Hg ( OAc ) 2 ,CH 3OH
(
CH 3CCH 2CH 3 ]
|
(ii ) NaBH 4 ,OH
OCH 3

i ) Hg ( OAc ) 2 ,CH 3CH 2OH


(

(ii ) NaBH 4 ,OH

HYDROBORATION-OXIDATION
Hydroboration has been developed by brown as a reaction of tremendous synthetic utility because alkyl
boranes are able to undergo a variety of transformation. Hydroboration is a one step, four centre, cis
addition process in accordance with M. rule but after oxidation it seems to be appear to violate M.rule.

OH ,H2 O2 , H2O
Hydroboration oxidation

(i ) BH3 THF

CH3CH = CH2 (CH3CH2CH2)3 B

CH3CH2CH 2OH

CH3 COOH
Hydroboration reduction

AgNO3
Dimerisation

CH3CH2CH 3
CH3CH2CH 2

NH2Cl

CH2CH2CH 3

CH3CH2CH 2NH2

Cl2

CH3CH2CH 2Cl

Mechanism of Hydroboration:
H

CH3CH=CH2 CH3CCH CH 3CH 2CH 2


|
HBH2
H BH2
BH 2

an alkylboran e

(CH3CH2CH2)3 B

More stable
transition state

Mechanism of oxidation :
HOOH + HO l HOO + H2O
R
RB

R
+

OOH

RBOOH

R
RBOR

repeat the two


preceding steps
two times

OR
ROBOR
HO

+ OH

OR
ROB OR
OH

3 ROH + BO3

repeat the two


preceding steps
two times

OR

OR

ROH + ROB

RO +

ROB

Q.1

(i ) BH

CH2 = CH2 3
(ii ) HO ,H 2O 2 , H 2O

Q.2

,THF
BH
3

H 2O / OH

OH

[Ans.CH3CH2OH]
H H

[Ans.
H

]
OH

[2]

[2]

CH3

Q.3

(i ) BH

[Ans.

CH3
H ]

OH ,

(ii ) HO ,H 2O 2 , H 2O

OH

CH3

OH
,THF
BH
3

Q.4

OH
H ]

CH3
CH3 ,

[Ans.

H 2O / OH

[2]

[2]

CH3

BD3, THF

Q.5

[2]

CH3COOH
BH3, THF
CH3COOD
BT3, THF
CH3COOD

CH2D
[Ans. (i) CH 3 CH CH 2 ,
|
|
D
H

(ii) CH 3 CH CH 2 , (iii) H
T
|
|
CH3
H
D

CH2D
T

H ]
CH3
CH2CHO

CCH
Q.6

i ) BH3 ,THF
(

(ii ) H 2O2 / HO

[Ans.

[2]

When God solves your problems, U have faith in his abilities. When he doesnt solve ur problems,
It means he has faith in ur abilities.

ASSIGNMENT-30
Q.1

Match the column :


Column I

[4]
Column II

Me
H
CCl4
Br
2 /

(A)
Me

(B)

(C)

(P)

Optically active

(Q)

Diastereoisomers

H /
OH

(R)

Optically inactive

CCl4
Br
2 /

(S)

Meso

(T)

Enantiomers

CCl4
Br
2 /

CH3
H

Me

(D)
Me

[Ans. (A) P,Q (B) R,T (C) R,T (D) R,S]


Q.2

Statement 1 : 1,1,1-trideutero-2-propanol reacts with conc. H2SO4 at high temperature to give only
one alkene, 3,3,3-trideutero propene.
[3]
because
Statement 2 : CD bond is stronger than CH bond.
(A) Statement-1 is true, statement-2 is true and statement-2 is correct explanation for statement-1.
(B) Statement-1 is true, statement-2 is true and statement-2 is NOT the correct explanation for statement-1.
(C) Statement-1 is true, statement-2 is false.
(D*) Statement-1 is false, statement-2 is true.

Q.3

Statement 1 : HCl in presence of peroxide will not undergo Antimarkonikov addition.


[3]
because
Statement 2 : Attack of chlorine radical on alkene is endothermic and attack of radical formed on
HCl is exothermic.
(A) Statement-1 is true, statement-2 is true and statement-2 is correct explanation for statement-1.
(B) Statement-1 is true, statement-2 is true and statement-2 is NOT the correct explanation for statement-1.
(C*) Statement-1 is true, statement-2 is false.
(D) Statement-1 is false, statement-2 is true.

Me

Et
HBr

Product.
Peroxide

Q.4

Et

[3]

Me
How many product will be formed in above reaction.
(A) 2
(B*) 4
(C) 6

(D) 5

Conc . H SO

2

4 A.

Q.5

[3]

Product A is:
(A)

Q.6

(B)

(C)

(D*)

Hydrolysis of ether proceed via carbocation intermediate as shown below :


H

Rate


determinin g step

[3]

+ CH3OH

Based on the above information, rationalize the relative rate of hydrolysis of compounds A to D below:
(I)
[Sol.

I > II > III > IV]

Q.7

In the given reaction

(II)

(III)

(IV)

[5]

C7H12 (A) HBr

(A) can be

Q.8

(A*)

(B*)

(C*)

(D)

Which one of the following compounds gives acetone (CH3)2C=O as one of the products of its ozonolysis?
[3]
(A)

(B)

(C)

(D*)

Q.9

Explain mechanism of following reaction.

[4]

HO

14

14

Q.10 The Corey-House Synthesis a way for coupling the alkyl groups of two alkyl halides to produce an
alkane:
[9]
several
R X + RX R R
steps

( 2 X )
diethyl
ether

RX + 2 Li

+ LiX

RLi

Alkyllithium

2 RLi + CuI R CuLi + LiI


2

Alkyllithium

Lithium
dialkylcuprate

R 2CuLi + R 'X
Lithium
dialkylcuprate

Alkyl halide

R ' R + RCu + LiX

Alkane

For the last step to give a good yield of the alkane, the alkyl halide RX must be either a methyl halide,
a primary alkyl halide, or a secondary cycloalkyl halide.
(i)

Complete the following reaction sequence:


CH 3CH 2 CH 2 CH 2 CH 2 I
Li
(C)
CH3 I
(A) CuI

(B)

Et 2O

Li
CH3CH2CH2CH2Br
(D) CuI

Et 2O

CH CH CH CH CH Br

2
2

22
(F)
(E) 3

[Ans. (A) CH3Li, (B) (CH3)2CuLi, (C) CH3CH2CH2CH2CH3, (D)CH3CH2CH2CH2Li,


(E) (CH3CH2CH2CH2)2CuLi , (F) CH3CH2CH2CH2 CH2CH2CH2CH2CH3
(ii)
How will you prepare following using Corey-House Synthesis.
(a) PhCMe3

[Sol.

(b)

(a)

CuLi + MeCl

(c)

CuLi + MeCl

(c)

(b)

CuLi + MeCl

(iii)

Identify the product of following reactions:


CH 2
||

(a) CH 3 C CH 2 Cl (CH 3 ) 2 CuLi

(b)

14

[Sol.

(a)

CH 3 C CH 2 CH 3
||
CH 2

(b)

CH 3
|
CH 3 CH 2 C CH CH CH 3
|
OLi

14

U may get Delayed 2 reach ur Targets. , But Every step towards target is equal 2 victory.

ASSIGNMENT-31
Q.1

Match the column:


Column I

Column II
(Product)

[4]

Br
Mg, 35C
Et2O

(A)

Product

(P)

Racemic mixture

(Q)

Diastereomers

(R)

Optically inactive product

(S)

Optically active product

(T)

Most reactive G.R.


[Ans. (A) Q,S (B) P,R (C) S,T (D) R]

CH3

(B)

H
H

CH3
(C) H

H
(D) CH3
H

CH3
Br
H
CH3

Mg, 35C
Et2O

Product

CH3
Mg, 35C
Br
Et2O
CH3
C2H5
CH2 Br
CH3
H
CH3

Mg(1 eq.)
35C, Et2O

Product

Product

CH2 Br

Q.2

Statement-1 : Aldehyde is more reactive than ketone toward nucleophilic attack.


[3]
Statement-2 : Due to less +I effect and less steric hinderance in aldehyde it is more reactive than ketone.
(A*) Statement-1 is true, statement-2 is true and statement-2 is correct explanation for statement-1.
(B) Statement-1 is true, statement-2 is true and statement-2 is NOT the correct explanation for statement-1.
(C) Statement-1 is true, statement-2 is false.
(D) Statement-1 is false, statement-2 is true.

Q.3

Consider the following reactions

[3]
CH I

CH3 I
LDA
M
L

3
NaH
X Y (LDA = lithiumdiisopropyl amide)

298 K

273 K

The products M and Y respectively are

(A)

(C*)

Q.4

and

and

(B)

and

(D)

and

NaNO

2
CH 3CH 2 C CH 3

?
HCl
||
O

[3]

Major product of this reaction is:


O
||
(A) CH 3CH C CH 3
|
NO
O
||
(C*) CH 3 C C CH 3
||
N OH
Q.5

(D) CH 3 CH 2 C CH 3
||
N OH

Among the following compounds which liberate two equivalent of CO2 gas on oxidative ozonolysis
followed by heating ?
[3]
(A)

Q.6

O
||
(B) CH 3 CH 2 C CH N OH

(B*)

(C)

(D)

What will be the product of hydration of 3-chloro-1-fluoro- propyne?

(A)

(B)

(C*)

[3]

(D)

Q.7

HOCl
HCCH A

[3]

( excess )

CH3CCH HOCl
B
( excess )

A & B are respectively


(A) Cl2CHCHO & CH3CCl2CHO

(B) Cl2CHCHO & CH 3C CHCl 2


||
O

(C) Cl2CHCOOH & CH3CCl2COOH

(D*) Cl2CHCOOH & CH 3C CHCl 2


||
O

H X

Q.8

[3]

X is:

(A*)

(B)

(C)

(D)

Q.9

Me2C CHCH2CH 2CH CHMe react with H2SO4 to give five structural isomer, C9H16, that react
with one eq. of H2/Pd. Give their structural formulas and account for their formation.

[Ans.

[5]

Q.10 Write major product in each case.

(a)

+ HBr A

(c)

[7]

H SO

(b)

( i ) CH MgBr ( excess )

E
(e) COOEt 3
(ii ) H
|
COOEt

2
4 B

(d)

HBr

(f)

HNO
2 F

H
G

(g)

[Sol.

(a)

(b)

, (c)

, (f)

If the 1st button of a shirt is wrongly put, all the rest are surely crooked. So always be careful on
your 1st step. Rest will come correct....

ASSIGNMENT-32

Q.1

Match the column :


Column I
(Reactions)
(A)

[4]
Column II
(Type of reaction)

CH3 CH = CH2 + HCl

(P)

Regioselective

(Q)

Stereoselective

(R)

Stereospecific

(S)

Anti Addition

(T)

Syn Addition

CH3
(B)

Cl

+ HBr

CH=CH2

CH3
C=C

(C)

(D)

CH3
H

+ Br2 CCl
4

( i ) B 2H 6

+ H2
(ii ) AcOH , H 2O

[Ans. (A) P, (B) P,Q,S (C) Q,R,S (D) Q,T]


Q.2

Statement-1 : If during a reaction

is generated, it quickly rearranges into

by hydride shift.

[3]

Statement-2 : Hydride is better migrator than methyl during carbocation rearrangements.


(A) Statement-1 is true, statement-2 is true and statement-2 is correct explanation for statement-1.
(B) Statement-1 is true, statement-2 is true and statement-2 is NOT the correct explanation for statement-1.
(C) Statement-1 is true, statement-2 is false.
(D*) Statement-1 is false, statement-2 is true.
Q.3

Statement-1 : Propan-2,2-diol is unstable.


[3]
Statement-2 : Repulsion between lone pairs of electron of two OH groups makes it unstable.
(A) Statement-1 is true, statement-2 is true and statement-2 is correct explanation for statement-1.
(B) Statement-1 is true, statement-2 is true and statement-2 is NOT the correct explanation for statement-1.
(C*) Statement-1 is true, statement-2 is false.
(D) Statement-1 is false, statement-2 is true.

Q.4

Q.5

Which of following on ozonolysis give 2-different product ?

[3]

(A*)

(B)

(C)

CH 3
|
(D) CH 3 C CH CH CH 2

Which of following hydrocarbon can react with maleic anhydride


(A)

(B*)

(C)

[3]
(D) none of these

Comprehension (Q.6 & Q.8)


[9]
Consider the following figure representing variation of heat of combustion of isomeric butenes.

Q.6

Q.7

Based on this answer the following questions:


Minimum dipole moment is of isomers.
(A) P
(B) Q

(C) R

(D*) S

Which gives fastest reaction with HBr?


(A) P
(B*) Q

(C) R

(D) S

Q.8

Which will give product showing no optical activity, when treated with HBr.
(A) P
(B) Q
(C) R
(D*) all of these

Q.9

Propose mechanism of following reaction:


H

[4]

Q.10 Provide a detailed, step-by-step mechanism for the following :

(a)

HCl

(b)

[Sol.

1equivalent

(c)

CH2=CHCHDCH=CH2
C

(d)

Ph C(OH ) C(I)Me 2
3 D

|
Me

(a)

[8]

AgNO

(b)

(d) Ph C (Me2) CO Me

Challenges are high, dreams are new, the world out there is waiting for you dare too dream dare
too try, no goal is too distant, no star too high !!

ASSIGNMENT-33

Q.1

Column I
(Reaction)

Column II
(Type of intermediate formed)

[4]

(A)

HO
PhCHCl2 (A)

(P)

Carbocation

(B)

Na

RBr
dry ether

(Q)

Carbanion

(R)

Free radical

(S)

Carbene

(C)

O
||
(i ) Mg / H 2O
CH 3 C CH 3

( ii ) H 2SO 4 ,

(D)

[Ans. (A) Q,S, (B) Q, R, (C) P,R (D) P]


Q.2

Statement-1 :
Propane on photochlorination gives 2-chloro propane as major product.
Statement-2 :
2 carbocation is more stable than 1 carbocation.
[3]
(A) Statement-1 is true, statement-2 is true and statement-2 is correct explanation for statement-1.
(B*) Statement-1 is true, statement-2 is true and statement-2 is NOT the correct explanation for statement-1.
(C) Statement-1 is true, statement-2 is false.
(D) Statement-1 is false, statement-2 is true.

Q.3

Statement-1 : Order of rate of decarboxylation in soda lime process of following compounds


is FCH2COOH > Cl CH2COOH
[3]
Statement-2 : F is a better I group than Cl.
(A) Statement-1 is true, statement-2 is true and statement-2 is correct explanation for statement-1.
(B) Statement-1 is true, statement-2 is true and statement-2 is NOT the correct explanation for statement-1.
(C) Statement-1 is true, statement-2 is false.
(D*) Statement-1 is false, statement-2 is true.

Q.4

Statement-1 :
Me MgBr give CH4 with acetone.
[3]
Statement-2 :
H of carbonyl compounds are active.
(A) Statement-1 is true, statement-2 is true and statement-2 is correct explanation for statement-1.
(B) Statement-1 is true, statement-2 is true and statement-2 is NOT the correct explanation for statement-1.
(C) Statement-1 is true, statement-2 is false.
(D*) Statement-1 is false, statement-2 is true.

||

Q.5

(B)

[3]

(A)

Give structure of (B)


O
(A)

(B*)

(C)

(D)

Me
O3
A+ B ;

Q.6

[3]

Total two type of Molozonides are formed identify relation between them?
(A) Diastereoisomers (B) Geometrical isomers (C*) Enantiomer
(D) Identical

OMDM

Q.7

[3]

OH
(A)

Q.8

OH

(B)

OH

OH
(C)

(D*)

Sentene ozonolysis

[3]

Which is the correct structure of Sentene.


(A)
Q.9

(B)

(C*)

(D)

The following equilibrium is driven to the right if the reaction is carried out in the presence of maleic
anhydride. What is the function of maleic anhydride?
[3]

Q.10
(a)
Prepare mechanism for following conversion.

[7]

( i ) CH MgBr

3
( ii ) H 2O

(b)

2O
+ HNO2 + H2SO4 H

Explain the mechanism


for this reaction

(1,2-dione)
1,2-cyclohexadione

When God leads you to the edge of the cliff, trust him fully. Either he will catch you when you
fall or he will teach you to fly.

ASSIGNMENT-38
OXIDATION OF ALKENES, ALCOHOLS & CARBONYL COMPOUNDS
(I)

OXIDATION OF ALKENES
OsO4

RCH=CR2

RCHCR2

H2O

OH OH

Cold dil.
alkaline
KMnO4

RCHCR2
OH OH

*
Cold dil. alkaline KMnO4 is called as Bayers reagent.
*
Overall syn addition
*
Given by alkenes & alkynes
*
Benzene & Cyclopropane can not give this reaction.
If we use acidic KMnO4 or warm KMnO4 or too concentrated KMnO4 the oxidative cleavage of
Glycol occurs resulting in mixture of Carboxylic acids & Ketones.

H ,KMnO 4
RCOOH + R2C = O
RCH = CR2

Hot acidic KMnO4, Hot acidic K2Cr2O7 & hot acidic NaIO4 gives same result with alkene. The effect
is similar to that of oxidative ozonolysis on alkenes.
Preilschaive reaction :
Epoxidation of alkenes is reaction of alkenes with peroxyacids.

O
O
||
||
CH2=CH2 + CH 3 C O O H CH2CH2 + CH 3 C OH
O
With the decrease in nucleophilicity of double bond, rate of reaction decreases.
With the decrease in er withdrawing substituents in leaving group, rate decreases.
R
CH
CH2

dil.H2SO4

CH

R
H2OCH

CH2

OH

R
H+ HOCH

CH2OH

CH2OH

H2O
RCO3H
RCO3H
H 3O
HCO3H

OH

R
CH +

OH

CH2

OH

OH

1 O Ag
2 2

R
CH
CH2

(II)
(1)

OXIDATION OF ALCOHOLS
Oxidising agents
Cu / 300C (or Red hot Cu tube)

(2)

H /KMnO4, (Strong oxidising agent)

(3)

H/K2Cr2O7, (Strong oxidising agent)

(4)

PCC (Pyridinium chloro chromate)

d
N CrO3 Cl or

CrO3 + HCl

(5)

Collins reagent
(

(6)

(2 mol) + CrO3 + CH2Cl2)

+ CrO3 + HCl + CH2Cl2

(7)

N
PDC

( Pyridinium dichromate )

(8)

Jones reagent (H2CrO4 in Anhydrous acetone)

(10)

or CrO3 + H2SO4 in acetone.


Sufficiently mild so that it oxidises alcohols
without oxidising or rearranging double bonds
(8 or 9)
MnO2-Oxidises only allylic or benzylicOH.i.e.

Cr2O7

(9)

Sarett reagent (i.e. PCC in CH2Cl2)

TsCl + DMSO + NaHCO3


Ts Cl

RCH2OTs

RCHO
RCH2OH
s DMSO

1 Allylic or benzylic OH MnO


2 Aldehyde

NaHCO3

Ts Cl

R2CHOTs

R2CO
R2CHOH
s DMSO

2 Allylic or benzylic OH MnO


2 Ketone

NaHCO3

DMSO

Ts
Cl
R3COTs
R3COH
s
NaHCO3

No effect on 3 ROH and on Carbon-carbon


multiple bond.

(11)

Periodic cleavage
(12)
A similar oxidation is obtained incase of HIO4
known as periodic cleavage.
(13)

O
RCHOH
+

HOI=O

R2COH

O
O

O
RCH
+
R2C=O
+
HIO3

R CH O I = O
R2COH

NBS
Openaur oxidation
R CH R

Al(OCMe3)3
O

OH

CH3 C CH3

O
RCR

R2CHOH + Al(OCMe3)3 Me3COH + Al(OCHR2)3


(R2CHO)2 AlO
CR2
O
3Me2C H

But reaction is only observed for Vic-diols.

3R2C=O + (Me2CO)3 Al

Oxidation of alcohol with aluminium tertiary


butoxide is Openaur oxidation.
OH

O
Al (OCMe3)3
acetone

Different oxidising agents are used to oxidise alcohols in corresponding carbonyl compounds and carboxylic acids.
e.g.

(I)

(II)

mild oxidising
R C H (Aldehyde) eg. 1,4,5,6,7,8,9,12
R CH 2 OH
agent
||
1alcohol
O

O
OH
||
|
mild oxidising
R CH R ' R C R ' (Ketone)
2alcohol

(III)

(IV)

(V)
(VI)

agent

strong oxidising

R C OH
R CH 2 OH
agent
||
1alcohol
O

CH 3
|
Cu 300C
CH 3 C OH
|
CH3
CH 3

eg.1,2,3,4,5,6,7,8,9,12,13
eg. 2,3

CH2
C

Dehydration takes place.

CH3

Double bond or Tripple bond is not affected by 1,4,5,6,7,8,9,10


No effect on 3 alcohol by 2,3,4,5,6,7,8,9,10,12,13

(III)

OXIDATION OF CARBONYL COMPOUNDS.

1.

RCHO + [Ag(NH3)2]OH
RCOOH + NH3 + Ag

Aldehyde acts as reducing agent, they can reduce mild oxidizing agents like Tollens Reagent. Tollens
test Gentle Heating for 20 to 25 mins.

2.

Fehlings Solutions
Fehlings A
aq. CuSO4

COONa
OH

Fehlings B
OH
H
Alk. solution of Roschelye
salt (sodium potassium tartrate)
COOK
It acts a carrier for Cu2+ as it make reversible complex with Cu2+
This test is also used is Blood and Urine test.
H 2O
RCOOH Cu
RCHO + Cu2+

RCOO Cu 2O ( red ppt.)

3.

Benedicts solution
Sodium Citrate + NaOH + NaHCO3 + CuSO4
H 2O
RCOOH Cu
RCHO + Cu2+

RCOO Cu 2O ( red ppt.)

4.

RCHO + HgCl2 + H2O


RCOOH + 2HCl + Hg2Cl2

RCHO + Hg2Cl2 + H2O RCOOH + 2HCl + 2Hg greyish solution

5.

Schiffs Reagent
Schiffs Reagent is aq. solution of following base decolourised by passing SO2.
Aldehyde restore pink colour of Schiffs reagent.
NH2

NH2

SO 2

Colourless RCHO
solution

(Schiffs Reagent)

RCOOH + Pink colour

+ NH

2 Cl
p-Rosaniline Hydrochloride
Magenta colour (Fuschin)

Ketons are not easy to oxidize so they do not give these 5 tests. These five tests can be used to distinguish
aldehyde and ketones. Both gives 2,4 DNP test
KETONES ARE DIFFICULT TO OXIDIZE
Ketones can be oxidized from their enolic form at high temperature with very strong oxidizing agent.
Oxidation of ketones is sometimes governed by Popoffs rule. According to this rule carbonyl group
remains with the smaller alkyl group. More electron rich alkene will be easy to oxidized.
O

[O]
MeCHO + CO 2 + H 2O

Allylic oxidation
SeO2 is a selective oxidizing agent with converts CH2 group agjacent to carbonyl group into carbonyl
group.The reagent, in general, oxidises active methylene and methyl groups to ketonic and aldehydic
groups respectively.
Me C Me

O
O
O
O O
|
|
|
|
|
|
||
||
SeO2
SeO2
C CHO
CH 2 C C C ; C CH 3
Double bonds, triple bonds and aromatic rings may also activate the methylene group.The methylene or
methyl group to the most highly substitued end of the double bond is hydroxylated according to the
order of preference of oxidation CH2 > CH3 > CH groups.
OH
|
SeO 2
CH

CH

CH

CH3= CHCH3
2
2
2 C H > 1 C H > 3 C H
Rate of reactivity order

CH3 CH = CH CH2CH3 SeO


2 CH 3 CH CH CH 2 CH 3
|
OH
CH3
CH CH3

SeO2

HOCH2
CH CH3
CH3

CH3

HO
SeO2

Q.1

H / KMNO
CH2 = CH2 4

(i)

(ii)

H / KMNO
CH3CH= CH2 4 [11]

/ KMNO 4
H

(iii)

/ KMNO 4
H

(iv)

/ KMNO 4
H

(v)

/ KMNO 4
H

(vi)

/ KMNO 4
H

(vii)

/ KMNO 4
H

(viii)

/ KMNO 4
H

(ix)

/ KMNO 4
H

(x)

/ KMNO 4
H

(xi)

Q.2

A to F alkenes with minimum possible carbon.

(i)

H / KMNO
A 4 MeCOOH as the only product

[7]

(ii)

H / KMNO
B 4

(iii)

H / KMNO
C 4 MeCH2COOH as the only organic (iv)

H / KMNO
D 4

O
||
H / KMNO 4

HOOC

C
C C C C COOH
E

||
O

(v)

(vi)

Q.3

H / KMNO
C10H10 4 HOOC C C C COOH

|
C COOH

1% alkaline

(i)

H / KMNO
(vii) F 4 acetone + ethanoic acid

mCPBA

(ii)

KMnO4

[6]

mCPBA\hydrolysis

Me

mCPBA
hydrolysis

(iii)

C=C

(iv)

Me

Me

Me

mCPBA
C=C
(v)
Me hydrolysis
H

Q.4

(i)

C=C

(vi)
H

KMnO / OH ,
?
CH3 CH2 CH2 OH 4

or KMnO 4 / H ,

K2Cr2O7 / H ,
or conc. HNO 3,

Me

Ph

mCPBA
hydrolysis
Ag2O or 2Ag +

1
O
2 2

[2]

(ii)

OH
|
KMnO ,H
?
CH 3 CH CH 2 CH 3 4

[1]

or K 2Cr2O 7 , H

OH

HO

(1)

(2) or (3)

(iii)

OH

or (7) or (8) or (9)

HO
Q.5

(4) or (5) or (6)


(10)

[4]

?
?

(i)

CH2 = CH (CH2)3 CH2 OH PCC

(ii)

C6H5 CH = CH CH2 OH PCC

(iii)

OH
|
(i ) Dil NaOH
CH 3 CH CH 2 CH 2 CH 2 OH PCC

(A) (B)

[9]

( ii )

OH

PCC

(iv)

(v)

CH2 = CH CH2OH MnO


2 ?

CH2OH

OH
(vi)

MnO
CHCH2 CH2 OH 2 ?
Acetone

CH3O
CH3O

(vii)

Q.6

CH 3
|
MnO 2
?
CH C CH C CH 2 OH Acetone

OH
|
(viii) C 6 H 5 CH CH 3 MnO
2 ?

(ix)

TsCl NaHCO3
C 6 H 5 CH CH CH 2 CH CH 2 OH DMSO

?
|
CH 3

(i)

HO

(ii)

OH
|
tert butoxide
CH 2 CH CH 2 CH CH 3 Aluminium

tert butoxide
Aluminium

?
Acetone

CH2OH

(iii)

tert butoxide
Aluminium

?
p benzoquinone

CCl 4

[3]

Q.7

Which one of the following alcohols are oxidised by MnO2?

[3]

(A) C6H5 CH2 CH2OH

OH
|
(B) CH 2 CH CH 2 CH CH 3

OH
|
(C*) CH 3 CH CH CH CH 3

(D) CH3CH2 CH2 OH

Q.8
(i)

[9]
HIO
Me CH CH 2 OH 4

|
OH

HIO
(ii) Me 2 C CH Et 4

|
|
OH OH

OH

HIO
4

(iii)

OH

(v)

CH 2 CH CH 2 CH 3 HIO
4

|
|
OH
OH

(vii)

CH 2 CH CH CH 2 HIO
4

|
|
|
|
OH
OH OH OH

(ix)

Me C C Me HIO
4

| | ||
O O
Which will give the Tollen test.

Q.9

(i)

OH
H

(ii)

O
Q.10 (a)

(viii)

Me C CH Me HIO
4

|| |
O OH

[3]

OMe

O OH
|| |
(iii) R C CH 2

Me2CHCMe

HO

(iv)

HO

[O]
C C C C

(b)

CH 2 OH HIO 4
(iv) HO CH 2 CH 2 CH

OH
(vi)
CH 2 CH CH CH 3 HIO
4

|
|
|
OH
OH OH

[O]
[Ans. 2CCOOH ]

[4]

O
[O]

Me CHCMe [Ans. Me2CO + MeCOOH ]

(c)

O
[O]
Me CCOOH + CO
HO
Me3CHCMe
Me +CHCMe
[Ans. Me3CCOOH + CO2 + H2O ]

O [O]

(d)

O [O]

[Ans.

]
O

Q.11

(a)

SeO2

CH3CHO

CH CHO

SeO
[Ans. CH CHO ]

[10]

O
(b)

Me2CO

SeO2

SeO
[Ans. Me C C H ]
O O

(c)

SeO2
CP1 CHCPBA
C
C P2 LAH
C C C C

P3

O
C C C C]
[Ans.

O O

O SeO
2

(d)

O SeO

[Ans.

]
O

HO
SeO2

(e)

O
CH3

(f)

CH3 CH = CH2

2 step
(1) step etard

(g)

Acrolein

SeO2
CH2 CH = CH2

MnO2

OH

SeO2

H C CH = CH2

OH

CH2

(h)

C H

CH3 C H

O O
SeO2

CH3 C C H

conc. NaOH

MnO2

P1

H /

P2

Q.12 How will you differentiate HCHO and PhCHO ?

[Ans. F and B test]

[2]

Q.13 How will you differentiate HCHO and MeCHO ?

[Ans. Iodoform test]

[1]

Success in not permanent & failure is not final. So, never stop working after success & never
stop trying after failure.

ASSIGNMENT-39
Give major products of following reactions:
Q.1

Py
Me C Cl ROH
||
O

Q.2

MeCOCl + EtOK

Q.3

MeCOCl + NH3

Q.4

Ph NH2 + MeCOCl

Q.5

Ph NH2 + PhCOCl

Q.6

PhOH + PhCOCl

Q.7

RNH2 + PhSO2Cl

Q.8

R2NH + PhSO2Cl

Q.9

ROH + KCN

Q.10 ROH +

Q.11

EtCOCl + EtCOONa

2
Q.12 EtCOOH + EtCOOH
4

KCN

Conc . H SO

Q.13

Q.14

H O

Q.15

3
Me C O C Me

||
||
O
O

Q.16

NaOH

Q.17

Me C OMe EtONa
||
O

Q.18

+ MeNH2

Q.19

Ph C NH 2 Me 2 NH
||
O

Q.20

Q.21

Q.23

Q.22

NH

Me C NH 2 NaOH

||
O

History proves that winners win because they refuse to be discouraged by temporary defeats!

ASSIGNMENT-40
Q.1

Write only the major product in each case:


AlCl

3 A

(a)

(c)

(e)

(g)

+ PhCH2Cl 3 G

AlCl

3 C
AlCl

3 E
AlCl

AlCl

3 I

(i)

(k)

Me
|
AlCl
+ Me C C Cl 3 K
| ||
H O

AlCl

3 M

(m)

(o)

(q)

(s)

AlCl

3 O

AlCl

3 B

(b)

(d)

(f)

+ CH2Cl2 3 F

(h)

+ CH2Cl2 3 H

(j)

(l)

(n)

AlCl

3 D
AlCl

AlCl

AlCl

+ Me C Cl 3 J
||
O
Me
|
AlCl
+ Me C C Cl 3 L
|
||
Me O

AlCl
+ Me C O C Me 3 N
||
||
O
O

AlCl

3 P

(p)

H
Q
+ HNO2

(r)

. H 2SO4
+ C C C OH conc

. H 2SO4
+ C C = C conc

(t)

. H 2SO4
+ PhCH2CH2CH2CH2OH conc

Oxidation of aromatic compounds.

(a)

(b)

VO

2 5

500C

(c)

VO

VO

(d)

2 5

2 5

500C

(e)

H K Cr O

H K Cr O

H KMnO

2 2 7

H K Cr O

(h)

2 2 7

H K Cr O

(f)

2 2 7

(g)

(i)

500C

2 2 7

(j) Ph ( CH 2 ) n CH3 , PhCHMe2, PhCH2OH, PhCH2Br,,


PhCHCl2, PhCHO, Ph

& all the compounds having at

least one H give PhCOOH.

(k) PhCMe3 HKMnO

4 Me3CCOOH

(l)

(m)

(n)

H KMnO

H KMnO

H KMnO

(m)

(o)

H KMnO

isophthalic acid

H O

(n)

KMnO 4

KMnO 4

Etard oxidation:
CrO Cl

2 2

AcOAc

Alb's persulphate oxidation:


K S O

2 8
2

Our attitude towards life determines lifes attitude towards us.

terephthalic acid

INORGANIC CHEMISTRY
ASSIGNMENTS

ASSIGNMENT-2

Q.1

What is the ratio of average formal charge of oxygen and Bond order of in CO32 .
(A*) 0.5
(B) +0.5
(C) 1.0
(D) None

[3]

Q.2

Which option is correct among the following


(A*) AlCl3(aq) is not hypovalent where as AlCl3 anhydrous is hypovalent
(B) AlCl3(aq) is hypovalent where as AlCl3 anhydrous is hyper valent
(C) AlCl3(aq) and AlCl3 anhydrous both are hypovalent
(D) None

[3]

Q.3

Compound X has eight atoms which has only four bond no -bond, no ionic bond, no-coordinate
bond, no H-bond.
[3]
(A) C2H6
(B*) B2H6
(C) C2(CN)6
(D) All the above

Q.4

Although the electronegativity of C is more in H C C H but it cannot form H-bond.


(A) C is linear
(B) C is sp-hybridised

(C*) C does not have lone pair e s


(D) None

Q.5

Statement-1 : S2F2 exist-in two forms where as S2Cl2 exist only is one form.
[3]
Statement-2 : Due to steric repulsion between two Cl and also in between S and S their is double
bond.
(A*) Statement-1 is true, statement-2 is true and statement-2 is correct explanation for statement-1.
(B) Statement-1 is true, statement-2 is true and statement-2 is NOT the correct explanation for statement-1.
(C) Statement-1 is true, statement-2 is false.
(D) Statement-1 is false, statement-2 is true.

Q.6

Calculate the percentage ionic character for flouride molecule where electronegativity of electropositive
element is 2.1. Write your answer in integer 1.
[3]
[Ans. 6]

Q.7

Which compound has 14 bond, 2 bond four coordinate bond and four hydrogen bond as well as
ionic bond :
[3]
(A*) Blue vitriol
(B) Glauber salt
(C) Candy fluid
(D) Norwagian salt petere

Q.8

Identify the element of third period whose successive I.E. are given below :
I.E1
I.E2
I.E3
X
513
4562
6920
Y
738
1451
7733
Z
1521
2666
3931

[3]

Q.9

Which one has large 4th I.E.


(A*) Ga
(B) Ge

[3]
(C) Both

[3]

(D) None

Q.10

Element A has 3e in the valence shell and its principal quantum number for last e is three and element
B has 4e in the valance shell and its principal Q.N. for last e is 2 what is the number of atom A and
number of atom B in compound of AB and what is this compound.
[3]
[Ans. 43]

Q.11

The X X bond length is 1.0 and C C bond length is 1.54 if electronegatives of X and C are 3.0
and 2.0 respectively. C X bond length is likely to be.
[3]

ASSIGNMENT-4

Q.1

Find the species / molecule is having maximum number of lone pair on the central atom.
(A) ClOF4
(B) ClOF2+
(C) BH4
(D*) XeOF2

Q.2

If y-axis is the approaching axis between two atoms, then which of the set of orbitals can not form the
bond between two atoms in general.
[3]
(A) pz pz
(B) px px
(C*) px py
(D) None of these

Q.3

The maximum number of bond and -bond can be formed between two atoms are respectively.[3]
(A) 4, 3
(B*) 3, 2
(C) 2, 3
(D) 3, 1

Q.4

Which of the following set of overlap can not provide -bond formation.
(A) 3d and 2p
(B) 2p and 3p
(C) 2p and 2p
(D*) 3p and 1s

[3]

Q.5

The ratio of number of -bond to -bond in N2 and CO molecules are

[3]

(A) 2.0 , 2.0


Q.6

Q.7

Q.8

Q.9

Q.10

(B) 2,

1
2

(C*)

1 1
,
2 2

(D)

[3]

1
,2
2

Which of the following overlapping is involved in formation of only -bond.


(A*) s p overlapping
(B) p d overlapping
(C*) s s overlapping
(D) p p overlapping

[5]

Paragraph for question nos. 7 to 9


Different types of bonds are formed in the chemical compounds. These bond have different strength and
bond energies associated with them. These bonds are formed with atoms in different environments.
Which of the following bond has highest bond energy?
[9]
(A*) -bond
(B) -bond
(C) Hydrogen bond (D) Metallic bond
Shape of the molecule is decided by
(A*) -bond
(C) both and -bond

(B) -bond
(D) Never Nor -bond

Which of the following hydrides is thermally least stable?


(A) H2O
(B*) H2Te
(C) H2S
Match the column :
Column I
(Orders)
(A)
Increasing order of ionic
mobility in water
(B)
Decreasing order of size
(C)
Decreasing order of ionisation
energy

(D) H2Se
[4]

(P)

Column II
(Sequences)
Si, Mg, Al, Na

(Q)
(R)

Li+, Na+, K+, Rb+, Cs+


O, O+, O2, O

(S)

Se, S, O, F
[Ans. (A) Q, (B) S, (C) P]

ASSIGNMENT-5

Q.1

Explain type of hybridisation, shape, bond angle and geometry of the following compounds.
[18]
(1)
XeF2
(11) IF7
(2)

XeF4

(12)

OF2

(3)

XeF5

(13)

NO 3

(4)

XeOF4

(14)

ClO 4

(15)

SF4

(16)

I3

(17)

ClO 3

(18)

OCl2

(5)
(6)
(7)

PCl3
PCl5
SF2

(8)

SF6

(9)

IF3

(10)

IF5

Q.2

What is hybridisation of central atom of anionic part of PBr5 in crystalline state.


[3]
2
3
(A) sp
(B) sp
(C) sp
(D*) not applicable

Q.3

What is the difference between bond angles in cationic species of PCl5 and PBr5 in solid state. [3]
(A) 60
(B) 10928
(C*) 0
(D) 90

Q.4

All possible bond angles in anionic part of PCl5 are.


(A) 109 28 only
(B*) 90, 180
(C) 90, 120, 180

[3]
(D) 72, 90, 180

Q.5

The hybridisation and shape of XeO3F2 molecule is


(A) sp3 and tetrahedral
(B) sp3d and see-saw
(C*) sp3d and TBP
(D) sp3d2 and octahedral

Q.6

TeF5, XeF2 , I3+ , XeF4 , PCl3


Which of the following shape does not describe to any of the above species ?
(A) Square pyramidal
(B) Square planar
(C*) Trigonal planar
(D) Linear

[3]

[3]

Q.7

Which of the following species does not exist?


(A*) XeF3

Q.8

Q.10

Q.11

(D) XeF6

(B) CO2 and SO2

(C*) SO2 and I3

[3]

(D*) ICl 2 and BeH2

Statement-1 :
CH4 and CH2F2 are having regular tetrahedron geometry.
[3]
Statement-2 :
Both are having same hybridization.
(A) Statement-1 is true, statement-2 is true and statement-2 is correct explanation for statement-1.
(B) Statement-1 is true, statement-2 is true and statement-2 is NOT the correct explanation for statement-1.
(C) Statement-1 is true, statement-2 is false.
(D*) Statement-1 is false, statement-2 is true.
Paragraph for question nos. 10 to 12
Hybridisation is the mixing of atomic orbital of comparable energy and the number of hybrid orbitals
formed is equal to the number of pure atomic orbitals mixed up and hybrid orbitals are occupied by bond pair and lone pair.
[9]
3
Which of the following geometry is most likely to not form from sp d hybridisation of the central atom.
(A) Linear
(B*) Tetrahedral
(C) T-Shaped
(D) See-Saw
The orbital is not participated in sp3d2 hybridisation.
(A) px

Q.12

(C) XeF5

Which of following pair of species is having different hybridisation but same shape.
(A) BeCl2 and CO2

Q.9

(B) XeF4

[3]

(B*) dxy

(C) d x 2 y 2

(D) pz

"The hybrid orbitals are at angle of X to one another" this statement is not valid for which of the
following hybridisation.
(A) sp3
(B) sp2
(C*) sp3d2
(D) sp

ASSIGNMENT-6
One or more than one correct:
Q.1
Select the correct statement for non polar molecule.
[3]
(A*) On time average the molecule is non-polar but at the particular moment it act as a dipole which is
equally probale in all directions.
(B) On time average the molecule is polar but at the particular moment it does not act as a dipole.
(C) On time average the molecule is non-polar and the particular moment it must not act as dipole.
(D) All are incorrect
Q.2

Select the correct order of B.P.


(A) BF3 > BMe3
(B*) BF3 < BMe3

[3]
(C) BF3 = BMe3

(D) None of these

Q.3

Which molecular geometry are most likely to result, from a octahedral electron geometry?
(A*) square planar
(B*) square pyramidal (C) linear
(D) Vshaped

[3]

Q.4

The correct order of hybridization of the central atom in the following species
NH3, XeO2F2, SeF4, NO2+

[3]

Q.5

(A) sp3, sp3, sp3d, sp

(B*) sp3, sp3d, sp3d, sp

(C) sp3, sp3d2, sp3d, sp2

(D) sp2, sp3d, sp3d2, sp

Halogens form compounds among themselves with the formula XX', XX & XX '7 (where X is the heavier
halogen) which of the following pair(s) represent(s) correct geometry with polar and non-polar nature
(theoretically)
[3]
(A*) XX' Linear Polar
(B) XX Linear Polar
(C) XX' Linear Non-polar

(D*) XX '7 Pentagonal bipyramidal Non-polar

Q.6

In molecules of the type AX2Ln (where L represents lone pairs and n is its number) there exists a bond
between element A and X. The X A X bond angle.
[3]
(A) Always decreases if n increases
(B) Always increases if n increases
(C*) Will be maximum for n = 3, 0
(D) Generally decreases if n decreases

Q.7

Which of the following pairs of species have identical shapes?


(A)

Q.8

Q.9

Q.10

NO 2

and

NO 2

(B) PCl5 and BrF5

(C*) XeF4 and

[3]
ICl4

(D) TeCl4 and XeO4

London force works in


[3]
(A) Polar molecule
(B) Non-polar molecule
(C*) All polar and non-polar molecule
(D) Ionic compounds
Choose the correct on the ClO bond length in NaClO4.
[3]
(A) All ClO bonds are of equal length.
(B) Three ClO bonds are of equal of length one longer.
(C) Two ClO bonds are of same length which are longer compound to other two ClO bond length.
(D) All are different.
Statement-1 :
Experimentally 100 % covalent bond formation is not possible
[3]
Statement-2 :
Non polar molecule has instantaneous dipole induced dipole interaction
(A*) Statement-1 is true, statement-2 is true and statement-2 is correct explanation for statement-1.

(B) Statement-1 is true, statement-2 is true and statement-2 is NOT the correct explanation for statement-1.
(C) Statement-1 is true, statement-2 is false.
(D) Statement-1 is false, statement-2 is true.
Q.11

Q.12

Spin pairing and overlapping theory can not explain the equal bond length as well as equal bond angles
in CH4 molecule. To explain the above facts we are in need of hybridization theory. Hybridization is the
mixing of atomic orbitals of comparable energy and the number of atomic orbitals involved is equal to the
number of hybrid orbitals formed of equal energy.
[3 3 = 9]
(i)
According to hybridization theory, the %s character in sp3d hybrid orbitals is
(A) 25%
(B) 33.33%
(C*) 20%
(D) 16.66%
(ii)

The number of axial and equitorial positions in octahedral geometry having sp3d2 hybridization
(A) 2, 4
(B) 4, 2
(C) 3, 3
(D*) 0, 0

(iii)

Increasing order of the energy of hybrid orbitals is


(A) sp3 < sp2 < sp
(B) sp2 < sp3 < sp
(C) sp < sp3 < sp2

Match the column :


Column I

(D*) sp < sp2 < sp3


[4]

Column II

(i)

IOF4

(P)

See-saw

(ii)

IO2 F2

(Q)

Trigonal bipyramidal

(iii)

XeO64

(R)

Linear

(iv)

XeF2

(S)

Square bipyramidal

(A) (i) P, (ii) S, (iii) R, (iv) Q


(C) (i) Q, (ii) P, (iii) S, (iv) R

(B*) (i) Q, (ii) P, (iii) S, (iv) R


(D) (i) Q, (ii) P, (iii) R, (iv) S

ASSIGNMENT-7
Q.1

Q.2

Q.3

Q.4

The correct order of d CH in the following option is


[3]
(A) CHF3 = CH2F2 = CH3F
(B) CHF3 > CH2F2 > CH3F
(C) CH2F2 > CH3F > CHF3
(D*) CH3F > CH2F2 > CHF3
O2F2 is an unstable yellow orange solid and H2O2 is a colourless liquid, both have OO bond.
OO bond length in H2O2 & O2F2 is respectively.
[3]
(A) 1.22, 1.48
(B*) 1.48, 1.22
(C) 1.22, 1.22
(D) 1.48, 1.48
The structure of O3 and N 3 are
(A) both linear
(B) Linear and bent respectively.
(C) both bent
(D*) Bent and linear respectively.
Select the correct order of following property.
(A) % s-character
:
sp3 > sp2 > sp

[3]

[3]

Q.5

^ bond angle
(B) ONO
:
NO3 > NO 2 > NO2
(C*) Second ionisation energy :
O>F>N>C
(D) C F bond length
:
CF4 > CH3F > CH2F2 > CF3H
Which of the following compounds is/are polar as well as non planar?

[3]
Cl

(A*) S2Cl2
Q.6
Q.7
Q.8

Q.9

Q.10

Q.11
Q.12

(B) B2H6

(C*) PCl2F3

The strongest PO bond is found in the molecule


(A*) F3PO
(B) Cl3PO
(C) Br3PO
F-As-F bond angle in AsF3Cl2 can be
(A*) 90 & 180 only (B) 120 only
(C) 90 & 120 only
The number of SS bonds in sulphur trioxide trimer (S3O9) is
(A) three
(B) two
(C) one

(D)

Cl

C=C=C=C

H
[3]

(D) (CH3)3PO
[3]
(D) 90 only
[3]
(D*) zero

Select the incorrect statement(s) about N2F4 and N2H4.


[3]
(A*) In N2F4 , d-orbitals are contracted by electronegative fluorine atoms, but d-orbital contraction is
not possible by H-atom in N2H4.
(B) The NN bond energy in N2F4 is more than NN bond energy in N2H4.
(C*) The NN bond length in N2F4 is more than that of in N2H4.
(D) The NN bond length in N2F4 is less than that of in N2H4.
Nodal planes of bond(s) in CH2=C=C=CH2 are located in
[3]
(A) All are in molecular plane
(B*) Two in molecular plane and one in a plane perpendicular to molecular plane which contains CC
-bond
(C) One in molecular plane and two in plane perpendicular to molecular plane which contains CC
-bond
(D) Two in molecular plane and one in a plane perpendicular to molecular plane which bisects CC
-bond at right angle
Which of the following shape can not be obtained from sp3d2 hybridisation.
[3]
(A) Square planar
(B) Square pyramidal (C*) Tetrahedral
(D) Octahedral
Which of the following statement(s) is/are correct about P4O6 and P4O10
[3]
(A*) Both oxides have closed cage like structure
(B*) Each oxide contains six equivalent POP bonds
(C) Both P4O6 and P4O10 molecules have pd bonds.
(D*) Both are the anhydrides of their respective acids.

ASSIGNMENT-4

Q.1

Find the species / molecule is having maximum number of lone pair on the central atom.
(A) ClOF4
(B) ClOF2+
(C) BH4
(D*) XeOF2

Q.2

If y-axis is the approaching axis between two atoms, then which of the set of orbitals can not form the
bond between two atoms in general.
[3]
(A) pz pz
(B) px px
(C*) px py
(D) None of these

Q.3

The maximum number of bond and -bond can be formed between two atoms are respectively.[3]
(A) 4, 3
(B*) 3, 2
(C) 2, 3
(D) 3, 1

Q.4

Which of the following set of overlap can not provide -bond formation.
(A) 3d and 2p
(B) 2p and 3p
(C) 2p and 2p
(D*) 3p and 1s

[3]

Q.5

The ratio of number of -bond to -bond in N2 and CO molecules are

[3]

(A) 2.0 , 2.0


Q.6

Q.7

Q.8

Q.9

Q.10

(B) 2,

1
2

(C*)

1 1
,
2 2

(D)

[3]

1
,2
2

Which of the following overlapping is involved in formation of only -bond.


(A*) s p overlapping
(B) p d overlapping
(C*) s s overlapping
(D) p p overlapping

[5]

Paragraph for question nos. 7 to 9


Different types of bonds are formed in the chemical compounds. These bond have different strength and
bond energies associated with them. These bonds are formed with atoms in different environments.
Which of the following bond has highest bond energy?
[9]
(A*) -bond
(B) -bond
(C) Hydrogen bond (D) Metallic bond
Shape of the molecule is decided by
(A*) -bond
(C) both and -bond

(B) -bond
(D) Never Nor -bond

Which of the following hydrides is thermally least stable?


(A) H2O
(B*) H2Te
(C) H2S
Match the column :
Column I
(Orders)
(A)
Increasing order of ionic
mobility in water
(B)
Decreasing order of size
(C)
Decreasing order of ionisation
energy

(D) H2Se
[4]

(P)

Column II
(Sequences)
Si, Mg, Al, Na

(Q)
(R)

Li+, Na+, K+, Rb+, Cs+


O, O+, O2, O

(S)

Se, S, O, F
[Ans. (A) Q, (B) S, (C) P]

ASSIGNMENT-5

Q.1

Explain type of hybridisation, shape, bond angle and geometry of the following compounds.
[18]
(1)
XeF2
(11) IF7
(2)

XeF4

(12)

OF2

(3)

XeF5

(13)

NO 3

(4)

XeOF4

(14)

ClO 4

(15)

SF4

(16)

I3

(17)

ClO 3

(18)

OCl2

(5)
(6)
(7)

PCl3
PCl5
SF2

(8)

SF6

(9)

IF3

(10)

IF5

Q.2

What is hybridisation of central atom of anionic part of PBr5 in crystalline state.


[3]
2
3
(A) sp
(B) sp
(C) sp
(D*) not applicable

Q.3

What is the difference between bond angles in cationic species of PCl5 and PBr5 in solid state. [3]
(A) 60
(B) 10928
(C*) 0
(D) 90

Q.4

All possible bond angles in anionic part of PCl5 are.


(A) 109 28 only
(B*) 90, 180
(C) 90, 120, 180

[3]
(D) 72, 90, 180

Q.5

The hybridisation and shape of XeO3F2 molecule is


(A) sp3 and tetrahedral
(B) sp3d and see-saw
(C*) sp3d and TBP
(D) sp3d2 and octahedral

Q.6

TeF5, XeF2 , I3+ , XeF4 , PCl3


Which of the following shape does not describe to any of the above species ?
(A) Square pyramidal
(B) Square planar
(C*) Trigonal planar
(D) Linear

[3]

[3]

Q.7

Which of the following species does not exist?


(A*) XeF3

Q.8

Q.10

Q.11

(D) XeF6

(B) CO2 and SO2

(C*) SO2 and I3

[3]

(D*) ICl 2 and BeH2

Statement-1 :
CH4 and CH2F2 are having regular tetrahedron geometry.
[3]
Statement-2 :
Both are having same hybridization.
(A) Statement-1 is true, statement-2 is true and statement-2 is correct explanation for statement-1.
(B) Statement-1 is true, statement-2 is true and statement-2 is NOT the correct explanation for statement-1.
(C) Statement-1 is true, statement-2 is false.
(D*) Statement-1 is false, statement-2 is true.
Paragraph for question nos. 10 to 12
Hybridisation is the mixing of atomic orbital of comparable energy and the number of hybrid orbitals
formed is equal to the number of pure atomic orbitals mixed up and hybrid orbitals are occupied by bond pair and lone pair.
[9]
3
Which of the following geometry is most likely to not form from sp d hybridisation of the central atom.
(A) Linear
(B*) Tetrahedral
(C) T-Shaped
(D) See-Saw
The orbital is not participated in sp3d2 hybridisation.
(A) px

Q.12

(C) XeF5

Which of following pair of species is having different hybridisation but same shape.
(A) BeCl2 and CO2

Q.9

(B) XeF4

[3]

(B*) dxy

(C) d x 2 y 2

(D) pz

"The hybrid orbitals are at angle of X to one another" this statement is not valid for which of the
following hybridisation.
(A) sp3
(B) sp2
(C*) sp3d2
(D) sp

ASSIGNMENT-6
One or more than one correct:
Q.1
Select the correct statement for non polar molecule.
[3]
(A*) On time average the molecule is non-polar but at the particular moment it act as a dipole which is
equally probale in all directions.
(B) On time average the molecule is polar but at the particular moment it does not act as a dipole.
(C) On time average the molecule is non-polar and the particular moment it must not act as dipole.
(D) All are incorrect
Q.2

Select the correct order of B.P.


(A) BF3 > BMe3
(B*) BF3 < BMe3

[3]
(C) BF3 = BMe3

(D) None of these

Q.3

Which molecular geometry are most likely to result, from a octahedral electron geometry?
(A*) square planar
(B*) square pyramidal (C) linear
(D) Vshaped

[3]

Q.4

The correct order of hybridization of the central atom in the following species
NH3, XeO2F2, SeF4, NO2+

[3]

Q.5

(A) sp3, sp3, sp3d, sp

(B*) sp3, sp3d, sp3d, sp

(C) sp3, sp3d2, sp3d, sp2

(D) sp2, sp3d, sp3d2, sp

Halogens form compounds among themselves with the formula XX', XX & XX '7 (where X is the heavier
halogen) which of the following pair(s) represent(s) correct geometry with polar and non-polar nature
(theoretically)
[3]
(A*) XX' Linear Polar
(B) XX Linear Polar
(C) XX' Linear Non-polar

(D*) XX '7 Pentagonal bipyramidal Non-polar

Q.6

In molecules of the type AX2Ln (where L represents lone pairs and n is its number) there exists a bond
between element A and X. The X A X bond angle.
[3]
(A) Always decreases if n increases
(B) Always increases if n increases
(C*) Will be maximum for n = 3, 0
(D) Generally decreases if n decreases

Q.7

Which of the following pairs of species have identical shapes?


(A)

Q.8

Q.9

Q.10

NO 2

and

NO 2

(B) PCl5 and BrF5

(C*) XeF4 and

[3]
ICl4

(D) TeCl4 and XeO4

London force works in


[3]
(A) Polar molecule
(B) Non-polar molecule
(C*) All polar and non-polar molecule
(D) Ionic compounds
Choose the correct on the ClO bond length in NaClO4.
[3]
(A) All ClO bonds are of equal length.
(B) Three ClO bonds are of equal of length one longer.
(C) Two ClO bonds are of same length which are longer compound to other two ClO bond length.
(D) All are different.
Statement-1 :
Experimentally 100 % covalent bond formation is not possible
[3]
Statement-2 :
Non polar molecule has instantaneous dipole induced dipole interaction
(A*) Statement-1 is true, statement-2 is true and statement-2 is correct explanation for statement-1.

(B) Statement-1 is true, statement-2 is true and statement-2 is NOT the correct explanation for statement-1.
(C) Statement-1 is true, statement-2 is false.
(D) Statement-1 is false, statement-2 is true.
Q.11

Q.12

Spin pairing and overlapping theory can not explain the equal bond length as well as equal bond angles
in CH4 molecule. To explain the above facts we are in need of hybridization theory. Hybridization is the
mixing of atomic orbitals of comparable energy and the number of atomic orbitals involved is equal to the
number of hybrid orbitals formed of equal energy.
[3 3 = 9]
(i)
According to hybridization theory, the %s character in sp3d hybrid orbitals is
(A) 25%
(B) 33.33%
(C*) 20%
(D) 16.66%
(ii)

The number of axial and equitorial positions in octahedral geometry having sp3d2 hybridization
(A) 2, 4
(B) 4, 2
(C) 3, 3
(D*) 0, 0

(iii)

Increasing order of the energy of hybrid orbitals is


(A) sp3 < sp2 < sp
(B) sp2 < sp3 < sp
(C) sp < sp3 < sp2

Match the column :


Column I

(D*) sp < sp2 < sp3


[4]

Column II

(i)

IOF4

(P)

See-saw

(ii)

IO2 F2

(Q)

Trigonal bipyramidal

(iii)

XeO64

(R)

Linear

(iv)

XeF2

(S)

Square bipyramidal

(A) (i) P, (ii) S, (iii) R, (iv) Q


(C) (i) Q, (ii) P, (iii) S, (iv) R

(B*) (i) Q, (ii) P, (iii) S, (iv) R


(D) (i) Q, (ii) P, (iii) R, (iv) S

ASSIGNMENT-7
Q.1

Q.2

Q.3

Q.4

The correct order of d CH in the following option is


[3]
(A) CHF3 = CH2F2 = CH3F
(B) CHF3 > CH2F2 > CH3F
(C) CH2F2 > CH3F > CHF3
(D*) CH3F > CH2F2 > CHF3
O2F2 is an unstable yellow orange solid and H2O2 is a colourless liquid, both have OO bond.
OO bond length in H2O2 & O2F2 is respectively.
[3]
(A) 1.22, 1.48
(B*) 1.48, 1.22
(C) 1.22, 1.22
(D) 1.48, 1.48
The structure of O3 and N 3 are
(A) both linear
(B) Linear and bent respectively.
(C) both bent
(D*) Bent and linear respectively.
Select the correct order of following property.
(A) % s-character
:
sp3 > sp2 > sp

[3]

[3]

Q.5

^ bond angle
(B) ONO
:
NO3 > NO 2 > NO2
(C*) Second ionisation energy :
O>F>N>C
(D) C F bond length
:
CF4 > CH3F > CH2F2 > CF3H
Which of the following compounds is/are polar as well as non planar?

[3]
Cl

(A*) S2Cl2
Q.6
Q.7
Q.8

Q.9

Q.10

Q.11
Q.12

(B) B2H6

(C*) PCl2F3

The strongest PO bond is found in the molecule


(A*) F3PO
(B) Cl3PO
(C) Br3PO
F-As-F bond angle in AsF3Cl2 can be
(A*) 90 & 180 only (B) 120 only
(C) 90 & 120 only
The number of SS bonds in sulphur trioxide trimer (S3O9) is
(A) three
(B) two
(C) one

(D)

Cl

C=C=C=C

H
[3]

(D) (CH3)3PO
[3]
(D) 90 only
[3]
(D*) zero

Select the incorrect statement(s) about N2F4 and N2H4.


[3]
(A*) In N2F4 , d-orbitals are contracted by electronegative fluorine atoms, but d-orbital contraction is
not possible by H-atom in N2H4.
(B) The NN bond energy in N2F4 is more than NN bond energy in N2H4.
(C*) The NN bond length in N2F4 is more than that of in N2H4.
(D) The NN bond length in N2F4 is less than that of in N2H4.
Nodal planes of bond(s) in CH2=C=C=CH2 are located in
[3]
(A) All are in molecular plane
(B*) Two in molecular plane and one in a plane perpendicular to molecular plane which contains CC
-bond
(C) One in molecular plane and two in plane perpendicular to molecular plane which contains CC
-bond
(D) Two in molecular plane and one in a plane perpendicular to molecular plane which bisects CC
-bond at right angle
Which of the following shape can not be obtained from sp3d2 hybridisation.
[3]
(A) Square planar
(B) Square pyramidal (C*) Tetrahedral
(D) Octahedral
Which of the following statement(s) is/are correct about P4O6 and P4O10
[3]
(A*) Both oxides have closed cage like structure
(B*) Each oxide contains six equivalent POP bonds
(C) Both P4O6 and P4O10 molecules have pd bonds.
(D*) Both are the anhydrides of their respective acids.

ASSIGNMENT-4

Q.1

Find the species / molecule is having maximum number of lone pair on the central atom.
(A) ClOF4
(B) ClOF2+
(C) BH4
(D*) XeOF2

Q.2

If y-axis is the approaching axis between two atoms, then which of the set of orbitals can not form the
bond between two atoms in general.
[3]
(A) pz pz
(B) px px
(C*) px py
(D) None of these

Q.3

The maximum number of bond and -bond can be formed between two atoms are respectively.[3]
(A) 4, 3
(B*) 3, 2
(C) 2, 3
(D) 3, 1

Q.4

Which of the following set of overlap can not provide -bond formation.
(A) 3d and 2p
(B) 2p and 3p
(C) 2p and 2p
(D*) 3p and 1s

[3]

Q.5

The ratio of number of -bond to -bond in N2 and CO molecules are

[3]

(A) 2.0 , 2.0


Q.6

Q.7

Q.8

Q.9

Q.10

(B) 2,

1
2

(C*)

1 1
,
2 2

(D)

[3]

1
,2
2

Which of the following overlapping is involved in formation of only -bond.


(A*) s p overlapping
(B) p d overlapping
(C*) s s overlapping
(D) p p overlapping

[5]

Paragraph for question nos. 7 to 9


Different types of bonds are formed in the chemical compounds. These bond have different strength and
bond energies associated with them. These bonds are formed with atoms in different environments.
Which of the following bond has highest bond energy?
[9]
(A*) -bond
(B) -bond
(C) Hydrogen bond (D) Metallic bond
Shape of the molecule is decided by
(A*) -bond
(C) both and -bond

(B) -bond
(D) Never Nor -bond

Which of the following hydrides is thermally least stable?


(A) H2O
(B*) H2Te
(C) H2S
Match the column :
Column I
(Orders)
(A)
Increasing order of ionic
mobility in water
(B)
Decreasing order of size
(C)
Decreasing order of ionisation
energy

(D) H2Se
[4]

(P)

Column II
(Sequences)
Si, Mg, Al, Na

(Q)
(R)

Li+, Na+, K+, Rb+, Cs+


O, O+, O2, O

(S)

Se, S, O, F
[Ans. (A) Q, (B) S, (C) P]

ASSIGNMENT-5

Q.1

Explain type of hybridisation, shape, bond angle and geometry of the following compounds.
[18]
(1)
XeF2
(11) IF7
(2)

XeF4

(12)

OF2

(3)

XeF5

(13)

NO 3

(4)

XeOF4

(14)

ClO 4

(15)

SF4

(16)

I3

(17)

ClO 3

(18)

OCl2

(5)
(6)
(7)

PCl3
PCl5
SF2

(8)

SF6

(9)

IF3

(10)

IF5

Q.2

What is hybridisation of central atom of anionic part of PBr5 in crystalline state.


[3]
2
3
(A) sp
(B) sp
(C) sp
(D*) not applicable

Q.3

What is the difference between bond angles in cationic species of PCl5 and PBr5 in solid state. [3]
(A) 60
(B) 10928
(C*) 0
(D) 90

Q.4

All possible bond angles in anionic part of PCl5 are.


(A) 109 28 only
(B*) 90, 180
(C) 90, 120, 180

[3]
(D) 72, 90, 180

Q.5

The hybridisation and shape of XeO3F2 molecule is


(A) sp3 and tetrahedral
(B) sp3d and see-saw
(C*) sp3d and TBP
(D) sp3d2 and octahedral

Q.6

TeF5, XeF2 , I3+ , XeF4 , PCl3


Which of the following shape does not describe to any of the above species ?
(A) Square pyramidal
(B) Square planar
(C*) Trigonal planar
(D) Linear

[3]

[3]

Q.7

Which of the following species does not exist?


(A*) XeF3

Q.8

Q.10

Q.11

(D) XeF6

(B) CO2 and SO2

(C*) SO2 and I3

[3]

(D*) ICl 2 and BeH2

Statement-1 :
CH4 and CH2F2 are having regular tetrahedron geometry.
[3]
Statement-2 :
Both are having same hybridization.
(A) Statement-1 is true, statement-2 is true and statement-2 is correct explanation for statement-1.
(B) Statement-1 is true, statement-2 is true and statement-2 is NOT the correct explanation for statement-1.
(C) Statement-1 is true, statement-2 is false.
(D*) Statement-1 is false, statement-2 is true.
Paragraph for question nos. 10 to 12
Hybridisation is the mixing of atomic orbital of comparable energy and the number of hybrid orbitals
formed is equal to the number of pure atomic orbitals mixed up and hybrid orbitals are occupied by bond pair and lone pair.
[9]
3
Which of the following geometry is most likely to not form from sp d hybridisation of the central atom.
(A) Linear
(B*) Tetrahedral
(C) T-Shaped
(D) See-Saw
The orbital is not participated in sp3d2 hybridisation.
(A) px

Q.12

(C) XeF5

Which of following pair of species is having different hybridisation but same shape.
(A) BeCl2 and CO2

Q.9

(B) XeF4

[3]

(B*) dxy

(C) d x 2 y 2

(D) pz

"The hybrid orbitals are at angle of X to one another" this statement is not valid for which of the
following hybridisation.
(A) sp3
(B) sp2
(C*) sp3d2
(D) sp

ASSIGNMENT-6
One or more than one correct:
Q.1
Select the correct statement for non polar molecule.
[3]
(A*) On time average the molecule is non-polar but at the particular moment it act as a dipole which is
equally probale in all directions.
(B) On time average the molecule is polar but at the particular moment it does not act as a dipole.
(C) On time average the molecule is non-polar and the particular moment it must not act as dipole.
(D) All are incorrect
Q.2

Select the correct order of B.P.


(A) BF3 > BMe3
(B*) BF3 < BMe3

[3]
(C) BF3 = BMe3

(D) None of these

Q.3

Which molecular geometry are most likely to result, from a octahedral electron geometry?
(A*) square planar
(B*) square pyramidal (C) linear
(D) Vshaped

[3]

Q.4

The correct order of hybridization of the central atom in the following species
NH3, XeO2F2, SeF4, NO2+

[3]

Q.5

(A) sp3, sp3, sp3d, sp

(B*) sp3, sp3d, sp3d, sp

(C) sp3, sp3d2, sp3d, sp2

(D) sp2, sp3d, sp3d2, sp

Halogens form compounds among themselves with the formula XX', XX & XX '7 (where X is the heavier
halogen) which of the following pair(s) represent(s) correct geometry with polar and non-polar nature
(theoretically)
[3]
(A*) XX' Linear Polar
(B) XX Linear Polar
(C) XX' Linear Non-polar

(D*) XX '7 Pentagonal bipyramidal Non-polar

Q.6

In molecules of the type AX2Ln (where L represents lone pairs and n is its number) there exists a bond
between element A and X. The X A X bond angle.
[3]
(A) Always decreases if n increases
(B) Always increases if n increases
(C*) Will be maximum for n = 3, 0
(D) Generally decreases if n decreases

Q.7

Which of the following pairs of species have identical shapes?


(A)

Q.8

Q.9

Q.10

NO 2

and

NO 2

(B) PCl5 and BrF5

(C*) XeF4 and

[3]
ICl4

(D) TeCl4 and XeO4

London force works in


[3]
(A) Polar molecule
(B) Non-polar molecule
(C*) All polar and non-polar molecule
(D) Ionic compounds
Choose the correct on the ClO bond length in NaClO4.
[3]
(A) All ClO bonds are of equal length.
(B) Three ClO bonds are of equal of length one longer.
(C) Two ClO bonds are of same length which are longer compound to other two ClO bond length.
(D) All are different.
Statement-1 :
Experimentally 100 % covalent bond formation is not possible
[3]
Statement-2 :
Non polar molecule has instantaneous dipole induced dipole interaction
(A*) Statement-1 is true, statement-2 is true and statement-2 is correct explanation for statement-1.

(B) Statement-1 is true, statement-2 is true and statement-2 is NOT the correct explanation for statement-1.
(C) Statement-1 is true, statement-2 is false.
(D) Statement-1 is false, statement-2 is true.
Q.11

Q.12

Spin pairing and overlapping theory can not explain the equal bond length as well as equal bond angles
in CH4 molecule. To explain the above facts we are in need of hybridization theory. Hybridization is the
mixing of atomic orbitals of comparable energy and the number of atomic orbitals involved is equal to the
number of hybrid orbitals formed of equal energy.
[3 3 = 9]
(i)
According to hybridization theory, the %s character in sp3d hybrid orbitals is
(A) 25%
(B) 33.33%
(C*) 20%
(D) 16.66%
(ii)

The number of axial and equitorial positions in octahedral geometry having sp3d2 hybridization
(A) 2, 4
(B) 4, 2
(C) 3, 3
(D*) 0, 0

(iii)

Increasing order of the energy of hybrid orbitals is


(A) sp3 < sp2 < sp
(B) sp2 < sp3 < sp
(C) sp < sp3 < sp2

Match the column :


Column I

(D*) sp < sp2 < sp3


[4]

Column II

(i)

IOF4

(P)

See-saw

(ii)

IO2 F2

(Q)

Trigonal bipyramidal

(iii)

XeO64

(R)

Linear

(iv)

XeF2

(S)

Square bipyramidal

(A) (i) P, (ii) S, (iii) R, (iv) Q


(C) (i) Q, (ii) P, (iii) S, (iv) R

(B*) (i) Q, (ii) P, (iii) S, (iv) R


(D) (i) Q, (ii) P, (iii) R, (iv) S

ASSIGNMENT-7
Q.1

Q.2

Q.3

Q.4

The correct order of d CH in the following option is


[3]
(A) CHF3 = CH2F2 = CH3F
(B) CHF3 > CH2F2 > CH3F
(C) CH2F2 > CH3F > CHF3
(D*) CH3F > CH2F2 > CHF3
O2F2 is an unstable yellow orange solid and H2O2 is a colourless liquid, both have OO bond.
OO bond length in H2O2 & O2F2 is respectively.
[3]
(A) 1.22, 1.48
(B*) 1.48, 1.22
(C) 1.22, 1.22
(D) 1.48, 1.48
The structure of O3 and N 3 are
(A) both linear
(B) Linear and bent respectively.
(C) both bent
(D*) Bent and linear respectively.
Select the correct order of following property.
(A) % s-character
:
sp3 > sp2 > sp

[3]

[3]

Q.5

^ bond angle
(B) ONO
:
NO3 > NO 2 > NO2
(C*) Second ionisation energy :
O>F>N>C
(D) C F bond length
:
CF4 > CH3F > CH2F2 > CF3H
Which of the following compounds is/are polar as well as non planar?

[3]
Cl

(A*) S2Cl2
Q.6
Q.7
Q.8

Q.9

Q.10

Q.11
Q.12

(B) B2H6

(C*) PCl2F3

The strongest PO bond is found in the molecule


(A*) F3PO
(B) Cl3PO
(C) Br3PO
F-As-F bond angle in AsF3Cl2 can be
(A*) 90 & 180 only (B) 120 only
(C) 90 & 120 only
The number of SS bonds in sulphur trioxide trimer (S3O9) is
(A) three
(B) two
(C) one

(D)

Cl

C=C=C=C

H
[3]

(D) (CH3)3PO
[3]
(D) 90 only
[3]
(D*) zero

Select the incorrect statement(s) about N2F4 and N2H4.


[3]
(A*) In N2F4 , d-orbitals are contracted by electronegative fluorine atoms, but d-orbital contraction is
not possible by H-atom in N2H4.
(B) The NN bond energy in N2F4 is more than NN bond energy in N2H4.
(C*) The NN bond length in N2F4 is more than that of in N2H4.
(D) The NN bond length in N2F4 is less than that of in N2H4.
Nodal planes of bond(s) in CH2=C=C=CH2 are located in
[3]
(A) All are in molecular plane
(B*) Two in molecular plane and one in a plane perpendicular to molecular plane which contains CC
-bond
(C) One in molecular plane and two in plane perpendicular to molecular plane which contains CC
-bond
(D) Two in molecular plane and one in a plane perpendicular to molecular plane which bisects CC
-bond at right angle
Which of the following shape can not be obtained from sp3d2 hybridisation.
[3]
(A) Square planar
(B) Square pyramidal (C*) Tetrahedral
(D) Octahedral
Which of the following statement(s) is/are correct about P4O6 and P4O10
[3]
(A*) Both oxides have closed cage like structure
(B*) Each oxide contains six equivalent POP bonds
(C) Both P4O6 and P4O10 molecules have pd bonds.
(D*) Both are the anhydrides of their respective acids.

ASSIGNMENT-4

Q.1

Find the species / molecule is having maximum number of lone pair on the central atom.
(A) ClOF4
(B) ClOF2+
(C) BH4
(D*) XeOF2

Q.2

If y-axis is the approaching axis between two atoms, then which of the set of orbitals can not form the
bond between two atoms in general.
[3]
(A) pz pz
(B) px px
(C*) px py
(D) None of these

Q.3

The maximum number of bond and -bond can be formed between two atoms are respectively.[3]
(A) 4, 3
(B*) 3, 2
(C) 2, 3
(D) 3, 1

Q.4

Which of the following set of overlap can not provide -bond formation.
(A) 3d and 2p
(B) 2p and 3p
(C) 2p and 2p
(D*) 3p and 1s

[3]

Q.5

The ratio of number of -bond to -bond in N2 and CO molecules are

[3]

(A) 2.0 , 2.0


Q.6

Q.7

Q.8

Q.9

Q.10

(B) 2,

1
2

(C*)

1 1
,
2 2

(D)

[3]

1
,2
2

Which of the following overlapping is involved in formation of only -bond.


(A*) s p overlapping
(B) p d overlapping
(C*) s s overlapping
(D) p p overlapping

[5]

Paragraph for question nos. 7 to 9


Different types of bonds are formed in the chemical compounds. These bond have different strength and
bond energies associated with them. These bonds are formed with atoms in different environments.
Which of the following bond has highest bond energy?
[9]
(A*) -bond
(B) -bond
(C) Hydrogen bond (D) Metallic bond
Shape of the molecule is decided by
(A*) -bond
(C) both and -bond

(B) -bond
(D) Never Nor -bond

Which of the following hydrides is thermally least stable?


(A) H2O
(B*) H2Te
(C) H2S
Match the column :
Column I
(Orders)
(A)
Increasing order of ionic
mobility in water
(B)
Decreasing order of size
(C)
Decreasing order of ionisation
energy

(D) H2Se
[4]

(P)

Column II
(Sequences)
Si, Mg, Al, Na

(Q)
(R)

Li+, Na+, K+, Rb+, Cs+


O, O+, O2, O

(S)

Se, S, O, F
[Ans. (A) Q, (B) S, (C) P]

ASSIGNMENT-5

Q.1

Explain type of hybridisation, shape, bond angle and geometry of the following compounds.
[18]
(1)
XeF2
(11) IF7
(2)

XeF4

(12)

OF2

(3)

XeF5

(13)

NO 3

(4)

XeOF4

(14)

ClO 4

(15)

SF4

(16)

I3

(17)

ClO 3

(18)

OCl2

(5)
(6)
(7)

PCl3
PCl5
SF2

(8)

SF6

(9)

IF3

(10)

IF5

Q.2

What is hybridisation of central atom of anionic part of PBr5 in crystalline state.


[3]
2
3
(A) sp
(B) sp
(C) sp
(D*) not applicable

Q.3

What is the difference between bond angles in cationic species of PCl5 and PBr5 in solid state. [3]
(A) 60
(B) 10928
(C*) 0
(D) 90

Q.4

All possible bond angles in anionic part of PCl5 are.


(A) 109 28 only
(B*) 90, 180
(C) 90, 120, 180

[3]
(D) 72, 90, 180

Q.5

The hybridisation and shape of XeO3F2 molecule is


(A) sp3 and tetrahedral
(B) sp3d and see-saw
(C*) sp3d and TBP
(D) sp3d2 and octahedral

Q.6

TeF5, XeF2 , I3+ , XeF4 , PCl3


Which of the following shape does not describe to any of the above species ?
(A) Square pyramidal
(B) Square planar
(C*) Trigonal planar
(D) Linear

[3]

[3]

Q.7

Which of the following species does not exist?


(A*) XeF3

Q.8

Q.10

Q.11

(D) XeF6

(B) CO2 and SO2

(C*) SO2 and I3

[3]

(D*) ICl 2 and BeH2

Statement-1 :
CH4 and CH2F2 are having regular tetrahedron geometry.
[3]
Statement-2 :
Both are having same hybridization.
(A) Statement-1 is true, statement-2 is true and statement-2 is correct explanation for statement-1.
(B) Statement-1 is true, statement-2 is true and statement-2 is NOT the correct explanation for statement-1.
(C) Statement-1 is true, statement-2 is false.
(D*) Statement-1 is false, statement-2 is true.
Paragraph for question nos. 10 to 12
Hybridisation is the mixing of atomic orbital of comparable energy and the number of hybrid orbitals
formed is equal to the number of pure atomic orbitals mixed up and hybrid orbitals are occupied by bond pair and lone pair.
[9]
3
Which of the following geometry is most likely to not form from sp d hybridisation of the central atom.
(A) Linear
(B*) Tetrahedral
(C) T-Shaped
(D) See-Saw
The orbital is not participated in sp3d2 hybridisation.
(A) px

Q.12

(C) XeF5

Which of following pair of species is having different hybridisation but same shape.
(A) BeCl2 and CO2

Q.9

(B) XeF4

[3]

(B*) dxy

(C) d x 2 y 2

(D) pz

"The hybrid orbitals are at angle of X to one another" this statement is not valid for which of the
following hybridisation.
(A) sp3
(B) sp2
(C*) sp3d2
(D) sp

ASSIGNMENT-6
One or more than one correct:
Q.1
Select the correct statement for non polar molecule.
[3]
(A*) On time average the molecule is non-polar but at the particular moment it act as a dipole which is
equally probale in all directions.
(B) On time average the molecule is polar but at the particular moment it does not act as a dipole.
(C) On time average the molecule is non-polar and the particular moment it must not act as dipole.
(D) All are incorrect
Q.2

Select the correct order of B.P.


(A) BF3 > BMe3
(B*) BF3 < BMe3

[3]
(C) BF3 = BMe3

(D) None of these

Q.3

Which molecular geometry are most likely to result, from a octahedral electron geometry?
(A*) square planar
(B*) square pyramidal (C) linear
(D) Vshaped

[3]

Q.4

The correct order of hybridization of the central atom in the following species
NH3, XeO2F2, SeF4, NO2+

[3]

Q.5

(A) sp3, sp3, sp3d, sp

(B*) sp3, sp3d, sp3d, sp

(C) sp3, sp3d2, sp3d, sp2

(D) sp2, sp3d, sp3d2, sp

Halogens form compounds among themselves with the formula XX', XX & XX '7 (where X is the heavier
halogen) which of the following pair(s) represent(s) correct geometry with polar and non-polar nature
(theoretically)
[3]
(A*) XX' Linear Polar
(B) XX Linear Polar
(C) XX' Linear Non-polar

(D*) XX '7 Pentagonal bipyramidal Non-polar

Q.6

In molecules of the type AX2Ln (where L represents lone pairs and n is its number) there exists a bond
between element A and X. The X A X bond angle.
[3]
(A) Always decreases if n increases
(B) Always increases if n increases
(C*) Will be maximum for n = 3, 0
(D) Generally decreases if n decreases

Q.7

Which of the following pairs of species have identical shapes?


(A)

Q.8

Q.9

Q.10

NO 2

and

NO 2

(B) PCl5 and BrF5

(C*) XeF4 and

[3]
ICl4

(D) TeCl4 and XeO4

London force works in


[3]
(A) Polar molecule
(B) Non-polar molecule
(C*) All polar and non-polar molecule
(D) Ionic compounds
Choose the correct on the ClO bond length in NaClO4.
[3]
(A) All ClO bonds are of equal length.
(B) Three ClO bonds are of equal of length one longer.
(C) Two ClO bonds are of same length which are longer compound to other two ClO bond length.
(D) All are different.
Statement-1 :
Experimentally 100 % covalent bond formation is not possible
[3]
Statement-2 :
Non polar molecule has instantaneous dipole induced dipole interaction
(A*) Statement-1 is true, statement-2 is true and statement-2 is correct explanation for statement-1.

(B) Statement-1 is true, statement-2 is true and statement-2 is NOT the correct explanation for statement-1.
(C) Statement-1 is true, statement-2 is false.
(D) Statement-1 is false, statement-2 is true.
Q.11

Q.12

Spin pairing and overlapping theory can not explain the equal bond length as well as equal bond angles
in CH4 molecule. To explain the above facts we are in need of hybridization theory. Hybridization is the
mixing of atomic orbitals of comparable energy and the number of atomic orbitals involved is equal to the
number of hybrid orbitals formed of equal energy.
[3 3 = 9]
(i)
According to hybridization theory, the %s character in sp3d hybrid orbitals is
(A) 25%
(B) 33.33%
(C*) 20%
(D) 16.66%
(ii)

The number of axial and equitorial positions in octahedral geometry having sp3d2 hybridization
(A) 2, 4
(B) 4, 2
(C) 3, 3
(D*) 0, 0

(iii)

Increasing order of the energy of hybrid orbitals is


(A) sp3 < sp2 < sp
(B) sp2 < sp3 < sp
(C) sp < sp3 < sp2

Match the column :


Column I

(D*) sp < sp2 < sp3


[4]

Column II

(i)

IOF4

(P)

See-saw

(ii)

IO2 F2

(Q)

Trigonal bipyramidal

(iii)

XeO64

(R)

Linear

(iv)

XeF2

(S)

Square bipyramidal

(A) (i) P, (ii) S, (iii) R, (iv) Q


(C) (i) Q, (ii) P, (iii) S, (iv) R

(B*) (i) Q, (ii) P, (iii) S, (iv) R


(D) (i) Q, (ii) P, (iii) R, (iv) S

ASSIGNMENT-7
Q.1

Q.2

Q.3

Q.4

The correct order of d CH in the following option is


[3]
(A) CHF3 = CH2F2 = CH3F
(B) CHF3 > CH2F2 > CH3F
(C) CH2F2 > CH3F > CHF3
(D*) CH3F > CH2F2 > CHF3
O2F2 is an unstable yellow orange solid and H2O2 is a colourless liquid, both have OO bond.
OO bond length in H2O2 & O2F2 is respectively.
[3]
(A) 1.22, 1.48
(B*) 1.48, 1.22
(C) 1.22, 1.22
(D) 1.48, 1.48
The structure of O3 and N 3 are
(A) both linear
(B) Linear and bent respectively.
(C) both bent
(D*) Bent and linear respectively.
Select the correct order of following property.
(A) % s-character
:
sp3 > sp2 > sp

[3]

[3]

Q.5

^ bond angle
(B) ONO
:
NO3 > NO 2 > NO2
(C*) Second ionisation energy :
O>F>N>C
(D) C F bond length
:
CF4 > CH3F > CH2F2 > CF3H
Which of the following compounds is/are polar as well as non planar?

[3]
Cl

(A*) S2Cl2
Q.6
Q.7
Q.8

Q.9

Q.10

Q.11
Q.12

(B) B2H6

(C*) PCl2F3

The strongest PO bond is found in the molecule


(A*) F3PO
(B) Cl3PO
(C) Br3PO
F-As-F bond angle in AsF3Cl2 can be
(A*) 90 & 180 only (B) 120 only
(C) 90 & 120 only
The number of SS bonds in sulphur trioxide trimer (S3O9) is
(A) three
(B) two
(C) one

(D)

Cl

C=C=C=C

H
[3]

(D) (CH3)3PO
[3]
(D) 90 only
[3]
(D*) zero

Select the incorrect statement(s) about N2F4 and N2H4.


[3]
(A*) In N2F4 , d-orbitals are contracted by electronegative fluorine atoms, but d-orbital contraction is
not possible by H-atom in N2H4.
(B) The NN bond energy in N2F4 is more than NN bond energy in N2H4.
(C*) The NN bond length in N2F4 is more than that of in N2H4.
(D) The NN bond length in N2F4 is less than that of in N2H4.
Nodal planes of bond(s) in CH2=C=C=CH2 are located in
[3]
(A) All are in molecular plane
(B*) Two in molecular plane and one in a plane perpendicular to molecular plane which contains CC
-bond
(C) One in molecular plane and two in plane perpendicular to molecular plane which contains CC
-bond
(D) Two in molecular plane and one in a plane perpendicular to molecular plane which bisects CC
-bond at right angle
Which of the following shape can not be obtained from sp3d2 hybridisation.
[3]
(A) Square planar
(B) Square pyramidal (C*) Tetrahedral
(D) Octahedral
Which of the following statement(s) is/are correct about P4O6 and P4O10
[3]
(A*) Both oxides have closed cage like structure
(B*) Each oxide contains six equivalent POP bonds
(C) Both P4O6 and P4O10 molecules have pd bonds.
(D*) Both are the anhydrides of their respective acids.

ASSIGNMENT-4

Q.1

Find the species / molecule is having maximum number of lone pair on the central atom.
(A) ClOF4
(B) ClOF2+
(C) BH4
(D*) XeOF2

Q.2

If y-axis is the approaching axis between two atoms, then which of the set of orbitals can not form the
bond between two atoms in general.
[3]
(A) pz pz
(B) px px
(C*) px py
(D) None of these

Q.3

The maximum number of bond and -bond can be formed between two atoms are respectively.[3]
(A) 4, 3
(B*) 3, 2
(C) 2, 3
(D) 3, 1

Q.4

Which of the following set of overlap can not provide -bond formation.
(A) 3d and 2p
(B) 2p and 3p
(C) 2p and 2p
(D*) 3p and 1s

[3]

Q.5

The ratio of number of -bond to -bond in N2 and CO molecules are

[3]

(A) 2.0 , 2.0


Q.6

Q.7

Q.8

Q.9

Q.10

(B) 2,

1
2

(C*)

1 1
,
2 2

(D)

[3]

1
,2
2

Which of the following overlapping is involved in formation of only -bond.


(A*) s p overlapping
(B) p d overlapping
(C*) s s overlapping
(D) p p overlapping

[5]

Paragraph for question nos. 7 to 9


Different types of bonds are formed in the chemical compounds. These bond have different strength and
bond energies associated with them. These bonds are formed with atoms in different environments.
Which of the following bond has highest bond energy?
[9]
(A*) -bond
(B) -bond
(C) Hydrogen bond (D) Metallic bond
Shape of the molecule is decided by
(A*) -bond
(C) both and -bond

(B) -bond
(D) Never Nor -bond

Which of the following hydrides is thermally least stable?


(A) H2O
(B*) H2Te
(C) H2S
Match the column :
Column I
(Orders)
(A)
Increasing order of ionic
mobility in water
(B)
Decreasing order of size
(C)
Decreasing order of ionisation
energy

(D) H2Se
[4]

(P)

Column II
(Sequences)
Si, Mg, Al, Na

(Q)
(R)

Li+, Na+, K+, Rb+, Cs+


O, O+, O2, O

(S)

Se, S, O, F
[Ans. (A) Q, (B) S, (C) P]

ASSIGNMENT-5

Q.1

Explain type of hybridisation, shape, bond angle and geometry of the following compounds.
[18]
(1)
XeF2
(11) IF7
(2)

XeF4

(12)

OF2

(3)

XeF5

(13)

NO 3

(4)

XeOF4

(14)

ClO 4

(15)

SF4

(16)

I3

(17)

ClO 3

(18)

OCl2

(5)
(6)
(7)

PCl3
PCl5
SF2

(8)

SF6

(9)

IF3

(10)

IF5

Q.2

What is hybridisation of central atom of anionic part of PBr5 in crystalline state.


[3]
2
3
(A) sp
(B) sp
(C) sp
(D*) not applicable

Q.3

What is the difference between bond angles in cationic species of PCl5 and PBr5 in solid state. [3]
(A) 60
(B) 10928
(C*) 0
(D) 90

Q.4

All possible bond angles in anionic part of PCl5 are.


(A) 109 28 only
(B*) 90, 180
(C) 90, 120, 180

[3]
(D) 72, 90, 180

Q.5

The hybridisation and shape of XeO3F2 molecule is


(A) sp3 and tetrahedral
(B) sp3d and see-saw
(C*) sp3d and TBP
(D) sp3d2 and octahedral

Q.6

TeF5, XeF2 , I3+ , XeF4 , PCl3


Which of the following shape does not describe to any of the above species ?
(A) Square pyramidal
(B) Square planar
(C*) Trigonal planar
(D) Linear

[3]

[3]

Q.7

Which of the following species does not exist?


(A*) XeF3

Q.8

Q.10

Q.11

(D) XeF6

(B) CO2 and SO2

(C*) SO2 and I3

[3]

(D*) ICl 2 and BeH2

Statement-1 :
CH4 and CH2F2 are having regular tetrahedron geometry.
[3]
Statement-2 :
Both are having same hybridization.
(A) Statement-1 is true, statement-2 is true and statement-2 is correct explanation for statement-1.
(B) Statement-1 is true, statement-2 is true and statement-2 is NOT the correct explanation for statement-1.
(C) Statement-1 is true, statement-2 is false.
(D*) Statement-1 is false, statement-2 is true.
Paragraph for question nos. 10 to 12
Hybridisation is the mixing of atomic orbital of comparable energy and the number of hybrid orbitals
formed is equal to the number of pure atomic orbitals mixed up and hybrid orbitals are occupied by bond pair and lone pair.
[9]
3
Which of the following geometry is most likely to not form from sp d hybridisation of the central atom.
(A) Linear
(B*) Tetrahedral
(C) T-Shaped
(D) See-Saw
The orbital is not participated in sp3d2 hybridisation.
(A) px

Q.12

(C) XeF5

Which of following pair of species is having different hybridisation but same shape.
(A) BeCl2 and CO2

Q.9

(B) XeF4

[3]

(B*) dxy

(C) d x 2 y 2

(D) pz

"The hybrid orbitals are at angle of X to one another" this statement is not valid for which of the
following hybridisation.
(A) sp3
(B) sp2
(C*) sp3d2
(D) sp

ASSIGNMENT-6
One or more than one correct:
Q.1
Select the correct statement for non polar molecule.
[3]
(A*) On time average the molecule is non-polar but at the particular moment it act as a dipole which is
equally probale in all directions.
(B) On time average the molecule is polar but at the particular moment it does not act as a dipole.
(C) On time average the molecule is non-polar and the particular moment it must not act as dipole.
(D) All are incorrect
Q.2

Select the correct order of B.P.


(A) BF3 > BMe3
(B*) BF3 < BMe3

[3]
(C) BF3 = BMe3

(D) None of these

Q.3

Which molecular geometry are most likely to result, from a octahedral electron geometry?
(A*) square planar
(B*) square pyramidal (C) linear
(D) Vshaped

[3]

Q.4

The correct order of hybridization of the central atom in the following species
NH3, XeO2F2, SeF4, NO2+

[3]

Q.5

(A) sp3, sp3, sp3d, sp

(B*) sp3, sp3d, sp3d, sp

(C) sp3, sp3d2, sp3d, sp2

(D) sp2, sp3d, sp3d2, sp

Halogens form compounds among themselves with the formula XX', XX & XX '7 (where X is the heavier
halogen) which of the following pair(s) represent(s) correct geometry with polar and non-polar nature
(theoretically)
[3]
(A*) XX' Linear Polar
(B) XX Linear Polar
(C) XX' Linear Non-polar

(D*) XX '7 Pentagonal bipyramidal Non-polar

Q.6

In molecules of the type AX2Ln (where L represents lone pairs and n is its number) there exists a bond
between element A and X. The X A X bond angle.
[3]
(A) Always decreases if n increases
(B) Always increases if n increases
(C*) Will be maximum for n = 3, 0
(D) Generally decreases if n decreases

Q.7

Which of the following pairs of species have identical shapes?


(A)

Q.8

Q.9

Q.10

NO 2

and

NO 2

(B) PCl5 and BrF5

(C*) XeF4 and

[3]
ICl4

(D) TeCl4 and XeO4

London force works in


[3]
(A) Polar molecule
(B) Non-polar molecule
(C*) All polar and non-polar molecule
(D) Ionic compounds
Choose the correct on the ClO bond length in NaClO4.
[3]
(A) All ClO bonds are of equal length.
(B) Three ClO bonds are of equal of length one longer.
(C) Two ClO bonds are of same length which are longer compound to other two ClO bond length.
(D) All are different.
Statement-1 :
Experimentally 100 % covalent bond formation is not possible
[3]
Statement-2 :
Non polar molecule has instantaneous dipole induced dipole interaction
(A*) Statement-1 is true, statement-2 is true and statement-2 is correct explanation for statement-1.

(B) Statement-1 is true, statement-2 is true and statement-2 is NOT the correct explanation for statement-1.
(C) Statement-1 is true, statement-2 is false.
(D) Statement-1 is false, statement-2 is true.
Q.11

Q.12

Spin pairing and overlapping theory can not explain the equal bond length as well as equal bond angles
in CH4 molecule. To explain the above facts we are in need of hybridization theory. Hybridization is the
mixing of atomic orbitals of comparable energy and the number of atomic orbitals involved is equal to the
number of hybrid orbitals formed of equal energy.
[3 3 = 9]
(i)
According to hybridization theory, the %s character in sp3d hybrid orbitals is
(A) 25%
(B) 33.33%
(C*) 20%
(D) 16.66%
(ii)

The number of axial and equitorial positions in octahedral geometry having sp3d2 hybridization
(A) 2, 4
(B) 4, 2
(C) 3, 3
(D*) 0, 0

(iii)

Increasing order of the energy of hybrid orbitals is


(A) sp3 < sp2 < sp
(B) sp2 < sp3 < sp
(C) sp < sp3 < sp2

Match the column :


Column I

(D*) sp < sp2 < sp3


[4]

Column II

(i)

IOF4

(P)

See-saw

(ii)

IO2 F2

(Q)

Trigonal bipyramidal

(iii)

XeO64

(R)

Linear

(iv)

XeF2

(S)

Square bipyramidal

(A) (i) P, (ii) S, (iii) R, (iv) Q


(C) (i) Q, (ii) P, (iii) S, (iv) R

(B*) (i) Q, (ii) P, (iii) S, (iv) R


(D) (i) Q, (ii) P, (iii) R, (iv) S

ASSIGNMENT-7
Q.1

Q.2

Q.3

Q.4

The correct order of d CH in the following option is


[3]
(A) CHF3 = CH2F2 = CH3F
(B) CHF3 > CH2F2 > CH3F
(C) CH2F2 > CH3F > CHF3
(D*) CH3F > CH2F2 > CHF3
O2F2 is an unstable yellow orange solid and H2O2 is a colourless liquid, both have OO bond.
OO bond length in H2O2 & O2F2 is respectively.
[3]
(A) 1.22, 1.48
(B*) 1.48, 1.22
(C) 1.22, 1.22
(D) 1.48, 1.48
The structure of O3 and N 3 are
(A) both linear
(B) Linear and bent respectively.
(C) both bent
(D*) Bent and linear respectively.
Select the correct order of following property.
(A) % s-character
:
sp3 > sp2 > sp

[3]

[3]

Q.5

^ bond angle
(B) ONO
:
NO3 > NO 2 > NO2
(C*) Second ionisation energy :
O>F>N>C
(D) C F bond length
:
CF4 > CH3F > CH2F2 > CF3H
Which of the following compounds is/are polar as well as non planar?

[3]
Cl

(A*) S2Cl2
Q.6
Q.7
Q.8

Q.9

Q.10

Q.11
Q.12

(B) B2H6

(C*) PCl2F3

The strongest PO bond is found in the molecule


(A*) F3PO
(B) Cl3PO
(C) Br3PO
F-As-F bond angle in AsF3Cl2 can be
(A*) 90 & 180 only (B) 120 only
(C) 90 & 120 only
The number of SS bonds in sulphur trioxide trimer (S3O9) is
(A) three
(B) two
(C) one

(D)

Cl

C=C=C=C

H
[3]

(D) (CH3)3PO
[3]
(D) 90 only
[3]
(D*) zero

Select the incorrect statement(s) about N2F4 and N2H4.


[3]
(A*) In N2F4 , d-orbitals are contracted by electronegative fluorine atoms, but d-orbital contraction is
not possible by H-atom in N2H4.
(B) The NN bond energy in N2F4 is more than NN bond energy in N2H4.
(C*) The NN bond length in N2F4 is more than that of in N2H4.
(D) The NN bond length in N2F4 is less than that of in N2H4.
Nodal planes of bond(s) in CH2=C=C=CH2 are located in
[3]
(A) All are in molecular plane
(B*) Two in molecular plane and one in a plane perpendicular to molecular plane which contains CC
-bond
(C) One in molecular plane and two in plane perpendicular to molecular plane which contains CC
-bond
(D) Two in molecular plane and one in a plane perpendicular to molecular plane which bisects CC
-bond at right angle
Which of the following shape can not be obtained from sp3d2 hybridisation.
[3]
(A) Square planar
(B) Square pyramidal (C*) Tetrahedral
(D) Octahedral
Which of the following statement(s) is/are correct about P4O6 and P4O10
[3]
(A*) Both oxides have closed cage like structure
(B*) Each oxide contains six equivalent POP bonds
(C) Both P4O6 and P4O10 molecules have pd bonds.
(D*) Both are the anhydrides of their respective acids.

ASSIGNMENT-4

Q.1

Find the species / molecule is having maximum number of lone pair on the central atom.
(A) ClOF4
(B) ClOF2+
(C) BH4
(D*) XeOF2

Q.2

If y-axis is the approaching axis between two atoms, then which of the set of orbitals can not form the
bond between two atoms in general.
[3]
(A) pz pz
(B) px px
(C*) px py
(D) None of these

Q.3

The maximum number of bond and -bond can be formed between two atoms are respectively.[3]
(A) 4, 3
(B*) 3, 2
(C) 2, 3
(D) 3, 1

Q.4

Which of the following set of overlap can not provide -bond formation.
(A) 3d and 2p
(B) 2p and 3p
(C) 2p and 2p
(D*) 3p and 1s

[3]

Q.5

The ratio of number of -bond to -bond in N2 and CO molecules are

[3]

(A) 2.0 , 2.0


Q.6

Q.7

Q.8

Q.9

Q.10

(B) 2,

1
2

(C*)

1 1
,
2 2

(D)

[3]

1
,2
2

Which of the following overlapping is involved in formation of only -bond.


(A*) s p overlapping
(B) p d overlapping
(C*) s s overlapping
(D) p p overlapping

[5]

Paragraph for question nos. 7 to 9


Different types of bonds are formed in the chemical compounds. These bond have different strength and
bond energies associated with them. These bonds are formed with atoms in different environments.
Which of the following bond has highest bond energy?
[9]
(A*) -bond
(B) -bond
(C) Hydrogen bond (D) Metallic bond
Shape of the molecule is decided by
(A*) -bond
(C) both and -bond

(B) -bond
(D) Never Nor -bond

Which of the following hydrides is thermally least stable?


(A) H2O
(B*) H2Te
(C) H2S
Match the column :
Column I
(Orders)
(A)
Increasing order of ionic
mobility in water
(B)
Decreasing order of size
(C)
Decreasing order of ionisation
energy

(D) H2Se
[4]

(P)

Column II
(Sequences)
Si, Mg, Al, Na

(Q)
(R)

Li+, Na+, K+, Rb+, Cs+


O, O+, O2, O

(S)

Se, S, O, F
[Ans. (A) Q, (B) S, (C) P]

ASSIGNMENT-5

Q.1

Explain type of hybridisation, shape, bond angle and geometry of the following compounds.
[18]
(1)
XeF2
(11) IF7
(2)

XeF4

(12)

OF2

(3)

XeF5

(13)

NO 3

(4)

XeOF4

(14)

ClO 4

(15)

SF4

(16)

I3

(17)

ClO 3

(18)

OCl2

(5)
(6)
(7)

PCl3
PCl5
SF2

(8)

SF6

(9)

IF3

(10)

IF5

Q.2

What is hybridisation of central atom of anionic part of PBr5 in crystalline state.


[3]
2
3
(A) sp
(B) sp
(C) sp
(D*) not applicable

Q.3

What is the difference between bond angles in cationic species of PCl5 and PBr5 in solid state. [3]
(A) 60
(B) 10928
(C*) 0
(D) 90

Q.4

All possible bond angles in anionic part of PCl5 are.


(A) 109 28 only
(B*) 90, 180
(C) 90, 120, 180

[3]
(D) 72, 90, 180

Q.5

The hybridisation and shape of XeO3F2 molecule is


(A) sp3 and tetrahedral
(B) sp3d and see-saw
(C*) sp3d and TBP
(D) sp3d2 and octahedral

Q.6

TeF5, XeF2 , I3+ , XeF4 , PCl3


Which of the following shape does not describe to any of the above species ?
(A) Square pyramidal
(B) Square planar
(C*) Trigonal planar
(D) Linear

[3]

[3]

Q.7

Which of the following species does not exist?


(A*) XeF3

Q.8

Q.10

Q.11

(D) XeF6

(B) CO2 and SO2

(C*) SO2 and I3

[3]

(D*) ICl 2 and BeH2

Statement-1 :
CH4 and CH2F2 are having regular tetrahedron geometry.
[3]
Statement-2 :
Both are having same hybridization.
(A) Statement-1 is true, statement-2 is true and statement-2 is correct explanation for statement-1.
(B) Statement-1 is true, statement-2 is true and statement-2 is NOT the correct explanation for statement-1.
(C) Statement-1 is true, statement-2 is false.
(D*) Statement-1 is false, statement-2 is true.
Paragraph for question nos. 10 to 12
Hybridisation is the mixing of atomic orbital of comparable energy and the number of hybrid orbitals
formed is equal to the number of pure atomic orbitals mixed up and hybrid orbitals are occupied by bond pair and lone pair.
[9]
3
Which of the following geometry is most likely to not form from sp d hybridisation of the central atom.
(A) Linear
(B*) Tetrahedral
(C) T-Shaped
(D) See-Saw
The orbital is not participated in sp3d2 hybridisation.
(A) px

Q.12

(C) XeF5

Which of following pair of species is having different hybridisation but same shape.
(A) BeCl2 and CO2

Q.9

(B) XeF4

[3]

(B*) dxy

(C) d x 2 y 2

(D) pz

"The hybrid orbitals are at angle of X to one another" this statement is not valid for which of the
following hybridisation.
(A) sp3
(B) sp2
(C*) sp3d2
(D) sp

ASSIGNMENT-6
One or more than one correct:
Q.1
Select the correct statement for non polar molecule.
[3]
(A*) On time average the molecule is non-polar but at the particular moment it act as a dipole which is
equally probale in all directions.
(B) On time average the molecule is polar but at the particular moment it does not act as a dipole.
(C) On time average the molecule is non-polar and the particular moment it must not act as dipole.
(D) All are incorrect
Q.2

Select the correct order of B.P.


(A) BF3 > BMe3
(B*) BF3 < BMe3

[3]
(C) BF3 = BMe3

(D) None of these

Q.3

Which molecular geometry are most likely to result, from a octahedral electron geometry?
(A*) square planar
(B*) square pyramidal (C) linear
(D) Vshaped

[3]

Q.4

The correct order of hybridization of the central atom in the following species
NH3, XeO2F2, SeF4, NO2+

[3]

Q.5

(A) sp3, sp3, sp3d, sp

(B*) sp3, sp3d, sp3d, sp

(C) sp3, sp3d2, sp3d, sp2

(D) sp2, sp3d, sp3d2, sp

Halogens form compounds among themselves with the formula XX', XX & XX '7 (where X is the heavier
halogen) which of the following pair(s) represent(s) correct geometry with polar and non-polar nature
(theoretically)
[3]
(A*) XX' Linear Polar
(B) XX Linear Polar
(C) XX' Linear Non-polar

(D*) XX '7 Pentagonal bipyramidal Non-polar

Q.6

In molecules of the type AX2Ln (where L represents lone pairs and n is its number) there exists a bond
between element A and X. The X A X bond angle.
[3]
(A) Always decreases if n increases
(B) Always increases if n increases
(C*) Will be maximum for n = 3, 0
(D) Generally decreases if n decreases

Q.7

Which of the following pairs of species have identical shapes?


(A)

Q.8

Q.9

Q.10

NO 2

and

NO 2

(B) PCl5 and BrF5

(C*) XeF4 and

[3]
ICl4

(D) TeCl4 and XeO4

London force works in


[3]
(A) Polar molecule
(B) Non-polar molecule
(C*) All polar and non-polar molecule
(D) Ionic compounds
Choose the correct on the ClO bond length in NaClO4.
[3]
(A) All ClO bonds are of equal length.
(B) Three ClO bonds are of equal of length one longer.
(C) Two ClO bonds are of same length which are longer compound to other two ClO bond length.
(D) All are different.
Statement-1 :
Experimentally 100 % covalent bond formation is not possible
[3]
Statement-2 :
Non polar molecule has instantaneous dipole induced dipole interaction
(A*) Statement-1 is true, statement-2 is true and statement-2 is correct explanation for statement-1.

(B) Statement-1 is true, statement-2 is true and statement-2 is NOT the correct explanation for statement-1.
(C) Statement-1 is true, statement-2 is false.
(D) Statement-1 is false, statement-2 is true.
Q.11

Q.12

Spin pairing and overlapping theory can not explain the equal bond length as well as equal bond angles
in CH4 molecule. To explain the above facts we are in need of hybridization theory. Hybridization is the
mixing of atomic orbitals of comparable energy and the number of atomic orbitals involved is equal to the
number of hybrid orbitals formed of equal energy.
[3 3 = 9]
(i)
According to hybridization theory, the %s character in sp3d hybrid orbitals is
(A) 25%
(B) 33.33%
(C*) 20%
(D) 16.66%
(ii)

The number of axial and equitorial positions in octahedral geometry having sp3d2 hybridization
(A) 2, 4
(B) 4, 2
(C) 3, 3
(D*) 0, 0

(iii)

Increasing order of the energy of hybrid orbitals is


(A) sp3 < sp2 < sp
(B) sp2 < sp3 < sp
(C) sp < sp3 < sp2

Match the column :


Column I

(D*) sp < sp2 < sp3


[4]

Column II

(i)

IOF4

(P)

See-saw

(ii)

IO2 F2

(Q)

Trigonal bipyramidal

(iii)

XeO64

(R)

Linear

(iv)

XeF2

(S)

Square bipyramidal

(A) (i) P, (ii) S, (iii) R, (iv) Q


(C) (i) Q, (ii) P, (iii) S, (iv) R

(B*) (i) Q, (ii) P, (iii) S, (iv) R


(D) (i) Q, (ii) P, (iii) R, (iv) S

ASSIGNMENT-7
Q.1

Q.2

Q.3

Q.4

The correct order of d CH in the following option is


[3]
(A) CHF3 = CH2F2 = CH3F
(B) CHF3 > CH2F2 > CH3F
(C) CH2F2 > CH3F > CHF3
(D*) CH3F > CH2F2 > CHF3
O2F2 is an unstable yellow orange solid and H2O2 is a colourless liquid, both have OO bond.
OO bond length in H2O2 & O2F2 is respectively.
[3]
(A) 1.22, 1.48
(B*) 1.48, 1.22
(C) 1.22, 1.22
(D) 1.48, 1.48
The structure of O3 and N 3 are
(A) both linear
(B) Linear and bent respectively.
(C) both bent
(D*) Bent and linear respectively.
Select the correct order of following property.
(A) % s-character
:
sp3 > sp2 > sp

[3]

[3]

Q.5

^ bond angle
(B) ONO
:
NO3 > NO 2 > NO2
(C*) Second ionisation energy :
O>F>N>C
(D) C F bond length
:
CF4 > CH3F > CH2F2 > CF3H
Which of the following compounds is/are polar as well as non planar?

[3]
Cl

(A*) S2Cl2
Q.6
Q.7
Q.8

Q.9

Q.10

Q.11
Q.12

(B) B2H6

(C*) PCl2F3

The strongest PO bond is found in the molecule


(A*) F3PO
(B) Cl3PO
(C) Br3PO
F-As-F bond angle in AsF3Cl2 can be
(A*) 90 & 180 only (B) 120 only
(C) 90 & 120 only
The number of SS bonds in sulphur trioxide trimer (S3O9) is
(A) three
(B) two
(C) one

(D)

Cl

C=C=C=C

H
[3]

(D) (CH3)3PO
[3]
(D) 90 only
[3]
(D*) zero

Select the incorrect statement(s) about N2F4 and N2H4.


[3]
(A*) In N2F4 , d-orbitals are contracted by electronegative fluorine atoms, but d-orbital contraction is
not possible by H-atom in N2H4.
(B) The NN bond energy in N2F4 is more than NN bond energy in N2H4.
(C*) The NN bond length in N2F4 is more than that of in N2H4.
(D) The NN bond length in N2F4 is less than that of in N2H4.
Nodal planes of bond(s) in CH2=C=C=CH2 are located in
[3]
(A) All are in molecular plane
(B*) Two in molecular plane and one in a plane perpendicular to molecular plane which contains CC
-bond
(C) One in molecular plane and two in plane perpendicular to molecular plane which contains CC
-bond
(D) Two in molecular plane and one in a plane perpendicular to molecular plane which bisects CC
-bond at right angle
Which of the following shape can not be obtained from sp3d2 hybridisation.
[3]
(A) Square planar
(B) Square pyramidal (C*) Tetrahedral
(D) Octahedral
Which of the following statement(s) is/are correct about P4O6 and P4O10
[3]
(A*) Both oxides have closed cage like structure
(B*) Each oxide contains six equivalent POP bonds
(C) Both P4O6 and P4O10 molecules have pd bonds.
(D*) Both are the anhydrides of their respective acids.

ASSIGNMENT-4

Q.1

Find the species / molecule is having maximum number of lone pair on the central atom.
(A) ClOF4
(B) ClOF2+
(C) BH4
(D*) XeOF2

Q.2

If y-axis is the approaching axis between two atoms, then which of the set of orbitals can not form the
bond between two atoms in general.
[3]
(A) pz pz
(B) px px
(C*) px py
(D) None of these

Q.3

The maximum number of bond and -bond can be formed between two atoms are respectively.[3]
(A) 4, 3
(B*) 3, 2
(C) 2, 3
(D) 3, 1

Q.4

Which of the following set of overlap can not provide -bond formation.
(A) 3d and 2p
(B) 2p and 3p
(C) 2p and 2p
(D*) 3p and 1s

[3]

Q.5

The ratio of number of -bond to -bond in N2 and CO molecules are

[3]

(A) 2.0 , 2.0


Q.6

Q.7

Q.8

Q.9

Q.10

(B) 2,

1
2

(C*)

1 1
,
2 2

(D)

[3]

1
,2
2

Which of the following overlapping is involved in formation of only -bond.


(A*) s p overlapping
(B) p d overlapping
(C*) s s overlapping
(D) p p overlapping

[5]

Paragraph for question nos. 7 to 9


Different types of bonds are formed in the chemical compounds. These bond have different strength and
bond energies associated with them. These bonds are formed with atoms in different environments.
Which of the following bond has highest bond energy?
[9]
(A*) -bond
(B) -bond
(C) Hydrogen bond (D) Metallic bond
Shape of the molecule is decided by
(A*) -bond
(C) both and -bond

(B) -bond
(D) Never Nor -bond

Which of the following hydrides is thermally least stable?


(A) H2O
(B*) H2Te
(C) H2S
Match the column :
Column I
(Orders)
(A)
Increasing order of ionic
mobility in water
(B)
Decreasing order of size
(C)
Decreasing order of ionisation
energy

(D) H2Se
[4]

(P)

Column II
(Sequences)
Si, Mg, Al, Na

(Q)
(R)

Li+, Na+, K+, Rb+, Cs+


O, O+, O2, O

(S)

Se, S, O, F
[Ans. (A) Q, (B) S, (C) P]

ASSIGNMENT-5

Q.1

Explain type of hybridisation, shape, bond angle and geometry of the following compounds.
[18]
(1)
XeF2
(11) IF7
(2)

XeF4

(12)

OF2

(3)

XeF5

(13)

NO 3

(4)

XeOF4

(14)

ClO 4

(15)

SF4

(16)

I3

(17)

ClO 3

(18)

OCl2

(5)
(6)
(7)

PCl3
PCl5
SF2

(8)

SF6

(9)

IF3

(10)

IF5

Q.2

What is hybridisation of central atom of anionic part of PBr5 in crystalline state.


[3]
2
3
(A) sp
(B) sp
(C) sp
(D*) not applicable

Q.3

What is the difference between bond angles in cationic species of PCl5 and PBr5 in solid state. [3]
(A) 60
(B) 10928
(C*) 0
(D) 90

Q.4

All possible bond angles in anionic part of PCl5 are.


(A) 109 28 only
(B*) 90, 180
(C) 90, 120, 180

[3]
(D) 72, 90, 180

Q.5

The hybridisation and shape of XeO3F2 molecule is


(A) sp3 and tetrahedral
(B) sp3d and see-saw
(C*) sp3d and TBP
(D) sp3d2 and octahedral

Q.6

TeF5, XeF2 , I3+ , XeF4 , PCl3


Which of the following shape does not describe to any of the above species ?
(A) Square pyramidal
(B) Square planar
(C*) Trigonal planar
(D) Linear

[3]

[3]

Q.7

Which of the following species does not exist?


(A*) XeF3

Q.8

Q.10

Q.11

(D) XeF6

(B) CO2 and SO2

(C*) SO2 and I3

[3]

(D*) ICl 2 and BeH2

Statement-1 :
CH4 and CH2F2 are having regular tetrahedron geometry.
[3]
Statement-2 :
Both are having same hybridization.
(A) Statement-1 is true, statement-2 is true and statement-2 is correct explanation for statement-1.
(B) Statement-1 is true, statement-2 is true and statement-2 is NOT the correct explanation for statement-1.
(C) Statement-1 is true, statement-2 is false.
(D*) Statement-1 is false, statement-2 is true.
Paragraph for question nos. 10 to 12
Hybridisation is the mixing of atomic orbital of comparable energy and the number of hybrid orbitals
formed is equal to the number of pure atomic orbitals mixed up and hybrid orbitals are occupied by bond pair and lone pair.
[9]
3
Which of the following geometry is most likely to not form from sp d hybridisation of the central atom.
(A) Linear
(B*) Tetrahedral
(C) T-Shaped
(D) See-Saw
The orbital is not participated in sp3d2 hybridisation.
(A) px

Q.12

(C) XeF5

Which of following pair of species is having different hybridisation but same shape.
(A) BeCl2 and CO2

Q.9

(B) XeF4

[3]

(B*) dxy

(C) d x 2 y 2

(D) pz

"The hybrid orbitals are at angle of X to one another" this statement is not valid for which of the
following hybridisation.
(A) sp3
(B) sp2
(C*) sp3d2
(D) sp

ASSIGNMENT-6
One or more than one correct:
Q.1
Select the correct statement for non polar molecule.
[3]
(A*) On time average the molecule is non-polar but at the particular moment it act as a dipole which is
equally probale in all directions.
(B) On time average the molecule is polar but at the particular moment it does not act as a dipole.
(C) On time average the molecule is non-polar and the particular moment it must not act as dipole.
(D) All are incorrect
Q.2

Select the correct order of B.P.


(A) BF3 > BMe3
(B*) BF3 < BMe3

[3]
(C) BF3 = BMe3

(D) None of these

Q.3

Which molecular geometry are most likely to result, from a octahedral electron geometry?
(A*) square planar
(B*) square pyramidal (C) linear
(D) Vshaped

[3]

Q.4

The correct order of hybridization of the central atom in the following species
NH3, XeO2F2, SeF4, NO2+

[3]

Q.5

(A) sp3, sp3, sp3d, sp

(B*) sp3, sp3d, sp3d, sp

(C) sp3, sp3d2, sp3d, sp2

(D) sp2, sp3d, sp3d2, sp

Halogens form compounds among themselves with the formula XX', XX & XX '7 (where X is the heavier
halogen) which of the following pair(s) represent(s) correct geometry with polar and non-polar nature
(theoretically)
[3]
(A*) XX' Linear Polar
(B) XX Linear Polar
(C) XX' Linear Non-polar

(D*) XX '7 Pentagonal bipyramidal Non-polar

Q.6

In molecules of the type AX2Ln (where L represents lone pairs and n is its number) there exists a bond
between element A and X. The X A X bond angle.
[3]
(A) Always decreases if n increases
(B) Always increases if n increases
(C*) Will be maximum for n = 3, 0
(D) Generally decreases if n decreases

Q.7

Which of the following pairs of species have identical shapes?


(A)

Q.8

Q.9

Q.10

NO 2

and

NO 2

(B) PCl5 and BrF5

(C*) XeF4 and

[3]
ICl4

(D) TeCl4 and XeO4

London force works in


[3]
(A) Polar molecule
(B) Non-polar molecule
(C*) All polar and non-polar molecule
(D) Ionic compounds
Choose the correct on the ClO bond length in NaClO4.
[3]
(A) All ClO bonds are of equal length.
(B) Three ClO bonds are of equal of length one longer.
(C) Two ClO bonds are of same length which are longer compound to other two ClO bond length.
(D) All are different.
Statement-1 :
Experimentally 100 % covalent bond formation is not possible
[3]
Statement-2 :
Non polar molecule has instantaneous dipole induced dipole interaction
(A*) Statement-1 is true, statement-2 is true and statement-2 is correct explanation for statement-1.

(B) Statement-1 is true, statement-2 is true and statement-2 is NOT the correct explanation for statement-1.
(C) Statement-1 is true, statement-2 is false.
(D) Statement-1 is false, statement-2 is true.
Q.11

Q.12

Spin pairing and overlapping theory can not explain the equal bond length as well as equal bond angles
in CH4 molecule. To explain the above facts we are in need of hybridization theory. Hybridization is the
mixing of atomic orbitals of comparable energy and the number of atomic orbitals involved is equal to the
number of hybrid orbitals formed of equal energy.
[3 3 = 9]
(i)
According to hybridization theory, the %s character in sp3d hybrid orbitals is
(A) 25%
(B) 33.33%
(C*) 20%
(D) 16.66%
(ii)

The number of axial and equitorial positions in octahedral geometry having sp3d2 hybridization
(A) 2, 4
(B) 4, 2
(C) 3, 3
(D*) 0, 0

(iii)

Increasing order of the energy of hybrid orbitals is


(A) sp3 < sp2 < sp
(B) sp2 < sp3 < sp
(C) sp < sp3 < sp2

Match the column :


Column I

(D*) sp < sp2 < sp3


[4]

Column II

(i)

IOF4

(P)

See-saw

(ii)

IO2 F2

(Q)

Trigonal bipyramidal

(iii)

XeO64

(R)

Linear

(iv)

XeF2

(S)

Square bipyramidal

(A) (i) P, (ii) S, (iii) R, (iv) Q


(C) (i) Q, (ii) P, (iii) S, (iv) R

(B*) (i) Q, (ii) P, (iii) S, (iv) R


(D) (i) Q, (ii) P, (iii) R, (iv) S

ASSIGNMENT-7
Q.1

Q.2

Q.3

Q.4

The correct order of d CH in the following option is


[3]
(A) CHF3 = CH2F2 = CH3F
(B) CHF3 > CH2F2 > CH3F
(C) CH2F2 > CH3F > CHF3
(D*) CH3F > CH2F2 > CHF3
O2F2 is an unstable yellow orange solid and H2O2 is a colourless liquid, both have OO bond.
OO bond length in H2O2 & O2F2 is respectively.
[3]
(A) 1.22, 1.48
(B*) 1.48, 1.22
(C) 1.22, 1.22
(D) 1.48, 1.48
The structure of O3 and N 3 are
(A) both linear
(B) Linear and bent respectively.
(C) both bent
(D*) Bent and linear respectively.
Select the correct order of following property.
(A) % s-character
:
sp3 > sp2 > sp

[3]

[3]

Q.5

^ bond angle
(B) ONO
:
NO3 > NO 2 > NO2
(C*) Second ionisation energy :
O>F>N>C
(D) C F bond length
:
CF4 > CH3F > CH2F2 > CF3H
Which of the following compounds is/are polar as well as non planar?

[3]
Cl

(A*) S2Cl2
Q.6
Q.7
Q.8

Q.9

Q.10

Q.11
Q.12

(B) B2H6

(C*) PCl2F3

The strongest PO bond is found in the molecule


(A*) F3PO
(B) Cl3PO
(C) Br3PO
F-As-F bond angle in AsF3Cl2 can be
(A*) 90 & 180 only (B) 120 only
(C) 90 & 120 only
The number of SS bonds in sulphur trioxide trimer (S3O9) is
(A) three
(B) two
(C) one

(D)

Cl

C=C=C=C

H
[3]

(D) (CH3)3PO
[3]
(D) 90 only
[3]
(D*) zero

Select the incorrect statement(s) about N2F4 and N2H4.


[3]
(A*) In N2F4 , d-orbitals are contracted by electronegative fluorine atoms, but d-orbital contraction is
not possible by H-atom in N2H4.
(B) The NN bond energy in N2F4 is more than NN bond energy in N2H4.
(C*) The NN bond length in N2F4 is more than that of in N2H4.
(D) The NN bond length in N2F4 is less than that of in N2H4.
Nodal planes of bond(s) in CH2=C=C=CH2 are located in
[3]
(A) All are in molecular plane
(B*) Two in molecular plane and one in a plane perpendicular to molecular plane which contains CC
-bond
(C) One in molecular plane and two in plane perpendicular to molecular plane which contains CC
-bond
(D) Two in molecular plane and one in a plane perpendicular to molecular plane which bisects CC
-bond at right angle
Which of the following shape can not be obtained from sp3d2 hybridisation.
[3]
(A) Square planar
(B) Square pyramidal (C*) Tetrahedral
(D) Octahedral
Which of the following statement(s) is/are correct about P4O6 and P4O10
[3]
(A*) Both oxides have closed cage like structure
(B*) Each oxide contains six equivalent POP bonds
(C) Both P4O6 and P4O10 molecules have pd bonds.
(D*) Both are the anhydrides of their respective acids.

ASSIGNMENT-4

Q.1

Find the species / molecule is having maximum number of lone pair on the central atom.
(A) ClOF4
(B) ClOF2+
(C) BH4
(D*) XeOF2

Q.2

If y-axis is the approaching axis between two atoms, then which of the set of orbitals can not form the
bond between two atoms in general.
[3]
(A) pz pz
(B) px px
(C*) px py
(D) None of these

Q.3

The maximum number of bond and -bond can be formed between two atoms are respectively.[3]
(A) 4, 3
(B*) 3, 2
(C) 2, 3
(D) 3, 1

Q.4

Which of the following set of overlap can not provide -bond formation.
(A) 3d and 2p
(B) 2p and 3p
(C) 2p and 2p
(D*) 3p and 1s

[3]

Q.5

The ratio of number of -bond to -bond in N2 and CO molecules are

[3]

(A) 2.0 , 2.0


Q.6

Q.7

Q.8

Q.9

Q.10

(B) 2,

1
2

(C*)

1 1
,
2 2

(D)

[3]

1
,2
2

Which of the following overlapping is involved in formation of only -bond.


(A*) s p overlapping
(B) p d overlapping
(C*) s s overlapping
(D) p p overlapping

[5]

Paragraph for question nos. 7 to 9


Different types of bonds are formed in the chemical compounds. These bond have different strength and
bond energies associated with them. These bonds are formed with atoms in different environments.
Which of the following bond has highest bond energy?
[9]
(A*) -bond
(B) -bond
(C) Hydrogen bond (D) Metallic bond
Shape of the molecule is decided by
(A*) -bond
(C) both and -bond

(B) -bond
(D) Never Nor -bond

Which of the following hydrides is thermally least stable?


(A) H2O
(B*) H2Te
(C) H2S
Match the column :
Column I
(Orders)
(A)
Increasing order of ionic
mobility in water
(B)
Decreasing order of size
(C)
Decreasing order of ionisation
energy

(D) H2Se
[4]

(P)

Column II
(Sequences)
Si, Mg, Al, Na

(Q)
(R)

Li+, Na+, K+, Rb+, Cs+


O, O+, O2, O

(S)

Se, S, O, F
[Ans. (A) Q, (B) S, (C) P]

ASSIGNMENT-5

Q.1

Explain type of hybridisation, shape, bond angle and geometry of the following compounds.
[18]
(1)
XeF2
(11) IF7
(2)

XeF4

(12)

OF2

(3)

XeF5

(13)

NO 3

(4)

XeOF4

(14)

ClO 4

(15)

SF4

(16)

I3

(17)

ClO 3

(18)

OCl2

(5)
(6)
(7)

PCl3
PCl5
SF2

(8)

SF6

(9)

IF3

(10)

IF5

Q.2

What is hybridisation of central atom of anionic part of PBr5 in crystalline state.


[3]
2
3
(A) sp
(B) sp
(C) sp
(D*) not applicable

Q.3

What is the difference between bond angles in cationic species of PCl5 and PBr5 in solid state. [3]
(A) 60
(B) 10928
(C*) 0
(D) 90

Q.4

All possible bond angles in anionic part of PCl5 are.


(A) 109 28 only
(B*) 90, 180
(C) 90, 120, 180

[3]
(D) 72, 90, 180

Q.5

The hybridisation and shape of XeO3F2 molecule is


(A) sp3 and tetrahedral
(B) sp3d and see-saw
(C*) sp3d and TBP
(D) sp3d2 and octahedral

Q.6

TeF5, XeF2 , I3+ , XeF4 , PCl3


Which of the following shape does not describe to any of the above species ?
(A) Square pyramidal
(B) Square planar
(C*) Trigonal planar
(D) Linear

[3]

[3]

Q.7

Which of the following species does not exist?


(A*) XeF3

Q.8

Q.10

Q.11

(D) XeF6

(B) CO2 and SO2

(C*) SO2 and I3

[3]

(D*) ICl 2 and BeH2

Statement-1 :
CH4 and CH2F2 are having regular tetrahedron geometry.
[3]
Statement-2 :
Both are having same hybridization.
(A) Statement-1 is true, statement-2 is true and statement-2 is correct explanation for statement-1.
(B) Statement-1 is true, statement-2 is true and statement-2 is NOT the correct explanation for statement-1.
(C) Statement-1 is true, statement-2 is false.
(D*) Statement-1 is false, statement-2 is true.
Paragraph for question nos. 10 to 12
Hybridisation is the mixing of atomic orbital of comparable energy and the number of hybrid orbitals
formed is equal to the number of pure atomic orbitals mixed up and hybrid orbitals are occupied by bond pair and lone pair.
[9]
3
Which of the following geometry is most likely to not form from sp d hybridisation of the central atom.
(A) Linear
(B*) Tetrahedral
(C) T-Shaped
(D) See-Saw
The orbital is not participated in sp3d2 hybridisation.
(A) px

Q.12

(C) XeF5

Which of following pair of species is having different hybridisation but same shape.
(A) BeCl2 and CO2

Q.9

(B) XeF4

[3]

(B*) dxy

(C) d x 2 y 2

(D) pz

"The hybrid orbitals are at angle of X to one another" this statement is not valid for which of the
following hybridisation.
(A) sp3
(B) sp2
(C*) sp3d2
(D) sp

ASSIGNMENT-6
One or more than one correct:
Q.1
Select the correct statement for non polar molecule.
[3]
(A*) On time average the molecule is non-polar but at the particular moment it act as a dipole which is
equally probale in all directions.
(B) On time average the molecule is polar but at the particular moment it does not act as a dipole.
(C) On time average the molecule is non-polar and the particular moment it must not act as dipole.
(D) All are incorrect
Q.2

Select the correct order of B.P.


(A) BF3 > BMe3
(B*) BF3 < BMe3

[3]
(C) BF3 = BMe3

(D) None of these

Q.3

Which molecular geometry are most likely to result, from a octahedral electron geometry?
(A*) square planar
(B*) square pyramidal (C) linear
(D) Vshaped

[3]

Q.4

The correct order of hybridization of the central atom in the following species
NH3, XeO2F2, SeF4, NO2+

[3]

Q.5

(A) sp3, sp3, sp3d, sp

(B*) sp3, sp3d, sp3d, sp

(C) sp3, sp3d2, sp3d, sp2

(D) sp2, sp3d, sp3d2, sp

Halogens form compounds among themselves with the formula XX', XX & XX '7 (where X is the heavier
halogen) which of the following pair(s) represent(s) correct geometry with polar and non-polar nature
(theoretically)
[3]
(A*) XX' Linear Polar
(B) XX Linear Polar
(C) XX' Linear Non-polar

(D*) XX '7 Pentagonal bipyramidal Non-polar

Q.6

In molecules of the type AX2Ln (where L represents lone pairs and n is its number) there exists a bond
between element A and X. The X A X bond angle.
[3]
(A) Always decreases if n increases
(B) Always increases if n increases
(C*) Will be maximum for n = 3, 0
(D) Generally decreases if n decreases

Q.7

Which of the following pairs of species have identical shapes?


(A)

Q.8

Q.9

Q.10

NO 2

and

NO 2

(B) PCl5 and BrF5

(C*) XeF4 and

[3]
ICl4

(D) TeCl4 and XeO4

London force works in


[3]
(A) Polar molecule
(B) Non-polar molecule
(C*) All polar and non-polar molecule
(D) Ionic compounds
Choose the correct on the ClO bond length in NaClO4.
[3]
(A) All ClO bonds are of equal length.
(B) Three ClO bonds are of equal of length one longer.
(C) Two ClO bonds are of same length which are longer compound to other two ClO bond length.
(D) All are different.
Statement-1 :
Experimentally 100 % covalent bond formation is not possible
[3]
Statement-2 :
Non polar molecule has instantaneous dipole induced dipole interaction
(A*) Statement-1 is true, statement-2 is true and statement-2 is correct explanation for statement-1.

(B) Statement-1 is true, statement-2 is true and statement-2 is NOT the correct explanation for statement-1.
(C) Statement-1 is true, statement-2 is false.
(D) Statement-1 is false, statement-2 is true.
Q.11

Q.12

Spin pairing and overlapping theory can not explain the equal bond length as well as equal bond angles
in CH4 molecule. To explain the above facts we are in need of hybridization theory. Hybridization is the
mixing of atomic orbitals of comparable energy and the number of atomic orbitals involved is equal to the
number of hybrid orbitals formed of equal energy.
[3 3 = 9]
(i)
According to hybridization theory, the %s character in sp3d hybrid orbitals is
(A) 25%
(B) 33.33%
(C*) 20%
(D) 16.66%
(ii)

The number of axial and equitorial positions in octahedral geometry having sp3d2 hybridization
(A) 2, 4
(B) 4, 2
(C) 3, 3
(D*) 0, 0

(iii)

Increasing order of the energy of hybrid orbitals is


(A) sp3 < sp2 < sp
(B) sp2 < sp3 < sp
(C) sp < sp3 < sp2

Match the column :


Column I

(D*) sp < sp2 < sp3


[4]

Column II

(i)

IOF4

(P)

See-saw

(ii)

IO2 F2

(Q)

Trigonal bipyramidal

(iii)

XeO64

(R)

Linear

(iv)

XeF2

(S)

Square bipyramidal

(A) (i) P, (ii) S, (iii) R, (iv) Q


(C) (i) Q, (ii) P, (iii) S, (iv) R

(B*) (i) Q, (ii) P, (iii) S, (iv) R


(D) (i) Q, (ii) P, (iii) R, (iv) S

ASSIGNMENT-7
Q.1

Q.2

Q.3

Q.4

The correct order of d CH in the following option is


[3]
(A) CHF3 = CH2F2 = CH3F
(B) CHF3 > CH2F2 > CH3F
(C) CH2F2 > CH3F > CHF3
(D*) CH3F > CH2F2 > CHF3
O2F2 is an unstable yellow orange solid and H2O2 is a colourless liquid, both have OO bond.
OO bond length in H2O2 & O2F2 is respectively.
[3]
(A) 1.22, 1.48
(B*) 1.48, 1.22
(C) 1.22, 1.22
(D) 1.48, 1.48
The structure of O3 and N 3 are
(A) both linear
(B) Linear and bent respectively.
(C) both bent
(D*) Bent and linear respectively.
Select the correct order of following property.
(A) % s-character
:
sp3 > sp2 > sp

[3]

[3]

Q.5

^ bond angle
(B) ONO
:
NO3 > NO 2 > NO2
(C*) Second ionisation energy :
O>F>N>C
(D) C F bond length
:
CF4 > CH3F > CH2F2 > CF3H
Which of the following compounds is/are polar as well as non planar?

[3]
Cl

(A*) S2Cl2
Q.6
Q.7
Q.8

Q.9

Q.10

Q.11
Q.12

(B) B2H6

(C*) PCl2F3

The strongest PO bond is found in the molecule


(A*) F3PO
(B) Cl3PO
(C) Br3PO
F-As-F bond angle in AsF3Cl2 can be
(A*) 90 & 180 only (B) 120 only
(C) 90 & 120 only
The number of SS bonds in sulphur trioxide trimer (S3O9) is
(A) three
(B) two
(C) one

(D)

Cl

C=C=C=C

H
[3]

(D) (CH3)3PO
[3]
(D) 90 only
[3]
(D*) zero

Select the incorrect statement(s) about N2F4 and N2H4.


[3]
(A*) In N2F4 , d-orbitals are contracted by electronegative fluorine atoms, but d-orbital contraction is
not possible by H-atom in N2H4.
(B) The NN bond energy in N2F4 is more than NN bond energy in N2H4.
(C*) The NN bond length in N2F4 is more than that of in N2H4.
(D) The NN bond length in N2F4 is less than that of in N2H4.
Nodal planes of bond(s) in CH2=C=C=CH2 are located in
[3]
(A) All are in molecular plane
(B*) Two in molecular plane and one in a plane perpendicular to molecular plane which contains CC
-bond
(C) One in molecular plane and two in plane perpendicular to molecular plane which contains CC
-bond
(D) Two in molecular plane and one in a plane perpendicular to molecular plane which bisects CC
-bond at right angle
Which of the following shape can not be obtained from sp3d2 hybridisation.
[3]
(A) Square planar
(B) Square pyramidal (C*) Tetrahedral
(D) Octahedral
Which of the following statement(s) is/are correct about P4O6 and P4O10
[3]
(A*) Both oxides have closed cage like structure
(B*) Each oxide contains six equivalent POP bonds
(C) Both P4O6 and P4O10 molecules have pd bonds.
(D*) Both are the anhydrides of their respective acids.

ASSIGNMENT-4

Q.1

Find the species / molecule is having maximum number of lone pair on the central atom.
(A) ClOF4
(B) ClOF2+
(C) BH4
(D*) XeOF2

Q.2

If y-axis is the approaching axis between two atoms, then which of the set of orbitals can not form the
bond between two atoms in general.
[3]
(A) pz pz
(B) px px
(C*) px py
(D) None of these

Q.3

The maximum number of bond and -bond can be formed between two atoms are respectively.[3]
(A) 4, 3
(B*) 3, 2
(C) 2, 3
(D) 3, 1

Q.4

Which of the following set of overlap can not provide -bond formation.
(A) 3d and 2p
(B) 2p and 3p
(C) 2p and 2p
(D*) 3p and 1s

[3]

Q.5

The ratio of number of -bond to -bond in N2 and CO molecules are

[3]

(A) 2.0 , 2.0


Q.6

Q.7

Q.8

Q.9

Q.10

(B) 2,

1
2

(C*)

1 1
,
2 2

(D)

[3]

1
,2
2

Which of the following overlapping is involved in formation of only -bond.


(A*) s p overlapping
(B) p d overlapping
(C*) s s overlapping
(D) p p overlapping

[5]

Paragraph for question nos. 7 to 9


Different types of bonds are formed in the chemical compounds. These bond have different strength and
bond energies associated with them. These bonds are formed with atoms in different environments.
Which of the following bond has highest bond energy?
[9]
(A*) -bond
(B) -bond
(C) Hydrogen bond (D) Metallic bond
Shape of the molecule is decided by
(A*) -bond
(C) both and -bond

(B) -bond
(D) Never Nor -bond

Which of the following hydrides is thermally least stable?


(A) H2O
(B*) H2Te
(C) H2S
Match the column :
Column I
(Orders)
(A)
Increasing order of ionic
mobility in water
(B)
Decreasing order of size
(C)
Decreasing order of ionisation
energy

(D) H2Se
[4]

(P)

Column II
(Sequences)
Si, Mg, Al, Na

(Q)
(R)

Li+, Na+, K+, Rb+, Cs+


O, O+, O2, O

(S)

Se, S, O, F
[Ans. (A) Q, (B) S, (C) P]

ASSIGNMENT-5

Q.1

Explain type of hybridisation, shape, bond angle and geometry of the following compounds.
[18]
(1)
XeF2
(11) IF7
(2)

XeF4

(12)

OF2

(3)

XeF5

(13)

NO 3

(4)

XeOF4

(14)

ClO 4

(15)

SF4

(16)

I3

(17)

ClO 3

(18)

OCl2

(5)
(6)
(7)

PCl3
PCl5
SF2

(8)

SF6

(9)

IF3

(10)

IF5

Q.2

What is hybridisation of central atom of anionic part of PBr5 in crystalline state.


[3]
2
3
(A) sp
(B) sp
(C) sp
(D*) not applicable

Q.3

What is the difference between bond angles in cationic species of PCl5 and PBr5 in solid state. [3]
(A) 60
(B) 10928
(C*) 0
(D) 90

Q.4

All possible bond angles in anionic part of PCl5 are.


(A) 109 28 only
(B*) 90, 180
(C) 90, 120, 180

[3]
(D) 72, 90, 180

Q.5

The hybridisation and shape of XeO3F2 molecule is


(A) sp3 and tetrahedral
(B) sp3d and see-saw
(C*) sp3d and TBP
(D) sp3d2 and octahedral

Q.6

TeF5, XeF2 , I3+ , XeF4 , PCl3


Which of the following shape does not describe to any of the above species ?
(A) Square pyramidal
(B) Square planar
(C*) Trigonal planar
(D) Linear

[3]

[3]

Q.7

Which of the following species does not exist?


(A*) XeF3

Q.8

Q.10

Q.11

(D) XeF6

(B) CO2 and SO2

(C*) SO2 and I3

[3]

(D*) ICl 2 and BeH2

Statement-1 :
CH4 and CH2F2 are having regular tetrahedron geometry.
[3]
Statement-2 :
Both are having same hybridization.
(A) Statement-1 is true, statement-2 is true and statement-2 is correct explanation for statement-1.
(B) Statement-1 is true, statement-2 is true and statement-2 is NOT the correct explanation for statement-1.
(C) Statement-1 is true, statement-2 is false.
(D*) Statement-1 is false, statement-2 is true.
Paragraph for question nos. 10 to 12
Hybridisation is the mixing of atomic orbital of comparable energy and the number of hybrid orbitals
formed is equal to the number of pure atomic orbitals mixed up and hybrid orbitals are occupied by bond pair and lone pair.
[9]
3
Which of the following geometry is most likely to not form from sp d hybridisation of the central atom.
(A) Linear
(B*) Tetrahedral
(C) T-Shaped
(D) See-Saw
The orbital is not participated in sp3d2 hybridisation.
(A) px

Q.12

(C) XeF5

Which of following pair of species is having different hybridisation but same shape.
(A) BeCl2 and CO2

Q.9

(B) XeF4

[3]

(B*) dxy

(C) d x 2 y 2

(D) pz

"The hybrid orbitals are at angle of X to one another" this statement is not valid for which of the
following hybridisation.
(A) sp3
(B) sp2
(C*) sp3d2
(D) sp

ASSIGNMENT-6
One or more than one correct:
Q.1
Select the correct statement for non polar molecule.
[3]
(A*) On time average the molecule is non-polar but at the particular moment it act as a dipole which is
equally probale in all directions.
(B) On time average the molecule is polar but at the particular moment it does not act as a dipole.
(C) On time average the molecule is non-polar and the particular moment it must not act as dipole.
(D) All are incorrect
Q.2

Select the correct order of B.P.


(A) BF3 > BMe3
(B*) BF3 < BMe3

[3]
(C) BF3 = BMe3

(D) None of these

Q.3

Which molecular geometry are most likely to result, from a octahedral electron geometry?
(A*) square planar
(B*) square pyramidal (C) linear
(D) Vshaped

[3]

Q.4

The correct order of hybridization of the central atom in the following species
NH3, XeO2F2, SeF4, NO2+

[3]

Q.5

(A) sp3, sp3, sp3d, sp

(B*) sp3, sp3d, sp3d, sp

(C) sp3, sp3d2, sp3d, sp2

(D) sp2, sp3d, sp3d2, sp

Halogens form compounds among themselves with the formula XX', XX & XX '7 (where X is the heavier
halogen) which of the following pair(s) represent(s) correct geometry with polar and non-polar nature
(theoretically)
[3]
(A*) XX' Linear Polar
(B) XX Linear Polar
(C) XX' Linear Non-polar

(D*) XX '7 Pentagonal bipyramidal Non-polar

Q.6

In molecules of the type AX2Ln (where L represents lone pairs and n is its number) there exists a bond
between element A and X. The X A X bond angle.
[3]
(A) Always decreases if n increases
(B) Always increases if n increases
(C*) Will be maximum for n = 3, 0
(D) Generally decreases if n decreases

Q.7

Which of the following pairs of species have identical shapes?


(A)

Q.8

Q.9

Q.10

NO 2

and

NO 2

(B) PCl5 and BrF5

(C*) XeF4 and

[3]
ICl4

(D) TeCl4 and XeO4

London force works in


[3]
(A) Polar molecule
(B) Non-polar molecule
(C*) All polar and non-polar molecule
(D) Ionic compounds
Choose the correct on the ClO bond length in NaClO4.
[3]
(A) All ClO bonds are of equal length.
(B) Three ClO bonds are of equal of length one longer.
(C) Two ClO bonds are of same length which are longer compound to other two ClO bond length.
(D) All are different.
Statement-1 :
Experimentally 100 % covalent bond formation is not possible
[3]
Statement-2 :
Non polar molecule has instantaneous dipole induced dipole interaction
(A*) Statement-1 is true, statement-2 is true and statement-2 is correct explanation for statement-1.

(B) Statement-1 is true, statement-2 is true and statement-2 is NOT the correct explanation for statement-1.
(C) Statement-1 is true, statement-2 is false.
(D) Statement-1 is false, statement-2 is true.
Q.11

Q.12

Spin pairing and overlapping theory can not explain the equal bond length as well as equal bond angles
in CH4 molecule. To explain the above facts we are in need of hybridization theory. Hybridization is the
mixing of atomic orbitals of comparable energy and the number of atomic orbitals involved is equal to the
number of hybrid orbitals formed of equal energy.
[3 3 = 9]
(i)
According to hybridization theory, the %s character in sp3d hybrid orbitals is
(A) 25%
(B) 33.33%
(C*) 20%
(D) 16.66%
(ii)

The number of axial and equitorial positions in octahedral geometry having sp3d2 hybridization
(A) 2, 4
(B) 4, 2
(C) 3, 3
(D*) 0, 0

(iii)

Increasing order of the energy of hybrid orbitals is


(A) sp3 < sp2 < sp
(B) sp2 < sp3 < sp
(C) sp < sp3 < sp2

Match the column :


Column I

(D*) sp < sp2 < sp3


[4]

Column II

(i)

IOF4

(P)

See-saw

(ii)

IO2 F2

(Q)

Trigonal bipyramidal

(iii)

XeO64

(R)

Linear

(iv)

XeF2

(S)

Square bipyramidal

(A) (i) P, (ii) S, (iii) R, (iv) Q


(C) (i) Q, (ii) P, (iii) S, (iv) R

(B*) (i) Q, (ii) P, (iii) S, (iv) R


(D) (i) Q, (ii) P, (iii) R, (iv) S

ASSIGNMENT-7
Q.1

Q.2

Q.3

Q.4

The correct order of d CH in the following option is


[3]
(A) CHF3 = CH2F2 = CH3F
(B) CHF3 > CH2F2 > CH3F
(C) CH2F2 > CH3F > CHF3
(D*) CH3F > CH2F2 > CHF3
O2F2 is an unstable yellow orange solid and H2O2 is a colourless liquid, both have OO bond.
OO bond length in H2O2 & O2F2 is respectively.
[3]
(A) 1.22, 1.48
(B*) 1.48, 1.22
(C) 1.22, 1.22
(D) 1.48, 1.48
The structure of O3 and N 3 are
(A) both linear
(B) Linear and bent respectively.
(C) both bent
(D*) Bent and linear respectively.
Select the correct order of following property.
(A) % s-character
:
sp3 > sp2 > sp

[3]

[3]

Q.5

^ bond angle
(B) ONO
:
NO3 > NO 2 > NO2
(C*) Second ionisation energy :
O>F>N>C
(D) C F bond length
:
CF4 > CH3F > CH2F2 > CF3H
Which of the following compounds is/are polar as well as non planar?

[3]
Cl

(A*) S2Cl2
Q.6
Q.7
Q.8

Q.9

Q.10

Q.11
Q.12

(B) B2H6

(C*) PCl2F3

The strongest PO bond is found in the molecule


(A*) F3PO
(B) Cl3PO
(C) Br3PO
F-As-F bond angle in AsF3Cl2 can be
(A*) 90 & 180 only (B) 120 only
(C) 90 & 120 only
The number of SS bonds in sulphur trioxide trimer (S3O9) is
(A) three
(B) two
(C) one

(D)

Cl

C=C=C=C

H
[3]

(D) (CH3)3PO
[3]
(D) 90 only
[3]
(D*) zero

Select the incorrect statement(s) about N2F4 and N2H4.


[3]
(A*) In N2F4 , d-orbitals are contracted by electronegative fluorine atoms, but d-orbital contraction is
not possible by H-atom in N2H4.
(B) The NN bond energy in N2F4 is more than NN bond energy in N2H4.
(C*) The NN bond length in N2F4 is more than that of in N2H4.
(D) The NN bond length in N2F4 is less than that of in N2H4.
Nodal planes of bond(s) in CH2=C=C=CH2 are located in
[3]
(A) All are in molecular plane
(B*) Two in molecular plane and one in a plane perpendicular to molecular plane which contains CC
-bond
(C) One in molecular plane and two in plane perpendicular to molecular plane which contains CC
-bond
(D) Two in molecular plane and one in a plane perpendicular to molecular plane which bisects CC
-bond at right angle
Which of the following shape can not be obtained from sp3d2 hybridisation.
[3]
(A) Square planar
(B) Square pyramidal (C*) Tetrahedral
(D) Octahedral
Which of the following statement(s) is/are correct about P4O6 and P4O10
[3]
(A*) Both oxides have closed cage like structure
(B*) Each oxide contains six equivalent POP bonds
(C) Both P4O6 and P4O10 molecules have pd bonds.
(D*) Both are the anhydrides of their respective acids.

ASSIGNMENT-4

Q.1

Find the species / molecule is having maximum number of lone pair on the central atom.
(A) ClOF4
(B) ClOF2+
(C) BH4
(D*) XeOF2

Q.2

If y-axis is the approaching axis between two atoms, then which of the set of orbitals can not form the
bond between two atoms in general.
[3]
(A) pz pz
(B) px px
(C*) px py
(D) None of these

Q.3

The maximum number of bond and -bond can be formed between two atoms are respectively.[3]
(A) 4, 3
(B*) 3, 2
(C) 2, 3
(D) 3, 1

Q.4

Which of the following set of overlap can not provide -bond formation.
(A) 3d and 2p
(B) 2p and 3p
(C) 2p and 2p
(D*) 3p and 1s

[3]

Q.5

The ratio of number of -bond to -bond in N2 and CO molecules are

[3]

(A) 2.0 , 2.0


Q.6

Q.7

Q.8

Q.9

Q.10

(B) 2,

1
2

(C*)

1 1
,
2 2

(D)

[3]

1
,2
2

Which of the following overlapping is involved in formation of only -bond.


(A*) s p overlapping
(B) p d overlapping
(C*) s s overlapping
(D) p p overlapping

[5]

Paragraph for question nos. 7 to 9


Different types of bonds are formed in the chemical compounds. These bond have different strength and
bond energies associated with them. These bonds are formed with atoms in different environments.
Which of the following bond has highest bond energy?
[9]
(A*) -bond
(B) -bond
(C) Hydrogen bond (D) Metallic bond
Shape of the molecule is decided by
(A*) -bond
(C) both and -bond

(B) -bond
(D) Never Nor -bond

Which of the following hydrides is thermally least stable?


(A) H2O
(B*) H2Te
(C) H2S
Match the column :
Column I
(Orders)
(A)
Increasing order of ionic
mobility in water
(B)
Decreasing order of size
(C)
Decreasing order of ionisation
energy

(D) H2Se
[4]

(P)

Column II
(Sequences)
Si, Mg, Al, Na

(Q)
(R)

Li+, Na+, K+, Rb+, Cs+


O, O+, O2, O

(S)

Se, S, O, F
[Ans. (A) Q, (B) S, (C) P]

ASSIGNMENT-5

Q.1

Explain type of hybridisation, shape, bond angle and geometry of the following compounds.
[18]
(1)
XeF2
(11) IF7
(2)

XeF4

(12)

OF2

(3)

XeF5

(13)

NO 3

(4)

XeOF4

(14)

ClO 4

(15)

SF4

(16)

I3

(17)

ClO 3

(18)

OCl2

(5)
(6)
(7)

PCl3
PCl5
SF2

(8)

SF6

(9)

IF3

(10)

IF5

Q.2

What is hybridisation of central atom of anionic part of PBr5 in crystalline state.


[3]
2
3
(A) sp
(B) sp
(C) sp
(D*) not applicable

Q.3

What is the difference between bond angles in cationic species of PCl5 and PBr5 in solid state. [3]
(A) 60
(B) 10928
(C*) 0
(D) 90

Q.4

All possible bond angles in anionic part of PCl5 are.


(A) 109 28 only
(B*) 90, 180
(C) 90, 120, 180

[3]
(D) 72, 90, 180

Q.5

The hybridisation and shape of XeO3F2 molecule is


(A) sp3 and tetrahedral
(B) sp3d and see-saw
(C*) sp3d and TBP
(D) sp3d2 and octahedral

Q.6

TeF5, XeF2 , I3+ , XeF4 , PCl3


Which of the following shape does not describe to any of the above species ?
(A) Square pyramidal
(B) Square planar
(C*) Trigonal planar
(D) Linear

[3]

[3]

Q.7

Which of the following species does not exist?


(A*) XeF3

Q.8

Q.10

Q.11

(D) XeF6

(B) CO2 and SO2

(C*) SO2 and I3

[3]

(D*) ICl 2 and BeH2

Statement-1 :
CH4 and CH2F2 are having regular tetrahedron geometry.
[3]
Statement-2 :
Both are having same hybridization.
(A) Statement-1 is true, statement-2 is true and statement-2 is correct explanation for statement-1.
(B) Statement-1 is true, statement-2 is true and statement-2 is NOT the correct explanation for statement-1.
(C) Statement-1 is true, statement-2 is false.
(D*) Statement-1 is false, statement-2 is true.
Paragraph for question nos. 10 to 12
Hybridisation is the mixing of atomic orbital of comparable energy and the number of hybrid orbitals
formed is equal to the number of pure atomic orbitals mixed up and hybrid orbitals are occupied by bond pair and lone pair.
[9]
3
Which of the following geometry is most likely to not form from sp d hybridisation of the central atom.
(A) Linear
(B*) Tetrahedral
(C) T-Shaped
(D) See-Saw
The orbital is not participated in sp3d2 hybridisation.
(A) px

Q.12

(C) XeF5

Which of following pair of species is having different hybridisation but same shape.
(A) BeCl2 and CO2

Q.9

(B) XeF4

[3]

(B*) dxy

(C) d x 2 y 2

(D) pz

"The hybrid orbitals are at angle of X to one another" this statement is not valid for which of the
following hybridisation.
(A) sp3
(B) sp2
(C*) sp3d2
(D) sp

ASSIGNMENT-6
One or more than one correct:
Q.1
Select the correct statement for non polar molecule.
[3]
(A*) On time average the molecule is non-polar but at the particular moment it act as a dipole which is
equally probale in all directions.
(B) On time average the molecule is polar but at the particular moment it does not act as a dipole.
(C) On time average the molecule is non-polar and the particular moment it must not act as dipole.
(D) All are incorrect
Q.2

Select the correct order of B.P.


(A) BF3 > BMe3
(B*) BF3 < BMe3

[3]
(C) BF3 = BMe3

(D) None of these

Q.3

Which molecular geometry are most likely to result, from a octahedral electron geometry?
(A*) square planar
(B*) square pyramidal (C) linear
(D) Vshaped

[3]

Q.4

The correct order of hybridization of the central atom in the following species
NH3, XeO2F2, SeF4, NO2+

[3]

Q.5

(A) sp3, sp3, sp3d, sp

(B*) sp3, sp3d, sp3d, sp

(C) sp3, sp3d2, sp3d, sp2

(D) sp2, sp3d, sp3d2, sp

Halogens form compounds among themselves with the formula XX', XX & XX '7 (where X is the heavier
halogen) which of the following pair(s) represent(s) correct geometry with polar and non-polar nature
(theoretically)
[3]
(A*) XX' Linear Polar
(B) XX Linear Polar
(C) XX' Linear Non-polar

(D*) XX '7 Pentagonal bipyramidal Non-polar

Q.6

In molecules of the type AX2Ln (where L represents lone pairs and n is its number) there exists a bond
between element A and X. The X A X bond angle.
[3]
(A) Always decreases if n increases
(B) Always increases if n increases
(C*) Will be maximum for n = 3, 0
(D) Generally decreases if n decreases

Q.7

Which of the following pairs of species have identical shapes?


(A)

Q.8

Q.9

Q.10

NO 2

and

NO 2

(B) PCl5 and BrF5

(C*) XeF4 and

[3]
ICl4

(D) TeCl4 and XeO4

London force works in


[3]
(A) Polar molecule
(B) Non-polar molecule
(C*) All polar and non-polar molecule
(D) Ionic compounds
Choose the correct on the ClO bond length in NaClO4.
[3]
(A) All ClO bonds are of equal length.
(B) Three ClO bonds are of equal of length one longer.
(C) Two ClO bonds are of same length which are longer compound to other two ClO bond length.
(D) All are different.
Statement-1 :
Experimentally 100 % covalent bond formation is not possible
[3]
Statement-2 :
Non polar molecule has instantaneous dipole induced dipole interaction
(A*) Statement-1 is true, statement-2 is true and statement-2 is correct explanation for statement-1.

(B) Statement-1 is true, statement-2 is true and statement-2 is NOT the correct explanation for statement-1.
(C) Statement-1 is true, statement-2 is false.
(D) Statement-1 is false, statement-2 is true.
Q.11

Q.12

Spin pairing and overlapping theory can not explain the equal bond length as well as equal bond angles
in CH4 molecule. To explain the above facts we are in need of hybridization theory. Hybridization is the
mixing of atomic orbitals of comparable energy and the number of atomic orbitals involved is equal to the
number of hybrid orbitals formed of equal energy.
[3 3 = 9]
(i)
According to hybridization theory, the %s character in sp3d hybrid orbitals is
(A) 25%
(B) 33.33%
(C*) 20%
(D) 16.66%
(ii)

The number of axial and equitorial positions in octahedral geometry having sp3d2 hybridization
(A) 2, 4
(B) 4, 2
(C) 3, 3
(D*) 0, 0

(iii)

Increasing order of the energy of hybrid orbitals is


(A) sp3 < sp2 < sp
(B) sp2 < sp3 < sp
(C) sp < sp3 < sp2

Match the column :


Column I

(D*) sp < sp2 < sp3


[4]

Column II

(i)

IOF4

(P)

See-saw

(ii)

IO2 F2

(Q)

Trigonal bipyramidal

(iii)

XeO64

(R)

Linear

(iv)

XeF2

(S)

Square bipyramidal

(A) (i) P, (ii) S, (iii) R, (iv) Q


(C) (i) Q, (ii) P, (iii) S, (iv) R

(B*) (i) Q, (ii) P, (iii) S, (iv) R


(D) (i) Q, (ii) P, (iii) R, (iv) S

ASSIGNMENT-7
Q.1

Q.2

Q.3

Q.4

The correct order of d CH in the following option is


[3]
(A) CHF3 = CH2F2 = CH3F
(B) CHF3 > CH2F2 > CH3F
(C) CH2F2 > CH3F > CHF3
(D*) CH3F > CH2F2 > CHF3
O2F2 is an unstable yellow orange solid and H2O2 is a colourless liquid, both have OO bond.
OO bond length in H2O2 & O2F2 is respectively.
[3]
(A) 1.22, 1.48
(B*) 1.48, 1.22
(C) 1.22, 1.22
(D) 1.48, 1.48
The structure of O3 and N 3 are
(A) both linear
(B) Linear and bent respectively.
(C) both bent
(D*) Bent and linear respectively.
Select the correct order of following property.
(A) % s-character
:
sp3 > sp2 > sp

[3]

[3]

Q.5

^ bond angle
(B) ONO
:
NO3 > NO 2 > NO2
(C*) Second ionisation energy :
O>F>N>C
(D) C F bond length
:
CF4 > CH3F > CH2F2 > CF3H
Which of the following compounds is/are polar as well as non planar?

[3]
Cl

(A*) S2Cl2
Q.6
Q.7
Q.8

Q.9

Q.10

Q.11
Q.12

(B) B2H6

(C*) PCl2F3

The strongest PO bond is found in the molecule


(A*) F3PO
(B) Cl3PO
(C) Br3PO
F-As-F bond angle in AsF3Cl2 can be
(A*) 90 & 180 only (B) 120 only
(C) 90 & 120 only
The number of SS bonds in sulphur trioxide trimer (S3O9) is
(A) three
(B) two
(C) one

(D)

Cl

C=C=C=C

H
[3]

(D) (CH3)3PO
[3]
(D) 90 only
[3]
(D*) zero

Select the incorrect statement(s) about N2F4 and N2H4.


[3]
(A*) In N2F4 , d-orbitals are contracted by electronegative fluorine atoms, but d-orbital contraction is
not possible by H-atom in N2H4.
(B) The NN bond energy in N2F4 is more than NN bond energy in N2H4.
(C*) The NN bond length in N2F4 is more than that of in N2H4.
(D) The NN bond length in N2F4 is less than that of in N2H4.
Nodal planes of bond(s) in CH2=C=C=CH2 are located in
[3]
(A) All are in molecular plane
(B*) Two in molecular plane and one in a plane perpendicular to molecular plane which contains CC
-bond
(C) One in molecular plane and two in plane perpendicular to molecular plane which contains CC
-bond
(D) Two in molecular plane and one in a plane perpendicular to molecular plane which bisects CC
-bond at right angle
Which of the following shape can not be obtained from sp3d2 hybridisation.
[3]
(A) Square planar
(B) Square pyramidal (C*) Tetrahedral
(D) Octahedral
Which of the following statement(s) is/are correct about P4O6 and P4O10
[3]
(A*) Both oxides have closed cage like structure
(B*) Each oxide contains six equivalent POP bonds
(C) Both P4O6 and P4O10 molecules have pd bonds.
(D*) Both are the anhydrides of their respective acids.

ASSIGNMENT10
Q.1

Column-I
(A)
(B)
(C)
(D)

Q.2

BH4
I2Cl6
AlCl4
BeCl2 (solid)

Column-II
(P)
(Q)
(R)
(S)

Choose the correct statements.


(A) CH3NCS molecule is linear
(C*) GeH3NCS molecule is bent

[12]

Central atom(s) of species/molecule is having sp3 hybridisation


The species / molecule is having no lone pair
All bond angles are identical
Species / molecule having co-ordinate bond.
[Ans. (A) P, Q, R, S, (B)S, (C) P, R, S, (D) P, S ]
[4]
(B*) SiH3NCS molecule is linear
(D*) P(SiH3)3 molecule is pyramidal

Q.3

Statement-1 :
SO42 and S52 have different structures.
[3]
Statement-2 :
O and S are of same group but of different period element.
(A) Statement-1 is true, statement-2 is true and statement-2 is correct explanation for statement-1.
(B*) Statement-1 is true, statement-2 is true and statement-2 is NOT the correct explanation for statement-1.
(C) Statement-1 is true, statement-2 is false.
(D) Statement-1 is false, statement-2 is true.

Q.4

Only one match is to be found out.


Column I
(A)
Dimer of BeCl2
(B)
Dimer of AlBr3
(C)

Q.5

Q.6

Q.7

XeF5

[9]
(P)
(Q)

Column II
All atoms are sp3 hybridised.
Non-planar and see-saw in shape

(R)

Central atoms are electron deficient

(S)

Bond angles are of 72


[Ans. (A) R, (B) P, (C) S]

In which of the following silicate structure, the number of corner shared is minimum.
(A*) Pyrosilicate
(B) Amphibole chain silicate
(C) 3D-silicate
(D) Six membered cyclic silicate

[3]

Paragraph for Question Nos. 6 to 8


Bridge bonding is a specific kind of bonding in pages of chemistry. In general -bond pair delocalisation is
very difficult. But electron deficiency of the central atom forces to delocalise and forms this kind of bond.
The B2H6 molecule is dissolved in tetrahydrofuran. Which atom(s) is/are having changes of hybridisation
with respect to reactant and final product of the process given.
[3]
(A) B only
(B) B and O
(C) B, O and C
(D*) None of these
In which of the dimerisation process, the achievement of the octet is not the driving force.
(A) 2AlCl3 Al2Cl6
(B) BeCl2 BeCl2 (solid)
(C*) 2ICl3 I2Cl6
(D) 2NO2 N2O4

[3]

Q.8

Q.9b

The molecule is not having 3c2e bond.


(A) BeH2 (dimer)
(B) BeH2 (solid)

[3]
(C*) C2H6

Only one match is to be found out.


Column I

(D) B2H6
[12]

Column II

(A)

ICl3

(P)

(B)
(C)
(D)

AlCl3
AlF3
NO 2

(Q)
(R)
(S)

Hybridisation of central atom is similar in both


dimer and monomer form.
Both Monomer and dimer forms are planar.
In dimer form all atoms are sp3 hybridised.
Does not exist in dimer form.
[Ans. (A) Q, (B) R, (C) S, (D) P ]

Comprehension
Silicate are existing mainly in the polymeric form. Several categories are available with us which depend
on the mode of sharing of corners of SiO44 tetrahedron.
Q.10

Which of the following pyroxene chain silicate is having same formula.


(A)

(B)

(C)

(D*) All of these

Q.11

Which of the following silicates are having same general formula.


(A) Sheet silicate and single chain silicate
(B) Single chain silicate and double chain silicate
(C*) 3-membered cyclic silicate and 5-membered cyclic silicate
(D) 2D-silicate and 3D-silicate.

Q.12

Which of the following silicates is having 2


(A*) Amphibole chain silicate
(C) Sheet silicate

1
o-atoms are shared per tetrahedron.
2
(B) Single chain silicate
(D) Pyro silicate

[3]

[3]

[3]

ASSIGNMENT-11
Choose only one correct option.
Q.1
Which of the following statement is incorrect regarding the complete hydrolysis of Marshall's acid.
(A) Caro's acid is an intermediate product.
[3]
(B) Two molecules of H2SO4 and one molecule of H2O2 are the final product.
(C) Hybridisation and oxidation state of central atom remain unchanged in the final product.
(D*) Both final products can act as oxidising as well as reducing agent.
Q.2

Q.3

Q.4

Which of the following is an uncommon hydrolysis product of XeF2 and XeF4?


(A) Xe
(B*) XeO3
(C) HF
(D) O2

[3]

Paragraph for question nos. 3 to 5


Covalent compounds undergo in hydrolysis via SN1 (unimolecular nucleophilic substitution) or SN2
(Bimolecular nucleophilic substitution) mechanism, for SN2 mechanism within the molecule atom should
have at least one vaccant orbital, if it is not there then hydrolysis takes place via SN1 mechanism (dissociative
step) in drastic condition.
What are the hydrolysed products of BeCl2 formed during hydrolysis
[3]
(I) [Be(OH)4]2
(II) Be(OH)2
(III) HCl
(IV) BeH2
(A) I, II, III
(B*) II, III
(C) I, III
(D) II, III and IV
Least probable product formed on hydrolysis of BCl3
(A*) [B(OH)4]
(B) HCl
(C) B(OH)3

[3]
(D) None of these

Q.5

CCl4 is inert towards hydrolysis under ordinary condition because


[3]
(I)
No vaccant orbital on attacking site of C-atom.
(II)
H2O molecule can not approach to the anti-bonding M.O. of CCl bond due to steric crowding
(III) Bond dissociation energy of CCl bond is very high
(IV) CCl4 is non polar and does not react with polar H2O molecule
Select correct code:
(A) I, II and IV
(B) I and IV
(C*) I and II
(D) I, II, III and IV

Q.6

Which of the following substance has the largest negative lattice energy ?
(A) NaCl
(B) CaBr2
(C) NaBr
(D*) CaCl2

[3]

Q.7

Which of the following is/are incorrect about solubility trend in group I & II?
Least soluble in water
Most soluble in water
(A) Hydroxides
:
LiOH
CsOH
(B*) Carbonates
:
Cs2 CO 3
Li2CO3
(C) Nitrate
:
Ba(NO3)2
Be(NO3)2
(D) Sulphates
:
BaSO4
BeSO4

[3]

Q.8

The stability of dihalides of Si, Ge, Sn and Pb increases in the sequence


(A) GeX2 < SiX2 < SnX2 < PbX2
(B) SiX2 < GeX2 < PbX2 < SnX2
(C*) SiX2 < GeX2 < SnX2 < PbX2
(D) PbX2 < SnX2 < GeX2 < SiX2

[3]

Paragraph for Question Nos. 9 to 11


Boric acid has three OH group, when it is involved in intermolecular H-bonding, results 2D-sheet like
structure in solid state.
Q.9

The number of H-atoms replaced from boric acid when it is dissolved in water is
(A) 2
(B) 1
(C) 3
(D*) zero

[3]

Q.10

It acts as what kind of acid in water.


(A) Arhenius acid
(B) Bronsted acid

[3]

Q.11

(C*) Lewis acid

Which of the following property is caused by 2D-layer structure of it.


(A) Very hard
(B*) Slippery nature (C) White colour

(D) Proton donar acid.


[3]
(D) Spongy nature

Match the column with multiple options:


Q.12

[12]

(A)

Column I
XeF5+

(P)

Column II
Two lone pairs

(B)

ICl4

(Q)

Planar

(C)

TeCl4

(R)

Non-planar

(D)

I3+

(S)

sp3d2 (Hybridization of central atom)


[Ans.

(A) R, S; (B) P, Q, S; (C) R; (D) P, Q]

ASSIGNMENT-12
One or more than one option(s) is/are correct
Q.1

Choose the correct option(s) :


(A) Order of increasing ionic size is O2 < F < Na+ < Mg+2
(B*) Order of increasing Lewis acid nature is BF3 < BCl3 < BBr3
(C*) Order of decreasing magnetic moment is Cr+2 > Co+2 > Ni+2 > Ti+3
(D) Order of decreasing Boiling point is GeH4 > CH4 > SiH4

[4]

Q.2

Which of the following statement is incorrect?


(A) Oxidizing power order : SiCl4 < SnCl4 < PbCl4
(B) Ionic character order : CsBr > RbBr > KBr > NaBr > LiBr
(C) The ionic character of lead (II) halides decreases with increase in atomic no. of halogen
(D*) The oxidation state of Tl in Tl I3 is +3.

[3]

Q.3

It has been observed that % 's' character in SbH bond in SbH3 is 0.5%. Predict the % 's' character in
the orbital occupied by the lone pair is
[3]
(A) 99.5 %
(B) 99.0 %
(C*) 98.5 %
(D) 98.0 %

Q.4

Choose the option(s) regarding correct order of acidic nature :


(A*) MgO < ZnO < P2O5 < SO3
(B) MgO < ZnO < SO3 < P2O5
(C*) Li2O < NO < CO2 < SO2
(D) Li2O < BeO < CO2 < NO

Q.5

Polarisation may be called as the distortion of the shape of an anion by an adjacently placed cation.
Which of the following statements is/are not correct?
[4]
(A*) Minimum polarization is brought about by a cation of low radius
(B*) A large cation is likely to bring about a large degree of polarisation
(C) Maximum polarization is brought about by a cation of high charge
(D*) A small anion is likely to undergo a large degree of polarisation

Q.6

Out of the following which one has the highest values of covalent character
(A) ZnCl2
(B) CdCl2
(C*) HgCl2
(D) CuCl

[3]

Q.7

Compound having lowest Melting point.


(A*) BeCl2
(B) MgCl2

[3]
(C) CaCl2

[3]

(D) SrCl2

Q.8

Which of the following properties is having incorrect order.


(A*) CaSO3 < SrSO3 < BaSO3 ; solubility order
(B) CaCO3 > SrCO3 > BaCO3 ; solubility order
(C) NaF > KF > LiF > RbF > CsF ; m.p. order
(D) CH3Cl > CH2Cl2 > CHCl3 > CCl4 ; dipole moment order

Q.9

For H3PO3 and H3PO4, the correct choice is


[4]
(A*) H3PO3 is dibasic and reducing agent.
(B) H3PO3 is dibasic and non reducing agent.
(C) H3PO4 is tribasic and reducing agent
(D*) H3PO4 is tribasic and non reducing agent.

[3]

Q.10

Statement-1 : The acidic strength order of hydra acids is : HF < HCl < HBr < HI
[3]
Statement-2 : The E.N. of halogens are F > Cl > Br > I.
(A) Statement-1 is true, statement-2 is true and statement-2 is correct explanation for statement-1.
(B*) Statement-1 is true, statement-2 is true and statement-2 is NOT the correct explanation for
statement-1.
(C) Statement-1 is true, statement-2 is false.
(D) Statement-1 is false, statement-2 is true.

Q.11

Choose the correct code for the following statements.


[3]
I.
The ()ve value of H for the dissolution of ionic compound is enough to predict the compound
is soluble in water at any temperature.
II.
For the alkali metals carbonate, solubility order decreases down the group.
III.
For the alkali metals ozonide, the thermal stability order increases down the group.
IV.
For the alkaline earth metals nitride, the thermal stability order increases down the group.
(A) T T F F
(B) T F F T
(C*) T F T F
(D) F T T F

Match the column with multiple options:


Q.12
(A)

Column I
Dithionous acid

(P)

Column II
SOS bond is not present

[8]

(B)

Thiosulphuric acid

(Q)

All S atom in the molecule has oxidation state +3

(C)

Caro's acid

(R)

Acidic strength of OH groups present in the


molecule is different

(D)

Pyrosulphurous acid

(S)

at least one S atom has oxidation state +5 in molecule


[Ans. (A) P,Q; (B)P; (C) P,R; (D) P,R,S]

ASSIGNMENT-13

One or more than one option(s) is/are correct


Question No. 1 to 4 (4 questions)
Schrondinger equation can be written for any molecule. However, it cannot be solved exactly for any
system containing more than one electron, molecular orbitals having one electron wave functions for
molecules are difficult to obtain directly from the solution of the Schrondinger wave equation. This
difficulty is overcome by resorting to an approximation method called linear combination of atomic
orbitals (LCAO) method to form molecular orbitals.
The molecular orbital formed by the addition of atomic orbitals is called the bonding molecular orbital
and the molecular orbital formed by the subtraction of atomic orbitals is called antibonding molecular
orbital. Qualitatively, the formation of molecular orbitals can be understood in terms of the constructive
or destructive interference of the electron waves of the combining atoms. In the formation of bonding
molecular orbital, the two electron waves of the bonding atoms reinforce each other (constructive
interference) while in the formation of antibonding molecular orbital, these electron waves cancel each
other (destructive interference). The result is that in a bonding molecular orbital most of the electron
density is located between the nuclei of the bonded atoms and hence the repulsion between the nuclei is
very low while in an antibonding molecular orbital, most of the electron density is located away from the
space between the nuclei, as a matter of fact there is a nodal plane (i.e., plane in which the electron
density is zero)
Q.1

How many nodal plane is/are present in *sp antibonding molecular orbital?
(A) zero

Q.2

(B*) 1

(C) 2

(D) 3

Which of the following combination of orbitals is correct?


(A)
+

(B)

(C*)

(D)

[3]

[3]

Q.3

Which of the following statements is not correct regarding bonding molecular orbitals?
[3]
(A) Bonding molecular orbitals possess less energy than the atomic orbitals from which they are formed
(B*) Bonding molecular orbitals have low electron density between the two nuclei
(C) Electron in bonding molecular orbitals contributes to the attraction between atoms
(D) They are formed when the lobes of the combining atomic orbitals have the same sign

Q.4

If X-axis is the molecular axis, then -molecular orbitals are formed by the overlap of
(A) s + pz
(B) px + py
(C*) pz + pz
(D) px + px

Q.5

Which statement is correct


[4]
(A*) Higher is the polarisation, higher will be relative solubility in non-polar solvent
(B*) Higher is the polarisation, higher will be the intensity of colour
(C*) Diamagnetic substances some times become coloured due to HOMO-LUMO transition
(D) Higher is the polarisation in metal oxide, higher will be the basic character

[3]

Q.6

If NB is the number of bonding electrons and NA is the number of antibonding electrons of a molecule.
Then choose the incorrect statement(s) for the given relationship,
[4]
NB > NA
(A) Molecule may be stable or unstable
(B*) Molecule may have any integral, fractional or zero value of bond order
(C*) Molecule is only paramagnetic species
(D*) Molecule does not exist

Q.7

In which of the following ionization processes, the bond order has increased and the magnetic behaviour
has changed?
[3]
(A) C2 C 2

Q.8

(B*) NO NO+

(D) N2 N 2

Which of the following pair of species is/are having same bond order and same magnetic moment.
(A*) HeH and H2
(C*) He2+ and Li2+

Q.9

(C) O2 O 2

(B*) O 2 and NO
(D*) N2+ and N2

According to MOT

[4]

[3]

the process N2 (g) N 2 (g) + e is more endothermic than the process O2(g) O 2 (g) + e
because
(A) N has half filled electronic configuration which is more stable.
(B) Nitrogen is less electronegative than oxygen.
(C) Electron is removed from anti bonding molecular orbital in N2 where as electron is removed from
bonding molecular orbital in O2.
(D*) Electron is removed from bonding molecular orbital in N2 where electron is removed from antibonding
molecular orbital in O2.
Q.10

Which of the following order is correct of the given property.


(A*) LiCl >NaCl > KCl > RbCl > CsCl
:
Thermal stability order
(B) BeF2 < MgF2 < CaF2 < SrF2 < BaF2
:
solubility order
(C) NO2 > NO > NO = NO2+ > NO+
:
bond length order
(D) BaO > SrO > CaO > BeO > MgO
:
basic character order

[3]

Match the column with multiple options:


Q.11

Column I
(A)

O2 O 2

Column II
(P)

[8]

An electron is removed from the * molecular


orbital.

(B)

N2 N 2

(Q)

An electron is removed from the molecular


orbital.

(C)

C2 C 2

(R)

Fractional bond order of molecular ion

(D)

NO NO+

(S)

Bond order decreases


[Ans. (A) P, R (B) R,S (C) Q,R,S (D) P]

ASSIGNMENT-14

Q.1

Find the molecule which is planar and polar.


(A) B3N3H6
(C*) BrF2Cl

[3]
(B) F2C = C = C = CF2
(D) F2C = C = CF2

Q.2

Find out the incorrect order of the dipole moment among the following pair of compound
(A) NH3 > NF3
(B*) p-nitrophenol < o-nitrophenol
(C) CH3Cl > CH2Cl2
(D) SiF4 < SF4

Q.3

For the molecule MA2Ln (where A is number of single bonded surrounding atoms, L indicates lone pair
and n is the number of lone pair and M is the central atom of s or p block element). the possible
range of n is
[3]
(A) 1 to 4
(B) zero to 4
(C) 1 to 3
(D*) 0 to 3

Q.4

The number of identical PO bonds in P2 O 46 is


(A) Four
(B*) Six
(C) Five

Q.5

All the following molecules are polar except


(A) CH2Cl2
(B) NBr3

[3]

[3]
(D) Three
[3]

(C*) XeF4

(D) FCN

Q.6

Statement-1 : Dipole moment of NF3 is less than that of NH3.


[3]
Statement-2 : Polarity of NF bond is less than that of NH bond.
(A) Statement-1 is true, statement-2 is true and statement-2 is correct explanation for statement-1.
(B) Statement-1 is true, statement-2 is true and statement-2 is NOT the correct explanation for statement-1.
(C*) Statement-1 is true, statement-2 is false.
(D) Statement-1 is false, statement-2 is true.

Q.7

Statement-1 :
Statement-2 :

Q.8

Statement-1 :P O bond length in POCl3 is 1.45 whereas the sum of single bond
[3]
covalent radii of phosphorus and oxygen is 1.83 .
Statement-2 : p d bond is present in P O bond.
(A*) Statement-1 is true, statement-2 is true and statement-2 is correct explanation for statement-1.
(B) Statement-1 is true, statement-2 is true and statement-2 is NOT the correct explanation for statement-1.
(C) Statement-1 is true, statement-2 is false.
(D) Statement-1 is false, statement-2 is true.

Dipole moment of H2O is more than that of OF2.


[3]
In H2O, the resultant bond dipole of O H bond and the resultant lone
pair moment are in opposite direction.
(A) Statement-1 is true, statement-2 is true and statement-2 is correct explanation for statement-1.
(B) Statement-1 is true, statement-2 is true and statement-2 is NOT the correct explanation for statement-1.
(C*) Statement-1 is true, statement-2 is false.
(D) Statement-1 is false, statement-2 is true.

Q.9

Statement-1 :
Allene is a non polar molecule.
[3]
Statement-2 :
Allene is non planar molecule.
(A) Statement-1 is true, statement-2 is true and statement-2 is correct explanation for statement-1.
(B*) Statement-1 is true, statement-2 is true and statement-2 is NOT the correct explanation for statement-1.
(C) Statement-1 is true, statement-2 is false.
(D) Statement-1 is false, statement-2 is true.

Q.10

Column-I

Column-II

(A)

2 lone pair

(P)

XeF5

(B)

Zero dipole moment

(Q)

NF3

(C)

Planar

(R)

ICl3

(D)

All adjacent bond angles are equal

(S)

XeF4

[10]

[Ans. (A) P, R, S; (B) P, S; (C) P, R, S; (D) P, Q, R, S ]


Q.11

Prove that dipole moment of C6H5Cl and m-C6H4Cl2 are same.

Q.12

Which of the following molecule will have the highest boiling point (in bracket D of the molecule is given)
(A) Dimethyl ether (1.3)
(B) Propane (0.0)
[3]
(C*) Methyl cyanide (3.9)
(D) Acetaldehyde (2.7)

Q.13

Match List-I with List-II and select the correct answer using the codes given below.
ListI
ListII
(a) H2O < H2S < H2Se < H2Te
1.
Bond angle
(b) H2O > H2S > H2Se > H2Te
2.
melting and boiling point
(c) H2O >> H2S < H2Se < H2Te
3.
acidic strength
(a)
(b)
(c)
(a)
(b)
(c)
(A)
1
3
2
(B)
3
2
1
(C*) 3
1
2
(D)
2
3
1

Q.14

Which of the following pair of molecule have same shape but different in polarity (Polar or nonpolar)
(A) H2O & NH3
(B) SnCl2 & SO2
(C*) CO2 & N2O
(D) SO2 & SO3
[3]

Q.15

Statement-1 :
The dipole moment of O2F2 is not zero.
[3]
Statement-2 :
All atoms are lying in the same plane.
(A) Statement-1 is true, statement-2 is true and statement-2 is correct explanation for statement-1.
(B) Statement-1 is true, statement-2 is true and statement-2 is NOT the correct explanation for statement-1.
(C*) Statement-1 is true, statement-2 is false.
(D) Statement-1 is false, statement-2 is true.

[5]

[3]

ASSIGNMENT-15

Q.1

Explain the structure of Boric acid in solid state.

Q.2

Boiling point of o-Nitrophenol is less than meta and para nitrophenol. Why?

Q.3

Maleic acid is more acidic than fumeric acid. Why?

Q.4

H F is only liquid among halogen acid. Why?

Q.5

Ammonia is more easily liquefied than HCl, explain.

Q.6

Why ice floats on water?

Q.7

Water shows maximum density at 4C. Why?

Q.8

HI is the strongest halogen acid, whereas HF is the weakest. Why?

Q.9

Wood pieces are used to hold ice-cream. Why?

Q.10

KHF2 is possible but not KHBr2 or KHI2. Why?

Q.11

O Nitrophenol is less soluble in H2O than p Nitrophenol. Why?

Q.12

o-Hydroxy benzaldehyde is a liquid at room temperature while p-hydroxy benzaldehyde is a high melting
solid.

Q.13

Glycerol is more viscous than ethanol. Explain.

Q.14

CH4 and H2O have nearly same molecular weight. Yet CH4 has a boiling point 112 K and water 373 K.
Explain.

Q.15

The experimental molecular weight of acetic acid in just double than theoretical molecular weight of
acetic acid. Why?

Q.16

Although chlorine has same electronegativity as nitrogen but the former does not form effective H-bonding.
Explain.

Q.17

Molar entropy change of vapourization of acetic acid is less than that of water. Explain

Q.18

Heat of vapourization of water is higher than HF, however strength of H-bond in HF is higher than water.
Explain

Q.19

Statement-1 :

The H-bond present in NH3 dissolved in water is best represented by


H
|
H N HO H and not by H NH O .
|
|
|
|
H
H
H
H

Statement-2 :
The OH bond polarity is more compared to that of NH bond.
(A) Statement-1 is true, statement-2 is true and statement-2 is correct explanation for statement-1.
(B) Statement-1 is true, statement-2 is true and statement-2 is NOT the correct explanation for statement-1.
(C) Statement-1 is true, statement-2 is false.
(D*) Statement-1 is false, statement-2 is true.
Q.20

Statement -1 : B(OH)3 is acidic while In(OH)3 is basic in nature.


Statement -2 : B(OH)3 has a highly Hbonding structure in solid state.
(A) Statement-1 is true, statement-2 is true and statement-2 is correct explanation for statement-1.
(B*) Statement-1 is true, statement-2 is true and statement-2 is NOT the correct explanation for
statement-1.
(C) Statement-1 is false, statement-2 is true.
(D) Statement-1 is true, statement-2 is false.

Q.21

Compare the basicity of


[Me4N]+ OH and [Me3NH]+ OH

ASSIGNMENT-18

Q.1

In which of the following metal carbonyls the C O bond order is lowest?


(A) [V(CO)6]
(B) [Mn(CO)6]+
(C) [Cr(CO)6]
(D*) [Ti(CO)6]2

Q.2

Which of the following names is/are correct for the compound Na[CoCl2(NO2)(-C3H5) (NH3)2]
(A*) Sodium allyldiamminedichloridonitrito-N-cobaltate (III)
[3]
(B) Sodium diamminedichloridoallylnitrito-N-cobaltate (III)
(C*) Sodium diamminedichloridocyclopropylnitrito-N-cobaltate (III)
(D) Sodium diamminecyclopropylnitrito-N-dichloridocobaltate (III)

Q.3

IUPAC name(s) of possible coordination positional isomer(s) of complex ion

Cl3(NH3) Pt

OH

[3]

[3]

2+

Pt(NH3)3Cl
is/are
OH
(A*) Di--hydroxido bis {diamminedichloridoplatinum (IV)} ion
(B) Bis{-hydroxidodiamminedichloridoplatinate (IV)} ion
(C*) Tetrachloridoplatinum (IV) di--hydroxidotetraammineplatinum (IV) ion
(D*) Diamminedichloridoplatinum (IV) di--hydroxidodiamminedichloridoplatinum (IV) ion
Q.4

Arrange the following compound according to dCC order.


(I)
C2 F 4
(II)
C2H4
(III) [PtCl3(C2H4)]
(A) I > II > III
(B*) III > II > I
(C) II > I > III
(D) II > III > I

Q.5

Statement-1 :
Statement-2 :

Q.6

Statement-1 :
[V(CO)6] can not act as oxidising agent.
[3]
Statement-2 :
It can not be reduced by reducing agent.
(A) Statement-1 is true, statement-2 is true and statement-2 is correct explanation for statement-1.
(B) Statement-1 is true, statement-2 is true and statement-2 is NOT the correct explanation for statement-1.
(C) Statement-1 is true, statement-2 is false.
(D*) Statement-1 is false, statement-2 is false.

[3]

In Zeise's salt C C bond order regarding ethylene molecule is less than two.
Ethylene accepts electron pair from filled d-orbital of Pt2+ into its vacant bonding
M.O.
[3]
(A) Statement-1 is true, statement-2 is true and statement-2 is correct explanation for statement-1.
(B) Statement-1 is true, statement-2 is true and statement-2 is NOT the correct explanation for statement-1.
(C*) Statement-1 is true, statement-2 is false.
(D) Statement-1 is false, statement-2 is true.

Q.7

Paragraph for question nos. 7 to 9


Sidwick EAN rule says that complex compound has the tendency to achieve the EAN of 36, 54 and 86
for first, second and third transition series elements.
[9]
Which of the following complex acts as reducing agent based on Sidwick EAN rule.
(A) Mn(CO)5
(B) Mn2(CO)10
(C*) Mn(CO)6
(D) [V(CO)6]

Q.8

Which of the following complex is following sidwick EAN rule.


(A) [Ag(S2O3)2]3 (when only 'S' atom is the donor atom)
(B*) [Cd(CN)4]2
(C) [Pt(en)2]2+
(D) [Mo( C3H5) Br(NH3)2]

Q.9

Which of following statement is not correct regarding complex "Ferrocene".


(A*) EAN of central atom in ferrocene is not equal to its nearest noble gas
(B) Molecule is having aromatic character
(C) It has sandwich like structure
(D) Two rings act as -donor ligand.

Q.10
(A)
(B)
(C)
(D)

Q.11
(i)
(ii)
(iii)
(iv)
(v)
(vi)
(vii)
(viii)
(ix)
(x)

Column I
K3[Fe(CN)5 (CO)]
K[PtCl3(C2H4)]
Na[Co(CO)4]
V(CO)6

(P)
(Q)
(R)
(S)

Column II
[12]
complex having lowest bond length of CO ligand
follow Sidgwick's rule of EAN
complex involved in synergic bonding
complex having highest bond length of CO ligand
[Ans. (A) P, Q, R; (B) R; (C) Q, R, S; (D) R]

Write down formula of the following complex compounds :


Pentaammine isothiocyanato cobalt (III) chloride
Sodium diaquobisoxalato manganate (III)
-amido--nitro-octaammine dicobalt (III) ion
Sodium bis (thiosulphato) argentate (I)
trans dichloridobis [ethylenediammine] cobalt (III) ion
-amido-bis-[pentammine cobalt (III)] nitrate
trans-Tetraamminedichloridoplatinum (IV) ion
cis-Dichloridobis (ethylenediamine) ruthenium (II)
Sodium tris (oxalato) aluminate (III)
Hexaamminecobalt (III) tris (oxalato) cobaltate (III)

[10]

ASSIGNMENT-19

(Choose the correct option, only one is correct)


Q.1
In the compound CoCl35NH3
(A) all the Cl show primary valency (PV) only
(B) two Cl show (PV) and one Cl secondary valency (SV)
(C*) two Cl show (PV) and one Cl (PV) as well as (SV)
(D) all the Cl show (SV)

[3]

Q.2

The two compounds pentaamminesulphatocobalt(III) bromide and


pentaamminesulphatocobalt(III) chloride represent
(A) Linkage isomerism
(B) Ionization isomerism
(C) Coordination isomerism
(D*) No isomerism

[3]

Q.3

Which of the following is an oxidizing agent?


(A*) Mn(CO)5
(B) Fe(CO)5

[3]

Q.4

(C) Mn2(CO)10

(D) Fe2(CO)9

EAN of the elements (*) are equal in:


*

(A*) N i (CO)4, [ Fe (CN)6]4


*

(C) [ Co (CN)6]3, [ Fe (CN)6]3

[3]
*

(B) [ N i (en)2]2+, [ Fe (H2O)6]2+


(D) [ N i (en)2]2+, [ Sc (H2O)6]3+

Q.5

Which of the following statements is incorrect?


(A) Co-ordination compounds and complexes are synonymous terms.
(B*) Complexes must give ions in the solution.
(C) Complexes may give ions in the solution or may not give ions in the solution.
(D) Generally complex ion does not dissociate into its component parts even in the solution.

[3]

Q.6

Which one of the following is an example of coordination isomerism?


(A) [Co(NH3)5Br]SO4 and [Co(NH3)5SO4]Br
(B) [Co(NH3)5NO2]Cl2 and [Co(NH3)5ONO]Cl2
(C) [Cr(H2O)6]Cl3 and [Cr(H2O)5Cl]Cl2 H2O
(D*) [Cr(NH3)6][Co(CN)6] and [Co(NH3)6][Cr(CN)6]

[3]

Q.7

Select correct code about complex [Cr(NO2)(NH3)5][ZnCl4]


[3]
(I) IUPAC name of the compound is Pentaamminenitrito-N-chromium (III) tetrachloridozincate (II)
(II) It shows geometrical isomerism
(III) It shows linkage isomerism
(IV) It shows co-ordination isomerism
(A) III, IV
(B*) I, III & IV
(C) II, III & IV
(D) I, II, III & IV

Q.8

Consider the following complexes:


(I) K2PtCl6
(II) PtCl4 2NH3
(III) PtCl4 3NH3
Their electrical conductances in an aqueous solutions are:
(A*) 256, 0, 97, 404 (B) 404, 0, 97, 256
(C) 256, 97, 0, 404

[3]
(IV) PtCl45NH3
(D) 404, 97, 256, 0

Q.9

Q.10

Statement-1 :
In Mn2(CO)10 molecule, there are total 70 electrons in both Mn atoms. [3]
Statement-2 :
Mn2(CO)10 molecule acts as oxidising agent.
(A) Statement-1 is true, statement-2 is true and statement-2 is correct explanation for statement-1.
(B) Statement-1 is true, statement-2 is true and statement-2 is NOT the correct explanation for statement-1.
(C*) Statement-1 is true, statement-2 is false.
(D) Statement-1 is false, statement-2 is true.
Paragraph for question nos. 10 to 12
Complex compounds which have same molecular formula but have different structural arrangements of
ligands around central metal atom or ion are called structural isomers and phenomenon is named as
structural isomerism.
[9]
Which of the following compounds is/are polymerisation isomer of [Fe(NO2)3(NH3)3].
(I) [Fe(NO2)(NH3)5] [Fe(NO2)5(NH3)]
(II) [Fe(NO2)2(NH3)4]2 [Fe(NO2)5(NH3)]
(III) [Fe(NO2)(NH3)5] [Fe(NO2)4(NH3)2]2 (IV) [Fe(NO2)2(NH3)4] [Fe(NO2)4(NH3)2]
Choose the correct code :
(A) II, III
(B) II, III, IV
(C) I, IV
(D*) I, II, III, IV

Q.11

How many more coordination isomers are possible of the following complex compound.
[PtCl2 (NH3)4] [Pt (SCN)4]
(A)
5
(B)
6
(C*) 8
(D)
9

Q.12

Select incorrect match


(A) [Co(NO2)(H2O)(en)2)]Cl2, [CoCl(NO2)(en)2] Cl . H2O Hydrate isomerism
(B*) [Cu(NH3)4][PtCl4], [CuCl2(NH3)2] [PtCl2(NH3)2] Coordination isomerism
(C) [Ni(CN)(H2O)(NH3)4]Cl, [NiCl(H2O)(NH3)4]CN Ionization isomerism
(D) [Cr(NCS)(NH3)5] [ZnCl4], [Cr(SCN)(NH3)5] [ZnCl4] Linkage isomerism

ASSIGNMENT-20
Q.1

Which of the following complexes can exist as diastereoisomers?


(A*) [Cr(NH3)2Cl4]
(B) [Co(NH3)5Br] 2+
2
(C) [FeCl2(NCS)2]
(D*) [PtCl2Br2]2

[3]

Q.2

Lead poisoning in the body can be removed by:


(A*) EDTA in the form of calcium dihydrogen salt
(B) cis-platin
(C) Zeisses salt
(D) DMG

[3]

Q.3

How many geometrical isomers are possible for [Pd2+ (NH2CH(CH3) CO 2 )2]o

[3]

(A) 2

(B) 3

(C*) 4

(D) 6

Q.4

Isomerisms exhibited by [Cr(NH3)2(H2O)2Cl2]+ are


(A) ionisation, optical
(B) hydrate, optical
(C*) geometrical, optical
(D) coordinate, geometrical

[3]

Q.5

Of the following complex ions, one exhibits isomerism. That is:


(A) [Ag(NH3)2]+
(B*) [Co(NH3)5NO2]2+
(C) [Pt(en)Cl2]
(D) [Co(NH3)5Cl]2+

[3]

Q.6

[Cr(NH3)5Br]Cl and [Cr(NH3)5Cl]Br can be distinguished by/and isomerism shown is:


(A) BaCl2, ionisation
(B*) AgNO3, ionisation
(C) AgNO3, coordinate
(D) BaCl2, linkage

[3]

Q.7

If excess of AgNO3 solution is added to 100 mL of a 0.024 M solution of dichlorobis (ethylene diamine)
cobalt (III) chloride, how many mol of AgCl be precipitated:
[3]
(A) 0.0012
(B) 0.0016
(C*) 0.0024
(D) 0.0048

Q.8

Which kind of isomerism is exhibited by [Co(EDTA)]


(A) Optical & geometrical isomerism
(B) Geometrical isomerism
(C*) Optical isomerism
(D) No isomerism

[3]

Q.9

Which of the following is correct IUPAC name of any complex compound.


(A) Tris (acetyl acetonato) iron (III) chloride
(B) Hexachloroplatinum (IV) tetraammine dicyano platinate (IV)
(C*) Ammine bromochloro methylamine platinum (II)
(D) Cis dichloro (ethylenediamine) platinum (II)

[3]

Q.10

[FeIICl(CN)4(O2)]4 is named as:


(A) chlorotetracyanodioxoferrate (II) ion
(B) chlorotetracyanoperoxoferrate (II) ion
(C*) chlorotetracyanosuperoxoferrate (II) ion (D) none is correct

[3]

Q.11

The total possible co-ordination isomers for the following compounds respectively are
[Co(en)3] [Cr(C2O4)3]
[Cu(NH3)4] [CuCl4]
[Ni(en)3] [Co(NO2)6]
(A) 4, 4, 4
(B) 2, 2, 2
(C) 2, 2, 4
(D*) 4, 2, 4

Q.12

Match The column:


Column I
(Co-ordination compound)
(A)
Na2[Pt(SCN)2(Ox)2]
(B)
[CrCl2(NH3)4]NO3
(C)
[Pt(NO2)(Gly)(NH3)]
(D)
K3[Fe(OH)2(C2O4)2 ]

(P)
(Q)
(R)
(S)

[3]

[12]
Column II
(Type of isomerism shown)
Ionization isomerism
Linkage isomerism
Geometrical isomerism
Optical isomerism
[Ans. (A) Q,R,S (B) P,R (C) Q,R (D) R,S]

ASSIGNMENT-21

Q.1

[Ni(CN)4]2 and [NiCl4]2 have similarity but not in


(A) magnetic moment (B) C.N. and O.N.
(C) structure

[3]
(D*) both (A) and (C)

Q.2

Which is true for [Ni(en)2]2+, Z(Ni) = 28?


(A) paramagnetism, dsp2, square planar, C.N. of Ni = 2
(B*) diamagnetism, dsp2, square planar, C.N. of Ni = 4
(C) diamagnetism, sp3, tetrahedral, C.N. of Ni = 4
(D) paramagnetism, sp3, square planar, C.N. of Ni = 4

[3]

Q.3

Among Ni(CO)4, [Ni(CN)4]2 and NiCl 24 ,

[3]

(A*) Ni(CO)4 and [Ni(CN)4]2 are diamagnetic and NiCl 24 is paramagnetic


(B) Ni(CO)4 and NiCl 24 are diamagnetic and [Ni(CN)4]2 is paramagnetic
(C) Ni(CO)4 is diamagnetic and [Ni(CN)4]2 and NiCl 24 are paramagnetic
(D) NiCl 24 and [Ni(CN)4]2 are diamagnetic and Ni(CO)4 is paramagnetic.
Q.4

Geometrical isomerism in coordination compunds is exhibited by


[3]
(A) Square planar and tetrahedral complexes (B*) Square planar and octahedral complexes
(C) Tetrahedral and octahedral complexes
(D) Squre planar, tetrahedral and octahedral complexes

Q.5

Which of the following is not optically active?


(A) [Co(en)3]3+
(B) [Cr(ox)3]3

Q.6

Arrange the following in order of decreasing number of unpaired electrons:


(I) [Fe(H2O)6]2+
(II) [Fe(CN)6]3
(III) [Fe(CN)6]4
(IV) [Fe(H2O)6]3+
(A*) IV, I, II, III
(B) I, II, III, IV
(C) III, II, I, IV
(D) II, III, I, IV

Q.7

For which of the following dn configuration of octahedral complex (es), can not exist in both high spin
and low spin forms.
[3]
3
5
6
8
(I) d
(II) d
(III) d
(IV) d
(A) II & III
(B) I & III
(C*) I & IV
(D) III & IV

Q.8

Which of the following statements is/are false


[4]
(A*) In [PtCl2(NH3)4]2+ complex ion, the cis-form is optically active, while trans-form is optically inactive
(B) In [Fe(C2O4)3]3, geometrical isomerism does not exist, while optical isomerism exists
(C*) [Mabcd]n square planar complexes exhibit both optical as well as geometrical isomerism
(D*) In [Mabcd]n tetrahedral complexes, optical isomerism cannot be observed

[3]
(C) cis-[CoCl2(en)2]+ (D*) trans-[CoCl2(en)2]+
[3]

COMPREHENSION
When a transition metal ion (usually) is involved in octahedral complex formation, the five degenerate
d-orbitals split into two set of degenerate orbitals (3 + 2). Three degenerate orbitals of lower energy
(dxy, dyz, dzx) and a set of degenerate orbitals of higher energy (d 2 2 and d 2 ) . The orbitals with
x y

lower energy are called t2g orbitals and those with higher energy are called eg orbitals.
In octahedral complexes, positive metal ion may be considered to be present at the centre and negative
ligands at the corner of a regular octahedron. As lobes of d 2 2 and d z2 lie along the axes, i.e., along
x y

Q.9

Q.10

the ligands the repulsions are more and so high is the energy. The lobes of the remaining three d-orbitals
lie between the axes i.e., between the ligands. The repulsion between them are less, so lesser the energy.
In the octahedral complexes, if metal ion has electrons more than 3 then for pairing them, the options are
(i)
Pairing may start with 4th electron in t2g orbitals.
(ii)
Pairing may start normally with 6th electrons when t2g and eg orbitals are singly filled.
In which of the following configurations, hybridisation and magnetic moment of octahedral complexes
are independent of nature of ligands.
[9]
3
(I)
d configuration of any metal cation
(II)
d6 configuration of IIIrd transition series metal cation
(III) d8 configuration of Ist transition series metal cation
(IV) d7 configuration of any metal cation
Select the correct code:
(A) III, IV
(B) I, III, IV
(C) I, II, IV
(D*) I, II, III
Which of the following electronic arrangement is / are possible for inner orbital oct complex.
(I) t 32g e g2

Q.11

(II) t 62g e1g

(III) t 32g e 0g

(IV) t 42g e g2

Select the correct code:


(A) I, IV
(B*) II, III

(C) III only

(D) III, IV

Select incorrect match for the following complexes.


(A) [IrF6]3
( > P)
(B*) [Co(H2O)6]3+
(C) Fe(CO)5

Q.12

( > P)

( < P)

2
(D) [PdCl 2 (SCN ) 2 ]
( > P)

Column-I
Column II
[12]
(A)
[Ma2bcde]n
(P)
3 optically inactive isomers
(B)
[Ma2b2c2]n
(Q)
4 geometrical isomers
(C)
[Ma3bcd]n
(R)
6 stereo(space)isomers
n

(D)
[M(AB)c2d2]
(S)
2 optically active isomers
(where AB Unsym. bidentate ligand, a,b,c,d & e monodentate ligands)
[Ans. (A) P ; (B) R,S ; (C) P, Q,S ; (D) Q, R]

ASSIGNMENT-22

One option is correct


Q.1

Which of the following complexes is diamagnetic?


(A*) [Fe(CN)6]4
(B) [Cu(NH3)4]2+
(C) [Ti(H2O)6]3+

[3]
(D) [Fe(CN)6

]3

Q.2

In nitroprusside ion the iron and NO exist as FeII and NO+ rather than FeIII and NO. These forms can
be differentiated by
[3]
(A) estimating the concentration of iron
(B) measuring the concentration of CN
(C*) measuring the solid state magnetic moment (D) thermally decomposing the compound

Q.3

The complex ions [Fe(CN)6]3 and [Fe(CN)6]4


(A) Are both octahedral and paramagnetic
(B) Are both octahedral and diamagnetic
(C*) Have same structure but opposite magnetic character
(D) Have different structure but opposite magnetic character.

Q.4

Of the following complex ions, the one that probably has the largest overall formation constant, Kf , is
(A) [Co(NH3)6]3+
(B) [Co(H2O)6]3+
[3]
3+
3+
(C) [Co(H2O)4(NH3)2]
(D*) [Co(en)3]

Q.5

Of the following complex ions, one is a Bronsted-Lowry acid. That one is


(A) [Cu(NH3)4]2+
(B) [FeCl4]
(C*) [Fe(H2O)6]3+
(D) [Zn(OH)4]2

[3]

Q.6

The number of unpaired electrons expected for the complex ion [Cr(NH3)6]2+ is:
(A*) 2
(B) 3
(C) 4
(D) 5

[3]

Q.7

The CFSE for [CoCl6]4 complex is 18000 cm1. The for [CoCl4]2 will be:
(A) 18000 cm1
(B) 16000 cm1
(C*) 8000 cm1
(D) 2000 cm1

[3]

More than one correct


Q.8
Which one of the following statement(s) is/are false?
(A*) Weak ligands like F, Cl and OH usually form low spin complexes.

[3]

[4]

(B*) Strong ligand like CN and NO 2 , generally form high spin complexes.
(C) [FeF6]3 is high spin complex.
(D*) [Ni(CO)4] is high spin complex
Q.9

Which is correct statement(s)?


(A*) [Ag(NH3)2]+ is linear with sp hybridised Ag+ ion

[4]

(B*) NiCl 24 , VO34 and MnO 4 have tetrahedral geometry


(C*) [Cu(NH3)4]2+ , [Pt(NH3)4]2+ & [Ni(CN)4]2 have dsp2 hybridisation of the metal ion
(D*) Fe(CO)5 has trigonal bipyramidal structure with d 2 sp3 hybridised iron.
z

Match the columne


Q.10 Match the column-I with column-II.
Note that column-I may have more than one matching options in column-II.
Column-I
Column-II
(A) [Ni(H2O)6]Cl2
(P)
d2sp3 hybridisation
(B) [Co(CN)2(NH3)4]OC2H5
(Q)
Ionisation isomerism
3
(C) [IrCl6]
(R)
= 2.83 B.M.
(D) [PtCl2(NH3)4]Br2
(S)
0 < P
[Ans. (A) R, S; (B) P, Q; (C) P; (D) P, Q]
Q.11

[12]

Match the column-I with column-II.


[12]
Note that column-I may have more than one matching options in column-II.
Column-I
Column-II
(A)
[Cr(gly)3]
(P)
Low spin complex
2
(B)
[CoBr2Cl2]
(Q)
high spin complex
(C)
[Fe(NH3)6]2+
(R)
optical isomerism
(D)
Na[PtBrCl(NO2)(NH3)]
(S)
geometrical isomerism
[Ans. (A) R, S; (B) Q; (C) Q; (D) P, S]

Subjective
Q.12 Explain the following with appropriate reasons :
[10]
2+
(a)
All octahedral complexes of Ni ion must be outer orbital complexes.
(b)
[Co(NH3)6]2+ and [Co(NO2)6]4 ions are easily oxidisable
(c)
4-coordinated complexes of Pd(II) and Pt(II) are diamagnetic square planar
(d)
Tetrahedral complexes do not show geometrical isomerism while square planar complexes do show this
kind of isomerism.
(e)
[Cu(NH3)4]2+ is coloured while [Cu(CN)4]3 is colour less.

ASSIGNMENT-23

One option is correct


Q.1
Complex compound [Cr(NCS)(NH3)5] [ZnCl4] will be
(A) Colourless & diamagnetic
(B) Green coloured & diamagnetic
(C*) Green coloured & shows coordination isomerism
(D) Diamagnetic & shows linkage isomerism
Q.2

The correct order of energies of d-orbitals of metal ion in a square planar complex is
(A) dxy = dyz = dzx > d
(C) d

Q.3

[3]

x 2 y 2

> d

z2

x 2 y 2

= d

z2

> dxy > dyz = dzx

(B) d

x 2 y 2

(D*) d

[3]

= d 2 > dxy = dyz = dzx

x 2 y 2

> dxy > d

z2

> dzx = dyz

MnO 4 is of intense pink colour, though Mn is in (7+) oxidation state, it is due to

[3]

(A) Oxygen gives colour to it


(B) Charge transfer when Mn (7+) gives its electron to oxygen & oxidise to Mn (8+) temporaily
(C*) Charge transfer when oxygen gives its electron to Mn (7+) changing in Mn (6+) temporaily
(D) none is correct explanation
Q.4

[CoCl2(NH3)4] + Cl [CoCl3(NH3)3] + NH3


[3]
If in the above reaction only one isomer of the product is obtained, which is true for the initial (reactant)
complex
(A) compound is in cis form
(B*) compound is in trans form
(C) compound is in both (cis and trans) forms (D) can't be predicted

Q.5

Aqueous solution of Ni2+ contains [Ni(H2O)6]2+ and its magnetic moment is 2.83 B.M.When ammonia
is added in it, comment on the magnetic moment of solution
[3]
(A*) It will remain same
(B) It increases from 2.83 B.M.
(C) It decreases from 2.83 B.M.
(D) It can not be predicted theoretically

Q.6

The complex [Fe(H2O)5NO]2+ is formed in the brown ring test for nitrates when freshly prepared
FeSO4 solution is added to aq solution of NO3 followed by addition of conc.H2SO4. Select correct
statement about this complex:
(A) colour change is due to charge transfer
(B) it has iron in +1 oxidate state and nitrosyl as NO+
(C) it has magnetic moment of 3.87 B.M. confirming three unpaired electrons in Fe
(D*) all are correct statements

[3]

Assertion Reason
Q.7 Statement-1 : Hunds rule violates in [Co(CN)6]3 complex ion
[3]
Statement-2 : Degeneracy of d orbitals is lost under any field ligand.
(A) Statement-1 is true, statement-2 is true and statement-2 is correct explanation for statement-1.
(B) Statement-1 is true, statement-2 is true and statement-2 is NOT the correct explanation for statement-1.
(C) Statement-1 is true, statement-2 is false.
(D*) Statement-1 is false, statement-2 is true.

Q.8

Statement-1 : Complex ion [Co(NH3)6]2+ is readily oxidized to [Co(NH3)6]3+.


[3]
2+
Statement-2 : Unpaired electron in complex ion [Co(NH3)6] is present in 4p orbital.
(A) Statement-1 is true, statement-2 is true and statement-2 is correct explanation for statement-1.
(B) Statement-1 is true, statement-2 is true and statement-2 is NOT the correct explanation for statement-1.
(C*) Statement-1 is true, statement-2 is false.
(D) Statement-1 is false, statement-2 is true.

More than one correct


Q.9
Select the correct statement.
[4]
(A*) Chelation effect is maximum for five & six membered rings
(B*) Greater the charge on the central metal cation, greater the value of (CFSE)
(C) In complex ion [CoF6]3, F is a weak field ligand, so that oct < P (Pairing energy) and it is low spin
complex.
(D*) [CoCl2(NH3)2 (en)]complex ion will have four different isomers.
Q.10

Which of the following statements is/are true


(A*) In Ferrocyanide ion, the effective atomic number is 36
(B*) Chelating ligands are atleast bidentate ligands

[4]

(C*) [CrCl2 (CN) 2 ( NH3 ) 2 ] and [CrCl3(NH3)3] both have d2sp3 hybridisation
(D*) As the number of rings in complex increases, stability of complex (chelate) also increases
Match the column:
Q.11
Column-I
Column-II
[12]
(Pair of complex compounds)
(Property which is different in given pair)
(A)
[Ni(CO)4] and K2[Ni(CN)4]
(P)
Magnetic moment
(B)
[Cu(NH3)4]SO4 and K3[Cu(CN)4]
(Q)
Oxidation no. of central metal
(C)
K2[NiCl4] and K4[Ni(CN)4]
(R)
Geometry
(D)
K2[NiCl4] and K2[PtCl4]
(S)
EAN of central metal
[Ans. (A) Q, R, S; (B) P, Q, R, S; (C) P, Q, S; (D) P, R, S]
Subjective:
Q.12 Explain the following with appropriate reasons :
[10]
(a)
NH2NH2 has two lone pairs. However, it does not act as chelating agent.
(b)
Cu(OH)2 is soluble in NH4OH where as it is insoluble in NaOH.
(c)
A blue coloured solution of [CoCl4]2 ion changes to pink on reaction with HgCl2.
(d)
What happens when [Ni(H2O)6]2+ is mixed with ethylenediamine in excess?
(e)
[Fe(CN)6]3 ion has magnetic moment 1.73 BM, while [Fe(H2O)6]3+ has a magnetic moment 5.92 BM.

ASSIGNMENT-24

OXY SALT ORE:

OXIDE ORE:
* ZnO

Zincite

(1)

* Fe2O3

Haematite

CaCO3

Lime stone

* Fe3O4

Magnetite

MgCO3

Magnesite

* Al2O32H2O

Bauxite

CaCO3MgCO3

Dolomite

* Fe2O33H2O

Limonite

* FeCO3

Siderite

* Cu2O

Cuprite or Ruby copper

Calamine

Pyrolusite

* ZnCO3

MnO2
SnO2

Tinstone or Casseterite

* Cu(OH)2CuCO3 Malachite or Basic Copper


Carbonate

TiO2

Rutile

Cu(OH)22CuCO3 Azurite

FeCr2 O4

CARBONATE ORE:

(FeO + Cr2O3) Chromite ore PbCO


3
Na2B4O710H2O Borax or Tincal
Ca2B6O115H2O

Colemanite

U3O8

Pitch Blende

FeO.TiO2

Ilmenite

SULPHURISED ORE:

(2)

Cerrusite

SULPHATE ORE:

CaSO4 2H2 O

Gypsum

MgSO47H2O

Epsom Salt

PbSO4

Anglesite

* PbS

Galena

BaSO4

Baryte

HgS

Cinnabar

Na2 SO410H2 O

Glauber Salt

* ZnS

Zinc blende/sphalerite

* Cu2S

Copper glance/Chalcocite

(3)

CuFeS2

Copper Pyrite (Chalcopyrite)

KNO 3

Indian Salt peter

* FeS2

Iron pyrite or Fool's gold

NaNO3

Chile Salt peter

Ag2S

Silver glance or Argentite

HALIDE ORE:
NaCl

Rock Salt

KCl

Sylvine

CaF2

Fluorspar

Na3AlF6

Cryolite

AgCl

Horn Silver

KClMgCl26H2O Carnalite

NITRATE ORE:

METALS IN LIVING ENTITIES :


(a)

Magnesium is found in chlorophyll.

(b)

Potassium is presnt in plant roots.

(c)

Manganese, iron and copper are present in


chloroplast.

(d)

Zinc is present in eyes of cats and cows.

(e)

Iron is present in haemoglobin.

(f)

Calcium is present in bones.

(g)

Vanadium is present in cucumbers.

(h)

Chromium is present in pron.

(i)

Cobalt is present in cynocobalamin (Vitamine)

ALLOYS
1.

NAME OF THE ALLOY


Magnelium

COMPOSITION
Al : 98%, Mg : 2%

USES
For making balance

2.

Duralumin

3.

Aluminium bronze

Al: 95%, Cu : 4 %
Mg : 0.5 %, Mn : 0.5%
Al :10%, Cu : 90 %

4.

Almica

5.

-Alloy

6.

Nickeloy

Al : 20%, Ni : 20 %
Co: 10%, Steel : 50%
Al : 92%, Cu : 4%
Mg : 1.5 %, Ni : 2.5 %
Al : 95%, Cu : 4 %, Ni : 1%

Air craft parts


boat machinary
Making coins, photo frames
utensils, golden paints
For making permanent magnet

7.

Pewter

Pb : 20, Sn : 80

Utensils

8.

Solder

Pb : 50, Sn : 50

Soldering

9.

Type metal

Pb : 75, Sn : 5, Sb:20

Printing type

10.

Bell metal

Cu : 80, Sn : 20

Bells making

11.

Babbit metal

Sn : 90, Sb : 7, Cu : 3

Bearing of machinary

12.

Frary metal

Pb : 97%, Ba: 2%, Ca: 1%

Bearing of machine

13.

Lino type metal

Pb : 83%, Sn : 3%, Sb:14%

Printing type

14.

Brass

Cu:70%, Zn:30%

15.

Bronze

Cu: 88-96%, Sn 4-12%

making utensils condenses tube


making
utensils, coins, statues

16.

Monel metal

Cu: 27%, Ni : 68%, Fe : 5%

17.

German silver

Cu: 50%, Zn: 30%, Ni: 20%

18.

Electron

Mg:95%, Zn:4.5, Cu: 0.5%

19.

Dutch metal

Cu: 80%, Zn: 20%

20

Nichrome

Ni, Cr, Fe

21.

Gun Metal

Cu : 87%, Zn:3%, Sn :10%

22.

Alnico

Al, Ni, Co

23.

Con Stantan

Cu : 60% , Ni : 40%

24.

Artifical Gold

Cu : 90%, Al : 10%

25.

14 Carat Gold

Au : 54%, Ag : 14% to 30%, Cu : 12-28%

26.

24 Carat Gold

100% Au
ALLOY OF STEEL

1.

Vanadium

V : 0.2-1%

2.

Chromium

Cr : 2- 4%

3.

Nickel

Ni : 3-5%

4.

Manganese steel

Mn : 10-18%

5.

Stainless steel

Cr : 12-14 % and Ni : 2-4%

6.

Tunguston

W : 10-20%

7.

Invar

Ni : 36%

Pistons and machine parts


Air craft parts

making pumps, turbines of


ships, boilers etc.
Flower Vase & ornaments
Parts of aeroplane and motor
cars
Golden yellow colour used for
decorative purpose

ASSIGNMENT-25

Q.1

Q.2

SnO2 is reduced to metallic Sn on smelting oxide with anthracite, limestone and sand. In this, function of
sand is :
[3]
(A) It acts as a flux
(B) It removes basic impurities as slag
(C*) Both are correct
(D) None is correct
Layer X
cool
and zinc


(Ag + Pb) alloy melt
(Ag + Pb + Zn) melt
is added
Layer Y

[3]

Select correct statement based on above scheme :


(A) Layer X contains zinc and silver
(B) Layer Y contains lead and silver but amount of silver in this layer is smaller than in the layer X.
(C) X and Y are immiscible layers
(D*) All are correct statements

AgCl + Na2CO3 Ag2CO3


X, X is :
(A) Ag2O and CO2
(B*) Ag, O2 and CO2
(C) Ag2O2 and CO2
(D) no effect

G(kJ/mol)

Q.3

Q.4

0
100
200
300
400
500
600
700
800
900
1000
1100
1200

2C +

g
2M

O2

+O 2

[3]

2C O

2M

gO

T1
T2
T3
Temperature

Incorrect statements about the plot is / are:


(A) T1 and T2 are melting point & boiling point of Mg respectively.
(B*) T1 and T2 are melting point & boiling point of MgO respectively.
(C) Reduction of MgO by coke is possible above T3
(D) Mg can be extracted from gaseous products by rapid cooling.

[3]

Comprehension (Q.5 to Q.8)


Questions given below are based on the given diagram for extractive metallurgy.
0
O
G,kJ

200

+O 2
n
Z
2

2C+O
2
2CO

400
600
800
1000

[12]

2Zn

b.p.
m.p.
O2 2
2Mg+

MgO

m.p.

b.p.

Q.5
Q.6

2000
500
1000
1500
Temperature, C
The points noted by arrows are the melting and boiling points of the metals zinc and magnesium. G as
a function of temperature for some reactions of extractive metallurgy.
At what approximate temperature, zinc and carbon have equal affinity for oxygen?
(A*) 1000C
(B) 1500C
(C) 500C
(D) 1200C

At this temperature G of the reaction is :


ZnO + C Zn + CO
(A) ve
(B) +ve

(C*) zero

(D) nothing can be said

Q.7

To make the following reduction process spontaneous, temperature should be :


ZnO + C Zn + CO
(A) < 1000C
(B*) > 1100C
(C) < 500C
(D) > 500C but < 1000C

Q.8

At 1100C, which reaction is spontaneous to a maximum extent?


(A) MgO + C Mg + CO
(B) ZnO + C Zn + CO
(C) MgO + Zn Mg + ZnO
(D*) ZnO + Mg MgO + Zn

Q.9

Give the correct order of initials T or F for following statements. Use T if statement is true and F if it is false.
(i)
Cu metal is extracted from its sulphide ore by reduction of Cu2O with FeS.
[3]
(ii)
An ore of Tin containing FeCrO4 is concentrated by magnetic separation method.
(iii)
Auto reduction process is used in the extraction of Cu & Hg.
(iv)
Cassiterite and Rutile are oxide ores of the metals.
(A) TFTT
(B) TTFT
(C*) FTTT
(D) FFFT

Q.10

Electrolyte reduction of alumina to aluminium by Hall-Heroult process is carried out:


(A) In the presence of NaCl
(B) In the presence of BaF2
(C*) In the presence of cryolite which forms a melt with lower melting temperature
(D) In the presence of cryolite which forms a melt with higher melting temperature

Q.11

During the process of electrorefining of copper some metals present as impurity settle as anode mud.
These are:
[3]
(A) Sn and Ag
(B) Pb and Zn
(C*) Ag and Au
(D) Fe and Ni

Q.12

The substance not likely to contain CaCO3 is:


(A) Sea shells
(B) Dolomite
(C) Marble statue

[3]

[3]
(D*) Calcined gypsum

Q.13

The most common elements present in the crust of the Earth are:
(A*) oxygen, silicon, aluminium
(B) oxygen, iron, magnesium
(C) silicon, iron, potassium
(D) oxygen, iron, silicon

[3]

Q.14

Extraction of silver from its ore involving NaCN, air and an active metal is known as:
(A) Pattinsons method
(B) Amalgamation method
(C*) Mc Arthur-forest method
(D) Parkes method

[3]

ASSIGNMENT-26
Q.1

Reduction of a metal oxide by excess carbon at high temperature is a method for the commercial preparation
of some metals. This method can be successfully applied in the case of
[3]
(A) BeO and Al2O3
(B*) ZnO and Fe2O3
(C) CaO and Cr2O3
(D) BaO and U3O8

Q.2

In Goldschmidt aluminothermic process, thermite mixture contains:


(A) 3 parts Fe2O3 and 2 parts Al
(B) 3 parts Al2O3 and 4 parts Al
(C) 1 part Fe2O3 and 12 part Al
(D*) 3 parts Fe2O3 and 1 part Al

[3]

Q.3

Electric furances are lined with magnesia because:


(A) It is not affected by acids
(B) It liberates oxygen on heating
(C*) It melts at very high temperature
(D) It has no effect of electricity\

[3]

Q.4

Which pair of elements can form alloy:


(A) Zn and Pb
(B) Fe and Hg

[3]
(C*) Fe and Cr

(D) C and Pt

Question No. 5 & 6 (2 questions)


Q.5

[6]

Step C (refining) involved in purification of Pb metal


(A) Distillation
(B) Bessemerization (C) Cupelation

(D*) Electrolytic refining

Q.6

Which of the following metals are obtained by auto reduction method:


Pb, Mn, Cu, Cr, Fe, Al.
(A) Cu, Fe
(B) Cu, Pb, Mn
(C) Mn, Cr, Pb
(D*) Pb, Cu

Q.7

Match List-I with List-II and select the correct answer using the codes given below the lists.
ListI (Metals)
ListII (Ores)
(a)
Tin
1.
Calamine
(b)
Zinc
2.
Cassiterite
(c)
Titanium
3.
Cerrusite
(d)
Lead
4.
Rutile
(a)
(b)
(c)
(d)
(a)
(b)
(c)
(d)
(A)
1
2
3
4
(B*) 2
1
4
3
(C)
4
3
2
1
(D)
2
1
3
4

Q.8

Match Column-I with Column-II and select the correct answer using the codes given below .
Column-I (Metals)
Column-II (Method used for refining)
(i)
Iron & copper
(P)
Poling
[3]
(ii)
Zirconium & Titanium
(Q)
Bessemerisation
(iii)
Lead & Tin
(R)
Van-Arkel
(iv)
Copper & Tin
(S)
Liquation
(A)
(B)
(C)
(D*)

(i)
P
Q
P
Q

(ii)
S
S
R
R

(iii)
R
R
S
S

(iv)
Q
P
Q
P

[3]

Q.9

On heating a mixture of Cu2O and Cu2S, we get:


(A*) Cu + SO2
(B) Cu + SO3
(C) CuO + CuS
(D) Cu2SO3

Q.10

Select incorrect statement regarding silver extraction process.


[3]
(A) When the lead-silver alloy is rich in silver, lead is removed by the cupellation process.
(B) When the lead-silver alloy is rich in lead, silver is removed by parke's or pattinson's process.
(C*) Zinc forms an alloy with lead, from which lead is separated by distillation.
(D) Zinc forms an alloy with silver, from which zinc is separated by distillation.

Q.11

Froth floatation process for the concentration of sulphide ores is an illustration of the practical application
of:
[3]
(A*) Adsorption
(B) Absorption
(C) Sedimentation
(D) Coagulation

Q.12

When ZnS and PbS minerals are present together, then NaCN is added to separate them in the froth
floatation process as a depressant, because
[3]
(A) Pb(CN)2 is precipitated while no effect on ZnS
(B*) ZnS forms soluble complex Na2[Zn(CN)4]
(C) PbS forms soluble complex Na2[Pb(CN)4]
(D) They cannot be separated by adding NaCN.

Q.13

When the sample of Cu with Zn impurity is to be purified by electrolysis, the appropriate electrodes are:
Cathode
Anode
[3]
(A)
Pure Zn
Pure Cu
(B)
Impure sample
Pure Cu
(C)
Impure Zn
Impure sample
(D*) Pure copper
Impure sample

Q.14

Match List-I (Metal) with List-II (Process of Extraction) and select the correct answer using the codes
given below the lists:
[3]
List-I (Metal)
List-II (Process of Extraction)
(a)
Aluminium
(1)
Blast furance
(b)
Iron
(2)
Mond process
(c)
Nickel
(3)
Bayer process
(d)
Copper
(4)
Cyanide process
(5)
Froth floatation
(a)
(b)
(c)
(d)
(A)
2
5
4
1
(B*) 3
1
2
5
(C)
2
1
4
5
(D)
3
5
2
1

[3]

ASSIGNMENT-27

Q.1

AgCl on fusion with Na2CO3 forms


(A) Ag2CO3
(B) Ag2O

[3]
(C*) Ag

(D) Ag2C2

Q.2
(i)
(ii)
(iii)
(iv)

Select the correct option for the given processes.


[3]
Process of heating steel to redness and then cooling it very slowly.
Process of heating steel in presence of NH3 and producing hard coating of Iron Nitride on the surface of steel.
Process of heating steel to redness and then cooling it suddenly by plunging it into water or oil.
Process of heating quenched steel to a temperature well below redness and then cooling it slowly.
(A) Tempering, Nitriding, Annealing & Quenching respectively
(B) Quenching, Nitriding, Annealing & Case Hardening respectively
(C) Tempering, Case harding, Quenching & Annealing respectively
(D*) Annealing, Nitriding, Quenching & Tempering respectively

Q.3

A solution of Na2SO4 in water is electrolysed using inert electrodes. The products at cathode and anode
are respectively
[3]
(A) O2 ; H2
(B) O2 ; Na
(C*) H2 ; O2
(D) O2 ; SO2

Q.4

If impurity in a metal has a greater affinity for oxygen, then the purification of metal may be carried out by
(A) Liquation
(B) Distillation
(C) Zone Refining
(D*) Cupellation
[3]

Q.5

Give the correct order of initials T or F for following statements. Use T if statement is true and F if it is
false.
[3]
(i)
In Gold Schmidt thermite process aluminium acts as a reducing agent.
(ii)
Mg is extracted by electrolysis of aq. solution MgCl2.
(iii)
Extraction of Pb is possible by smelting.
(iv)
Red Bauxite is purified by Serpeck's process.
(A) TTTF
(B) TFFT
(C) FTTT
(D*) TFTF

Q.6

Among the following groups of oxides, the group containing oxides that cannot be reduced by C to give
the respective metal is
[3]
(A*) CaO and K2O (B) Fe2O3 and ZnO (C) Cu2O and SnO2 (D) PbO and Pb3O4

Q.7

In the cyanide process for extraction of gold and silver from ores, the cyanide solution acts as a [3]
(A) reducing agent to reduce the gold and silver compounds present in the ores into the metallic states
(B*) leaching agent to bring the gold and silver into solution as cyanide complexes and thus separate
these metals from the ores
(C) leaching agent to dissolve all the other constituents of the ores leaving the gold and silver as metals
(D) leaching agent to bring the ores into solution.

Q.8

By which process Pb and Sn are extracted respectively are:


(A) Carbon reduction and self reduction
(B*) Self reduction and carbon reduction
(C) Electrolytic reduction and cyanide process
(D) Cyanide process and electrolytic reduction

[3]

Q.9

Match the following:


(I)
Bauxite
(II)
Monazite
(III) Malachite
(IV) Pitch blends
(A) I-d, II-b, III-c, IV-d
(C) I-c, II-a, III-b, IV-d

[3]
(a)
(b)
(c)
(d)

Copper
Uranium
Thorium
Aluminium
(B) I-b, II-c, III-d, IV-a
(D*) I-d, II-c, III-a, IV-b

Q.10

Match List-I with List-II and select the correct answer using the codes given below the lists:
List-I (Ore)
List-II (Metal)
(1)
Carnallite
(P)
Zinc
(2)
Calamine
(Q)
Titanium
(3)
IImenite
(R)
Magnesium
(4)
Chalcopyrite
(S)
Copper
(1)
(2)
(3)
(4)
(A)
P
R
Q
S
(B)
P
R
S
Q
(C)
R
P
S
Q
(D*) R
P
Q
S

[3]

Q.11

During the production of iron and steel.


[4]
(A) The oxide ore is primarily reduced to iron by solid coke according to the reaction.
2Fe2O3 + 3C 4Fe + 3CO2
(B*) The oxide ore is reduced by the carbon monoxide according to the reaction
Fe2O3 + 3CO 2Fe + 3CO2
(C*) Major silica impurities are removed as calcium silicate slag by addition of a fluxing agent lime stone.
(D*) The converter slag containing phosphorus is used as a fertilizer.

Q.12

Consider the following metallurgical processes:


[4]
(I) Heating impure metal with CO and distilling the resulting volatile carbonyl (b.p. 43C) and finally
decomposing at 150200C to get the pure metal
(II) Heating the sulphide ore in air until a part is converted to oxide and then further heating in the
absence of air to let the oxide react with unchanged metal sulphide.
(III) Electrolysis of the molten electrolyte containing approximately equal amounts of the metal chloride
and NaCl to obtain the metal
The processes used for obtaining magnesium, nickel and copper are respectively:
(A) (I), (II) and (III) (B) (II), (III) and (I) (C*) (III), (I) and (II) (D) (II), (I) and (III)

Q.13

What products are formed during, the electrolysis of a concentrated aqueous solution of NaCl? [4]
(A*) Cl2 (g)
(B*) NaOH (aq)
(C*) H2 (g)
(D) None

Q.14
(A)
(B)
(C)
(D)

Column-I (Ore)
Ilmenite
Dolomite
Carnalite
Chromite

Column-II (Metal in Ore)


[12]
(P)
Iron
(Q)
Magnesium
(R)
Potassium
(S)
Titanium
[Ans. (A) P, S (B) Q (C) Q, R (D) P]

ASSIGNMENT-25

Q.1

Q.2

SnO2 is reduced to metallic Sn on smelting oxide with anthracite, limestone and sand. In this, function of
sand is :
[3]
(A) It acts as a flux
(B) It removes basic impurities as slag
(C*) Both are correct
(D) None is correct
Layer X
cool
and zinc


(Ag + Pb) alloy melt
(Ag + Pb + Zn) melt
is added
Layer Y

[3]

Select correct statement based on above scheme :


(A) Layer X contains zinc and silver
(B) Layer Y contains lead and silver but amount of silver in this layer is smaller than in the layer X.
(C) X and Y are immiscible layers
(D*) All are correct statements

AgCl + Na2CO3 Ag2CO3


X, X is :
(A) Ag2O and CO2
(B*) Ag, O2 and CO2
(C) Ag2O2 and CO2
(D) no effect

G(kJ/mol)

Q.3

Q.4

0
100
200
300
400
500
600
700
800
900
1000
1100
1200

2C +

g
2M

O2

+O 2

[3]

2C O

2M

gO

T1
T2
T3
Temperature

Incorrect statements about the plot is / are:


(A) T1 and T2 are melting point & boiling point of Mg respectively.
(B*) T1 and T2 are melting point & boiling point of MgO respectively.
(C) Reduction of MgO by coke is possible above T3
(D) Mg can be extracted from gaseous products by rapid cooling.

[3]

Comprehension (Q.5 to Q.8)


Questions given below are based on the given diagram for extractive metallurgy.
0
O
G,kJ

200

+O 2
n
Z
2

2C+O
2
2CO

400
600
800
1000

[12]

2Zn

b.p.
m.p.
O2 2
2Mg+

MgO

m.p.

b.p.

Q.5
Q.6

2000
500
1000
1500
Temperature, C
The points noted by arrows are the melting and boiling points of the metals zinc and magnesium. G as
a function of temperature for some reactions of extractive metallurgy.
At what approximate temperature, zinc and carbon have equal affinity for oxygen?
(A*) 1000C
(B) 1500C
(C) 500C
(D) 1200C

At this temperature G of the reaction is :


ZnO + C Zn + CO
(A) ve
(B) +ve

(C*) zero

(D) nothing can be said

Q.7

To make the following reduction process spontaneous, temperature should be :


ZnO + C Zn + CO
(A) < 1000C
(B*) > 1100C
(C) < 500C
(D) > 500C but < 1000C

Q.8

At 1100C, which reaction is spontaneous to a maximum extent?


(A) MgO + C Mg + CO
(B) ZnO + C Zn + CO
(C) MgO + Zn Mg + ZnO
(D*) ZnO + Mg MgO + Zn

Q.9

Give the correct order of initials T or F for following statements. Use T if statement is true and F if it is false.
(i)
Cu metal is extracted from its sulphide ore by reduction of Cu2O with FeS.
[3]
(ii)
An ore of Tin containing FeCrO4 is concentrated by magnetic separation method.
(iii)
Auto reduction process is used in the extraction of Cu & Hg.
(iv)
Cassiterite and Rutile are oxide ores of the metals.
(A) TFTT
(B) TTFT
(C*) FTTT
(D) FFFT

Q.10

Electrolyte reduction of alumina to aluminium by Hall-Heroult process is carried out:


(A) In the presence of NaCl
(B) In the presence of BaF2
(C*) In the presence of cryolite which forms a melt with lower melting temperature
(D) In the presence of cryolite which forms a melt with higher melting temperature

Q.11

During the process of electrorefining of copper some metals present as impurity settle as anode mud.
These are:
[3]
(A) Sn and Ag
(B) Pb and Zn
(C*) Ag and Au
(D) Fe and Ni

Q.12

The substance not likely to contain CaCO3 is:


(A) Sea shells
(B) Dolomite
(C) Marble statue

[3]

[3]
(D*) Calcined gypsum

Q.13

The most common elements present in the crust of the Earth are:
(A*) oxygen, silicon, aluminium
(B) oxygen, iron, magnesium
(C) silicon, iron, potassium
(D) oxygen, iron, silicon

[3]

Q.14

Extraction of silver from its ore involving NaCN, air and an active metal is known as:
(A) Pattinsons method
(B) Amalgamation method
(C*) Mc Arthur-forest method
(D) Parkes method

[3]

ASSIGNMENT-26
Q.1

Reduction of a metal oxide by excess carbon at high temperature is a method for the commercial preparation
of some metals. This method can be successfully applied in the case of
[3]
(A) BeO and Al2O3
(B*) ZnO and Fe2O3
(C) CaO and Cr2O3
(D) BaO and U3O8

Q.2

In Goldschmidt aluminothermic process, thermite mixture contains:


(A) 3 parts Fe2O3 and 2 parts Al
(B) 3 parts Al2O3 and 4 parts Al
(C) 1 part Fe2O3 and 12 part Al
(D*) 3 parts Fe2O3 and 1 part Al

[3]

Q.3

Electric furances are lined with magnesia because:


(A) It is not affected by acids
(B) It liberates oxygen on heating
(C*) It melts at very high temperature
(D) It has no effect of electricity\

[3]

Q.4

Which pair of elements can form alloy:


(A) Zn and Pb
(B) Fe and Hg

[3]
(C*) Fe and Cr

(D) C and Pt

Question No. 5 & 6 (2 questions)


Q.5

[6]

Step C (refining) involved in purification of Pb metal


(A) Distillation
(B) Bessemerization (C) Cupelation

(D*) Electrolytic refining

Q.6

Which of the following metals are obtained by auto reduction method:


Pb, Mn, Cu, Cr, Fe, Al.
(A) Cu, Fe
(B) Cu, Pb, Mn
(C) Mn, Cr, Pb
(D*) Pb, Cu

Q.7

Match List-I with List-II and select the correct answer using the codes given below the lists.
ListI (Metals)
ListII (Ores)
(a)
Tin
1.
Calamine
(b)
Zinc
2.
Cassiterite
(c)
Titanium
3.
Cerrusite
(d)
Lead
4.
Rutile
(a)
(b)
(c)
(d)
(a)
(b)
(c)
(d)
(A)
1
2
3
4
(B*) 2
1
4
3
(C)
4
3
2
1
(D)
2
1
3
4

Q.8

Match Column-I with Column-II and select the correct answer using the codes given below .
Column-I (Metals)
Column-II (Method used for refining)
(i)
Iron & copper
(P)
Poling
[3]
(ii)
Zirconium & Titanium
(Q)
Bessemerisation
(iii)
Lead & Tin
(R)
Van-Arkel
(iv)
Copper & Tin
(S)
Liquation
(A)
(B)
(C)
(D*)

(i)
P
Q
P
Q

(ii)
S
S
R
R

(iii)
R
R
S
S

(iv)
Q
P
Q
P

[3]

Q.9

On heating a mixture of Cu2O and Cu2S, we get:


(A*) Cu + SO2
(B) Cu + SO3
(C) CuO + CuS
(D) Cu2SO3

Q.10

Select incorrect statement regarding silver extraction process.


[3]
(A) When the lead-silver alloy is rich in silver, lead is removed by the cupellation process.
(B) When the lead-silver alloy is rich in lead, silver is removed by parke's or pattinson's process.
(C*) Zinc forms an alloy with lead, from which lead is separated by distillation.
(D) Zinc forms an alloy with silver, from which zinc is separated by distillation.

Q.11

Froth floatation process for the concentration of sulphide ores is an illustration of the practical application
of:
[3]
(A*) Adsorption
(B) Absorption
(C) Sedimentation
(D) Coagulation

Q.12

When ZnS and PbS minerals are present together, then NaCN is added to separate them in the froth
floatation process as a depressant, because
[3]
(A) Pb(CN)2 is precipitated while no effect on ZnS
(B*) ZnS forms soluble complex Na2[Zn(CN)4]
(C) PbS forms soluble complex Na2[Pb(CN)4]
(D) They cannot be separated by adding NaCN.

Q.13

When the sample of Cu with Zn impurity is to be purified by electrolysis, the appropriate electrodes are:
Cathode
Anode
[3]
(A)
Pure Zn
Pure Cu
(B)
Impure sample
Pure Cu
(C)
Impure Zn
Impure sample
(D*) Pure copper
Impure sample

Q.14

Match List-I (Metal) with List-II (Process of Extraction) and select the correct answer using the codes
given below the lists:
[3]
List-I (Metal)
List-II (Process of Extraction)
(a)
Aluminium
(1)
Blast furance
(b)
Iron
(2)
Mond process
(c)
Nickel
(3)
Bayer process
(d)
Copper
(4)
Cyanide process
(5)
Froth floatation
(a)
(b)
(c)
(d)
(A)
2
5
4
1
(B*) 3
1
2
5
(C)
2
1
4
5
(D)
3
5
2
1

[3]

ASSIGNMENT-27

Q.1

AgCl on fusion with Na2CO3 forms


(A) Ag2CO3
(B) Ag2O

[3]
(C*) Ag

(D) Ag2C2

Q.2
(i)
(ii)
(iii)
(iv)

Select the correct option for the given processes.


[3]
Process of heating steel to redness and then cooling it very slowly.
Process of heating steel in presence of NH3 and producing hard coating of Iron Nitride on the surface of steel.
Process of heating steel to redness and then cooling it suddenly by plunging it into water or oil.
Process of heating quenched steel to a temperature well below redness and then cooling it slowly.
(A) Tempering, Nitriding, Annealing & Quenching respectively
(B) Quenching, Nitriding, Annealing & Case Hardening respectively
(C) Tempering, Case harding, Quenching & Annealing respectively
(D*) Annealing, Nitriding, Quenching & Tempering respectively

Q.3

A solution of Na2SO4 in water is electrolysed using inert electrodes. The products at cathode and anode
are respectively
[3]
(A) O2 ; H2
(B) O2 ; Na
(C*) H2 ; O2
(D) O2 ; SO2

Q.4

If impurity in a metal has a greater affinity for oxygen, then the purification of metal may be carried out by
(A) Liquation
(B) Distillation
(C) Zone Refining
(D*) Cupellation
[3]

Q.5

Give the correct order of initials T or F for following statements. Use T if statement is true and F if it is
false.
[3]
(i)
In Gold Schmidt thermite process aluminium acts as a reducing agent.
(ii)
Mg is extracted by electrolysis of aq. solution MgCl2.
(iii)
Extraction of Pb is possible by smelting.
(iv)
Red Bauxite is purified by Serpeck's process.
(A) TTTF
(B) TFFT
(C) FTTT
(D*) TFTF

Q.6

Among the following groups of oxides, the group containing oxides that cannot be reduced by C to give
the respective metal is
[3]
(A*) CaO and K2O (B) Fe2O3 and ZnO (C) Cu2O and SnO2 (D) PbO and Pb3O4

Q.7

In the cyanide process for extraction of gold and silver from ores, the cyanide solution acts as a [3]
(A) reducing agent to reduce the gold and silver compounds present in the ores into the metallic states
(B*) leaching agent to bring the gold and silver into solution as cyanide complexes and thus separate
these metals from the ores
(C) leaching agent to dissolve all the other constituents of the ores leaving the gold and silver as metals
(D) leaching agent to bring the ores into solution.

Q.8

By which process Pb and Sn are extracted respectively are:


(A) Carbon reduction and self reduction
(B*) Self reduction and carbon reduction
(C) Electrolytic reduction and cyanide process
(D) Cyanide process and electrolytic reduction

[3]

Q.9

Match the following:


(I)
Bauxite
(II)
Monazite
(III) Malachite
(IV) Pitch blends
(A) I-d, II-b, III-c, IV-d
(C) I-c, II-a, III-b, IV-d

[3]
(a)
(b)
(c)
(d)

Copper
Uranium
Thorium
Aluminium
(B) I-b, II-c, III-d, IV-a
(D*) I-d, II-c, III-a, IV-b

Q.10

Match List-I with List-II and select the correct answer using the codes given below the lists:
List-I (Ore)
List-II (Metal)
(1)
Carnallite
(P)
Zinc
(2)
Calamine
(Q)
Titanium
(3)
IImenite
(R)
Magnesium
(4)
Chalcopyrite
(S)
Copper
(1)
(2)
(3)
(4)
(A)
P
R
Q
S
(B)
P
R
S
Q
(C)
R
P
S
Q
(D*) R
P
Q
S

[3]

Q.11

During the production of iron and steel.


[4]
(A) The oxide ore is primarily reduced to iron by solid coke according to the reaction.
2Fe2O3 + 3C 4Fe + 3CO2
(B*) The oxide ore is reduced by the carbon monoxide according to the reaction
Fe2O3 + 3CO 2Fe + 3CO2
(C*) Major silica impurities are removed as calcium silicate slag by addition of a fluxing agent lime stone.
(D*) The converter slag containing phosphorus is used as a fertilizer.

Q.12

Consider the following metallurgical processes:


[4]
(I) Heating impure metal with CO and distilling the resulting volatile carbonyl (b.p. 43C) and finally
decomposing at 150200C to get the pure metal
(II) Heating the sulphide ore in air until a part is converted to oxide and then further heating in the
absence of air to let the oxide react with unchanged metal sulphide.
(III) Electrolysis of the molten electrolyte containing approximately equal amounts of the metal chloride
and NaCl to obtain the metal
The processes used for obtaining magnesium, nickel and copper are respectively:
(A) (I), (II) and (III) (B) (II), (III) and (I) (C*) (III), (I) and (II) (D) (II), (I) and (III)

Q.13

What products are formed during, the electrolysis of a concentrated aqueous solution of NaCl? [4]
(A*) Cl2 (g)
(B*) NaOH (aq)
(C*) H2 (g)
(D) None

Q.14
(A)
(B)
(C)
(D)

Column-I (Ore)
Ilmenite
Dolomite
Carnalite
Chromite

Column-II (Metal in Ore)


[12]
(P)
Iron
(Q)
Magnesium
(R)
Potassium
(S)
Titanium
[Ans. (A) P, S (B) Q (C) Q, R (D) P]

ASSIGNMENT-25

Q.1

Q.2

SnO2 is reduced to metallic Sn on smelting oxide with anthracite, limestone and sand. In this, function of
sand is :
[3]
(A) It acts as a flux
(B) It removes basic impurities as slag
(C*) Both are correct
(D) None is correct
Layer X
cool
and zinc


(Ag + Pb) alloy melt
(Ag + Pb + Zn) melt
is added
Layer Y

[3]

Select correct statement based on above scheme :


(A) Layer X contains zinc and silver
(B) Layer Y contains lead and silver but amount of silver in this layer is smaller than in the layer X.
(C) X and Y are immiscible layers
(D*) All are correct statements

AgCl + Na2CO3 Ag2CO3


X, X is :
(A) Ag2O and CO2
(B*) Ag, O2 and CO2
(C) Ag2O2 and CO2
(D) no effect

G(kJ/mol)

Q.3

Q.4

0
100
200
300
400
500
600
700
800
900
1000
1100
1200

2C +

g
2M

O2

+O 2

[3]

2C O

2M

gO

T1
T2
T3
Temperature

Incorrect statements about the plot is / are:


(A) T1 and T2 are melting point & boiling point of Mg respectively.
(B*) T1 and T2 are melting point & boiling point of MgO respectively.
(C) Reduction of MgO by coke is possible above T3
(D) Mg can be extracted from gaseous products by rapid cooling.

[3]

Comprehension (Q.5 to Q.8)


Questions given below are based on the given diagram for extractive metallurgy.
0
O
G,kJ

200

+O 2
n
Z
2

2C+O
2
2CO

400
600
800
1000

[12]

2Zn

b.p.
m.p.
O2 2
2Mg+

MgO

m.p.

b.p.

Q.5
Q.6

2000
500
1000
1500
Temperature, C
The points noted by arrows are the melting and boiling points of the metals zinc and magnesium. G as
a function of temperature for some reactions of extractive metallurgy.
At what approximate temperature, zinc and carbon have equal affinity for oxygen?
(A*) 1000C
(B) 1500C
(C) 500C
(D) 1200C

At this temperature G of the reaction is :


ZnO + C Zn + CO
(A) ve
(B) +ve

(C*) zero

(D) nothing can be said

Q.7

To make the following reduction process spontaneous, temperature should be :


ZnO + C Zn + CO
(A) < 1000C
(B*) > 1100C
(C) < 500C
(D) > 500C but < 1000C

Q.8

At 1100C, which reaction is spontaneous to a maximum extent?


(A) MgO + C Mg + CO
(B) ZnO + C Zn + CO
(C) MgO + Zn Mg + ZnO
(D*) ZnO + Mg MgO + Zn

Q.9

Give the correct order of initials T or F for following statements. Use T if statement is true and F if it is false.
(i)
Cu metal is extracted from its sulphide ore by reduction of Cu2O with FeS.
[3]
(ii)
An ore of Tin containing FeCrO4 is concentrated by magnetic separation method.
(iii)
Auto reduction process is used in the extraction of Cu & Hg.
(iv)
Cassiterite and Rutile are oxide ores of the metals.
(A) TFTT
(B) TTFT
(C*) FTTT
(D) FFFT

Q.10

Electrolyte reduction of alumina to aluminium by Hall-Heroult process is carried out:


(A) In the presence of NaCl
(B) In the presence of BaF2
(C*) In the presence of cryolite which forms a melt with lower melting temperature
(D) In the presence of cryolite which forms a melt with higher melting temperature

Q.11

During the process of electrorefining of copper some metals present as impurity settle as anode mud.
These are:
[3]
(A) Sn and Ag
(B) Pb and Zn
(C*) Ag and Au
(D) Fe and Ni

Q.12

The substance not likely to contain CaCO3 is:


(A) Sea shells
(B) Dolomite
(C) Marble statue

[3]

[3]
(D*) Calcined gypsum

Q.13

The most common elements present in the crust of the Earth are:
(A*) oxygen, silicon, aluminium
(B) oxygen, iron, magnesium
(C) silicon, iron, potassium
(D) oxygen, iron, silicon

[3]

Q.14

Extraction of silver from its ore involving NaCN, air and an active metal is known as:
(A) Pattinsons method
(B) Amalgamation method
(C*) Mc Arthur-forest method
(D) Parkes method

[3]

ASSIGNMENT-26
Q.1

Reduction of a metal oxide by excess carbon at high temperature is a method for the commercial preparation
of some metals. This method can be successfully applied in the case of
[3]
(A) BeO and Al2O3
(B*) ZnO and Fe2O3
(C) CaO and Cr2O3
(D) BaO and U3O8

Q.2

In Goldschmidt aluminothermic process, thermite mixture contains:


(A) 3 parts Fe2O3 and 2 parts Al
(B) 3 parts Al2O3 and 4 parts Al
(C) 1 part Fe2O3 and 12 part Al
(D*) 3 parts Fe2O3 and 1 part Al

[3]

Q.3

Electric furances are lined with magnesia because:


(A) It is not affected by acids
(B) It liberates oxygen on heating
(C*) It melts at very high temperature
(D) It has no effect of electricity\

[3]

Q.4

Which pair of elements can form alloy:


(A) Zn and Pb
(B) Fe and Hg

[3]
(C*) Fe and Cr

(D) C and Pt

Question No. 5 & 6 (2 questions)


Q.5

[6]

Step C (refining) involved in purification of Pb metal


(A) Distillation
(B) Bessemerization (C) Cupelation

(D*) Electrolytic refining

Q.6

Which of the following metals are obtained by auto reduction method:


Pb, Mn, Cu, Cr, Fe, Al.
(A) Cu, Fe
(B) Cu, Pb, Mn
(C) Mn, Cr, Pb
(D*) Pb, Cu

Q.7

Match List-I with List-II and select the correct answer using the codes given below the lists.
ListI (Metals)
ListII (Ores)
(a)
Tin
1.
Calamine
(b)
Zinc
2.
Cassiterite
(c)
Titanium
3.
Cerrusite
(d)
Lead
4.
Rutile
(a)
(b)
(c)
(d)
(a)
(b)
(c)
(d)
(A)
1
2
3
4
(B*) 2
1
4
3
(C)
4
3
2
1
(D)
2
1
3
4

Q.8

Match Column-I with Column-II and select the correct answer using the codes given below .
Column-I (Metals)
Column-II (Method used for refining)
(i)
Iron & copper
(P)
Poling
[3]
(ii)
Zirconium & Titanium
(Q)
Bessemerisation
(iii)
Lead & Tin
(R)
Van-Arkel
(iv)
Copper & Tin
(S)
Liquation
(A)
(B)
(C)
(D*)

(i)
P
Q
P
Q

(ii)
S
S
R
R

(iii)
R
R
S
S

(iv)
Q
P
Q
P

[3]

Q.9

On heating a mixture of Cu2O and Cu2S, we get:


(A*) Cu + SO2
(B) Cu + SO3
(C) CuO + CuS
(D) Cu2SO3

Q.10

Select incorrect statement regarding silver extraction process.


[3]
(A) When the lead-silver alloy is rich in silver, lead is removed by the cupellation process.
(B) When the lead-silver alloy is rich in lead, silver is removed by parke's or pattinson's process.
(C*) Zinc forms an alloy with lead, from which lead is separated by distillation.
(D) Zinc forms an alloy with silver, from which zinc is separated by distillation.

Q.11

Froth floatation process for the concentration of sulphide ores is an illustration of the practical application
of:
[3]
(A*) Adsorption
(B) Absorption
(C) Sedimentation
(D) Coagulation

Q.12

When ZnS and PbS minerals are present together, then NaCN is added to separate them in the froth
floatation process as a depressant, because
[3]
(A) Pb(CN)2 is precipitated while no effect on ZnS
(B*) ZnS forms soluble complex Na2[Zn(CN)4]
(C) PbS forms soluble complex Na2[Pb(CN)4]
(D) They cannot be separated by adding NaCN.

Q.13

When the sample of Cu with Zn impurity is to be purified by electrolysis, the appropriate electrodes are:
Cathode
Anode
[3]
(A)
Pure Zn
Pure Cu
(B)
Impure sample
Pure Cu
(C)
Impure Zn
Impure sample
(D*) Pure copper
Impure sample

Q.14

Match List-I (Metal) with List-II (Process of Extraction) and select the correct answer using the codes
given below the lists:
[3]
List-I (Metal)
List-II (Process of Extraction)
(a)
Aluminium
(1)
Blast furance
(b)
Iron
(2)
Mond process
(c)
Nickel
(3)
Bayer process
(d)
Copper
(4)
Cyanide process
(5)
Froth floatation
(a)
(b)
(c)
(d)
(A)
2
5
4
1
(B*) 3
1
2
5
(C)
2
1
4
5
(D)
3
5
2
1

[3]

ASSIGNMENT-27

Q.1

AgCl on fusion with Na2CO3 forms


(A) Ag2CO3
(B) Ag2O

[3]
(C*) Ag

(D) Ag2C2

Q.2
(i)
(ii)
(iii)
(iv)

Select the correct option for the given processes.


[3]
Process of heating steel to redness and then cooling it very slowly.
Process of heating steel in presence of NH3 and producing hard coating of Iron Nitride on the surface of steel.
Process of heating steel to redness and then cooling it suddenly by plunging it into water or oil.
Process of heating quenched steel to a temperature well below redness and then cooling it slowly.
(A) Tempering, Nitriding, Annealing & Quenching respectively
(B) Quenching, Nitriding, Annealing & Case Hardening respectively
(C) Tempering, Case harding, Quenching & Annealing respectively
(D*) Annealing, Nitriding, Quenching & Tempering respectively

Q.3

A solution of Na2SO4 in water is electrolysed using inert electrodes. The products at cathode and anode
are respectively
[3]
(A) O2 ; H2
(B) O2 ; Na
(C*) H2 ; O2
(D) O2 ; SO2

Q.4

If impurity in a metal has a greater affinity for oxygen, then the purification of metal may be carried out by
(A) Liquation
(B) Distillation
(C) Zone Refining
(D*) Cupellation
[3]

Q.5

Give the correct order of initials T or F for following statements. Use T if statement is true and F if it is
false.
[3]
(i)
In Gold Schmidt thermite process aluminium acts as a reducing agent.
(ii)
Mg is extracted by electrolysis of aq. solution MgCl2.
(iii)
Extraction of Pb is possible by smelting.
(iv)
Red Bauxite is purified by Serpeck's process.
(A) TTTF
(B) TFFT
(C) FTTT
(D*) TFTF

Q.6

Among the following groups of oxides, the group containing oxides that cannot be reduced by C to give
the respective metal is
[3]
(A*) CaO and K2O (B) Fe2O3 and ZnO (C) Cu2O and SnO2 (D) PbO and Pb3O4

Q.7

In the cyanide process for extraction of gold and silver from ores, the cyanide solution acts as a [3]
(A) reducing agent to reduce the gold and silver compounds present in the ores into the metallic states
(B*) leaching agent to bring the gold and silver into solution as cyanide complexes and thus separate
these metals from the ores
(C) leaching agent to dissolve all the other constituents of the ores leaving the gold and silver as metals
(D) leaching agent to bring the ores into solution.

Q.8

By which process Pb and Sn are extracted respectively are:


(A) Carbon reduction and self reduction
(B*) Self reduction and carbon reduction
(C) Electrolytic reduction and cyanide process
(D) Cyanide process and electrolytic reduction

[3]

Q.9

Match the following:


(I)
Bauxite
(II)
Monazite
(III) Malachite
(IV) Pitch blends
(A) I-d, II-b, III-c, IV-d
(C) I-c, II-a, III-b, IV-d

[3]
(a)
(b)
(c)
(d)

Copper
Uranium
Thorium
Aluminium
(B) I-b, II-c, III-d, IV-a
(D*) I-d, II-c, III-a, IV-b

Q.10

Match List-I with List-II and select the correct answer using the codes given below the lists:
List-I (Ore)
List-II (Metal)
(1)
Carnallite
(P)
Zinc
(2)
Calamine
(Q)
Titanium
(3)
IImenite
(R)
Magnesium
(4)
Chalcopyrite
(S)
Copper
(1)
(2)
(3)
(4)
(A)
P
R
Q
S
(B)
P
R
S
Q
(C)
R
P
S
Q
(D*) R
P
Q
S

[3]

Q.11

During the production of iron and steel.


[4]
(A) The oxide ore is primarily reduced to iron by solid coke according to the reaction.
2Fe2O3 + 3C 4Fe + 3CO2
(B*) The oxide ore is reduced by the carbon monoxide according to the reaction
Fe2O3 + 3CO 2Fe + 3CO2
(C*) Major silica impurities are removed as calcium silicate slag by addition of a fluxing agent lime stone.
(D*) The converter slag containing phosphorus is used as a fertilizer.

Q.12

Consider the following metallurgical processes:


[4]
(I) Heating impure metal with CO and distilling the resulting volatile carbonyl (b.p. 43C) and finally
decomposing at 150200C to get the pure metal
(II) Heating the sulphide ore in air until a part is converted to oxide and then further heating in the
absence of air to let the oxide react with unchanged metal sulphide.
(III) Electrolysis of the molten electrolyte containing approximately equal amounts of the metal chloride
and NaCl to obtain the metal
The processes used for obtaining magnesium, nickel and copper are respectively:
(A) (I), (II) and (III) (B) (II), (III) and (I) (C*) (III), (I) and (II) (D) (II), (I) and (III)

Q.13

What products are formed during, the electrolysis of a concentrated aqueous solution of NaCl? [4]
(A*) Cl2 (g)
(B*) NaOH (aq)
(C*) H2 (g)
(D) None

Q.14
(A)
(B)
(C)
(D)

Column-I (Ore)
Ilmenite
Dolomite
Carnalite
Chromite

Column-II (Metal in Ore)


[12]
(P)
Iron
(Q)
Magnesium
(R)
Potassium
(S)
Titanium
[Ans. (A) P, S (B) Q (C) Q, R (D) P]

ASSIGNMENT-25

Q.1

Q.2

SnO2 is reduced to metallic Sn on smelting oxide with anthracite, limestone and sand. In this, function of
sand is :
[3]
(A) It acts as a flux
(B) It removes basic impurities as slag
(C*) Both are correct
(D) None is correct
Layer X
cool
and zinc


(Ag + Pb) alloy melt
(Ag + Pb + Zn) melt
is added
Layer Y

[3]

Select correct statement based on above scheme :


(A) Layer X contains zinc and silver
(B) Layer Y contains lead and silver but amount of silver in this layer is smaller than in the layer X.
(C) X and Y are immiscible layers
(D*) All are correct statements

AgCl + Na2CO3 Ag2CO3


X, X is :
(A) Ag2O and CO2
(B*) Ag, O2 and CO2
(C) Ag2O2 and CO2
(D) no effect

G(kJ/mol)

Q.3

Q.4

0
100
200
300
400
500
600
700
800
900
1000
1100
1200

2C +

g
2M

O2

+O 2

[3]

2C O

2M

gO

T1
T2
T3
Temperature

Incorrect statements about the plot is / are:


(A) T1 and T2 are melting point & boiling point of Mg respectively.
(B*) T1 and T2 are melting point & boiling point of MgO respectively.
(C) Reduction of MgO by coke is possible above T3
(D) Mg can be extracted from gaseous products by rapid cooling.

[3]

Comprehension (Q.5 to Q.8)


Questions given below are based on the given diagram for extractive metallurgy.
0
O
G,kJ

200

+O 2
n
Z
2

2C+O
2
2CO

400
600
800
1000

[12]

2Zn

b.p.
m.p.
O2 2
2Mg+

MgO

m.p.

b.p.

Q.5
Q.6

2000
500
1000
1500
Temperature, C
The points noted by arrows are the melting and boiling points of the metals zinc and magnesium. G as
a function of temperature for some reactions of extractive metallurgy.
At what approximate temperature, zinc and carbon have equal affinity for oxygen?
(A*) 1000C
(B) 1500C
(C) 500C
(D) 1200C

At this temperature G of the reaction is :


ZnO + C Zn + CO
(A) ve
(B) +ve

(C*) zero

(D) nothing can be said

Q.7

To make the following reduction process spontaneous, temperature should be :


ZnO + C Zn + CO
(A) < 1000C
(B*) > 1100C
(C) < 500C
(D) > 500C but < 1000C

Q.8

At 1100C, which reaction is spontaneous to a maximum extent?


(A) MgO + C Mg + CO
(B) ZnO + C Zn + CO
(C) MgO + Zn Mg + ZnO
(D*) ZnO + Mg MgO + Zn

Q.9

Give the correct order of initials T or F for following statements. Use T if statement is true and F if it is false.
(i)
Cu metal is extracted from its sulphide ore by reduction of Cu2O with FeS.
[3]
(ii)
An ore of Tin containing FeCrO4 is concentrated by magnetic separation method.
(iii)
Auto reduction process is used in the extraction of Cu & Hg.
(iv)
Cassiterite and Rutile are oxide ores of the metals.
(A) TFTT
(B) TTFT
(C*) FTTT
(D) FFFT

Q.10

Electrolyte reduction of alumina to aluminium by Hall-Heroult process is carried out:


(A) In the presence of NaCl
(B) In the presence of BaF2
(C*) In the presence of cryolite which forms a melt with lower melting temperature
(D) In the presence of cryolite which forms a melt with higher melting temperature

Q.11

During the process of electrorefining of copper some metals present as impurity settle as anode mud.
These are:
[3]
(A) Sn and Ag
(B) Pb and Zn
(C*) Ag and Au
(D) Fe and Ni

Q.12

The substance not likely to contain CaCO3 is:


(A) Sea shells
(B) Dolomite
(C) Marble statue

[3]

[3]
(D*) Calcined gypsum

Q.13

The most common elements present in the crust of the Earth are:
(A*) oxygen, silicon, aluminium
(B) oxygen, iron, magnesium
(C) silicon, iron, potassium
(D) oxygen, iron, silicon

[3]

Q.14

Extraction of silver from its ore involving NaCN, air and an active metal is known as:
(A) Pattinsons method
(B) Amalgamation method
(C*) Mc Arthur-forest method
(D) Parkes method

[3]

ASSIGNMENT-26
Q.1

Reduction of a metal oxide by excess carbon at high temperature is a method for the commercial preparation
of some metals. This method can be successfully applied in the case of
[3]
(A) BeO and Al2O3
(B*) ZnO and Fe2O3
(C) CaO and Cr2O3
(D) BaO and U3O8

Q.2

In Goldschmidt aluminothermic process, thermite mixture contains:


(A) 3 parts Fe2O3 and 2 parts Al
(B) 3 parts Al2O3 and 4 parts Al
(C) 1 part Fe2O3 and 12 part Al
(D*) 3 parts Fe2O3 and 1 part Al

[3]

Q.3

Electric furances are lined with magnesia because:


(A) It is not affected by acids
(B) It liberates oxygen on heating
(C*) It melts at very high temperature
(D) It has no effect of electricity\

[3]

Q.4

Which pair of elements can form alloy:


(A) Zn and Pb
(B) Fe and Hg

[3]
(C*) Fe and Cr

(D) C and Pt

Question No. 5 & 6 (2 questions)


Q.5

[6]

Step C (refining) involved in purification of Pb metal


(A) Distillation
(B) Bessemerization (C) Cupelation

(D*) Electrolytic refining

Q.6

Which of the following metals are obtained by auto reduction method:


Pb, Mn, Cu, Cr, Fe, Al.
(A) Cu, Fe
(B) Cu, Pb, Mn
(C) Mn, Cr, Pb
(D*) Pb, Cu

Q.7

Match List-I with List-II and select the correct answer using the codes given below the lists.
ListI (Metals)
ListII (Ores)
(a)
Tin
1.
Calamine
(b)
Zinc
2.
Cassiterite
(c)
Titanium
3.
Cerrusite
(d)
Lead
4.
Rutile
(a)
(b)
(c)
(d)
(a)
(b)
(c)
(d)
(A)
1
2
3
4
(B*) 2
1
4
3
(C)
4
3
2
1
(D)
2
1
3
4

Q.8

Match Column-I with Column-II and select the correct answer using the codes given below .
Column-I (Metals)
Column-II (Method used for refining)
(i)
Iron & copper
(P)
Poling
[3]
(ii)
Zirconium & Titanium
(Q)
Bessemerisation
(iii)
Lead & Tin
(R)
Van-Arkel
(iv)
Copper & Tin
(S)
Liquation
(A)
(B)
(C)
(D*)

(i)
P
Q
P
Q

(ii)
S
S
R
R

(iii)
R
R
S
S

(iv)
Q
P
Q
P

[3]

Q.9

On heating a mixture of Cu2O and Cu2S, we get:


(A*) Cu + SO2
(B) Cu + SO3
(C) CuO + CuS
(D) Cu2SO3

Q.10

Select incorrect statement regarding silver extraction process.


[3]
(A) When the lead-silver alloy is rich in silver, lead is removed by the cupellation process.
(B) When the lead-silver alloy is rich in lead, silver is removed by parke's or pattinson's process.
(C*) Zinc forms an alloy with lead, from which lead is separated by distillation.
(D) Zinc forms an alloy with silver, from which zinc is separated by distillation.

Q.11

Froth floatation process for the concentration of sulphide ores is an illustration of the practical application
of:
[3]
(A*) Adsorption
(B) Absorption
(C) Sedimentation
(D) Coagulation

Q.12

When ZnS and PbS minerals are present together, then NaCN is added to separate them in the froth
floatation process as a depressant, because
[3]
(A) Pb(CN)2 is precipitated while no effect on ZnS
(B*) ZnS forms soluble complex Na2[Zn(CN)4]
(C) PbS forms soluble complex Na2[Pb(CN)4]
(D) They cannot be separated by adding NaCN.

Q.13

When the sample of Cu with Zn impurity is to be purified by electrolysis, the appropriate electrodes are:
Cathode
Anode
[3]
(A)
Pure Zn
Pure Cu
(B)
Impure sample
Pure Cu
(C)
Impure Zn
Impure sample
(D*) Pure copper
Impure sample

Q.14

Match List-I (Metal) with List-II (Process of Extraction) and select the correct answer using the codes
given below the lists:
[3]
List-I (Metal)
List-II (Process of Extraction)
(a)
Aluminium
(1)
Blast furance
(b)
Iron
(2)
Mond process
(c)
Nickel
(3)
Bayer process
(d)
Copper
(4)
Cyanide process
(5)
Froth floatation
(a)
(b)
(c)
(d)
(A)
2
5
4
1
(B*) 3
1
2
5
(C)
2
1
4
5
(D)
3
5
2
1

[3]

ASSIGNMENT-27

Q.1

AgCl on fusion with Na2CO3 forms


(A) Ag2CO3
(B) Ag2O

[3]
(C*) Ag

(D) Ag2C2

Q.2
(i)
(ii)
(iii)
(iv)

Select the correct option for the given processes.


[3]
Process of heating steel to redness and then cooling it very slowly.
Process of heating steel in presence of NH3 and producing hard coating of Iron Nitride on the surface of steel.
Process of heating steel to redness and then cooling it suddenly by plunging it into water or oil.
Process of heating quenched steel to a temperature well below redness and then cooling it slowly.
(A) Tempering, Nitriding, Annealing & Quenching respectively
(B) Quenching, Nitriding, Annealing & Case Hardening respectively
(C) Tempering, Case harding, Quenching & Annealing respectively
(D*) Annealing, Nitriding, Quenching & Tempering respectively

Q.3

A solution of Na2SO4 in water is electrolysed using inert electrodes. The products at cathode and anode
are respectively
[3]
(A) O2 ; H2
(B) O2 ; Na
(C*) H2 ; O2
(D) O2 ; SO2

Q.4

If impurity in a metal has a greater affinity for oxygen, then the purification of metal may be carried out by
(A) Liquation
(B) Distillation
(C) Zone Refining
(D*) Cupellation
[3]

Q.5

Give the correct order of initials T or F for following statements. Use T if statement is true and F if it is
false.
[3]
(i)
In Gold Schmidt thermite process aluminium acts as a reducing agent.
(ii)
Mg is extracted by electrolysis of aq. solution MgCl2.
(iii)
Extraction of Pb is possible by smelting.
(iv)
Red Bauxite is purified by Serpeck's process.
(A) TTTF
(B) TFFT
(C) FTTT
(D*) TFTF

Q.6

Among the following groups of oxides, the group containing oxides that cannot be reduced by C to give
the respective metal is
[3]
(A*) CaO and K2O (B) Fe2O3 and ZnO (C) Cu2O and SnO2 (D) PbO and Pb3O4

Q.7

In the cyanide process for extraction of gold and silver from ores, the cyanide solution acts as a [3]
(A) reducing agent to reduce the gold and silver compounds present in the ores into the metallic states
(B*) leaching agent to bring the gold and silver into solution as cyanide complexes and thus separate
these metals from the ores
(C) leaching agent to dissolve all the other constituents of the ores leaving the gold and silver as metals
(D) leaching agent to bring the ores into solution.

Q.8

By which process Pb and Sn are extracted respectively are:


(A) Carbon reduction and self reduction
(B*) Self reduction and carbon reduction
(C) Electrolytic reduction and cyanide process
(D) Cyanide process and electrolytic reduction

[3]

Q.9

Match the following:


(I)
Bauxite
(II)
Monazite
(III) Malachite
(IV) Pitch blends
(A) I-d, II-b, III-c, IV-d
(C) I-c, II-a, III-b, IV-d

[3]
(a)
(b)
(c)
(d)

Copper
Uranium
Thorium
Aluminium
(B) I-b, II-c, III-d, IV-a
(D*) I-d, II-c, III-a, IV-b

Q.10

Match List-I with List-II and select the correct answer using the codes given below the lists:
List-I (Ore)
List-II (Metal)
(1)
Carnallite
(P)
Zinc
(2)
Calamine
(Q)
Titanium
(3)
IImenite
(R)
Magnesium
(4)
Chalcopyrite
(S)
Copper
(1)
(2)
(3)
(4)
(A)
P
R
Q
S
(B)
P
R
S
Q
(C)
R
P
S
Q
(D*) R
P
Q
S

[3]

Q.11

During the production of iron and steel.


[4]
(A) The oxide ore is primarily reduced to iron by solid coke according to the reaction.
2Fe2O3 + 3C 4Fe + 3CO2
(B*) The oxide ore is reduced by the carbon monoxide according to the reaction
Fe2O3 + 3CO 2Fe + 3CO2
(C*) Major silica impurities are removed as calcium silicate slag by addition of a fluxing agent lime stone.
(D*) The converter slag containing phosphorus is used as a fertilizer.

Q.12

Consider the following metallurgical processes:


[4]
(I) Heating impure metal with CO and distilling the resulting volatile carbonyl (b.p. 43C) and finally
decomposing at 150200C to get the pure metal
(II) Heating the sulphide ore in air until a part is converted to oxide and then further heating in the
absence of air to let the oxide react with unchanged metal sulphide.
(III) Electrolysis of the molten electrolyte containing approximately equal amounts of the metal chloride
and NaCl to obtain the metal
The processes used for obtaining magnesium, nickel and copper are respectively:
(A) (I), (II) and (III) (B) (II), (III) and (I) (C*) (III), (I) and (II) (D) (II), (I) and (III)

Q.13

What products are formed during, the electrolysis of a concentrated aqueous solution of NaCl? [4]
(A*) Cl2 (g)
(B*) NaOH (aq)
(C*) H2 (g)
(D) None

Q.14
(A)
(B)
(C)
(D)

Column-I (Ore)
Ilmenite
Dolomite
Carnalite
Chromite

Column-II (Metal in Ore)


[12]
(P)
Iron
(Q)
Magnesium
(R)
Potassium
(S)
Titanium
[Ans. (A) P, S (B) Q (C) Q, R (D) P]

ASSIGNMENT-25

Q.1

Q.2

SnO2 is reduced to metallic Sn on smelting oxide with anthracite, limestone and sand. In this, function of
sand is :
[3]
(A) It acts as a flux
(B) It removes basic impurities as slag
(C*) Both are correct
(D) None is correct
Layer X
cool
and zinc


(Ag + Pb) alloy melt
(Ag + Pb + Zn) melt
is added
Layer Y

[3]

Select correct statement based on above scheme :


(A) Layer X contains zinc and silver
(B) Layer Y contains lead and silver but amount of silver in this layer is smaller than in the layer X.
(C) X and Y are immiscible layers
(D*) All are correct statements

AgCl + Na2CO3 Ag2CO3


X, X is :
(A) Ag2O and CO2
(B*) Ag, O2 and CO2
(C) Ag2O2 and CO2
(D) no effect

G(kJ/mol)

Q.3

Q.4

0
100
200
300
400
500
600
700
800
900
1000
1100
1200

2C +

g
2M

O2

+O 2

[3]

2C O

2M

gO

T1
T2
T3
Temperature

Incorrect statements about the plot is / are:


(A) T1 and T2 are melting point & boiling point of Mg respectively.
(B*) T1 and T2 are melting point & boiling point of MgO respectively.
(C) Reduction of MgO by coke is possible above T3
(D) Mg can be extracted from gaseous products by rapid cooling.

[3]

Comprehension (Q.5 to Q.8)


Questions given below are based on the given diagram for extractive metallurgy.
0
O
G,kJ

200

+O 2
n
Z
2

2C+O
2
2CO

400
600
800
1000

[12]

2Zn

b.p.
m.p.
O2 2
2Mg+

MgO

m.p.

b.p.

Q.5
Q.6

2000
500
1000
1500
Temperature, C
The points noted by arrows are the melting and boiling points of the metals zinc and magnesium. G as
a function of temperature for some reactions of extractive metallurgy.
At what approximate temperature, zinc and carbon have equal affinity for oxygen?
(A*) 1000C
(B) 1500C
(C) 500C
(D) 1200C

At this temperature G of the reaction is :


ZnO + C Zn + CO
(A) ve
(B) +ve

(C*) zero

(D) nothing can be said

Q.7

To make the following reduction process spontaneous, temperature should be :


ZnO + C Zn + CO
(A) < 1000C
(B*) > 1100C
(C) < 500C
(D) > 500C but < 1000C

Q.8

At 1100C, which reaction is spontaneous to a maximum extent?


(A) MgO + C Mg + CO
(B) ZnO + C Zn + CO
(C) MgO + Zn Mg + ZnO
(D*) ZnO + Mg MgO + Zn

Q.9

Give the correct order of initials T or F for following statements. Use T if statement is true and F if it is false.
(i)
Cu metal is extracted from its sulphide ore by reduction of Cu2O with FeS.
[3]
(ii)
An ore of Tin containing FeCrO4 is concentrated by magnetic separation method.
(iii)
Auto reduction process is used in the extraction of Cu & Hg.
(iv)
Cassiterite and Rutile are oxide ores of the metals.
(A) TFTT
(B) TTFT
(C*) FTTT
(D) FFFT

Q.10

Electrolyte reduction of alumina to aluminium by Hall-Heroult process is carried out:


(A) In the presence of NaCl
(B) In the presence of BaF2
(C*) In the presence of cryolite which forms a melt with lower melting temperature
(D) In the presence of cryolite which forms a melt with higher melting temperature

Q.11

During the process of electrorefining of copper some metals present as impurity settle as anode mud.
These are:
[3]
(A) Sn and Ag
(B) Pb and Zn
(C*) Ag and Au
(D) Fe and Ni

Q.12

The substance not likely to contain CaCO3 is:


(A) Sea shells
(B) Dolomite
(C) Marble statue

[3]

[3]
(D*) Calcined gypsum

Q.13

The most common elements present in the crust of the Earth are:
(A*) oxygen, silicon, aluminium
(B) oxygen, iron, magnesium
(C) silicon, iron, potassium
(D) oxygen, iron, silicon

[3]

Q.14

Extraction of silver from its ore involving NaCN, air and an active metal is known as:
(A) Pattinsons method
(B) Amalgamation method
(C*) Mc Arthur-forest method
(D) Parkes method

[3]

ASSIGNMENT-26
Q.1

Reduction of a metal oxide by excess carbon at high temperature is a method for the commercial preparation
of some metals. This method can be successfully applied in the case of
[3]
(A) BeO and Al2O3
(B*) ZnO and Fe2O3
(C) CaO and Cr2O3
(D) BaO and U3O8

Q.2

In Goldschmidt aluminothermic process, thermite mixture contains:


(A) 3 parts Fe2O3 and 2 parts Al
(B) 3 parts Al2O3 and 4 parts Al
(C) 1 part Fe2O3 and 12 part Al
(D*) 3 parts Fe2O3 and 1 part Al

[3]

Q.3

Electric furances are lined with magnesia because:


(A) It is not affected by acids
(B) It liberates oxygen on heating
(C*) It melts at very high temperature
(D) It has no effect of electricity\

[3]

Q.4

Which pair of elements can form alloy:


(A) Zn and Pb
(B) Fe and Hg

[3]
(C*) Fe and Cr

(D) C and Pt

Question No. 5 & 6 (2 questions)


Q.5

[6]

Step C (refining) involved in purification of Pb metal


(A) Distillation
(B) Bessemerization (C) Cupelation

(D*) Electrolytic refining

Q.6

Which of the following metals are obtained by auto reduction method:


Pb, Mn, Cu, Cr, Fe, Al.
(A) Cu, Fe
(B) Cu, Pb, Mn
(C) Mn, Cr, Pb
(D*) Pb, Cu

Q.7

Match List-I with List-II and select the correct answer using the codes given below the lists.
ListI (Metals)
ListII (Ores)
(a)
Tin
1.
Calamine
(b)
Zinc
2.
Cassiterite
(c)
Titanium
3.
Cerrusite
(d)
Lead
4.
Rutile
(a)
(b)
(c)
(d)
(a)
(b)
(c)
(d)
(A)
1
2
3
4
(B*) 2
1
4
3
(C)
4
3
2
1
(D)
2
1
3
4

Q.8

Match Column-I with Column-II and select the correct answer using the codes given below .
Column-I (Metals)
Column-II (Method used for refining)
(i)
Iron & copper
(P)
Poling
[3]
(ii)
Zirconium & Titanium
(Q)
Bessemerisation
(iii)
Lead & Tin
(R)
Van-Arkel
(iv)
Copper & Tin
(S)
Liquation
(A)
(B)
(C)
(D*)

(i)
P
Q
P
Q

(ii)
S
S
R
R

(iii)
R
R
S
S

(iv)
Q
P
Q
P

[3]

Q.9

On heating a mixture of Cu2O and Cu2S, we get:


(A*) Cu + SO2
(B) Cu + SO3
(C) CuO + CuS
(D) Cu2SO3

Q.10

Select incorrect statement regarding silver extraction process.


[3]
(A) When the lead-silver alloy is rich in silver, lead is removed by the cupellation process.
(B) When the lead-silver alloy is rich in lead, silver is removed by parke's or pattinson's process.
(C*) Zinc forms an alloy with lead, from which lead is separated by distillation.
(D) Zinc forms an alloy with silver, from which zinc is separated by distillation.

Q.11

Froth floatation process for the concentration of sulphide ores is an illustration of the practical application
of:
[3]
(A*) Adsorption
(B) Absorption
(C) Sedimentation
(D) Coagulation

Q.12

When ZnS and PbS minerals are present together, then NaCN is added to separate them in the froth
floatation process as a depressant, because
[3]
(A) Pb(CN)2 is precipitated while no effect on ZnS
(B*) ZnS forms soluble complex Na2[Zn(CN)4]
(C) PbS forms soluble complex Na2[Pb(CN)4]
(D) They cannot be separated by adding NaCN.

Q.13

When the sample of Cu with Zn impurity is to be purified by electrolysis, the appropriate electrodes are:
Cathode
Anode
[3]
(A)
Pure Zn
Pure Cu
(B)
Impure sample
Pure Cu
(C)
Impure Zn
Impure sample
(D*) Pure copper
Impure sample

Q.14

Match List-I (Metal) with List-II (Process of Extraction) and select the correct answer using the codes
given below the lists:
[3]
List-I (Metal)
List-II (Process of Extraction)
(a)
Aluminium
(1)
Blast furance
(b)
Iron
(2)
Mond process
(c)
Nickel
(3)
Bayer process
(d)
Copper
(4)
Cyanide process
(5)
Froth floatation
(a)
(b)
(c)
(d)
(A)
2
5
4
1
(B*) 3
1
2
5
(C)
2
1
4
5
(D)
3
5
2
1

[3]

ASSIGNMENT-27

Q.1

AgCl on fusion with Na2CO3 forms


(A) Ag2CO3
(B) Ag2O

[3]
(C*) Ag

(D) Ag2C2

Q.2
(i)
(ii)
(iii)
(iv)

Select the correct option for the given processes.


[3]
Process of heating steel to redness and then cooling it very slowly.
Process of heating steel in presence of NH3 and producing hard coating of Iron Nitride on the surface of steel.
Process of heating steel to redness and then cooling it suddenly by plunging it into water or oil.
Process of heating quenched steel to a temperature well below redness and then cooling it slowly.
(A) Tempering, Nitriding, Annealing & Quenching respectively
(B) Quenching, Nitriding, Annealing & Case Hardening respectively
(C) Tempering, Case harding, Quenching & Annealing respectively
(D*) Annealing, Nitriding, Quenching & Tempering respectively

Q.3

A solution of Na2SO4 in water is electrolysed using inert electrodes. The products at cathode and anode
are respectively
[3]
(A) O2 ; H2
(B) O2 ; Na
(C*) H2 ; O2
(D) O2 ; SO2

Q.4

If impurity in a metal has a greater affinity for oxygen, then the purification of metal may be carried out by
(A) Liquation
(B) Distillation
(C) Zone Refining
(D*) Cupellation
[3]

Q.5

Give the correct order of initials T or F for following statements. Use T if statement is true and F if it is
false.
[3]
(i)
In Gold Schmidt thermite process aluminium acts as a reducing agent.
(ii)
Mg is extracted by electrolysis of aq. solution MgCl2.
(iii)
Extraction of Pb is possible by smelting.
(iv)
Red Bauxite is purified by Serpeck's process.
(A) TTTF
(B) TFFT
(C) FTTT
(D*) TFTF

Q.6

Among the following groups of oxides, the group containing oxides that cannot be reduced by C to give
the respective metal is
[3]
(A*) CaO and K2O (B) Fe2O3 and ZnO (C) Cu2O and SnO2 (D) PbO and Pb3O4

Q.7

In the cyanide process for extraction of gold and silver from ores, the cyanide solution acts as a [3]
(A) reducing agent to reduce the gold and silver compounds present in the ores into the metallic states
(B*) leaching agent to bring the gold and silver into solution as cyanide complexes and thus separate
these metals from the ores
(C) leaching agent to dissolve all the other constituents of the ores leaving the gold and silver as metals
(D) leaching agent to bring the ores into solution.

Q.8

By which process Pb and Sn are extracted respectively are:


(A) Carbon reduction and self reduction
(B*) Self reduction and carbon reduction
(C) Electrolytic reduction and cyanide process
(D) Cyanide process and electrolytic reduction

[3]

Q.9

Match the following:


(I)
Bauxite
(II)
Monazite
(III) Malachite
(IV) Pitch blends
(A) I-d, II-b, III-c, IV-d
(C) I-c, II-a, III-b, IV-d

[3]
(a)
(b)
(c)
(d)

Copper
Uranium
Thorium
Aluminium
(B) I-b, II-c, III-d, IV-a
(D*) I-d, II-c, III-a, IV-b

Q.10

Match List-I with List-II and select the correct answer using the codes given below the lists:
List-I (Ore)
List-II (Metal)
(1)
Carnallite
(P)
Zinc
(2)
Calamine
(Q)
Titanium
(3)
IImenite
(R)
Magnesium
(4)
Chalcopyrite
(S)
Copper
(1)
(2)
(3)
(4)
(A)
P
R
Q
S
(B)
P
R
S
Q
(C)
R
P
S
Q
(D*) R
P
Q
S

[3]

Q.11

During the production of iron and steel.


[4]
(A) The oxide ore is primarily reduced to iron by solid coke according to the reaction.
2Fe2O3 + 3C 4Fe + 3CO2
(B*) The oxide ore is reduced by the carbon monoxide according to the reaction
Fe2O3 + 3CO 2Fe + 3CO2
(C*) Major silica impurities are removed as calcium silicate slag by addition of a fluxing agent lime stone.
(D*) The converter slag containing phosphorus is used as a fertilizer.

Q.12

Consider the following metallurgical processes:


[4]
(I) Heating impure metal with CO and distilling the resulting volatile carbonyl (b.p. 43C) and finally
decomposing at 150200C to get the pure metal
(II) Heating the sulphide ore in air until a part is converted to oxide and then further heating in the
absence of air to let the oxide react with unchanged metal sulphide.
(III) Electrolysis of the molten electrolyte containing approximately equal amounts of the metal chloride
and NaCl to obtain the metal
The processes used for obtaining magnesium, nickel and copper are respectively:
(A) (I), (II) and (III) (B) (II), (III) and (I) (C*) (III), (I) and (II) (D) (II), (I) and (III)

Q.13

What products are formed during, the electrolysis of a concentrated aqueous solution of NaCl? [4]
(A*) Cl2 (g)
(B*) NaOH (aq)
(C*) H2 (g)
(D) None

Q.14
(A)
(B)
(C)
(D)

Column-I (Ore)
Ilmenite
Dolomite
Carnalite
Chromite

Column-II (Metal in Ore)


[12]
(P)
Iron
(Q)
Magnesium
(R)
Potassium
(S)
Titanium
[Ans. (A) P, S (B) Q (C) Q, R (D) P]

ASSIGNMENT-25

Q.1

Q.2

SnO2 is reduced to metallic Sn on smelting oxide with anthracite, limestone and sand. In this, function of
sand is :
[3]
(A) It acts as a flux
(B) It removes basic impurities as slag
(C*) Both are correct
(D) None is correct
Layer X
cool
and zinc


(Ag + Pb) alloy melt
(Ag + Pb + Zn) melt
is added
Layer Y

[3]

Select correct statement based on above scheme :


(A) Layer X contains zinc and silver
(B) Layer Y contains lead and silver but amount of silver in this layer is smaller than in the layer X.
(C) X and Y are immiscible layers
(D*) All are correct statements

AgCl + Na2CO3 Ag2CO3


X, X is :
(A) Ag2O and CO2
(B*) Ag, O2 and CO2
(C) Ag2O2 and CO2
(D) no effect

G(kJ/mol)

Q.3

Q.4

0
100
200
300
400
500
600
700
800
900
1000
1100
1200

2C +

g
2M

O2

+O 2

[3]

2C O

2M

gO

T1
T2
T3
Temperature

Incorrect statements about the plot is / are:


(A) T1 and T2 are melting point & boiling point of Mg respectively.
(B*) T1 and T2 are melting point & boiling point of MgO respectively.
(C) Reduction of MgO by coke is possible above T3
(D) Mg can be extracted from gaseous products by rapid cooling.

[3]

Comprehension (Q.5 to Q.8)


Questions given below are based on the given diagram for extractive metallurgy.
0
O
G,kJ

200

+O 2
n
Z
2

2C+O
2
2CO

400
600
800
1000

[12]

2Zn

b.p.
m.p.
O2 2
2Mg+

MgO

m.p.

b.p.

Q.5
Q.6

2000
500
1000
1500
Temperature, C
The points noted by arrows are the melting and boiling points of the metals zinc and magnesium. G as
a function of temperature for some reactions of extractive metallurgy.
At what approximate temperature, zinc and carbon have equal affinity for oxygen?
(A*) 1000C
(B) 1500C
(C) 500C
(D) 1200C

At this temperature G of the reaction is :


ZnO + C Zn + CO
(A) ve
(B) +ve

(C*) zero

(D) nothing can be said

Q.7

To make the following reduction process spontaneous, temperature should be :


ZnO + C Zn + CO
(A) < 1000C
(B*) > 1100C
(C) < 500C
(D) > 500C but < 1000C

Q.8

At 1100C, which reaction is spontaneous to a maximum extent?


(A) MgO + C Mg + CO
(B) ZnO + C Zn + CO
(C) MgO + Zn Mg + ZnO
(D*) ZnO + Mg MgO + Zn

Q.9

Give the correct order of initials T or F for following statements. Use T if statement is true and F if it is false.
(i)
Cu metal is extracted from its sulphide ore by reduction of Cu2O with FeS.
[3]
(ii)
An ore of Tin containing FeCrO4 is concentrated by magnetic separation method.
(iii)
Auto reduction process is used in the extraction of Cu & Hg.
(iv)
Cassiterite and Rutile are oxide ores of the metals.
(A) TFTT
(B) TTFT
(C*) FTTT
(D) FFFT

Q.10

Electrolyte reduction of alumina to aluminium by Hall-Heroult process is carried out:


(A) In the presence of NaCl
(B) In the presence of BaF2
(C*) In the presence of cryolite which forms a melt with lower melting temperature
(D) In the presence of cryolite which forms a melt with higher melting temperature

Q.11

During the process of electrorefining of copper some metals present as impurity settle as anode mud.
These are:
[3]
(A) Sn and Ag
(B) Pb and Zn
(C*) Ag and Au
(D) Fe and Ni

Q.12

The substance not likely to contain CaCO3 is:


(A) Sea shells
(B) Dolomite
(C) Marble statue

[3]

[3]
(D*) Calcined gypsum

Q.13

The most common elements present in the crust of the Earth are:
(A*) oxygen, silicon, aluminium
(B) oxygen, iron, magnesium
(C) silicon, iron, potassium
(D) oxygen, iron, silicon

[3]

Q.14

Extraction of silver from its ore involving NaCN, air and an active metal is known as:
(A) Pattinsons method
(B) Amalgamation method
(C*) Mc Arthur-forest method
(D) Parkes method

[3]

ASSIGNMENT-26
Q.1

Reduction of a metal oxide by excess carbon at high temperature is a method for the commercial preparation
of some metals. This method can be successfully applied in the case of
[3]
(A) BeO and Al2O3
(B*) ZnO and Fe2O3
(C) CaO and Cr2O3
(D) BaO and U3O8

Q.2

In Goldschmidt aluminothermic process, thermite mixture contains:


(A) 3 parts Fe2O3 and 2 parts Al
(B) 3 parts Al2O3 and 4 parts Al
(C) 1 part Fe2O3 and 12 part Al
(D*) 3 parts Fe2O3 and 1 part Al

[3]

Q.3

Electric furances are lined with magnesia because:


(A) It is not affected by acids
(B) It liberates oxygen on heating
(C*) It melts at very high temperature
(D) It has no effect of electricity\

[3]

Q.4

Which pair of elements can form alloy:


(A) Zn and Pb
(B) Fe and Hg

[3]
(C*) Fe and Cr

(D) C and Pt

Question No. 5 & 6 (2 questions)


Q.5

[6]

Step C (refining) involved in purification of Pb metal


(A) Distillation
(B) Bessemerization (C) Cupelation

(D*) Electrolytic refining

Q.6

Which of the following metals are obtained by auto reduction method:


Pb, Mn, Cu, Cr, Fe, Al.
(A) Cu, Fe
(B) Cu, Pb, Mn
(C) Mn, Cr, Pb
(D*) Pb, Cu

Q.7

Match List-I with List-II and select the correct answer using the codes given below the lists.
ListI (Metals)
ListII (Ores)
(a)
Tin
1.
Calamine
(b)
Zinc
2.
Cassiterite
(c)
Titanium
3.
Cerrusite
(d)
Lead
4.
Rutile
(a)
(b)
(c)
(d)
(a)
(b)
(c)
(d)
(A)
1
2
3
4
(B*) 2
1
4
3
(C)
4
3
2
1
(D)
2
1
3
4

Q.8

Match Column-I with Column-II and select the correct answer using the codes given below .
Column-I (Metals)
Column-II (Method used for refining)
(i)
Iron & copper
(P)
Poling
[3]
(ii)
Zirconium & Titanium
(Q)
Bessemerisation
(iii)
Lead & Tin
(R)
Van-Arkel
(iv)
Copper & Tin
(S)
Liquation
(A)
(B)
(C)
(D*)

(i)
P
Q
P
Q

(ii)
S
S
R
R

(iii)
R
R
S
S

(iv)
Q
P
Q
P

[3]

Q.9

On heating a mixture of Cu2O and Cu2S, we get:


(A*) Cu + SO2
(B) Cu + SO3
(C) CuO + CuS
(D) Cu2SO3

Q.10

Select incorrect statement regarding silver extraction process.


[3]
(A) When the lead-silver alloy is rich in silver, lead is removed by the cupellation process.
(B) When the lead-silver alloy is rich in lead, silver is removed by parke's or pattinson's process.
(C*) Zinc forms an alloy with lead, from which lead is separated by distillation.
(D) Zinc forms an alloy with silver, from which zinc is separated by distillation.

Q.11

Froth floatation process for the concentration of sulphide ores is an illustration of the practical application
of:
[3]
(A*) Adsorption
(B) Absorption
(C) Sedimentation
(D) Coagulation

Q.12

When ZnS and PbS minerals are present together, then NaCN is added to separate them in the froth
floatation process as a depressant, because
[3]
(A) Pb(CN)2 is precipitated while no effect on ZnS
(B*) ZnS forms soluble complex Na2[Zn(CN)4]
(C) PbS forms soluble complex Na2[Pb(CN)4]
(D) They cannot be separated by adding NaCN.

Q.13

When the sample of Cu with Zn impurity is to be purified by electrolysis, the appropriate electrodes are:
Cathode
Anode
[3]
(A)
Pure Zn
Pure Cu
(B)
Impure sample
Pure Cu
(C)
Impure Zn
Impure sample
(D*) Pure copper
Impure sample

Q.14

Match List-I (Metal) with List-II (Process of Extraction) and select the correct answer using the codes
given below the lists:
[3]
List-I (Metal)
List-II (Process of Extraction)
(a)
Aluminium
(1)
Blast furance
(b)
Iron
(2)
Mond process
(c)
Nickel
(3)
Bayer process
(d)
Copper
(4)
Cyanide process
(5)
Froth floatation
(a)
(b)
(c)
(d)
(A)
2
5
4
1
(B*) 3
1
2
5
(C)
2
1
4
5
(D)
3
5
2
1

[3]

ASSIGNMENT-27

Q.1

AgCl on fusion with Na2CO3 forms


(A) Ag2CO3
(B) Ag2O

[3]
(C*) Ag

(D) Ag2C2

Q.2
(i)
(ii)
(iii)
(iv)

Select the correct option for the given processes.


[3]
Process of heating steel to redness and then cooling it very slowly.
Process of heating steel in presence of NH3 and producing hard coating of Iron Nitride on the surface of steel.
Process of heating steel to redness and then cooling it suddenly by plunging it into water or oil.
Process of heating quenched steel to a temperature well below redness and then cooling it slowly.
(A) Tempering, Nitriding, Annealing & Quenching respectively
(B) Quenching, Nitriding, Annealing & Case Hardening respectively
(C) Tempering, Case harding, Quenching & Annealing respectively
(D*) Annealing, Nitriding, Quenching & Tempering respectively

Q.3

A solution of Na2SO4 in water is electrolysed using inert electrodes. The products at cathode and anode
are respectively
[3]
(A) O2 ; H2
(B) O2 ; Na
(C*) H2 ; O2
(D) O2 ; SO2

Q.4

If impurity in a metal has a greater affinity for oxygen, then the purification of metal may be carried out by
(A) Liquation
(B) Distillation
(C) Zone Refining
(D*) Cupellation
[3]

Q.5

Give the correct order of initials T or F for following statements. Use T if statement is true and F if it is
false.
[3]
(i)
In Gold Schmidt thermite process aluminium acts as a reducing agent.
(ii)
Mg is extracted by electrolysis of aq. solution MgCl2.
(iii)
Extraction of Pb is possible by smelting.
(iv)
Red Bauxite is purified by Serpeck's process.
(A) TTTF
(B) TFFT
(C) FTTT
(D*) TFTF

Q.6

Among the following groups of oxides, the group containing oxides that cannot be reduced by C to give
the respective metal is
[3]
(A*) CaO and K2O (B) Fe2O3 and ZnO (C) Cu2O and SnO2 (D) PbO and Pb3O4

Q.7

In the cyanide process for extraction of gold and silver from ores, the cyanide solution acts as a [3]
(A) reducing agent to reduce the gold and silver compounds present in the ores into the metallic states
(B*) leaching agent to bring the gold and silver into solution as cyanide complexes and thus separate
these metals from the ores
(C) leaching agent to dissolve all the other constituents of the ores leaving the gold and silver as metals
(D) leaching agent to bring the ores into solution.

Q.8

By which process Pb and Sn are extracted respectively are:


(A) Carbon reduction and self reduction
(B*) Self reduction and carbon reduction
(C) Electrolytic reduction and cyanide process
(D) Cyanide process and electrolytic reduction

[3]

Q.9

Match the following:


(I)
Bauxite
(II)
Monazite
(III) Malachite
(IV) Pitch blends
(A) I-d, II-b, III-c, IV-d
(C) I-c, II-a, III-b, IV-d

[3]
(a)
(b)
(c)
(d)

Copper
Uranium
Thorium
Aluminium
(B) I-b, II-c, III-d, IV-a
(D*) I-d, II-c, III-a, IV-b

Q.10

Match List-I with List-II and select the correct answer using the codes given below the lists:
List-I (Ore)
List-II (Metal)
(1)
Carnallite
(P)
Zinc
(2)
Calamine
(Q)
Titanium
(3)
IImenite
(R)
Magnesium
(4)
Chalcopyrite
(S)
Copper
(1)
(2)
(3)
(4)
(A)
P
R
Q
S
(B)
P
R
S
Q
(C)
R
P
S
Q
(D*) R
P
Q
S

[3]

Q.11

During the production of iron and steel.


[4]
(A) The oxide ore is primarily reduced to iron by solid coke according to the reaction.
2Fe2O3 + 3C 4Fe + 3CO2
(B*) The oxide ore is reduced by the carbon monoxide according to the reaction
Fe2O3 + 3CO 2Fe + 3CO2
(C*) Major silica impurities are removed as calcium silicate slag by addition of a fluxing agent lime stone.
(D*) The converter slag containing phosphorus is used as a fertilizer.

Q.12

Consider the following metallurgical processes:


[4]
(I) Heating impure metal with CO and distilling the resulting volatile carbonyl (b.p. 43C) and finally
decomposing at 150200C to get the pure metal
(II) Heating the sulphide ore in air until a part is converted to oxide and then further heating in the
absence of air to let the oxide react with unchanged metal sulphide.
(III) Electrolysis of the molten electrolyte containing approximately equal amounts of the metal chloride
and NaCl to obtain the metal
The processes used for obtaining magnesium, nickel and copper are respectively:
(A) (I), (II) and (III) (B) (II), (III) and (I) (C*) (III), (I) and (II) (D) (II), (I) and (III)

Q.13

What products are formed during, the electrolysis of a concentrated aqueous solution of NaCl? [4]
(A*) Cl2 (g)
(B*) NaOH (aq)
(C*) H2 (g)
(D) None

Q.14
(A)
(B)
(C)
(D)

Column-I (Ore)
Ilmenite
Dolomite
Carnalite
Chromite

Column-II (Metal in Ore)


[12]
(P)
Iron
(Q)
Magnesium
(R)
Potassium
(S)
Titanium
[Ans. (A) P, S (B) Q (C) Q, R (D) P]

ASSIGNMENT-25

Q.1

Q.2

SnO2 is reduced to metallic Sn on smelting oxide with anthracite, limestone and sand. In this, function of
sand is :
[3]
(A) It acts as a flux
(B) It removes basic impurities as slag
(C*) Both are correct
(D) None is correct
Layer X
cool
and zinc


(Ag + Pb) alloy melt
(Ag + Pb + Zn) melt
is added
Layer Y

[3]

Select correct statement based on above scheme :


(A) Layer X contains zinc and silver
(B) Layer Y contains lead and silver but amount of silver in this layer is smaller than in the layer X.
(C) X and Y are immiscible layers
(D*) All are correct statements

AgCl + Na2CO3 Ag2CO3


X, X is :
(A) Ag2O and CO2
(B*) Ag, O2 and CO2
(C) Ag2O2 and CO2
(D) no effect

G(kJ/mol)

Q.3

Q.4

0
100
200
300
400
500
600
700
800
900
1000
1100
1200

2C +

g
2M

O2

+O 2

[3]

2C O

2M

gO

T1
T2
T3
Temperature

Incorrect statements about the plot is / are:


(A) T1 and T2 are melting point & boiling point of Mg respectively.
(B*) T1 and T2 are melting point & boiling point of MgO respectively.
(C) Reduction of MgO by coke is possible above T3
(D) Mg can be extracted from gaseous products by rapid cooling.

[3]

Comprehension (Q.5 to Q.8)


Questions given below are based on the given diagram for extractive metallurgy.
0
O
G,kJ

200

+O 2
n
Z
2

2C+O
2
2CO

400
600
800
1000

[12]

2Zn

b.p.
m.p.
O2 2
2Mg+

MgO

m.p.

b.p.

Q.5
Q.6

2000
500
1000
1500
Temperature, C
The points noted by arrows are the melting and boiling points of the metals zinc and magnesium. G as
a function of temperature for some reactions of extractive metallurgy.
At what approximate temperature, zinc and carbon have equal affinity for oxygen?
(A*) 1000C
(B) 1500C
(C) 500C
(D) 1200C

At this temperature G of the reaction is :


ZnO + C Zn + CO
(A) ve
(B) +ve

(C*) zero

(D) nothing can be said

Q.7

To make the following reduction process spontaneous, temperature should be :


ZnO + C Zn + CO
(A) < 1000C
(B*) > 1100C
(C) < 500C
(D) > 500C but < 1000C

Q.8

At 1100C, which reaction is spontaneous to a maximum extent?


(A) MgO + C Mg + CO
(B) ZnO + C Zn + CO
(C) MgO + Zn Mg + ZnO
(D*) ZnO + Mg MgO + Zn

Q.9

Give the correct order of initials T or F for following statements. Use T if statement is true and F if it is false.
(i)
Cu metal is extracted from its sulphide ore by reduction of Cu2O with FeS.
[3]
(ii)
An ore of Tin containing FeCrO4 is concentrated by magnetic separation method.
(iii)
Auto reduction process is used in the extraction of Cu & Hg.
(iv)
Cassiterite and Rutile are oxide ores of the metals.
(A) TFTT
(B) TTFT
(C*) FTTT
(D) FFFT

Q.10

Electrolyte reduction of alumina to aluminium by Hall-Heroult process is carried out:


(A) In the presence of NaCl
(B) In the presence of BaF2
(C*) In the presence of cryolite which forms a melt with lower melting temperature
(D) In the presence of cryolite which forms a melt with higher melting temperature

Q.11

During the process of electrorefining of copper some metals present as impurity settle as anode mud.
These are:
[3]
(A) Sn and Ag
(B) Pb and Zn
(C*) Ag and Au
(D) Fe and Ni

Q.12

The substance not likely to contain CaCO3 is:


(A) Sea shells
(B) Dolomite
(C) Marble statue

[3]

[3]
(D*) Calcined gypsum

Q.13

The most common elements present in the crust of the Earth are:
(A*) oxygen, silicon, aluminium
(B) oxygen, iron, magnesium
(C) silicon, iron, potassium
(D) oxygen, iron, silicon

[3]

Q.14

Extraction of silver from its ore involving NaCN, air and an active metal is known as:
(A) Pattinsons method
(B) Amalgamation method
(C*) Mc Arthur-forest method
(D) Parkes method

[3]

ASSIGNMENT-26
Q.1

Reduction of a metal oxide by excess carbon at high temperature is a method for the commercial preparation
of some metals. This method can be successfully applied in the case of
[3]
(A) BeO and Al2O3
(B*) ZnO and Fe2O3
(C) CaO and Cr2O3
(D) BaO and U3O8

Q.2

In Goldschmidt aluminothermic process, thermite mixture contains:


(A) 3 parts Fe2O3 and 2 parts Al
(B) 3 parts Al2O3 and 4 parts Al
(C) 1 part Fe2O3 and 12 part Al
(D*) 3 parts Fe2O3 and 1 part Al

[3]

Q.3

Electric furances are lined with magnesia because:


(A) It is not affected by acids
(B) It liberates oxygen on heating
(C*) It melts at very high temperature
(D) It has no effect of electricity\

[3]

Q.4

Which pair of elements can form alloy:


(A) Zn and Pb
(B) Fe and Hg

[3]
(C*) Fe and Cr

(D) C and Pt

Question No. 5 & 6 (2 questions)


Q.5

[6]

Step C (refining) involved in purification of Pb metal


(A) Distillation
(B) Bessemerization (C) Cupelation

(D*) Electrolytic refining

Q.6

Which of the following metals are obtained by auto reduction method:


Pb, Mn, Cu, Cr, Fe, Al.
(A) Cu, Fe
(B) Cu, Pb, Mn
(C) Mn, Cr, Pb
(D*) Pb, Cu

Q.7

Match List-I with List-II and select the correct answer using the codes given below the lists.
ListI (Metals)
ListII (Ores)
(a)
Tin
1.
Calamine
(b)
Zinc
2.
Cassiterite
(c)
Titanium
3.
Cerrusite
(d)
Lead
4.
Rutile
(a)
(b)
(c)
(d)
(a)
(b)
(c)
(d)
(A)
1
2
3
4
(B*) 2
1
4
3
(C)
4
3
2
1
(D)
2
1
3
4

Q.8

Match Column-I with Column-II and select the correct answer using the codes given below .
Column-I (Metals)
Column-II (Method used for refining)
(i)
Iron & copper
(P)
Poling
[3]
(ii)
Zirconium & Titanium
(Q)
Bessemerisation
(iii)
Lead & Tin
(R)
Van-Arkel
(iv)
Copper & Tin
(S)
Liquation
(A)
(B)
(C)
(D*)

(i)
P
Q
P
Q

(ii)
S
S
R
R

(iii)
R
R
S
S

(iv)
Q
P
Q
P

[3]

Q.9

On heating a mixture of Cu2O and Cu2S, we get:


(A*) Cu + SO2
(B) Cu + SO3
(C) CuO + CuS
(D) Cu2SO3

Q.10

Select incorrect statement regarding silver extraction process.


[3]
(A) When the lead-silver alloy is rich in silver, lead is removed by the cupellation process.
(B) When the lead-silver alloy is rich in lead, silver is removed by parke's or pattinson's process.
(C*) Zinc forms an alloy with lead, from which lead is separated by distillation.
(D) Zinc forms an alloy with silver, from which zinc is separated by distillation.

Q.11

Froth floatation process for the concentration of sulphide ores is an illustration of the practical application
of:
[3]
(A*) Adsorption
(B) Absorption
(C) Sedimentation
(D) Coagulation

Q.12

When ZnS and PbS minerals are present together, then NaCN is added to separate them in the froth
floatation process as a depressant, because
[3]
(A) Pb(CN)2 is precipitated while no effect on ZnS
(B*) ZnS forms soluble complex Na2[Zn(CN)4]
(C) PbS forms soluble complex Na2[Pb(CN)4]
(D) They cannot be separated by adding NaCN.

Q.13

When the sample of Cu with Zn impurity is to be purified by electrolysis, the appropriate electrodes are:
Cathode
Anode
[3]
(A)
Pure Zn
Pure Cu
(B)
Impure sample
Pure Cu
(C)
Impure Zn
Impure sample
(D*) Pure copper
Impure sample

Q.14

Match List-I (Metal) with List-II (Process of Extraction) and select the correct answer using the codes
given below the lists:
[3]
List-I (Metal)
List-II (Process of Extraction)
(a)
Aluminium
(1)
Blast furance
(b)
Iron
(2)
Mond process
(c)
Nickel
(3)
Bayer process
(d)
Copper
(4)
Cyanide process
(5)
Froth floatation
(a)
(b)
(c)
(d)
(A)
2
5
4
1
(B*) 3
1
2
5
(C)
2
1
4
5
(D)
3
5
2
1

[3]

ASSIGNMENT-27

Q.1

AgCl on fusion with Na2CO3 forms


(A) Ag2CO3
(B) Ag2O

[3]
(C*) Ag

(D) Ag2C2

Q.2
(i)
(ii)
(iii)
(iv)

Select the correct option for the given processes.


[3]
Process of heating steel to redness and then cooling it very slowly.
Process of heating steel in presence of NH3 and producing hard coating of Iron Nitride on the surface of steel.
Process of heating steel to redness and then cooling it suddenly by plunging it into water or oil.
Process of heating quenched steel to a temperature well below redness and then cooling it slowly.
(A) Tempering, Nitriding, Annealing & Quenching respectively
(B) Quenching, Nitriding, Annealing & Case Hardening respectively
(C) Tempering, Case harding, Quenching & Annealing respectively
(D*) Annealing, Nitriding, Quenching & Tempering respectively

Q.3

A solution of Na2SO4 in water is electrolysed using inert electrodes. The products at cathode and anode
are respectively
[3]
(A) O2 ; H2
(B) O2 ; Na
(C*) H2 ; O2
(D) O2 ; SO2

Q.4

If impurity in a metal has a greater affinity for oxygen, then the purification of metal may be carried out by
(A) Liquation
(B) Distillation
(C) Zone Refining
(D*) Cupellation
[3]

Q.5

Give the correct order of initials T or F for following statements. Use T if statement is true and F if it is
false.
[3]
(i)
In Gold Schmidt thermite process aluminium acts as a reducing agent.
(ii)
Mg is extracted by electrolysis of aq. solution MgCl2.
(iii)
Extraction of Pb is possible by smelting.
(iv)
Red Bauxite is purified by Serpeck's process.
(A) TTTF
(B) TFFT
(C) FTTT
(D*) TFTF

Q.6

Among the following groups of oxides, the group containing oxides that cannot be reduced by C to give
the respective metal is
[3]
(A*) CaO and K2O (B) Fe2O3 and ZnO (C) Cu2O and SnO2 (D) PbO and Pb3O4

Q.7

In the cyanide process for extraction of gold and silver from ores, the cyanide solution acts as a [3]
(A) reducing agent to reduce the gold and silver compounds present in the ores into the metallic states
(B*) leaching agent to bring the gold and silver into solution as cyanide complexes and thus separate
these metals from the ores
(C) leaching agent to dissolve all the other constituents of the ores leaving the gold and silver as metals
(D) leaching agent to bring the ores into solution.

Q.8

By which process Pb and Sn are extracted respectively are:


(A) Carbon reduction and self reduction
(B*) Self reduction and carbon reduction
(C) Electrolytic reduction and cyanide process
(D) Cyanide process and electrolytic reduction

[3]

Q.9

Match the following:


(I)
Bauxite
(II)
Monazite
(III) Malachite
(IV) Pitch blends
(A) I-d, II-b, III-c, IV-d
(C) I-c, II-a, III-b, IV-d

[3]
(a)
(b)
(c)
(d)

Copper
Uranium
Thorium
Aluminium
(B) I-b, II-c, III-d, IV-a
(D*) I-d, II-c, III-a, IV-b

Q.10

Match List-I with List-II and select the correct answer using the codes given below the lists:
List-I (Ore)
List-II (Metal)
(1)
Carnallite
(P)
Zinc
(2)
Calamine
(Q)
Titanium
(3)
IImenite
(R)
Magnesium
(4)
Chalcopyrite
(S)
Copper
(1)
(2)
(3)
(4)
(A)
P
R
Q
S
(B)
P
R
S
Q
(C)
R
P
S
Q
(D*) R
P
Q
S

[3]

Q.11

During the production of iron and steel.


[4]
(A) The oxide ore is primarily reduced to iron by solid coke according to the reaction.
2Fe2O3 + 3C 4Fe + 3CO2
(B*) The oxide ore is reduced by the carbon monoxide according to the reaction
Fe2O3 + 3CO 2Fe + 3CO2
(C*) Major silica impurities are removed as calcium silicate slag by addition of a fluxing agent lime stone.
(D*) The converter slag containing phosphorus is used as a fertilizer.

Q.12

Consider the following metallurgical processes:


[4]
(I) Heating impure metal with CO and distilling the resulting volatile carbonyl (b.p. 43C) and finally
decomposing at 150200C to get the pure metal
(II) Heating the sulphide ore in air until a part is converted to oxide and then further heating in the
absence of air to let the oxide react with unchanged metal sulphide.
(III) Electrolysis of the molten electrolyte containing approximately equal amounts of the metal chloride
and NaCl to obtain the metal
The processes used for obtaining magnesium, nickel and copper are respectively:
(A) (I), (II) and (III) (B) (II), (III) and (I) (C*) (III), (I) and (II) (D) (II), (I) and (III)

Q.13

What products are formed during, the electrolysis of a concentrated aqueous solution of NaCl? [4]
(A*) Cl2 (g)
(B*) NaOH (aq)
(C*) H2 (g)
(D) None

Q.14
(A)
(B)
(C)
(D)

Column-I (Ore)
Ilmenite
Dolomite
Carnalite
Chromite

Column-II (Metal in Ore)


[12]
(P)
Iron
(Q)
Magnesium
(R)
Potassium
(S)
Titanium
[Ans. (A) P, S (B) Q (C) Q, R (D) P]

ASSIGNMENT-25

Q.1

Q.2

SnO2 is reduced to metallic Sn on smelting oxide with anthracite, limestone and sand. In this, function of
sand is :
[3]
(A) It acts as a flux
(B) It removes basic impurities as slag
(C*) Both are correct
(D) None is correct
Layer X
cool
and zinc


(Ag + Pb) alloy melt
(Ag + Pb + Zn) melt
is added
Layer Y

[3]

Select correct statement based on above scheme :


(A) Layer X contains zinc and silver
(B) Layer Y contains lead and silver but amount of silver in this layer is smaller than in the layer X.
(C) X and Y are immiscible layers
(D*) All are correct statements

AgCl + Na2CO3 Ag2CO3


X, X is :
(A) Ag2O and CO2
(B*) Ag, O2 and CO2
(C) Ag2O2 and CO2
(D) no effect

G(kJ/mol)

Q.3

Q.4

0
100
200
300
400
500
600
700
800
900
1000
1100
1200

2C +

g
2M

O2

+O 2

[3]

2C O

2M

gO

T1
T2
T3
Temperature

Incorrect statements about the plot is / are:


(A) T1 and T2 are melting point & boiling point of Mg respectively.
(B*) T1 and T2 are melting point & boiling point of MgO respectively.
(C) Reduction of MgO by coke is possible above T3
(D) Mg can be extracted from gaseous products by rapid cooling.

[3]

Comprehension (Q.5 to Q.8)


Questions given below are based on the given diagram for extractive metallurgy.
0
O
G,kJ

200

+O 2
n
Z
2

2C+O
2
2CO

400
600
800
1000

[12]

2Zn

b.p.
m.p.
O2 2
2Mg+

MgO

m.p.

b.p.

Q.5
Q.6

2000
500
1000
1500
Temperature, C
The points noted by arrows are the melting and boiling points of the metals zinc and magnesium. G as
a function of temperature for some reactions of extractive metallurgy.
At what approximate temperature, zinc and carbon have equal affinity for oxygen?
(A*) 1000C
(B) 1500C
(C) 500C
(D) 1200C

At this temperature G of the reaction is :


ZnO + C Zn + CO
(A) ve
(B) +ve

(C*) zero

(D) nothing can be said

Q.7

To make the following reduction process spontaneous, temperature should be :


ZnO + C Zn + CO
(A) < 1000C
(B*) > 1100C
(C) < 500C
(D) > 500C but < 1000C

Q.8

At 1100C, which reaction is spontaneous to a maximum extent?


(A) MgO + C Mg + CO
(B) ZnO + C Zn + CO
(C) MgO + Zn Mg + ZnO
(D*) ZnO + Mg MgO + Zn

Q.9

Give the correct order of initials T or F for following statements. Use T if statement is true and F if it is false.
(i)
Cu metal is extracted from its sulphide ore by reduction of Cu2O with FeS.
[3]
(ii)
An ore of Tin containing FeCrO4 is concentrated by magnetic separation method.
(iii)
Auto reduction process is used in the extraction of Cu & Hg.
(iv)
Cassiterite and Rutile are oxide ores of the metals.
(A) TFTT
(B) TTFT
(C*) FTTT
(D) FFFT

Q.10

Electrolyte reduction of alumina to aluminium by Hall-Heroult process is carried out:


(A) In the presence of NaCl
(B) In the presence of BaF2
(C*) In the presence of cryolite which forms a melt with lower melting temperature
(D) In the presence of cryolite which forms a melt with higher melting temperature

Q.11

During the process of electrorefining of copper some metals present as impurity settle as anode mud.
These are:
[3]
(A) Sn and Ag
(B) Pb and Zn
(C*) Ag and Au
(D) Fe and Ni

Q.12

The substance not likely to contain CaCO3 is:


(A) Sea shells
(B) Dolomite
(C) Marble statue

[3]

[3]
(D*) Calcined gypsum

Q.13

The most common elements present in the crust of the Earth are:
(A*) oxygen, silicon, aluminium
(B) oxygen, iron, magnesium
(C) silicon, iron, potassium
(D) oxygen, iron, silicon

[3]

Q.14

Extraction of silver from its ore involving NaCN, air and an active metal is known as:
(A) Pattinsons method
(B) Amalgamation method
(C*) Mc Arthur-forest method
(D) Parkes method

[3]

ASSIGNMENT-26
Q.1

Reduction of a metal oxide by excess carbon at high temperature is a method for the commercial preparation
of some metals. This method can be successfully applied in the case of
[3]
(A) BeO and Al2O3
(B*) ZnO and Fe2O3
(C) CaO and Cr2O3
(D) BaO and U3O8

Q.2

In Goldschmidt aluminothermic process, thermite mixture contains:


(A) 3 parts Fe2O3 and 2 parts Al
(B) 3 parts Al2O3 and 4 parts Al
(C) 1 part Fe2O3 and 12 part Al
(D*) 3 parts Fe2O3 and 1 part Al

[3]

Q.3

Electric furances are lined with magnesia because:


(A) It is not affected by acids
(B) It liberates oxygen on heating
(C*) It melts at very high temperature
(D) It has no effect of electricity\

[3]

Q.4

Which pair of elements can form alloy:


(A) Zn and Pb
(B) Fe and Hg

[3]
(C*) Fe and Cr

(D) C and Pt

Question No. 5 & 6 (2 questions)


Q.5

[6]

Step C (refining) involved in purification of Pb metal


(A) Distillation
(B) Bessemerization (C) Cupelation

(D*) Electrolytic refining

Q.6

Which of the following metals are obtained by auto reduction method:


Pb, Mn, Cu, Cr, Fe, Al.
(A) Cu, Fe
(B) Cu, Pb, Mn
(C) Mn, Cr, Pb
(D*) Pb, Cu

Q.7

Match List-I with List-II and select the correct answer using the codes given below the lists.
ListI (Metals)
ListII (Ores)
(a)
Tin
1.
Calamine
(b)
Zinc
2.
Cassiterite
(c)
Titanium
3.
Cerrusite
(d)
Lead
4.
Rutile
(a)
(b)
(c)
(d)
(a)
(b)
(c)
(d)
(A)
1
2
3
4
(B*) 2
1
4
3
(C)
4
3
2
1
(D)
2
1
3
4

Q.8

Match Column-I with Column-II and select the correct answer using the codes given below .
Column-I (Metals)
Column-II (Method used for refining)
(i)
Iron & copper
(P)
Poling
[3]
(ii)
Zirconium & Titanium
(Q)
Bessemerisation
(iii)
Lead & Tin
(R)
Van-Arkel
(iv)
Copper & Tin
(S)
Liquation
(A)
(B)
(C)
(D*)

(i)
P
Q
P
Q

(ii)
S
S
R
R

(iii)
R
R
S
S

(iv)
Q
P
Q
P

[3]

Q.9

On heating a mixture of Cu2O and Cu2S, we get:


(A*) Cu + SO2
(B) Cu + SO3
(C) CuO + CuS
(D) Cu2SO3

Q.10

Select incorrect statement regarding silver extraction process.


[3]
(A) When the lead-silver alloy is rich in silver, lead is removed by the cupellation process.
(B) When the lead-silver alloy is rich in lead, silver is removed by parke's or pattinson's process.
(C*) Zinc forms an alloy with lead, from which lead is separated by distillation.
(D) Zinc forms an alloy with silver, from which zinc is separated by distillation.

Q.11

Froth floatation process for the concentration of sulphide ores is an illustration of the practical application
of:
[3]
(A*) Adsorption
(B) Absorption
(C) Sedimentation
(D) Coagulation

Q.12

When ZnS and PbS minerals are present together, then NaCN is added to separate them in the froth
floatation process as a depressant, because
[3]
(A) Pb(CN)2 is precipitated while no effect on ZnS
(B*) ZnS forms soluble complex Na2[Zn(CN)4]
(C) PbS forms soluble complex Na2[Pb(CN)4]
(D) They cannot be separated by adding NaCN.

Q.13

When the sample of Cu with Zn impurity is to be purified by electrolysis, the appropriate electrodes are:
Cathode
Anode
[3]
(A)
Pure Zn
Pure Cu
(B)
Impure sample
Pure Cu
(C)
Impure Zn
Impure sample
(D*) Pure copper
Impure sample

Q.14

Match List-I (Metal) with List-II (Process of Extraction) and select the correct answer using the codes
given below the lists:
[3]
List-I (Metal)
List-II (Process of Extraction)
(a)
Aluminium
(1)
Blast furance
(b)
Iron
(2)
Mond process
(c)
Nickel
(3)
Bayer process
(d)
Copper
(4)
Cyanide process
(5)
Froth floatation
(a)
(b)
(c)
(d)
(A)
2
5
4
1
(B*) 3
1
2
5
(C)
2
1
4
5
(D)
3
5
2
1

[3]

ASSIGNMENT-27

Q.1

AgCl on fusion with Na2CO3 forms


(A) Ag2CO3
(B) Ag2O

[3]
(C*) Ag

(D) Ag2C2

Q.2
(i)
(ii)
(iii)
(iv)

Select the correct option for the given processes.


[3]
Process of heating steel to redness and then cooling it very slowly.
Process of heating steel in presence of NH3 and producing hard coating of Iron Nitride on the surface of steel.
Process of heating steel to redness and then cooling it suddenly by plunging it into water or oil.
Process of heating quenched steel to a temperature well below redness and then cooling it slowly.
(A) Tempering, Nitriding, Annealing & Quenching respectively
(B) Quenching, Nitriding, Annealing & Case Hardening respectively
(C) Tempering, Case harding, Quenching & Annealing respectively
(D*) Annealing, Nitriding, Quenching & Tempering respectively

Q.3

A solution of Na2SO4 in water is electrolysed using inert electrodes. The products at cathode and anode
are respectively
[3]
(A) O2 ; H2
(B) O2 ; Na
(C*) H2 ; O2
(D) O2 ; SO2

Q.4

If impurity in a metal has a greater affinity for oxygen, then the purification of metal may be carried out by
(A) Liquation
(B) Distillation
(C) Zone Refining
(D*) Cupellation
[3]

Q.5

Give the correct order of initials T or F for following statements. Use T if statement is true and F if it is
false.
[3]
(i)
In Gold Schmidt thermite process aluminium acts as a reducing agent.
(ii)
Mg is extracted by electrolysis of aq. solution MgCl2.
(iii)
Extraction of Pb is possible by smelting.
(iv)
Red Bauxite is purified by Serpeck's process.
(A) TTTF
(B) TFFT
(C) FTTT
(D*) TFTF

Q.6

Among the following groups of oxides, the group containing oxides that cannot be reduced by C to give
the respective metal is
[3]
(A*) CaO and K2O (B) Fe2O3 and ZnO (C) Cu2O and SnO2 (D) PbO and Pb3O4

Q.7

In the cyanide process for extraction of gold and silver from ores, the cyanide solution acts as a [3]
(A) reducing agent to reduce the gold and silver compounds present in the ores into the metallic states
(B*) leaching agent to bring the gold and silver into solution as cyanide complexes and thus separate
these metals from the ores
(C) leaching agent to dissolve all the other constituents of the ores leaving the gold and silver as metals
(D) leaching agent to bring the ores into solution.

Q.8

By which process Pb and Sn are extracted respectively are:


(A) Carbon reduction and self reduction
(B*) Self reduction and carbon reduction
(C) Electrolytic reduction and cyanide process
(D) Cyanide process and electrolytic reduction

[3]

Q.9

Match the following:


(I)
Bauxite
(II)
Monazite
(III) Malachite
(IV) Pitch blends
(A) I-d, II-b, III-c, IV-d
(C) I-c, II-a, III-b, IV-d

[3]
(a)
(b)
(c)
(d)

Copper
Uranium
Thorium
Aluminium
(B) I-b, II-c, III-d, IV-a
(D*) I-d, II-c, III-a, IV-b

Q.10

Match List-I with List-II and select the correct answer using the codes given below the lists:
List-I (Ore)
List-II (Metal)
(1)
Carnallite
(P)
Zinc
(2)
Calamine
(Q)
Titanium
(3)
IImenite
(R)
Magnesium
(4)
Chalcopyrite
(S)
Copper
(1)
(2)
(3)
(4)
(A)
P
R
Q
S
(B)
P
R
S
Q
(C)
R
P
S
Q
(D*) R
P
Q
S

[3]

Q.11

During the production of iron and steel.


[4]
(A) The oxide ore is primarily reduced to iron by solid coke according to the reaction.
2Fe2O3 + 3C 4Fe + 3CO2
(B*) The oxide ore is reduced by the carbon monoxide according to the reaction
Fe2O3 + 3CO 2Fe + 3CO2
(C*) Major silica impurities are removed as calcium silicate slag by addition of a fluxing agent lime stone.
(D*) The converter slag containing phosphorus is used as a fertilizer.

Q.12

Consider the following metallurgical processes:


[4]
(I) Heating impure metal with CO and distilling the resulting volatile carbonyl (b.p. 43C) and finally
decomposing at 150200C to get the pure metal
(II) Heating the sulphide ore in air until a part is converted to oxide and then further heating in the
absence of air to let the oxide react with unchanged metal sulphide.
(III) Electrolysis of the molten electrolyte containing approximately equal amounts of the metal chloride
and NaCl to obtain the metal
The processes used for obtaining magnesium, nickel and copper are respectively:
(A) (I), (II) and (III) (B) (II), (III) and (I) (C*) (III), (I) and (II) (D) (II), (I) and (III)

Q.13

What products are formed during, the electrolysis of a concentrated aqueous solution of NaCl? [4]
(A*) Cl2 (g)
(B*) NaOH (aq)
(C*) H2 (g)
(D) None

Q.14
(A)
(B)
(C)
(D)

Column-I (Ore)
Ilmenite
Dolomite
Carnalite
Chromite

Column-II (Metal in Ore)


[12]
(P)
Iron
(Q)
Magnesium
(R)
Potassium
(S)
Titanium
[Ans. (A) P, S (B) Q (C) Q, R (D) P]

ASSIGNMENT-28

Solubility chart of ionic compounds in aqueous solution.


Cation

Anion
OCl , ClO 2 , ClO3 ,

Any

Solublility
All are soluble

Exception
(i) ClO 4 of K+, Rb+, Cs+
are insoluble.
(ii) AgNO2 is insoluble
(iii) CH3COOAg is
partially soluble.

All are soluble

[Pt Cl6]2 &


[Co(NO2)6]3
are insoluble
----+
(i) Ag , Pb2+, Hg 2 2 ,

ClO 4 , NO 2 , NO 3
CH3COO, HSO 3 ,

K+, Rb+,
Cs+ & NH 4
Na+
Any

HCO3 , HS
Any

Any
Cl , Br, I

All are soluble


All are soluble

Any

SO 42

All are soluble

Any

SO 32 , CO 32 ,

All are
insoluble

C 2 O 4 2 , PO 4 3 ,
O2 ,OH , F

Any
Any

CN, OCN,SCN,
S2
CrO 42 is similar to

All are
insoluble
Same as SO 42

SO 42
Any

MnO 41 is similar
to ClO 4

1.
2.

Same as ClO 4

Cu 22 are insoluble
(ii) PbCl2, CuBr2, Cu2I2
are soluble on warming
(iii) HgBr2 & HgI2
remain insoluble on
warming.
(i)Ba+2 ,Sr2+, Pb2+ are
insoluble
(ii) CaSO4, Ag2SO4 ,
SnSO4 & HgSO4 are
partially soluble
(i) BeF2 & AgF are
soluble
(ii) K+ to Cs+ & NH 4 are
soluble
(iii) O2 & OH of Sr 2
& Ba+2 are soluble
I group cation & II group
cation are soluble
Same as SO 42 but
SrCrO4 is soluble
NaMnO4, KMnO4 are
soluble

3.

All the cuprous salts are insoluble.


Sulphides are usually insoluble, leaving those of alkali metals and ammonium sulphide, which are soluble.
Sulphides of alkaline earth metals and of ammonium are decomposed in aqueous solution.
Compounds of alkali metals and ammonium salts are generally soluble.

4.

2
CO32 , OH, CrO 24 , PO 34 & SO 3 are found generally insoluble, leaving those of alkali metals and

ammonium cation, which are soluble.

ASSIGNMENT-29

COLOURS OF DIFFERENT COMPOUNDS


Black Colour Compounds
1. PbS
2. Ag2S
3. CuS
4. Cu2S
5. NiS
6. CoS
7. HgS
8. FeS
9. NiO
10. MnO
11. FeO
12. CuO
13. PbO2
14. MnO2
15. Mn3O4
16. Fe3O4
17. Co3O4
18. Ni(OH)3
19. Cu3P2
20. BiI3
21. Hg + Hg(NH2)Cl
Blue Colour Compounds
(a) Light Blue Compunds
1. Cu(OH)2
2. Cu(NO3)2
3. CuCl2
4. CuSO45H2O (Blue Vitriol)
5. Co(OH)2
(b) Deep Blue Compounds
1. [Cu(NH3)4]SO4(Switzers reagent)
2. [Cu(NH3)4](NO3)2
3. Fe4[Fe(CN)6]3 (Prussians blue)
4. Fe3[Fe(CN)6]2 (Turnbulls blue)
5. Na4[Fe(CN)5(NOS)](Violet)

Green Colour Compounds


1. Ni(OH)2 (green ppt.)
2. Hg2I2 (green ppt.)
3. Cr2O3 (green solid)
4. Cr(OH)3 (green ppt.)
5. Cr2(SO4)3
6. CrCl3
7. FeSO4 7H2O
8. FeCl2
9. FeSO4 (NH4)2SO46H2O(Mohrs salt)
10. Na2MnO4
11. K2MnO4
12. B(OC2H5)3 (Burns with green edge flame)
13. CoO ZnO (Riemanns green)
White Colour Compounds
1. AgCl
2. Ag2CO3 (white)
3. AgNO2
4. Ag2SO4
5. PbCl2
6. PbBr2 (White crystalline ppt.)
7. Cu2I2 (White ppt.)
8. PbSO4
9. SrSO4
10. BaSO4
11. Hg2SO4
12. BaCO3
13. SrCO3
14. CaCO3
15. MgCO3
16. Be(OH)2
17. Mg(OH)2
18. Ca(OH)2
19. Al(OH)3 (White gelatinous ppt.)
20. Cd(OH)2 (Dirty White)
21. Bi(OH)3 (White ppt.)
22. Zn(OH)2
23. ZnCO3
24. ZnS
25. ZnO
26. CaC2O4
27. Ag2C2O4
28. Ag2S2O3
29. PbS2O3
30. Zn2[Fe(CN)6] (white ppt.)

Yellow Coloured Compounds


1. As2S3
2. As2S5
3. CdS
4. SnS2(Artificial gold)
5. FeS2(Fools gold)
6. (NH4)2Sx (where x = 2 to 5)(YAS)
7. PbCrO4
8. BaCrO4
9. SrCrO4
10. AgBr (light yellow)
11. AgI (Dark yellow)
12. PbI2
13. PbO (in Cold)
14. ZnO (in Hot)
15. HgO (Yellow ppt.)
16. Na2O2 (Pale yellow)
17. Ag3PO4
18. Ag3AsO3
19. Cu(CN)2
20. K3[Co(NO2)6] (Fischers salt)
21. (NH4)3PO4 12MoO3
22. (NH4)3 AsO4 12 MoO3
23. Na2CrO4
24. CrO42 (Yellow in solution)
Red Colour Compounds
1. Ag2CrO4 (Brick red)
2. Hg2CrO4 (Brick red)
3. HgI2 (Scarlet red)
4. Pb3O4 (2PbO + PbO2)
5. CrO2Cl2 (Reddish brown)
6. Fe(CH3COO)3 (Blood red)
7. Fe(SCN)3 (Blood red)
8. AsI3
9. SbI3
10. SnI2
11. CuBr2
12. [Ni(DMG)2] (Rosy red)

Brown Coloured Compounds


1. SnS
2. Bi2S3
3. CdO
4. PbO2
5. Fe(OH)3 (Reddish Brown)
6. Fe2O3 (Reddish Brown solid)
7. Fe2(CO3)3
8. Cu2O (Reddish Brown)
9. Ag3AsO4 (Reddish Brown)
10. O

NH2I

11. Cu2I2 + I3 (Brown ppt.)


12. Cu2[Fe(CN)6] (Chocolate brown)
13. NO2 (Brown gas)
14. [Fe(H2O)5 (NO)]SO4 (Brown ring)
Orange Coloured Compounds
1. Sb2S3
2. Sb2S5
3. KO3
4. CsO2
5. Cr2O 72 (Orange in aq. solution)
Pink Coloured Compounds
1. Mn(OH)2
2. MnS
3. MnO 4 (Pink or purple in aq. solution)
4. CO(CN)2
5. (NH4)2 SnCl6
6.COCl26H2O (Pink on melting black also known as
sympathetic ink)
Gas
1. Colourless odourless gas CO2, N2, O2
2. Colourless gas having pungent smell
NH3, H2S, SO2
3. Coloured gas Cl2 (Yellowish green)
Br2 (Brown)
NO2 (Brown)
I2 (Violet fumes)

ASSIGNMENT-30

Choose the correct option. (Only one option is correct):


Q.1
Which of the following pair of compounds cannot co-exist in aqueous solution.
(A) Na2CO3, K2CO3 (B*) NaHCO3, NaOH (C) Rb2CO3, KHCO3 (D) NaOH and KCl
Q.2

Q.3

CO 32 and SO 32 cannot be distinguished by


(A) H2O2 solution
(B) lime water

(C) Cl2 water

(D*) all of these

Ag2CO3
A & B are
(A) dil. HNO3 and boiling in water respectively.
(B*) Direct heating and boiling in water respectively.
(C) dil. HNO3 and NH3 respectively.
(D) hot water and NH3 respectively.

Q.4

Calcium hydride, also known as hydrolith, is hydrolysed by water to produce


(A*) Ca(OH)2 + H2 (B) CaO + O2
(C) CaO + H2
(D) does not hydrolyse

Q.5

Which of the following options is correct


I.

HgCl2 can be used for the distinction of HCO 3 and CO 32

II.
Bisulphite solution is neutral towards litmus paper and pH is 7
III.
Aq. suspension of Ag2CO3 and Ag2SO3 both produces Ag2O on heating
IV.
SO2 acts as reducing agent when passed through FeCl3 solution
(A) FFTT
(B) TFTT
(C*) TTFT
(D) FTFF
Q.6

Q.7

Which of the following produces colourless gas with dil. H2SO4


(A) Na2S2O3
(B) NaHCO3
(C) CH3CO2NH4

(D*) All

Which of the following pair of salt produces odourless gas with dil. H2SO4
(A) HCO 3 and HSO 3

(B*) HCO 3 and CO 32

(C) S2O32 and CH 3CO 2

(D) CO 32 and CH 3CO 2

Q.8

Na2SO3 and Na2S2O3 cannot be distinguished by


(A) addition of Pb(OAc)2 followed by heating
(B) addition CaCl2 solution
(C*) addition of AgNO3 followed by heating
(D) All of these

Q.9

The colour of KMnO4/Hwill be decolourised by


(A) S2 solution

(B) SO 32 solution

(C*) both (A) and (B) (D) None

Q.10

Comment True (T) or False (F) for the following statements.


(I)
Same gas comes out when Na2S is treated with dil. HCl and Na2SO3 is treated with
(Zn + dil. H2SO4)
(II)
Same observation when acidified nitrite solution is treated with urea or thiourea followed by
addition of FeCl3 solution.
(III) Same observation when nitrite solution or acetate solution is treated with dil. H2SO4 solution
(IV) With CaCl2 and BaCl2 both acetate and formate do not react.
(A) FTTF
(B) TFTT
(C*) TFFT
(D) FTTT
Question No. 11 to 15 (5 questions)
Questions given below consist of two statements each printed as Assertion (A) and Reason (R); while
answering these questions you are required to choose any one of the following four responses:
(A) if both (A) and (R) are true and (R) is the correct explanation of (A)
(B) if both (A) and (R) are true but (R) is not correct explanation of (A)
(C) if (A) is true but (R) is false
(D) if (A) is false and (R) is true

Q.11

Q.12
Q.13

Q.14

Q.15

Assertion : Initially there is no ppt. when AgNO3 is added to Na2SO3 solution


Reason: Localised formation of Ag2SO3 is destroyed by soluble complex formation like [AgSO3]
[Ans. A]
Assertion : NaHCO3 is the least soluble alkali bicarbonate.
Reason : massive H-bonding present in solid NaHCO3.
[Ans. A]
Assertion : LiHCO3 cannot exist in solid form.
Reason : Li2O has the highest thermal stability among the alkali metals oxide.

[Ans. B]

Assertion : CO2 + K2Cr2O7 no reaction


Reason : C is already in maximum oxidation state

[Ans. A]

Assertion : When HCO 3 and CO 32 ions are present together, CaCl2 is to be used in excess to
distinguished both
Reason : Excess amount of CaCl2 forms soluble complex with HCO 3 of the formula of
[Ca (HCO 3 )6 ]4

[Ans. C]

ASSIGNMENT-31

Choose the correct option. (Only one option is correct):


Q.1
Soda extract is useful when given mixture has any insoluble salt, it is prepared by
(A) Fusing soda and mixture and then extracting with water
(B) Dissolving NaHCO3 and mixture in dil HCl
(C) Boiling Na2CO3 and mixture in dil HCl
(D*) Boiling Na2CO3 and mixture in distilled water.
Q.2

Which of the following salts will not produce any observable changes when H2S is passed through its
aqueous solution
(A) (CH3CO2)2Pb
(B*) Na2[Fe(CN)5NO] (C) AgNO3
(D) none of these

Q.3

Choose the correct option for the following statement


I.
SnCl4 gives the chromyl chloride test
II.
Layer test is not applicable for chloride ion
III.
In the layer test, if red colour comes first, it is confirmed that iodide is absent
IV.

For the removal of I from a mixture of I + NO3 , AgNO3 can be used and then NO3 can be

tested
(A*) FTTF

(B) FTFT

(C) FFTF

(D) TTTF

Q.4

In the K2Cr2O7 solution when alkali solution of BaCl2 is added, the yellow ppt. obtained is of
(A) BaCr2O7
(B*) BaCrO4
(C) BaCrO42H2O
(D) none

Q.5

Aq . solution of a yellow substance (A)

KI

conc . solution

Compound (A) is
(A) PbI2
Q.6

Q.7

Q.8

NH Cl

colourless solution(B) 4Brown ppt .

(B) AgI

solution added

(made alk .)

(C*) HgI2

(D) Hg2I2

(X) KOH
(Y) (gas turns red litmus blue) +(Z) Zn
KOH
(Y) (gas)

(X)
gas (supports in combustion)
Identify (X) to (Z).
(A) X = NH4NO2
Y = NH3

Z = KNO2

(B) X = (NH4)2Cr2O7

Y = NH3

Z = Cr2O3

(C) X = (NH4)2SO4

Y = NH3

Z = K2SO4

(D*) X = NH4NO3
Y = NH3
Z = KNO3
The co-ordination number of central ion of the complex obtained in the Na-nitroprusite test of sulphide
ion is
(A) 5
(B*) 6
(C) 7
(D) 4
BeC2O4 and BaC2O4 are heated separately with bunsen burner. The solid residue obtained are
respectively
(A) BeO and BaO
(B*) BeO and BaCO3 (C) BeCO3 and BaO (D) BeCO3 and BaCO3

Q.9

Unknown salt 'A' + solid K2Cr2O7 + conc. H2SO4 Reddish brown fumes.
Which is the correct statement regarding the above observation
(A) It confirms the presence of Cl ion
(B) It confirms the presence of Br ion
(C) It confirms the presence of both
(D*) It neither confirms Cl nor Br unless it is passed through NaOH solution
Question No. 10 to 12 (3 questions)
Questions given below consist of two statements each printed as Assertion (A) and Reason (R); while
answering these questions you are required to choose any one of the following four responses:
(A) if both (A) and (R) are true and (R) is the correct explanation of (A)
(B) if both (A) and (R) are true but (R) is not correct explanation of (A)
(C) if (A) is true but (R) is false
(D) if (A) is false and (R) is true

Q.10

(A) NO3 and NO 2 both do not give brown fumes with dil. H2SO4
(R) Protonation of NO 2 is more easier compared to NO3 since NO3 is more stable by resonance
[Ans. D]

Q.11

Q.12

Q.13

(A) Oxalate evolves gas with conc. H2SO4 in presence of MnO2


(R) MnO2 acts as catalyst over here

[Ans. C]

(A) In the brown ring compound, Fe is in the +1 oxidation state


(R) Spin only magnetic moment found for this compound is 3.87 BM

[Ans. A]

Question No. 13 to 15( 3 questions)


Acetic acid is added to the solution of sodium carbonate the gas evolved does not turn purple colour of
KMnO4 but turns lime water milky forming a compound (M) which becomes soluble by passing the
same gas in excess forming another compound (N). But same observation is not obtained with boric
acid
Purple colour of KMnO4 has not changed because
(A) the gas has no oxidising property since central atom is with minimum oxidation state
(B*) the gas has no reducing property since the central atom is with maximum oxidation state
(C) the gas has no precipitation characteristics
(D) the gas precipitates CaCO3 from lime water

Q.14

The compound formed in the above sequence (M) and (N) are respectively
(A) water soluble CaCO3 and water soluble Ca(HCO3)2
(B) water insoluble CaCO3 and water insoluble Ca(HCO3)2
(C*) water insoluble CaCO3 and water soluble Ca(HCO3)2
(D) water soluble CaCO3 and water insoluble Ca(HCO3)2

Q.15

The correct order of increasing acidity is


(A) Boric acid < Acetic acid < Carbonic acid (B) Acetic acid < Boric acid < Carbonic acid
(C) Carbonic acid < Acetic acid < Boric acid (D*) Boric acid < Carbonic acid < Acetic acid

ASSIGNMENT-32
Choose the correct option. (Only one option is correct):
Q.1
In the reaction sequence:
lead
Cl
O 2 so ln
CrCl3 NH

(C). In this reaction sequence compound C is


4 (A) Na
2

(B) acetate
NH 4OH

(A) Na2CrO4
Q.2

Q.5
(i)
(ii)
(iii)
(iv)

Q.6

Q.7

(D*) PbCrO4

(B) SO 24

(C*) NO3

(D) CO 32

A sodium salt on treatment with MgCl2 gives white precipitate only on heating. The anion of the sodium
salt is:
(A*) HCO 3

Q.4

(C) Cr(OH)3

Which is not easily precipitated from aqueous solution


(A) Cl

Q.3

(B) Na2Cr2O7

(B) CO 32

Which reaction is possible


(A) KMnO4 + Na2SO4
(C) SrSO4 + Ni(NO3)2

(C) NO3

(D) SO 24

(B) BaSO4 + KCl


(D*) ZnSO4 + BaS

Give the correct order of initials T or F for following statements. Use T if statement is true and F if it is
false.
Cu+ undergoes disproportionation to Cu and Cu2+ in aq. solution
Hg2Cl2 does not impart chromyl chloride test
[Fe(H2O)5NO]2+ complex is highly unstable due to the presence of weak field ligand
Bond length of CO+ is greater than CO
(A) TFFT
(B) TTFT
(C) FTTF
(D*) TTFF
Fe2+ and Fe3+ can be distinguished by
(A) K3[Fe(CN)6]
(B) K4[Fe(CN)6]

(C) KSCN

CuO
B + C(g) + H O
2O
Mg3N2 H

A(gas)
2

C(g) can be obtained by heating


(A) (NH4)2SO4
(B*) (NH4)2Cr2O7
(C) NH4NO3

(D*) All

(D) NH4Cl

Q.8

When K2Cr2O7 is treated with H2O2 in acidic medium, in presence and in absence of org. solvent. The
oxidation state of chromium finally:
(A) increases and decreases respectively
(B) decreases and increases respectively
(C*) retained same and decreases respectively (D) retained same in both cases.

Q.9

Which of the following acid radical does not produce white ppt with Pb(OAc)2 solution.
(A) Br

Q.10

(B) S2O32

(C) CO 32

(D*) None of these

(C) HgCl2 solution

(D*) All of these

CO 32 and HCO 3 can be distinguished by

(A) Phenolphthalein

(B) BaCl2 solution

Q.11
Q.12

H2S is passed into BaCl2 solution. The ppt formed is


(A) BaS
(B) Ba(SH)2
(C) Ba(OH)2
Colour obtained in test of S2O32 + [Ni(en)3] NO3 is
(A) Red

Q.13

(D*) no ppt.

(B*) Violet

(C) Indigo

Which of following is / are reduced by thiosulphate solution.


(I) Fe+3 solution
(II) I2 solution
(III) Cu+2 solution
(A*) I, II, III only
(B) I, III only
(C) IV only

(D) Yellow

(IV) Hg2+
(D) I, III, IV only

Question No. 14 to 15 (2 questions)

Aq. solution of A

on
Solid A
anh. A
heating

Q.14

Q.15

A is
(A) K2S2O3

(B) CaS2O3

(C) PbS2O3

(D*) Na2S2O3

A on strong heating produces compound(s) has/have


(A) chain structure
(B) Tetrahedral structure
(C*) both
(D) none

ASSIGNMENT-33

Choose the correct option. (Only one option is correct):


Q.1

Na2S2O3. 5H2O is heated strongly to produce M and N and water. Both are consisting of S. Mention
the average oxidation state of 'S' in M and N respectively.
(A) 2, +5

Q.2

Q.3

(B) 2 ,+ 4

2
(C*) , +6
5

(D) 2, +6

A compound (X) on decomposition gives a colourless gas. The residue is dissolved in water to obtain
(Y). Excess CO2 is bubbled through aqueous solution of (Y) and (Z) is formed. (Z) are gentle heating
gives back (X). The (X) is
(A*) CaCO3
(B) Ca(HCO3)2
(C) NaHCO3
(D) Na2CO3
Na 2 C 2O 4 Solution

?
BaCO3(s) + AcOH

Comment on the product of this reaction.


(A) BaCO3 remains unaffected.
(B) BaC2O4 will be precipitated as white precipitate
(C) Ba(OAc)2 will be precipitated as white precipitate
(D*) Clear solution
Q.4

Statement-1: Brown ring test can be done for NO3 in presence of NO 2


Statement-2: Oxidation state of iron is changing from +2 to +1 in the brown ring complex.
(A) Statement-1 is true, statement-2 is true and statement-2 is correct explanation for statement-1.
(B) Statement-1 is true, statement-2 is true and statement-2 is NOT the correct explanation for statement-1.
(C) Statement-1 is true, statement-2 is false.
(D*) Statement-1 is false, statement-2 is true.

Q.5

Statement-1: NH 4 and K+ cations can be distinguished by using Nesseler's reagent followed by


adding HClO4.
Statement-2: NH 4 gives brown ppt. with Nesseler's reagent where as K+ forms white ppt. of KClO4
with perchloric acid.
(A*) Statement-1 is true, statement-2 is true and statement-2 is correct explanation for statement-1.
(B) Statement-1 is true, statement-2 is true and statement-2 is NOT the correct explanation for statement-1.
(C*) Statement-1 is false, statement-2 is true.
(D) Statement-1 is true, statement-2 is false.

Question No. 6 & 7 (2 questions)

Q.6

Q.7

Water of crystallisation in compound P


(A*) 4 moles
(B) 7 moles

(C) 10 moles

(D) 5 moles

Blue compound Q is
(A) Co(BO2)2

(C) CoOAl2O3

(D) CoOMgO

Compound (D) + I + H+ Gas


Evolved gas is similar to
(A) Gas-B
(B) Gas-G

(C*) Gas-H

(D) None

Yellow ppt. of compound (I) is insoluble in


(A) NaOH
(B*) CH3COOH

(C) dil. HNO3

(D) None

Type of hybridization of complex (E)


(A*) sp3d2
(B) d2sp3

(C) sp3

(D) dsp2

(B*) NaCoPO4

Question No. 8 to 11 (4 questions)

Q.8

Q.9

Q.10

Q.11

Q.12

Type of hybridization of central atom of gas (B)


(A) sp
(B*) sp2
(C) sp3

(D) No hybridization

Match the column-I with column-II.


Note that column-I may have more than one matching options in column-II.
Column - I
Column -II
(A)
Sodium nitroprusside
(P)
= 0 B.M.
(B)
Brown ring complex
(Q)
octahedral
(C)
(D)

Complex of Ag formed during its extraction


Potassium ferrocyanide

(R)
= 15 B.M.
(S)
NO+ ligand
[Ans. (A) P,Q,S, (B) Q,R,S, (C) P, (D) P,Q]

ASSIGNMENT-34

Q.1

Q.2

Choose the correct option. (Only one option is correct):


Match the following list and choose the correct option.
List A
List B
(I)
HgI2
(a)
Yellow ppt obtained when H2S is passed in its dil. HCl solution
(II)
Na+
(b)
Yellow solution
(III) FeCl3
(c)
Yellow flame
(IV) As2O3
(d)
Yellow sublimate
(A) I-(a), II-(c), III-(d), IV-(b)
(B*) I-(d), II-(c), III-(b), IV-(a)
(C) II-(d), III-(c), I-(b), IV-(a)
(D) II-(d), I-(c), III-(b), IV-(a)

A (white substance)
swells up first strong
contracted into amorphous powder..
heating

A is
(A) Na2B4O7.10H2O
(C*) K2SO4.Al2(SO4)3.24H2O

(B) Na2B4O7
(D) Na(NH4)HPO4.4H2O

Q.3

Aqueous solution of A + AcOH + K2CrO4 yellow ppt. The above information is not correct for
which of the following cations:
(I) Pb+2
(II) Ba2+
(III) Ca2+
(IV) Sr2+
(A) I & III
(B) I, II & IV
(C) I, II
(D*) III, IV

Q.4

Calcium imide on hydrolysis will give gas (B) which on oxidation by bleaching powder gives gas (C) gas
(C) on reaction with magnesium give compound (D). (D) on hydrolysis gives again gas (B). (B), (C) and
(D) are
(A*) NH3, N2, Mg3N2
(B) N2, NH3, MgNH
(C) N2 , N2O5 , Mg(NO3)2
(D) NH3, NO2 , Mg(NO2)2

Q.5

The compound present in brorax bead is


(A) B2O3
(B) NaBO2

(C) NaBO3

(D*) NaBO2 + B2O3

Q.6

When NH4OH is added in Hg2(NO3)2 solution, the ppt formed is


(A) Hg2O
(B) Hg + HgO
(C*) 2Hg + HgOHg(NH2)NO3
(D) HgOHg(NH2)NO3

Q.7

A reddish pink substance on heating gives off a vapour which condenses on the sides of the test tube and
the substance turns blue. It on cooling water is added to the residue it turns to its original colour. The
substance is
(A) Iodine crystals
(B) Copper sulphate crystals
(C*) Cobalt chloride crystals
(D) Zinc oxide

Q.8

Brown ppt. (A) dissolve in HNO3 gives (B) which gives white ppt (C) with NH4OH. (C) on reaction
with HCl gives solution (D) which gives white turbidity on addition of water. What is (D).
(A*) BiCl3
(B) Bi(OH)3
(C) BiOCl
(D) Bi(NO3)3

Q.9

When conc. H2SO4 was treated with K4[Fe(CN)6], CO gas was evolved. By mistake, somebody used
dilute H2SO4 instead of conc. H2SO4, then the gas evolved was
(A) CO
(B*) HCN
(C) N2
(D) CO2

Q.10

An inorganic Red colour compound (A) on heating, gives a compound (B) and a gas (C). (A) on
treatment with conc. HNO3, gives compound (D), brown colour substance (E) and a neutral oxide (F).
Compound (D) on warming gives off again gas (C). Then, (E) will be:
(A) Mn3O4
(B*) PbO2
(C) Pb3O4
(D) Fe2O3

Q.11

In this sequence X, Y, Z are respectively


(A) Acidified H2O2 ; Alkaline H2O2 ; Acidified H2O2
(B) Alkaline H2O2 ; Acidified H2O2 : Zn / HCl
(C) Acidified H2O2 ; Heat ; Alkaline H2O2
(D*) Alkaline H2O2 ; Acidified H2O2 ; On standing

PHYSICAL CHEMISTRY
ASSIGNMENTS

ASSIGNMENT-1
Q.1

Q.2

Q.3

Q.4

Q.5

50 gm ethylamine reacts with 60 gm Na NO 2 & 30 gm HCl. Where N is N15 (isotope). After the
completion of reaction, gas is evolved. Then select correct option.
[3]
(A) Pure 10 gm N2 evolved out
(B) 10 gm of hybrid N2 evolved out
(C) 5 gm C2H5OH is formed
(D*) None of these
Which of the following mixtures can be regarded as buffer ?
[3]
(A) 500 ml of 0.2 M AcOH + 1000 ml 0.2 M NaOH
(B*) 500 ml of 0.2 M AcOH + 300 mL of 0.2 M NaOH
(C*) 0.2 M Na2B4O7 10H2O
(D) 500 ml of 0.2M AcOH + 500 ml 0.2 M NaOH
If the value of Kp for the reaction
[5]
2H2S(g)
2H2(g) + S2(g) is 50% of total pressure at equilibrium than show that degree of
dissociation of H2S will be less than 66.6% but more than 50%.
100 gm Ca2B6O11 is mixed with 50gm Na2CO3. The resultant is filtered and washed. The mother liquor
collected and CO2 gas is passed through it. Find out the total weight of sodium compound formed in the
solution.
[7]
[Ans. 71 gm ]
Separate the extensive and intensive variables enthalpy, refractive index, density, visocity, volume, pressure,
dipolemoment, heat capacity, elevation in BP, chemical potential universal gas constant R, molar volume,
vapor pressure.
[5]
[Ans. Intensive : ref index, density, visocity, pressure, dipole, elevation in boiling point chemical point, R,
molar vol, vapor pressure ]

Q.6

FeS2 on oxidation by air yields Fe2O3 and SO2. If the equivalent of oxygen used in the reaction is X than
how many equivalent of Fe2O3 and SO2 are formed after the reaction
[5]
[Ans. 10x/11 ]

Q.7

Certain amount of ion X reacts with cyclopentadiene in basic solution and forms complex Y. X is obtained
by the reaction of FeCl3 with Na2S2O3. If 100 gm of ion X reacts with 200 gm of cyclopentadiene in
basic solution. How much Y is obtained if reaction efficiency is 63 %. Also comment in the symmetry of
Y & its stability.
[7]
[Ans. 0.9689 mol. ]

Q.8

A current of 1.7 A is passed through 300 mL of 0.160 M solution of ZnSO4 for 230 sec. with current
efficiency of 90%. Find the molarity of Zn2+ after the deposition of Zn. Assume volume remains constant.
[5]
[Ans. 0.154 M]

Q.9

How much amount of Fe0.93 O1.00 is completely oxidized by 50 ml 0.02 M Ba(MnO4)2 in presence of
non oxidising acid.
[5]
[Ans. 0.86 gms ]
Q.10 Consider the following reaction,
As2S3 + NO3 AsO43 + SO42 + NO
If number of equivalent of NO3 used is X. Calculate the number of equivalent of AsO43 & SO42
formed.
[5]
[Ans. 24x/28 ]

ASSIGNMENT-2

Q.1

Which of the following statement are true.


(A) CV is independent of temperature for a perfect gas.
(B) It neither heat nor matter can enter or leave a system, that system must be isolated.

[3]

(C*) dF = 0 where F = E, H, G, A, S, C
(D) A process in which final temperature equals to initial temperature must be an isothermal process.
Q.2

Predict the sign of q, w, U & H for given process.

[2 7 = 14]
Sign of

Process

Q.3

Q.4

Remark
q

DU

DH

Melting of solid benzene at 1 atm and


normal melting point.

>0

<0

>0

>0

Melting of ice at 1 atm and 0C.

>0

>0

>0

>0

Adiabatic expansion of one mole of ideal gas.

<0

<0

<0

Adiabatic expansion of ideal gas into vacuum.

Isothermal expansion of a perfect gas.

>0

<0

Heating of perfect gas at constant P.

>0

<0

>0

>0

Cooling of perfect gas at constant volume.

<0

<0

<0

Which of the statement are true.

[5]

E
V
(A*) CP CV = V P T
T
P

H P
(B*) CP CV = V P T

T V

(C*) CP CV = nR for gases only

H
(D*) V = 0 for ideal gas

Select the False statement


(A*) E = q + w for every thermodynamic system at rest in the absence of external field.
(B*) q = 0 for every cyclic process
(C) In isothermal free expansion q = 0, w = 0, E = 0, H = 0

H
(D) P = Vm where Vm is molar volume of solid, liquid or gas.

[5]

Q.5

Which of following statement are true.


[5]
(A) (w)rev (w)irrev > 0 for isothermal expansion.
(B) Heat absorbed in isothermal reversible process is greater than isothermal irreversible process.
(C) (w)irev > (w) rev for isothermal compression.
(D) Heat rejected in isothermal process is greater in (n1)th stage as compare to nth stage work.

Q.6

Two moles of an ideal monoatomic gas undergoes cyclic process ABCA as shown in figure. Find the
ratio of temperature at B & A.
[5]
[Ans. 27/4]
B

3P0

P0

V0

60

30

C
6V0

Q.7

The average degree of freedom per molecule for a gas is 6. The gas perform 25 J of work when it
expands at constant pressure. Find the heat absorbed by the gas.
[5]
[Ans. 100 J]

Q.8

A transition metal X forms an oxide of formula X2O3. It is found that only 50% of X atoms in this
compound are in +3 oxidation state. The only other stable oxidation state of X are +2 and +5. What %
of X atoms are in the +2 oxidation state in this compound.
[5]

ASSIGNMENT-3
Q.1

Q.2

Q.3

Q.4

Q.5

The minimum work which must be done to compress 16 gm of oxygen at 300 K from a pressure of
1.01325 103 N/m2 to 1.01325 105 N/m2 is :
[3]
(A*) 5744 J
(B) 8622 J
(C) 3872 J
(D) 7963 J
An ideal gaseous sample at initial state i (P0, V0, T0 ) is allowed to expand to volume 2V0 using two
different process, in the first process the equation of process is 2PV2 = K1 and in second process the
equation of the process is PV = K2. Then
[3]
(A) Work done in first process will be greater than work in second process (magnitude wise)
(B) The order of values of work done cannot be compared unless we know the value of K1 and K2
(C*) Value of work done (magnitude) in second process is greater in above expansion irrespective of
the value of K1 and K2.
(D) 1st process is not possible.
Select the correct statement.
[3]
(A*) When equilibrium is established macroscopic properties do not change with time.
(B*) At equilibrium the system should be at rest without the assistance of any external source.
(C) Steady state and equilibrium state is same
(D*) Steady state is dynamic on macroscopic as well as microscopic level but equilibrium is dynamic on
microscopic level only.
Internal energy includes.
[3]
(A*) Rotational energy (B*) Bond energy
(C*) Relativistic energy (D*) Vibrational energy
In the PV-diagram shown in figure, ABC is a semicircle. Find the work
done in the process ABC. Where P is in atm and V in litres.
[4]

3
P

B
A

Q.6

1
2
V
[Ans. /2 ]
PV graph for an ideal gas undergoing polytropic process PVm = constant is shown is here. Find the
value of m.
[4]

P
5

210 (Pa)

[Ans. 3/2 ]
37
5

410 (m )

Q.7

A portion of helium gas in a vertical cylindrical container is in thermodynamic equilibrium with the
surroundings. The gas is confined by a movable heavy piston. The piston is slowly elevated by a distance
H from its equilibrium position and then kept in the elevated position long enough for the thermodynamic
equilibrium to be reestablished. After that, the container is insulated and then the piston is released. After
the piston comes to rest, what is the new equilibrium position of the piston with respect to initial position?
[Ans. The piston ends up 0.4 H above its initial position.][4]
Q.8 An ideal gas has a molar heat capacity CV at constant volume. What is molar capacity of this gas in a
function of volume if gas undergoes process T = T0eV.
[4]
Q.9 A 10 g mixture of isobutene and isobutane requires 20 g of Br2 (in CCl4) for complete addition. If 10 g
of the mixture is catalytically hydrogenated and the entire alkane is monobrominated in the presence of
light at 127C, which exclusive product and how much of it would be formed? (Atomic weight of
bromine= 80)
[Ans. 24.51 g C4H9Br]
[4]
Q.10 How much (in gms) of AlCl3 (solid) must be added to 250 ml 0.01 M HCl solution so that Cl ion
concentration is increased to 220 %.
[4]
[Ans. 0.1335 gms.]

ASSIGNMENT-4

Explain
Q.1 How was the absolute zero of temperature determined ?
Q.2 Can a tyre be inflated without a rise in temperature, If yes, how ?
Q.1

2 mol of 4 atoms combine to form 1 mol of H2 molecule. If the internal energy change for the process is
432.6 kJ. What will be the theoretical decrease in mass for the process?
[3]
2
12
12
6
(A) 3 10 kg.
(B*) 4.8 10 kg (C) 1.4 10 kg (D) 1.4 10 kg
Comprehension
Consider the system
CH4(g) +
Rigid impermeable
cylinder

2O2(g)
CO2(g)+2H2O(g)

C4H10(g)
13
+ O2(g)
2
4CO2(g)+5H2O(g)
Massless Frictonless
Adiabatic piston

Q.2

Q.3

The temperature is maintained at 400 K and the complete combustion of methane and butane with
requisite amount of O2 is done., as shown. If initially the piston was present at the centre of the cylinder
shown above. Given : Total volume of cylinder = 10 litre R = 8.3 mol/k.
Calculate the final position of the piston after the combustion has taken place, if c/s area of
cylinder = 1 dm2.
[3]
(A) 25 cm from Right (B) 75 cm from Left (C*) 75 cm from Right (D) 125 cm from Left
Calculate the net work done during the process ?
(A) + 6.9 kJ
(B) 12.11 kJ
(C*) 5.21 kJ

[3]
(D) + 0.42 kJ

5
R is in thermal contact with a
2
water bath. Treat the vessel, gas and water bath as being in thermal equilibrium initially at 298 k, and as
separated by adiabatic walls from the rest of the universe. The vessel gas and water bath have an
average heat capacity of Cp = 7500 J/K. The gas is compressed reversibly to Pf = 10.5 bar. What is the
temperature of the system after thermal equilibrium has been established?
[3]
(A*) 299.2 K.
(B) 763.2 K.
(C) 479.2 K
(D) 182.9 K

Q.4

A vessel containing 1 mol of an ideal gas with Pi = 1 bar and Cpm =

Q.5

Which of the following can be considered true in regard to quasi-static process?


[5]
(A*) An example of quasi-static process can be the movement of heat from hot body to cold body
separated through an almost vacuum.
(B*) The system undergoes a major change in terms of a directed path along a sequence of states in
which the system and surrounding are in internal equilibrium.
(C*) Reversible and irreversible processes are two classes of quasi-static process
(D)
A quasi-static process can only be plotted on a P-V-T curve.

Q.6

A bottle at 21C contains an ideal gas at a pressure of 126.4 103 Pa. The rubber stopper closing the
bottle is removed. The gas expands adiabatically against Pexternal = 101.9 103 Pa and some gas is
expelled from the bottle in the process when P = Pexternal , the stopper is quickly replaced. The gas
remaining in the bottle slowly warms up to 21C what is the final pressure[in Pa] in the bottle for a
monoatomic gas.
[3]
[Ans. 110.5 103 Pa.]

Q.7

If a monoatomic ideal gas undergoes the cyclic process as shown below.


P

[5]

1
2
4
3
V

(a)

V3
Calculate the efficiency of this cycle if V = 2.
1

(b)

Plot the same cycle on PT and VT graph.

[Ans. (a) 68.35 %]

Q.8

One mole of N2 in a stale defined by Ti = 300 k and Vi = 2.5 litre undergoes an isothermal reversible
expansion until Vf = 23 litre, What is the percentage error, in calculation of work, using the ideal gas
equation instead of the Vander Waals equation. Given a = 1.370 dm6 bar mol2, b = 0.0387 dm3mol1.
[5]
[Ans 0.25%]

Q.9

Regard H as a function of T and P use the Cyclic rule to obtain the expression.

H
Cp

P T

[3]

T

P H

C
Q.10 U is a state function and obeys Eulers reciprocity theorem. Using this statement show that v 0
V T
for an ideal gas.
[3]

Q.1

Q.2

Q.3

Q.4

Q.5

Q.6
Q.7

ASSIGNMENT-5
For a perfectly crystalline solid Cp,m = aT3, where a is constant. If Cp,m is 0.42 J/K-mol at 10 K, molar
entropy at 10 K is
[3]
(A) 0.42 J/K-mol
(B*) 0.14 J/K-mol
(C) 4.2 J/K-mol
(D) zero
Entropy change for process at 1 atm and 10C, H 2 O (l ) H 2 O (s) is
[Given: Hfusion(0C) = 6000 J/mol, Hfusion(10C) = 5523 J/mol

[3]

273
Cp,m(H2O, s) = 36 J/K-mol, Cp,m(H2O, l) = 75 J/K-mol, ln
= 0.04]
263
(A) 23.47 J/K-mol
(B*) 20.41 J/K-mol
(C) 21 J/K-mol
(D) 20.47 J/K-mol
Select the correct statement.
[3]
(A) Entropy of XeF5+ is maximum when structure is square pyramidal (perfect)
(B) Entropy of XeF5+ is maximum when structure is distorted square pyramidal.
(C) Entropy of XeF5+ is maximum when structure is distorted octahedral.
(D) Entropy of XeF5+ is greater in distorted octahedral as compared to distorted square pyramidal.
Select the correct statement.
[3]
(A) Every system which is left to itself, will on average change towards condition of maximum probability.
(B) Gain in information is loss in entropy.
(C) On increasing temperature entropy will always increases.
(D) At 0 K only one micro state is possible.
A diatomic gas is used in a carnot engine as a working substance. In an adiabatic expansion, volume of
gas increases two times then what is the percentage efficiency of carnot engine?
[4]
[Ans. 0024]
3
2
A diatomic gas (ideal) undergoes change according to law T V = constant from initial volume 2.5 litres
to 4.5 litres at initial temperature T1 = 300K. Find out entropy change in the system.
[4]
Two identical bodies of constant heat capacity one at same initial temperature. A refrigerator operates
between these two bodies until one is cooled to temperature T2. if the bodies remains at constant
pressure and undergoes no change in phase. Then calculate the minimum work needed.
[4]

T2

W = CP 1 T2 2T1
T2

Q.8 An ideal gas undergoes change such that pressure drop to 4 times and volume increases to 2 times. Then
calculate the entropy change in system during the process.
[4]
Q.9 The expansion process of n = 2 moles of argon proceeds so that gas pressure increases in direct proportion
to it volume. Find the entropy increment of the gas in this process provided its volume increases two
times.
[4]
Q.10 The enthalpy of vaporization of liquid A at 42C is 32 kJ/mole. If the normal boiling point of liquid is
47C, using the given data, find Svaporisation at 42C (in J/K mole)
[4]
CP [A (l)] = 62 J/K mole
CP [A (g)] = 30 J/K mole
A(l) A(g)

315 5
=

take ln
320 320

[Ans. 100 J/K mole]

ASSIGNMENT-6

Q.1

Consider the system shown below :

[3]

A
B
C
Rigid
CO2(g) O2(g) He(g)
Adiabatic
Boundary
1mtr.
Massless-frictionless-Non-Rigid
Adiabatic piston

The three chambers shown in the figure contain one mole of each gas and piston A and B are adjusted
mechanically (using pin/holders) in such a way that each gas occupies same volume. A student by mistake
pushes piston C towards left such that the pin/holder breaks and all the three pistons are free to move.

V
The ratio
V

1.66

He( Final )

1 .4

CO2 ( Final )

1
. Calculate x .
x
3

Consider the c/s area of piston (A) and (B) to be negligible.


(A) 2.5
(B) 3
(C) 4.5
Q.2

Which of the following reactions is an example of complete combustion ?


(A) N2(g) + 2O2 (g) N2O4 (g)
(C) Cl2(g) +

Q.3

3
O (g) Cl2O3(g)
2 2

Calculate the Enthalpy of hydrogenation of


mol respectively.
(A) 199 kJ/mol

Q.4

(D) 1.5

(B*)

[3]

1
3
S8(g) + O2(g) SO3(g)
8
2

(D) All of these

,if the f H of

and

are 37 and 156 kJ/


[3]

(B*) 238 kJ/mol

(C) 59.5 kJ/mol

(D) 476 kJ/mol

Which of the following statements is/are incorrect ?


[3]
(A) The enthalpy of formation of elements in standard state is taken as zero, because the total energy of
the universe is constant.
(B) The absolute entropy of elements in standard state can not be taken as zero, because the entropy of
the universe is always increasing.
(C*) Refrigerator can be considered equivalent to Carnot Engine.
(D*) Cr+ in its ground state has 6 possible microstates, considering quantum mechanical model of atom.

Q.5

Which of the following can be a criteria of spontaneity ?


(A*) S(U,V) > 0
(B*) U(S,U) < 0
(C*) A(T,V) < 0

[3]
(D*) G(T,P) < 0

Q.6

It is estimated that an average human brain consumes the equivalents of 10 gm glucose per hour estimate
the power output of Brain in Watt. If 1W =1 J/sec. (combH(glucose) = 2818.8 kJ/mol).
[Ans. 43.5 W]
[4]

Q.7

The free energy change for the conversion of malate into fumarate is 3 kJ. In metabolism this reaction is
coupled with fumarate asparate. Which is exergonic by 15.5 kJ. Calculate G for the overall
reaction and interpret the result.
[4]

Q.8

The vapor pressure of decane is 10 torr at 55.7 C and 400 torr at 150.6 C. Calculate Hvap., Svap.
at 100 C. Also find out normal boiling point.
[4]

Q.9

Compound (A) is mixed with (B) in presence of acid catalyst and converted into cyclic compound (C).
Calculate the enthalpy change for the reaction.
[4]
A+BC
Compound (B) is prepared by NaNO2 + NaHSO3 + SO2 + H2O NaHSO4 + (B). (A) is used in
the preparation of Caprolactum.
If
ECH = 414 kJ/mol
ECN = 230 kJ/mol
EC=O = 312 kJ/mol
ECC = 340 kJ/mol
ENH = 198 kJ/mol
ENO = 210 kJ/mol
EOH = 150 kJ/mol

Q.10 n-Butane is heated with AlCl3. Calculate the enthalpy change for the reaction if ECH = 414 kJ/mol and
ECC = 340 kJ/mol.
[4]

ASSIGNMENT-7

Q.1

In the following graph :

[3]

Select the correct options(s)


(A) PA > PB
(B) VB > VA
Q.2

(C) PB > PA

(D) VA > VB

The free energy of gas vs temperature is represented as follows

Gg

[3]

C
D

B
Gg at P
A
Gg at P
O

Which of the following is/are true?


(A) P > P
(B) P > P
Q.3

(C) SC > SD

(D) SD > SC

Two identical bodies A and B of constant heat capacity at temperature TAand TB[TA > TB], are connected
using a heat engine. Which of the following statements is/are correct in this regard ?
[3]
(A) At maximum attainable temperature, the work done by the engine is minimum.
(B) At minimum attainable temperature, the Suniv.= 0.
(C) The final temperature ranges between

T1 T2
and T1.T2 .
2

(D) The total work delivered by the engine is the difference between heat absorbed from hot body and
heat rejected to cold body.
Q.4

Statement - 1 : On increasing the temperature, enthalpy of products and reactants changes by different
extents.
[3]
Statement - 2 : CP values of products and reactants are different.
(A*) Statement-1 is true, statement-2 is true and statement-2 is correct explanation for statement-1.
(B) Statement-1 is true, statement-2 is true and statement-2 is NOT the correct explanation for statement-1.
(C) Statement-1 is true, statement-2 is false.
(D) Statement-1 is false, statement-2 is true.

Q.5

500 ml decimolar ACOH is mixed with 1000 ml M/20 NH3. What is heat of neutralization of the
mixture if ionization constant of acid and base are same.
[4]

Q.6

When glucose is burnt in a bomb calorimeter, the reaction is found to be exothermic and
E = 2880 kJ/mol the entropy change is found to be 182.4 J/K-mol. Calculate free energy change at
298 K for the process. How much of the energy change can be extracted as Heat and How much can be
extracted as work.
[4]

Q.7

CaC2 an hydrolysis gives a gas (A) which is also hydrolysed in presence of promoters and catalyst and
results a liquid which is purified and added in base catalyst and product (B) is obtained after slight
heating. Find out enthalpy change in the reaction (A) to (B) assuming reaction take place in gas phase. If
ECC= 900 kJ/mol, ECC= 600 kJ/mol, EC= 700 kJ/mol , EOH= 243 kJ/mol, ECC= 300 kJ/mol. [4]

Q.8

An ideal gas cycle is represented by a rectangle on PV diagram. If P1 and P2 are the lower and higher
pressure and V1, V2 are smaller and larger volumes respectively, then show that
[4]

Q.9

1
P2
V1

(P2 P1 ) (V2 V1 )

RBr is mixed with NaSCN, using polar protic solvent (water) resulting in the formation of 79% A and
16% B. Calculate the f H of A and B, Given
[4]
Assume , R = CH3
ECS = 380 kJ/mol
ECH = 414 kJ/mol
EC=S = 520 kJ/mol
ECN = 680 kJ/mol
EC=N = 620 kJ/mol
What are compounds A and B and justify its percentage ?

Q.10 What is the probability that 2 moles of water originally at 50 C will spontaneously separate into 1 mole
water at 49C and 1 mol water at 51C. CP,m = 75 J/K mol
[4]

ASSIGNMENT-12

Q.1
(a)

Find the n factor in following non-redox interaction.


Of base
(i)
Al(OH)3 + HCl Al(OH)Cl2 + H2O
(ii)
Al(OH)3 + H2SO4 Al(OH)(HSO4)2 + H2O
(iii)
Ba(OH)2 + HCl BaCl2 + H2O

[10]

CH 2 NH 3 Cl
CH 2 NH 2
|
+ HCl |
CH 2 NH 2
CH 2 NH 3 Cl

(iv)

(v)

+ HCl
[Ans. (a) (i) n = 2, (ii) n = 2, (iii) 2, (iv) 2, (v) 1 ]

(b)

Of acid
(ii)
(iii)

H3SbO4 KOH
KH2SbO4 + H2O
H3SbO4 + KOH K2HSbO4 + H2O
H2SO4 + KOH KHSO4 + H2O

(iv)

KOH

(v)

KOH

(i)

[Ans.(b) (i) 1, (ii) 2, (iii) 1, (iv) 1, (v) 2]


Q.2

Find the n factor of underlined compound in following redox interaction


(i)
Pb(NO3)2 + Cr2(SO4)3 PbSO4 + Cr(NO3)3
(ii)
KMnO4 + MnSO4MnO2
(iii)
P4 H2PO2 + PH3
(iv)
N2H4 N2 + NH3
(v)
KMnO4 K2MnO4 + MnO2 + O2
(vi)
CH4 + O2 CO2 + H2O
[Ans. (i) 2,6,2,3 (ii) 3,2,6/5 (iii) nf = 3, nf = 1 , nf = 3, (iv) n = 4/3, (v) n = 2, (vi) n = 8, ]

[6]

ASSIGNMENT-13
Q.1

Find the n factor in following non-redox interaction.


(i)
MgAl(OH)5 + HCl MgCl + AlCl3 + H2O
(ii)
Ba(OH)2 + HCl Ba(OH)Cl + H2O
(iii)
RNH2 + HCl RNH3 Cl
(iv)
H2SO4 + KOH K2SO4 + H2O
(v)
H3PO4 + NaOH Na3PO4 + H2O
(vi)

+ NaOH

(vii)

NaOH

[7]

+ H2O
+ H2O
[Ans. (i) 5, (ii) 1,(iii) 1, (iv) 2, (v) 3, (vi) 1, (vii) 2]

Q.2

Find the n factor of underlined compound in following redox interaction


[2]
(a)
Cl2 ClO4 + Cl
(b)
S + NaOH Na2S + Na2S2O3 + H2O
[Ans. (a) n = 7/4 (b) n = 1]

Q.3

Given the reaction:


S2O82 + 2e 2SO42
Mn2+ + 4H2O MnO4 + 8H+ + 5e
How many moles of S2O82 ions are required to oxidise 1 mole of Mn2+ ?

Q.4

NH2OH reacts with ferric sulphate as follows:


2NH2OH + 4Fe3+ N2O + H2O + 4Fe2+ + 4H+
What is the equivalent weight of NH2OH in this reaction?

[4]

[Ans. 2.5 mol]


[4]
[Ans. M/2]

Q.5

20 ml of 0.1 M solution of metal ion reacted with 20 ml of 0.1 M SO2 solution. SO2 reacted according
to the equation:
[4]
SO2 + 2H2O SO42 + 4H+ + 2e
If the oxidation number of metal ion was +3, then what is the new oxidation number of the metal?
[Ans. +1]

Q.6

How many ml of 0.15 M Na2CrO4 will be required to oxidise 40 ml of 0.5 M Na2S2O3.


[4]
2
2
2
Given: CrO4 + S2O3 Cr(OH)4 + SO4
[Ans. 355 ml]

Q.7

It requires 40.05 ml of 1M Ce4+ to titrate 20ml of 1M Sn2+ to Sn4+. What is the oxidation state of the
cerium in the product.
[4]
[Ans. + 3]

Q.8

A volume of 12.53 ml of 0.05093 M SeO2 reacted with exactly 25.52 ml of 0.1M CrSO4. In the
reaction, Cr2+ was oxidized to Cr3+. To what oxidation state was selenium converted by the reaction.
[4]
[Ans. zero]

Q.9

Potassium acid oxalate K2C2O4 3H2C2O44H2O can be oxidized by MnO4 in acid medium. Calculate
the volume of 0.1M KMnO4 reacting in acid solution with one gram of the acid oxalate.
[4]
[Ans. V = 31.68 ml]

Q.10 50gm of a sample of Ca(OH)2 is dissolved in 50ml of 0.5N HCl solution. The excess of HCl was titrated
with 0.3N NaOH. The volume of NaOH used was 20cc. Calculate % purity of Ca(OH)2.
[4]
[Ans. 1.406%]

ASSIGNMENT-14
Q.1

Calculate the number of moles of KMnO4 which will react with 180 gm H2C2O4 according to given
reaction
KMnO4 + H2C2O4 2CO2 + Mn2+
and also calculate the volume of CO2 produced at STP.
[4]

Q.2

Calculate the mass of K2Cr2O7 required to produced 254 gm I2 from KI solution.


Given :
K2Cr2O7 + 2KI 2Cr3+ + I2

[4]

Q.3

Calculate the moles of KCl required to produced 10 mol Cl2 by the reaction with KClO3.

[4]

Q.4

3 mol of a mixture of FeSO4 and Fe2(SO4)3 required 100 ml of 2M KMnO4 solution in acidic medium.
What is the mole fraction of FeSO4 in the mixture?
[4]
[Ans. 1/3]

Q.5

5 ml of NHCl, 20 ml of

Q.6

A 1 g sample of H2O2 solution containing x % H2O2 by mass requires x cm3 of a KMnO4 solution for
complete oxidation under acidic conditions. Calculate the normality of KMnO4 solution.
[4]
[Ans. 0.588 N]

Q.7

A mixture of FeO and Fe2O3 is reacted with acidified KMnO4 solution having a concentration of 0.2278
M, 100 ml of which was used. The solution was then titrated with Zn dust which converted Fe3+ of the
solution to Fe2+. The Fe2+ required 1000 ml of 0.13 M K2Cr2O7 solution. Find the % of FeO & Fe2O3.
[4]
[Ans. FeO = 13.34%; Fe2O3 = 86.66%]

Q.8

25 mL of a solution containing HCl was treated with excess of M/5 KIO3 and KI solution of unknown
concentration where I2 liberated is titrated against a standard solution of 0.021M Na2S2O3 solution
whose 24 mL were used up. Find the strength of HCl and volume of KIO3 solution consumed. [4]
[Ans. VKIO = 0.42 mL, [HCl] = 0.0168N]

N
N
H2SO4 and 30 ml of HNO3 are mixed together and the volume made
2
3
to 1 litre. What is the weight of pure NaOH required to neutralize the solution?
[4]
[Ans. 1 gm]

Q.9

A 0.517g sample containing Ba(SCN)2 was dissolved in a bicarbonate solution. 50.0 mL of 0.107 N
iodine was added, and the mixture was allowed to stand for five minutes. The solution was then acidified,
and the excess I2 was titrated with 16.3 mL of 0.0965 M sodium thiosulphate. Write a balanced equation
for the oxidation of SCN into SO42 and HCN . Calculate the percent Ba(SCN)2 in the sample. [4]
[Ans. SCN+3I2+ 4H2O l SO42 + HCN + 7H+ + 6I, 15.68%]

Q.10 The neutralization of a solution of 1.2 g of a substance containing a mixture of H2C2O4. 2H2O, KHC2O4.
H2O and different impurities of a neutral salt consumed 18.9 ml of 0.5 N NaOH solution. On titration
with KMnO4 solution, 0.4 g of the same substance needed 21.55 ml of 0.25 N KMnO4. Calculate the
% composition of the substance.
[4]
[Ans. H2C2O4. 2H2O = 14.36%, KHC2O4. H2O = 81.71% ]

ASSIGNMENT-15
Q.1

520 gm mixture of Fe2O3 & FeO reacts completely with 158 gm KMnO4 in acidic medium. Calculate
the composition of the mixture by moles.
[4]
(Given : Mn+7 Mn+2)

Q.2

Calculate the concentration of H2O2 solution if 20 ml of H2O2 solution reacts completely with 10ml of 2
M KMnO4 in acidic medium.
[4]

Q.3

How many moles of thiosulphate (S2O32) will be required to react completely with one mole I2 in acidic
medium (where it gets oxidised to SO42)?
[4]
[Ans. 1/4]

Q.4

Metallic tin in the presence of HCI is oxidized by K2Cr2O7 to stannic chloride, SnCl4. What volume of
deci-normal dichromate solution would be reduced by 1g of tin.
[4]
[Ans. 337 mL]

Q.5

5g sample of brass was dissolved in one litre dil. H2SO4. 20 ml of this solution were mixed with KI,
liberating I2 and Cu+ and the I2 required 20 ml of 0.0327 N hypo solution for complete titration. Calculate
the percentage of Cu in the alloy.
[4]
[Ans. 41.53%]

Q.6

A 1.0 g sample of Fe2O3 solid of 55.2% purity is dissolved in acid and reduced by heating the solution
with zinc dust. The resultant solution is cooled and made upto 100.0 mL. An aliquot of 25.0 mL of this
solution requires 17.0 mL of 0.0167 M solution of an oxidant for titration. Calculate the number of moles
of electrons taken up by the oxidant in the reaction of the above titration.
[4]
[Ans. 6.07 6]

Q.7

5g of pyrolusite (impure MnO2) were heated with conc. HCl and Cl2 evolved was passed through
N
excess of KI solution. The iodine liberated required 40 mL of hypo solution. Find the % of MnO2 in
10
the pyrolusite.
[4]
[Ans. 0.174g; 3.48%]
Question No. 8 to 10 (3 questions)
A steel sample is to be analysed for Cr and Mn simultaneously. By suitable treatment Cr is oxidized as
Cr2O72 and the Mn to MnO4.
Cr Cr2O72
MnMnO4
A 10 gm sample of steel is used to produce 250.0 mL of a solution containing Cr2O72 and MnO4.
A 10 mL portion of this solution is added to a BaCl2 solution and by proper adjustment of the acidity, the
chromium is completely precipitated as BaCrO4; 0.0549 g is obtained.

Q.8

Q.9

Cr2O72 H
BaCrO4
A second 10 mL portion of this solution requires exactly 15.95 mL of 0.0750M standard Fe2+ solution
for its titration (in acid solution).
[9]
% of chromium in the steel sample
(A) 1.496
(B*) 2.82
(C) 1.96
(D) 5
Equivalent of Fe2+ required for reduction of MnO4 is
(A*) 5.44 104
(B) 0.544 102
(C) 1.196 103

(D) 11.96 104

Q.10 Amount of BaCl2 required for conversion of Cr2O72 to BaCrO4 in steel sample
(A) 0.045
(B) 0.0549
(C*) 1.125
(D) 2.82

ASSIGNMENT-16

Single Correct
Q.1 How many times solubility of CaF2 is decreased in 4 103 M KF (aq.) solution as compare to pure
water at 25C. Given Ksp (CaF2) = 3.2 1011
[3]
(A) 50
(B*) 100
(C) 500
(D) 1000
Q.2

During the electrolysis of aqueous Zn (NO3)2 solution:


(A) Zn plates out at the cathode
(B) Zn plates out at the anode
(C) H2 gas is evolved at the anode
(D*) O2 gas is evolved at the anode

[3]

Q.3

The salt Al(OH)3 is involved in the following two equilibria,


Al(OH)3(s)
Al3+ (aq.) + 3OH (aq.), Ksp

[3]

Al (OH)3(s) + OH (aq.)
Al(OH)4 (aq.), Kc
Which of the following relationship is correct at which solubility is minimum ?

(A) OH

(B) OH

1/ 3

K sp


Kc

K sp


K
c

1/ 4

(B) OH

K
c
K sp

1/ 4

(D*) None of these

Q.4

The mass of P4O10 produced if 440 gm of P4S3 is mixed with 384 gm of O2 is


P4S3 + O2 P4O10 + SO2
(A) 568 gm
(B*) 426 gm
(C) 284 gm
(D) 396 gm

Q.5

The minimum mass of mixture of A2 and B4 required to produce at least 1 kg of each product is :
(Given At. mass of 'A' = 10 ; At. mass of 'B' = 120)
[3]

2AB2 + 4A2B
5A2 + 2B4
(A*) 2120 gm
(B) 1060 gm

(C) 560 gm

[3]

(D) 1660 gm

More than one correct


Q.6

The reduction potential of hydrogen half-cell will be positive if


(A) p(H2) =1 atm and [H+] = 1 M
(B*) p(H2) = 1 atm and [H+] = 2M
(C) p(H2) = 2 atm and [H+] =1 M
(D*) p(H2) = 2 atm and [H+] = 2M

[4]

Comprehension
NaBr, used to produce AgBr for use in photography can be self prepared as follows :
[9]
Fe + Br2 FeBr2
....(i)
FeBr2 + Br2 Fe3Br8
....(ii)
(not balanced)
Fe3Br8 + Na2CO3 NaBr + CO2 + Fe3O4
....(iii) (not balanced)
Q.7 Mass of iron required to produce 2.06 103 kg NaBr
(A) 420 gm
(B*) 420 kg
(C) 4.2 105 kg
(D) 4.2 108 gm
Q.8

If the yield of (ii) is 60% & (iii) reaction is 70% then mass of iron required to produce 2.06 103 kg
NaBr
(A) 105 kg
(B) 105 gm
(C*) 103 kg
(D) None

Q.9

If yield of (iii) reaction is 90% then mole of CO2 formed when 2.06 103 gm NaBr is formed
(A) 20
(B*) 10
(C) 40
(D) None

Subjective:
Q.10 Find the EMF of cell formed by connecting two half cells:

[5]

Pt(s) | MnO 4 (0.1 M) ; MnCl2 (0.2 M) HCl (1M)


Pt(s) | Cr2O 72 (0.1 M) ; CrCl3 (0.2 M) HCl (0.7 M)
Given E

MnO 4 / Mn 2

= 1.51 volt

Cr2O 27 / Cr 3

= 1.33 volt

[Ans. 0.2 Volt]

ASSIGNMENT-17
Single Correct
Q.1 Calculate the pH of 6.66 103 M solution of Al(OH)3, it's first dissociation is 100 % where as second
dissociation is 50% and third dissociation is negligible.
[3]
(A) 2
(B*) 12
(C) 11
(D) 3
Q.2

10 ml of 0.1 M tribasic acid H3A is titrated with 0.1 M NaOH solution. What is the ratio of
2nd equivalence point. Given K1 = 7.5 104, K2 = 108, K3 = 1012
(A) ~104
(B) ~ 10+4
(C*) ~ 107
(D) ~ 10+6

[ H 3 A]
[ A 3 ]
[3]

at

Q.3

Mass of sucrose C12H22O11 produced by mixing 84 gm of carbon, 12 gm of hydrogen and


56 lit. O2 at 1 atm & 273 K according to given reaction, is
[3]
C(s) + H2(g) + O2 (g) C12H22O11(s)
(A) 138.5
(B*) 155.5
(C) 172.5
(D) 199.5

Q.4

The number of carbon atoms present in a signature, if a signature written by carbon pencil weights
1.2 103 g is
[3]
20
19
19
20
(A) 12.04 10
(B*) 6.02 10
(C) 3.01 10
(D) 6.02 10

Q.5

An aqueous solution of Na2SO4 was electrolysed for 10 min. 82 ml of a gas was produced at anode
over water at 27C at a total pressure of 580 torr. Determine the current that was used.
[3]
(Vapour pressure of H2O at 27C = 10 torr) (R = 0.082 atm lit./ mol / K)
(A) 0.1 Amp
(B) 1.25 Amp
(C) 2.23 Amp
(D*) 1.61 Amp

More than one correct


Q.6
A + 2B C
2C 3D
Which of the following option(s) is / are incorrect for above reaction.

[4]

(A) maximum no. of moles of D which can be produced by 2 moles of A is 3.


(B*) maximum no. of moles of D which can be produced by 6 moles of B is 4.
(C*) maximum no. of moles of D which can be produced by 2 moles of A and 6 moles of B is 4.5
(D*) All above options are correct.
Comprehension
For questions No. 7 & 8 use following information:

Q.7

[6]
378
8 mole of a mixture of N2, NO2 and N2O4 has a mean molecular mass of
. On heating to a
8
temperature at which N2O4 dissociated completely (N2O4 2NO2), the mean molecular mass
378
becomes
. [Given : Atomic mass N= 14, O = 16]
9
The ratio of number of moles of N2 : NO2 : N2O4 in original mixture is
(A) 3 : 4 : 1

Q.8

(B*) 2 : 5 : 1

(C) 3 : 5 : 2

(D) 4 : 2 : 1

The ratio of number of moles of N2 and NO2 after heating is


(A) 1 : 1

(B) 3 : 9

(C*) 2 : 7

(D) 1 : 2

Match the Column


Q.9 Column-I and column-II contains four entries each. Entry of column-I is to be matched with only one
entry of column-II. [Given : Atomic mass Ag = 108, S = 32]
[10]
Column I
Column II
(A)
5 m NaOH solution
(P)
16 M
(dsolution = 0.6 gm /ml).
Molarity of solution is
(B)

250 ml of H2O2 solution


provides 64 gm O2 .
Molarity of H2O2 solution is

(Q)

1M

(C)

100 ml , 1 M H2SO4 solution


(dsolution = 1.5 gm/ml) is mixed
with 400 ml of water, density of final
solution 1.25 gm/ml
Final Molarity of H2SO4 solution is

(R)

2.5 M

(D)

100 ml, 6 M NaCl solution is mixed with


100 ml of 17% (w/w) AgNO3
[dsolution= 8 gm/ml] solution.
The molarity of Ag+ ions in final solution is

(S)

0.227 M

[Ans. (A) R; (B) P; (C) S; (D) Q]

Subjective
Q.10 1.0 L of solution which was in equilibrium with solid mixture of AgCl and Ag2CrO4 was found to contain
1 104 moles of Ag+ ions, 1.0 106 moles of Cl ions and 8.0 104 moles of CrO 24 ions. Ag+ ions
are added slowly to the above mixture (keeping volume constant) till 8.0 107 moles of AgCl got
precipitated. How many moles of Ag2CrO4 were also precipitated?
[5]
4
[Ans.7.68 10 mole]

ASSIGNMENT-18

Single Correct
Q.1 The half life of the unimolecular elementary reaction
A(g)B(g) + C(g) is 6.93 min. How long will it take for the concentration of A to be reduced to 10
% of the initial value?
[3]
(A) 10.053 min
Q.2

Q.3

(C) 46 min

(D*) 23.03 min

Volume of water which must be added to 8M HCl to make 2 lit. of 7.3 % w/v HCl solution is [3]
(A*) 1.5 lit.
(B) 0.5 lit.
(C) 1 lit.
(D) 2 lit.
dA
= k[A].
For the reaction A (aq) B (g) + C (g), the rate law is
dt
Find the value 'X' in data table provided
time (min)
0
10
30
conc. of A(M) 1
0.7
X

(A) 0.1
Q.4

(B) 4.6 min

(B) 0.243

(C*) 0.343

[3]

(D) None of these

The instantaneous rate of disappearance of the MnO4 ion in the following reaction is 4.56 103 Ms1.
2MnO4 + 10 I + 16 H+ 2Mn2+ + 5I2 + 8H2O
[3]
The rate of appearance of I2 is
(A) 1.14 103 Ms1
(C) 4.56 104 Ms1

(B) 5.7 103 Ms1


(D*) 1.14 102 Ms1

Q.5

For which of the following solution, minimum amount of NaOH is required for complete neutralisation.
[Assume complete dissociation of acid ]
[3]
(A) 50 ml H2SO4 solution [basicity = 2] (dsolution = 2 gm/ml) , % w/w = 98
(B) 50 ml H3PO4 solution [basicity = 3] (dsolution = 2 gm/ml) , % w/w = 98
(C) 100 ml HCl solution [basicity = 1] (dsolution = 1.5 gm/ml) , % w/w = 36.5
(D*) 100 ml HNO3 solution [basicity = 1] (dsolution = 1.5 gm/ml) , % w/w = 36.5

Q.6

For the reaction


[3]
[Ag(CN)2] l Ag+ + 2CN ; Kc at 25C is 4 1019
The equilibrium concentration of Ag+ is, in a solution which is 0.1 M in KCN & 0.03 M in AgNO3
originally.
(A) 7.5 1016 M
(B*)7.5 1018 M
(C) 1.25 1019 M
(D) 1.25 1017 M

Q.7

2 mole of an ideal monoatomic gas at 27C undergoes adiabatic free expansion until it volume becomes
5 times of initial volume. The change in entropy of the gas is ( ln 5 = 1.60 ; R = 8.3 J/K-mole ) [3]
(A) Zero
(B*) 26.56 J/K
(C) 7968 J/K
(D) 39.84 J/K

Q.8

For a gas reaction A P at T (K) the rate is given by

[3]

rate = k1 p 2A atm/hr
rate = k 2 C 2A mol / litre / hr
the relation between k1 and k2 is
(A) k2 = k1

(B*) k2 = k1 RT

(C) k2 = k1/RT

(D) k2 = k1 (RT)2

Assertion & Reason


Q.9 Statement -1 : For a very dilute solution molality & molarity are always approximately equal
Statement -2 : Mass of solution is always approximately equal to mass of solvent for a very dilute
solution.
[3]
(A) Statement-1 is true, statement-2 is true and statement-2 is correct explanation for statement-1.
(B) Statement-1 is true, statement-2 is true and statement-2 is NOT the correct explanation for statement-1.
(C*) Statement-1 is false, statement-2 is true.
(D) Statement-1 is true, statement-2 is false.
More than one correct:
Q.10 A 1 litre solution containing NH4Cl and NH4OH has hydroxide ion concentration of 106 mol/lit. Which
of the following hydroxides could not be precipitated when the solution is added to 1 litre solution of 0.1
M metal ions
[5]
3
6
(A*) Ag(OH) (KSP = 5 10 )
(B*) Ca(OH)2 (KSP = 8 10 )
(C*) Mg(OH)2 (KSP = 3 1011)
(D) Fe(OH)2 (KSP = 8 1016)

ASSIGNMENT-19

Single correct
Q.1 For acid catalysed hydrolysis of ester rate law obtained is rate = k [ester] [H+],
where k = 0.01 M1 hr 1.What is the half-life if the initial concentrations are 0.02 M for the ester and
0.05 M for the catalyzing acid :
[3]
(A) 1429 hours
(B) 5000 hours
(C*) 1386 hours
(D) 2 hours

Q.3

100 ml of H2SO4 solution having molarity 1M is mixed with 400 ml of water. Resultant molarity of
H2SO4 solution is:
[3]
(A) 4.4 M
(B) 0.145 M
(C) 0.227 M
(D*) 0.2 M

Q.4

For the reactions

KI
(i) A
P..

&

II
(ii) B K
Q, following observation is made.

1M

0.5 M
[A]

0.25 M
0

30

Concentration

For a reaction of order n, what is the relationship between t3/4 and t1/2 , where t3/4 is the time required for
concentration (C) to become 1/4 C0 where C0 and C are the values of the reactant concentration at the
start and after time t, respectively.
[3]
n1
n1
(A*) t3/4 = t1/2 [2 + 1]
(B) t3/4 = t1/2 [2 1]
(C) t3/4 = t1/2 [2n+1 1]
(C) t3/4 = t1/2 [2n+1 + 1]

Concentration

Q.2

[3]

1M
0.5 M
[B]

60

60

Time (min)

Time (min)

KI
Calculate K , where KI and KII are rate constant for the respective reaction.
II

(A*) 2.772
Q.5

(B) 1

(C) 0.36

(D) None of these

There exist an equilibrium between solid BaSO4 and Ba2+ and SO 24 ion in aqueous medium. The
possible equilibrium state are shown in figure as thick line. Now, if equilibrium is disturbed by addition of
BaNO3 and (b) K2SO4 and dotted line represent approach of system towards equilibrium. Match the
columns given below:
[3]

(a) addition of BaNO3

(c)

(e)

(b) addition of K2SO4

(d)

(f)

(A) ac, bd

(B*) ad, be

(C) af, be

(D) ad, bc

More than one may be correct


Q.6 In a chemical reaction A(g) is converted to B(g), following observation is made. Identify the correct
statement(s).
[4]
Pressure (atm)

PB
8
6
4

PA

2
10

30 time (min.)

20

1
A(g) B(g).
2
(B) t1/2 of the reaction is independent of initial partial pressure of A.
(C*) Reaction must be complex reaction.
(D*) Time of completion for the reaction is 40 minutes.

(A*) Chemical reaction should be

Comprehension
Comprehension (Q.7 to Q.9)
Concentrations measured as a function of time when gaseous N2O5 at initial concentration of 0.0200 M
decomposes to gaseous NO2 and O2 at 50C. The change in concentration with time is given by the
following graph.
[9]
NO2

0.03

Concentration(M)

[NO2]
=0.003 M

0.02

[NO2]=0.004M

0.01
O2
N2O5
= tan1 (2.5 105)

0.00
0

100 200

300 400 500


Time(s)

600 700

800

Q.7

The instantaneous rate of disappearance of N2O5 at the begining of the reaction is


(A) 2 105 M/s
(B) 1 105 M/s
(C*) 5 105 M/s
(D) zero

Q.8

The rate of formation of O2 during the period 600 700 s is


(A) 4 105 M/s
(B) 3 105 M/s
(C) 5 105 M/s

(D*) 7.5 106 M/s

The rate of decomposition of N2O5 during the period 300 400 s is


(A) 4 104 M/s
(B) 3 105 M/s
(C*) 2 105 M/s

(D) 4 108 M/s

Q.9

Subjective
Q.10 50 ml of a gaseous hydrocarbon is mixed with 200 ml oxygen. After sparking, evolved gaseous mixture
was passed through KOH solution which result in 100 ml contraction, when further passed through
alkaline pyrogallcol solution shows a contraction of 50 ml. Find molecular formula of hydrocarbon. [5]
[Ans. C2H4 ]

ASSIGNMENT-20
Single Correct
Q.1 The dissociation of HI molecules as shown below, occurs at a temperature of 629 K. The rate constant,
k = 3.02 105 M1s1
HI (g )

1
1
H 2 (g ) I 2 (g )
2
2

what is the reaction order.


(A) 0
(B) 1
Q.2

[3]
(D) Can not be predicted

(C*) 2

For the first order reaction

[3]

N NCl

Cl + N2(g)

following observation is made :


8
7
6
log (V Vt) 5
4
3
2

time (min.)
10 20 30 40

where Vt(in cc) is volume of N2 collected at time t & V (in cc) is volume of N2 collected after a long
time. What is the time taken in minute for 75% reaction ?
(A) 2.5
(B) 5.0
(C*) 3
(D) 10
Q.3

At 1800 K ethane gas decomposes to ethene and hydrogen. Rate constant for the reaction is
103 Pa1 sec1. If initial pressure of ethane is 3 105 Pa, how many sec. would it take for the pressure
to reach 5 105 Pa?
[3]
2
3
(A) 1800.2 sec.
(B) 3.33 10 sec. (C*) 6.66 10 sec.
(D) 1000.4 sec.

Q.4

Two first order reaction have half-lives in the ratio 8 : 1. Calculate the ratio of time intervals t1 : t2. The
th

th

1
3
time t1 and t2 are the time period for and completion.
4
4
(A) 1 : 0.301
(B) 0.125 : 0.602
(C*) 1 : 0.602

[3]
(D) None of these

Q.5

10 ml of 0.1 M CH3COONa+ is mixed with 11 ml of 0.1 M HCl. The concentration of CH3COO ion
at equilibrium is [Take Ka (CH3COOH) = 105]
[3]
4
4
4
8
(A) 2.1 10 M
(B) 21 10 M
(C*) 10 M
(D) 10 M

Q.6

For any sparingly soluble salt [M(NH3)4 Br2] H2PO2, What will be the solubility product constant ?
[Given 0M ( NH

= 100 S-m2- mol1, 0H

3 ) 4 Br2

2 PO 2

= 50 S-m2- mol1

Specific resistance of saturated [M(NH3)4 Br2] H2PO2 solution is 200 -cm ]


(A*) 1.11 1011
(B) 1.11 103
(C) 3.33 106
(D) None of these

[3]

Assertion & Reason


Q.7 Statement -1 : Molality of pure ethanol is lesser than pure water.
[3]
Statement -2 : As density of ethanol is lesser than density of water.
[Given : dethanol = 0.789 gm/ml; dwater = 1 gm/ml]
(A) Statement-1 is true, statement-2 is true and statement-2 is correct explanation for statement-1.
(B*) Statement-1 is true, statement-2 is true and statement-2 is NOT the correct explanation for
statement-1.
(C) Statement-1 is false, statement-2 is true.
(D) Statement-1 is true, statement-2 is false.
More than one may be correct
Q.8 Select the option(s) which have equal number of 'O' atom.
(A) 10 ml H2O(l)
(B*) 0.15 mole of V2O5
(C*) 12 gm O3(g)
(D) 12.044 1022 molecules of CO2

[4]

Subjective
Q.9 The following data were obtained during the first order thermal decomposition of SO2Cl2 at a constant
volume.
[5]
SO2 Cl2 (g) SO2 (g) + Cl2 (g)

Experiment Time(s) Total pressure / atm


1
0
0.5
2
100
0.6
Calculate the rate of the reaction when total pressure is 0.65 atm.

[Ans. 7.8 104 atm s1]

Q.10 Would H2O2 behave as oxidant or reductant with respect to the following couples at standard condition.
(b) S2O82 / SO 24

(a) I2 / I
Given E

O2 ,H / H2O2

E I

/ I

= + 0.69 volt

= 0.535 volt

Fe3 / Fe2

(c) Fe3+ / Fe2+

H 2O 2 , H / H 2O

S2O82 / SO 24

[7]
= + 1.77 volt

= 2.0 volt

= 0.77 volt
[Ans. (a) oxidising agent, (b) reducing agent, (c) both oxidising and reducing agent]

ASSIGNMENT-21

Single Correct
Q.1 The acid catalysed hydrolysis of an organic compound A at 30C has a time for half change of 100 min.
when carried out in a buffer solution at pH = 5, and 10 min. when carried out at pH = 4. Both times of
half change are independent of the initial concentration of A. If the rate constant K is given by
d[ A]
=K[A]a[H+]b , what are the values of a and b?
dt
(A*) a = 1, b = 1
(B) a = 2, b = 1
(C) a = 0, b = 1

Q.2

(D) a = 1, c = 0

What is the aq. ammonia concentration of a solution prepared by dissolving 0.15 mole of NH 4 CH3COO
in 1L H2O.[ K a (CH

3COOH )

(A*) 8.3 104


Q.3

[3]

=1.8105 ; K b( NH

4OH )

=1.8105]

(C) 6.4 104

(B) 0.15

[3]
(D) 3.8 104

AgCl can be obtained:

[3]

(A) by extrapolation of the graph and C to zero concentration


(B*) by known values of of AgNO3, HCl and HNO3
(C) both (A) and (B)
(D) none of these
Q.4

Benzoic acid solution is titrated with NaOH conductometrically, graphical representation of the titration is:

(A)

(B)

(C)

(D*)

[3]

Q.5

C6H5OH (g) + O2 (g) CO2 (g) + H2O (l)


[3]
Magnitude of volume change if 30 ml of C6H5OH (g) is burnt with excess amount of oxygen, is
(A) 30 ml
(B*) 60 ml
(C) 20 ml
(D) 10 ml

Q.6

Two radioactive materials X1 and X2 have decay constants 10 and respectively. If initially they have the
same number of nuclei, then the ratio of the number of nuclei of X1 to that of X2 will be
1/e after a time
[3]
(A) 1/(10)
(B) 1/(11)
(C) 11/(10)
(D*) 1/(9)

Q.7

In a radioactive decay sequence

[3]

1
2
A
B
C
What is the ratio of numbers of atoms of A to the number of atoms of B after long time interval starting
with pure A. The mean lives of A and B are 20 hours and 30 minutes respectively.
(A) 2/3
(B) 10
(C) 20
(D*) 40

Q.8

A certain weak acid has a dissociation constant of 1.0 104. The equilibrium constant for its reaction
with a strong base is :
[3]
4
10
10
14
(A) 1.0 10
(B) 1.0 10
(C*) 1.0 10
(D) 1.0 10

More than One may be correct


Q.9 Which of the following statements is correct.
[4]
14
(A*) On bombarding 7N Nuclei with -particle, the nuclei of the product formed after release of
proton would be 8O17.
(B*) Nuclide and it's decay product after -emission are called isobars
(C*) Nuclide and it's decay product after -emission are called isodiaphers.
(D) Half life of radium is 1580 years. Its average life will be 1097.22 years.
Subjective
Q.10 A 5.0 gm sample containing Pb3O4 , PbO2 and some inert impurity is dissolved in 250 ml
dil. HNO3 solution and 2.68 g of Na2C2 O4 was added so that all lead converted into Pb2+.
A 10 ml portion of this solution required 8.0 ml, 0.02 M KMnO4 for titration of excess of oxalate. In an
another experiment, 25 ml of solution was taken and excess oxalate was removed by extraction, this
required 10 ml of a permanganate solution for oxidation of Pb2+ to Pb4+, 10 ml of this permanganate
solution is equivalent to 4.48 ml, 5V H2O2 solution.
Calculate mass % of PbO2 & Pb3O4 in the original sample. [At. wt. of Pb = 207]
[6]
[Ans. % Pb3O4 = 68.5, % PbO2= 23.9]

ASSIGNMENT-22
Single Correct
Q.1 A radioactive nuclide is produced at a constant rate of -per second. It decay constant is . If N0 be the
no. of nulcei at time t = 0, then maximum no. of nuclei possible are
[3]
(A*)
Q.2

(B) N0

(C) /

(D)

+ N0

Among the following ion which is best oxidising agent?


Given

E 0[ Fe( CN)6 ]3 /[Fe (CN)6 ]4

[3]

= 0.36 V

E 0 Fe3 / Fe 2 = 0.77 V
(A*) Fe3+
(B) Fe2+

(C) [Fe(CN)6]4

(D) [Fe(CN)6]3

Q.3

The reaction: 2NO + 2H2 N2 + 2H2O has been assigned to follow following mechanism [3]
I.
NO + NO l N2O2 (fast)
II.
N2O2 + H2 N2O + H2O (slow)
III.
N2O + H2 N2 + H2O (fast)
The rate constant of step II is 1.2 104 mole1L min1 while equilibrium constant of step I is
1.4 102. What is the rate of reaction when concentration of NO and H2 each is 0.5 mole L1.
(A*) 2.1 107 mol L1 min1
(B) 3.2 106 mol L1 min1
4
1
1
(C) 3.5 10 mol L min
(D) none of these

Q.4

When a car starts following chemical reaction takes place in the battery for 5 sec. to deliver 48.25 A
current:[3]

Pb (s) + PbO2 (s) + 2H2SO4 + 2e 2PbSO4(s) + 2H2O + 2e


Calculate mass of H2SO4 consumed during the process (in mg)
(A) 0.245
(B*) 245
(C) 0.490
(D) 490

Comprehension
Question No. 5 to 7 (3 questions)
Following titration method is taken to compute stepwise ionisation constant of a weak dibasic acid

A:
p-hydroxybenzoic acid
A has two ionisation proton and there can be stepwise neutralisation by NaOH
25 ml of a dilute aqueous solution of A is titrated with 0.02 M NaOH (aq) and pH measured.
Step Volume of NaOH added
pH
I
8.12 ml
4.57
II
16.24 ml
7.02 (at first equivalent point)

[9]

Q.5

Which H+ is removed in step I?

(A)

+ H2O l

+ H 3O

(C) both 50% in each part

(B*)

+ H2O l

+ H 3O

(D) H2O + H2O l H3O + OHr


(autoprotolysis of H2O)

Q.6

pK a (= log K a ) of p-hydroxybenzoic acids is:


1
1

(A*) 4.57
Q.7

(B) 9.47

(C) 4.90

(D) 7.00

pK a (= log K a ) of p-hydroxy benzoic acid is nearly


2
2

(A) 4.5

(B) 7

(C*) 9.5

(D) 5

More than one may be correct


Q.8 Choose the correct option(s):
(A*) Melting point of ice decreased on increasing pressure
(B*) 2O3(g) 3O2(g), S > 0
(C) pH is an extensive property
(D) Decrease in atomic no. by one unit due to emission of positron

[4]

e is as a result of conversion of
0
1

one neutron into proton.


Q.9

Iodine liberated was titrated against 0.1 M hypo solution 40 ml of which was required. Concentration of
H2O2 solution may be expressed as.
[5]
(A*) 0.68 gm / L

(B*) 0.02 M

(C*) 0.04 N

(D) 6.8 % w/v

Subjective
Q.10 A gaseous compound 'A' reacts by three independent first order processes
(as shown in figure) with rate constant 2 103 , 3 103 and
1.93 103 sec1 respectively for products B, C and D respectively. If
initially pure 'A' was taken in a closed container with P = 2 atm, find
the partial pressure of 'B' (in atm) after 100 sec from start of experiment. [5]
[Ans. 2.9 101]

ASSIGNMENT-23

Single Correct
Q.1 During electrolysis at acidified water, O2 gas is formed at the anode. To produce O2 gas at the anode at
the rate of 0.224 cm3 per second at STP, current passed is :
[3]
(A) 0.224 A
(B) 2.24 A
(C) 9.65 A
(D*) 3.86 A
Q.2

Ionic conductances of H+ and SO 24 at infinite dilution are x and y S cm2 equiv1. Hence, equivalent
conductance of H2SO4 at infinite dilution is:
(A*) x + y
(B) 2(x + y)

Q.3

[3]
(C) 2x + y

(D) x + 2y

The radioisotope 15 P32 is used in biochemical studies. A sample of this isotope has an activity 1000
times the detectable limit. How long could an experiment be run with the sample before the radioactivity
could no longer be detected ( t1 / 2 =14.2 days)
(A) ~ 28 days
(B) ~ 102 days
(C*) ~ 142 days

[3]
(D) ~ 50 days

Q.4

A radioactive mixture containing a short lived species A and short lived species B. Both emitting particles at a given instant, emits at rate 10,000 -particles per minute. 10 minutes later, it emits at the
rate of 7000 particles per minute. If half lives of the species are 10 min and 100 hours respectively, then
the ratio of activities of A : B in the initial mixture was
[3]
(A) 3 : 7
(B) 4 : 6
(C*) 6 : 4
(D) 10 : 4

Q.5

1.5 gm mixture of SiO2 and Fe2O3 on very strong heating leave a residue weighing 1.46 gm. The
reaction responsible for loss of weight is
Fe2O3 (s) Fe3O4 (s) + O2(g)
What is the percentage by mass of Fe2O3 in original sample.
[3]
(A*) 80%

Q.6

(B) 20%

(C) 40%

(D) 60%

For the reaction

[3]

R OH + X
r
e
n d rd
II o
k2
I sot r
der R OH + X

k1
R X + OH

the rate is given by


Rate = k1 [RX] [OH] + k2 [RX]
Calculate % of RX which reacts by 2nd order mechanism when [OH] is 0.1 M.
k1 100
(A) k k
1
2

k 2 100
(B) k k
1
2

10 k1
(C) k k
1
2

100 k1
(D*) k 10k
1
2

Comprehension
Paragraph for question nos. 7 to 9

A factory, producing methanol, is based on the reaction :


CO + 2H2 l CH3OH
Hydrogen & carbon monoxide are obtained by the reaction
CH4 + H2O l CO + 3H2
Three units of factory namely, the "reformer" for the H2 and CO production, the "methanol reactor" for
production of methanol and a "separator" to separate CH3OH from CO and H2 are schematically
shown in figure.

Reformer

Valve-3

Valve-2

Valve-1

Reactor

Separator

CH3OH

The flow of methanol from Valve-3 is 103 mol/sec. The factory is so designed that

2
of the CO is
3

converted to CH3OH. Assume that the reformer reaction goes to completion.


CO + 2H2 l CH3OH
Hr = 100 R
Q.7

What is the flow of CO and H2 at Valve-1?


(A) CO : 1500 mol / sec.; H2 : 2000 mol/sec.
(B) CO : 1500 mol / sec.; H2 : 3000 mol/sec.
(C) CO : 1000 mol / sec.; H2 : 2000 mol/sec.
(D*) CO : 1500 mol / sec.; H2 : 4500 mol/sec.

[3]

Q.8

What is the flow of CO and H2 at Valve-2?


(A) CO : 500 mol / sec.; H2 : 1000 mol/sec.
(C) CO : 500 mol / sec.; H2 : 2000 mol/sec.

[3]

Q.9

(B*) CO : 500 mol / sec.; H2 : 2500 mol/sec.


(D) CO : 500 mol / sec.; H2 : 1500 mol/sec.

Amount of energy released in methanol reactor in 1 minute?


(A)1200 kcal
(B*) 12000 kcal
(C) 6000 kcal

[3]
(D) None of these

More than one may be correct


Q.10 The order of reaction A product can be given by the expression(s)
[where r = rate of reaction ; [A]1 = concentration at time t1 ; [A]2 = concentration at time t2 ]
ln r2 ln r1
(A*) ln[ A] ln [A]
2
1

ln [A 0 ]2 ln [A 0 ]1
(B) ln[ t ] ln [ t ]
1/ 2 2
1/ 2 1

d[A]
/ ln [A]
(C*) ln
k.dt

(D*)

ln (r / k )
ln[ A]

[5]

ASSIGNMENT-24

Single Correct
Q.1 In the redox reaction
a CrCl3 + b H2O2 + c NaOH 2 Na2CrO4 + x NaCl + y H2O
Find the value of a, b, c in the given equation :
(A*) a = 2, b = 3, c = 10
(C) a = 2, b = 2, c = 8
Q.2

(B) 1 : 2

(C*) 4 : 5

(D) None of these

Ionisation constant of a weak acid (HA) in terms of m and m is:


(A) Ka =

(C) Ka =
Q.4

(B) a = 1, b = 2, c = 8
(D) a = 2, b = 3, c = 4

An equimolar mixture of Na2C2O4 and H2C2O4 required V1 litre of 0.1 M KMnO4 in acidic medium
for complete oxidation. The same amount of the mixture required V2 litre of 0.2 M NaOH for
neutralization. The ratio of V1 to V2 (V1/V2) is :
[3]
(A) 2 : 5

Q.3

[3]

Cm

(B*) Ka =

( m )
C( m ) 2

[3]

C2m
m ( m m )

(D) none of these

m ( m m )

1 mole of AgNO3 is added to 10 litre of 1 M NH3. What is the concentration of Ag(NH3)+ in solution?
[Given : For Ag( NH 3 ) 2 , K f1 2.0 103 ; K f 2 10 4 ]
(A) 8

Q.5

105

(B*) 1.25

105

(C) 4

106

The enthalpy changes of the following reactions at 27C are


1
Cl (g) NaCl (s)
2 2
H2(g) + S (s) + 2O2 (g) H2SO4 (l)
2Na(s) + S(s) + 2O2 (g) Na2SO4 (s)

Na(s) +

[3]
(D) 1.25

104
[3]

rH = 411 kJ/mol
rH = 811 kJ/mol
rH = 1382 kJ/mol

1
1
H2(g) + Cl2(g) HCl (g)
rH = 92 kJ/mol;
R = 8.3 J/K-mol
2
2
from these data, the heat change of reaction at constant volume ( in kJ/mol) at 27C for the process
2NaCl (s) + H2SO4 (l) Na2SO4 (s) + 2HCl (g) is
(A) 67
(B*) 62.02
(C) 71.98
(D) None

Q.6

What volume of 0.2 M RNH3Cl solution should be added to 100 ml of 0.1 M RNH2 solution to
produce a buffer solution of pH = 8.7?
[3]
Given : pKb of RNH2 = 5 ; log 2 = 0.3
(A) 50 ml
(B*) 100 ml
(C) 200 ml
(D) None of these

Assertion & Reason


Q.7

7
litre and 1 litree respectively. The
11
mass of CO is same at that of CO2 in the mixture.
[3]
Statement -2 : Ratio of total number of atoms of C to O is 7 : 11 in above mixture.
(Assuming gases to be ideal)
(A) Statement-1 is true, statement-2 is true and statement-2 is correct explanation for statement-1.
(B) Statement-1 is true, statement-2 is true and statement-2 is NOT the correct explanation for statement-1.
(C*) Statement-1 is true, statement-2 is false.
(D) Statement-1 is false, statement-2 is true.

Statement -1 : Partial volume of CO2 and CO in a mixture are

More than One may correct :


Q.8 For the first order parallel reactions of A starting with 1M initial concentration of A :
3k
A

2k

[4]

B
2C

k
3D

Which of the following is / are true.


(A*) [B]t : [C]t : [D]t is independent of time
(B*) [C]t > [B]t = [D]t
(C*) [C] = 0.67 M
(D) If k = 0.5 min1 then [B] = 1 M after 1.386 min.
Q.9

NO2 is involved in the formation of smog and acid rain. A reaction that is important in the formation of
NO2 is O3(g) + NO(g) l O2(g) + NO2(g) Kc= 6 1034 , if the air over Bansal Classes contained
1 106M O3, 1105 M NO, 2.5 104 M NO2 and 8.2 103 M O2, then which is incorrect ?
(A*) There will be a tendency to form more NO and O3
[4]
(B) There will be a tendency to form more NO2 and O2
(C*) There will be a tendency to form more NO2 and O3
(D*) There will be no tendency for change because the reaction is at equilibrium.

Subjective
Q.10 10 mole of liquid 'A' and 20 mole of liquid 'B' is mixed in a cylindrical vessel containing a piston arrangement. Initially a pressure of 2 atm is maintained on the solution. Now, the piston is raised slowly and
isothermally. Assume ideal behaviour of solution and A and B are completely miscible.
PAo 0.6 atm and PBo 0.9 atm

What is the pressure at which


present in the vapour form.
[Given : 57 = 7.55 ]

1
th of the total amount (by mol) of liquid solution taken initially, will be
3
[6]

[Ans. 0.791 atm]

ASSIGNMENT-25

Single Correct
Q.1 15 g of a solute in 100 g water makes a solution freeze at 1C. 30 g of a solute in 100 g of water will
give a depression in freezing point equal to
[3]
(A) 2C
(B) 0.5C
(C*) 2C
(D) 1C
Q.2

In cold countries, ethylene glycol is added to water in the radiators of cars during winters. It results in
(A) lowering of boiling point
(B) reduced viscosity
[3]
(C) reduced specific heat
(D*) lowering of freezing point

Q.3

Which represents correct difference for an ideal solution containing non-volatile solute.

(I)

[3]

(II)

(III)
(A*) I, II, III

(B) I, III

(C) II, III

(D) I, II

Q.4

The reaction N2 + O2 l 2NO, follows the kinetics Rate [N2][O2]. The order of reactions when both
N2 and O2 are in small amount, N2 is in large excess as compared to O2 and both N2 and O2 are in large
excess are
[3]
(A*) 2, 1 and 2
(B) 2, 1 and 1
(C) 2, 1 and 0
(D) 1, 1 and 0

Q.5

During an experiment an ideal gas is found to obey an additional law PV3 = constant. The initial temperature
of gas is 600 K what will be final temperature if gas expands to its double volume.
[3]
(A) 1200 K
(B) 2400 K
(C) 300 K
(D*) 150 K

Q.6

For two parallel first order reactions, what is the overall activation energy of reaction? The yields of B
and C in products are 40% and 60%, respectively.
[3]
a 20 kcal / mol
A E

B
(A) 60 kcal/mol
(B*) 32 kcal/mol

a 40 kcal / mol
A E

C
(C) 28 kcal/mol
(D) 20 kcal / mol

More than one may be correct


Q.7 The cell potential for the unbalanced chemical reaction.
Hg22+ + NO3 + H+ Hg2+ + HNO2 + H2O

[4]
;

E ocell 0.02 V

2.303 RT

0.06
is measured under standard state conditions in the electrochemical cell.
F

Then which is the correct statement:


(A*) Solution in cathodic compartment is acidic.
(B) Solution in Anodic compartment is acidic
2
, if activity of other components are equal to one.
9
(D*) 0.6 moles of electron pass through the circuit when 0.6 moles of Hg2+ are produced in the cell.

(C*) Cell potential will be zero at pH =

Q.8

Q.9

E = 0 for which process, must be zero


(A*) Cyclic process
(C) Isochoric process

[5]
(B*) Isothermal ideal gas expansion
(D) Adiabatic process

The incorrect statement(s) regarding 2M MgCl2 aqueous solution is/are (dsolution = 1.09 gm/ml) [4]
(A) Molality of Cl is 4.44 m
(B*) Mole fraction of MgCl2 is exactly 0.035
(C) The conc. of MgCl2 is 19% w/v
(D*) The conc. of MgCl2 is 19 104 ppm

Subjective
Q.10 Kinetic data for hydrolysis of a 0.3 M solution of ethyl acetate in presence of HCl as catalyst is given in
the following table. A constant volume of reaction mixture is taken at different time and titrated against
standard alkali solution at different time.
Time / min
0
10
t=
Vol. of Alkali / ml
25.00
28.00
40.00
If the rate law is given by rate = k[Ester]1[H+]1, where H+ is a catalyst ,find the rate (in M/min) of ester
hydrolysis in a solution which is 0.1 M each in ester and acid HCl (M/min)
5
Use: ln = 0.225
4

[5]
[Ans. 4.5 1004]

ASSIGNMENT-26

Single Correct
Q.1 A graph was plotted between the molar conductance of various electrolytes (HCl, KCl and CH3COOH)
and root of their concentrations in mole per litre.
[3]

Which of the following is correct match?


(A*) I (CH3COOH) ; II (KCl) ; III (HCl)
(C) I (CH3COOH) ; II (HCl) ; III (KCl)

(B) I (HCl) ; II (KCl) ; III (CH3COOH)


(D) I (KCl) ; II (CH3COOH) ; III (HCl)

Q.2

For a zero order reaction and a Ist order reaction half life are in ratio of 4 : 1. Calculate ratio of time taken
to complete 87.5 % reaction for zero order : first order reaction respectively.
[3]
(A*) 7 : 3
(B) 3 : 7
(C) 4 : 1
(D) 5 : 3

Q.3

One litre water in a bucket is placed in a closed dry air room having dimension 4 2 1.5 m3 at 300 K.
If vapour pressure of water at 300 K is 38 mm & density of water at 300 K is 0.9 gm/ml. Then volume
of water left in liquid state will be (R = 0.08 L atm mol1)
[3]
(A) 450 ml
(B*) 500 ml
(C) 875 ml
(D) 900 ml

More than one may be correct


Q.4 Which of the following statement true for one mole of an ideal gas

H E
(A*) T T R

P V

H
E
(B*) T T

P V

E
=0
(C*)
V T

H
0
(D*)
V T

[3]

Comprehension
Question No. 5 to 8 (4 questions)
Properties whose values depend only on the concentration of solute particles in solution and not on the
identify of the solute are called colligative properties.
Q.5 Acetic acid in benzene solution forms dimer due to intermolecular H-bonding. For this case vant Hoff
factor is:
[3]
(A) i =1
(B) i > 1
(C*) i < 1
(D) None of these
Q.6

An aqueous solution of 0.01 M CH3COOH has vant Hoff factor of 1.01. If pH = log [H+], pH of
0.01 M CH3COOH solution would be:
[3]
(A) 2
(B) 3
(C*) 4
(D) 5

Q.7

In which case vant Hoff factor is maximum?


(A) KCl, 50% ionised
(C) SnCl4, 20% ionised

[3]
(B) K2SO4 40% ionised
(D*) FeCl3, 30% ionised

Q.8

A complex containing K+, Pt (IV) and Cl is 100% ionised giving i = 3. Thus, complex is:
(A) K2 [PtCl4]
(B*) K2[PtCl6]
(C) K3[PtCl5]
(D) K[PtCl3]

[3]

Q.9

Match the column :


Column I (Order)

[8]
Column II
1

(A)

Zero

(P)

(ax)

log(t1/2)

(B)

First

45

(Q)
log a

(ax)

(C)

Second

(R)

log(t1/2)

(D)

Third

(S)
log a

(ax)

(T)

t
[Ans. (A) Q,R (B) S,T, (C) T, (D) P,T]
Subjective
Q.10 A solution of Na2S2O3 is to be standardised by titration against iodine liberated from standard KIO3
solution. The later is prepared by dissolving 4.28 gm of KIO3 in water and making upto 500 ml. 20 ml
of this solution are then mixed with excess KI solution and the following reaction occurs.
[6]

IO3 + 5I + 6H+ 3I2 + 3H2O


The resulting iodine is titrated with the Na2S2O3 solution according to
I2 + 2S2O32 2 I + S4O 62
and it is found that 48 ml are needed.
Calculate the concentration (in millimoles / L) of Na2S2O3 solution.

[Ans. 0100.00 Ans. ]

ASSIGNMENT-27

Single Correct
Q.1 An element has FCC structure with edge length 200 pm. Calculate density if 200 g of this element
contains 24 1023 atoms.
[3]
(A) 4.16 g cm3
(B*) 41.6 g cm3
(C) 4.16 kg m3
(D) 41.6 kg m3
Q.2

The boiling point of 1.0 m K4[Fe(CN)6] is (Kb for water = 0.52 K kg mol1)
(A) 100.52C
(B) 100.10C
(C) 100.26C
(D*) 102.6C

Q.3

For CrCl3. xNH3, elevation in boiling point of one molal solution is double than that of one molal urea
solution, hence x would be (complex is 100% ionized)
[3]
(A*) 4
(B) 5
(C) 6
(D) none of these

Q.4

The solubility of gases in liquids:


(A) increases with increase in pressure and temperature
(B) decreases with increase in pressure and temperature
(C*) increases with increase in pressure and decrease in temperature
(D) decreases with increase in pressure and increase in temperature

Q.5

A reaction at 300 K with G = 1743 J /mol consists of 3 mole of A (g), 6 mole of B (g) and 3 mole
of C (g). If A, B and C are in equilbrium in 1 litre container then the reaction may be
[3]
[Given : 2 = e0.7, R = 8.3 J/K - mol]
(A) A + B l C
(B) A l B + 2C
(C*) 2A l B + C
(D) A + B l 2C

Q.6

pH of 0.1 M BOH (a weak base) is found to be 12. The solution at temperature TK will display an
osmatic pressure equal to:
[3]
(A) 0.01 RT
(B) 0.01 RT
(C*) 0.11 RT
(D) 1.1 RT

Q.7

Equal volumes of 1.0 M KCl and 1.0 M AgNO3 are mixed. The depression of freezing point of the
resulting solution will be (Kf(H2O) = 1.86 K kg mol1, 1 M = 1m)
[3]
(A) 3.72 K
(B*) 1.86 K
(C) 0.93 K
(D) none of these

[3]

[3]

More than One may be correct :


Q.9

The activation energies of two reactions I and II are Ea and 2Ea respectively. If the temperature of the
reacting systems is increased from T to T ', predict which of the following alternative is/are correct?
(k represent rate constant)
[5]
(A*) k 'I / k I 1

(B*) k 'II / k II 1

(C*) k 'I / k I k 'II / k II (D) k 'I / k I 2 k 'II / k II

Subjective :
Q.10 A gas has been subjected to an isochoric and isobaric cycle. Plot the graph
of this cycle in the pressure-density, V-T and P-T co-ordinates.
[6]

P
2

[Ans. (i)

(ii)

(iii)

1
4
T

ASSIGNMENT-28
Single correct
Q.1 Select incorrect statements:
[3]
(A) NH3 is soluble in water due to hydrogen bonding as well as due to formation of ions
(B) Gases which can be liquefied easily are more soluble in water than the gases which cannot be
liquefied
(C*) The solute follows Henry law at all pressure of gas
(D) The solute follows Henry law at low pressure of gas
Q.2

For an ionic crystal of the general formula AX and the coordination number 6, the values of radius ratio
will be
[3]
(A) Greater than 0.73
(B*) In between 0.73 and 0.41
(C) In between 0.41 and 0.22
(D) Less than 0.22

Q.3

The standard EMF of the cell in which the reaction, MnO 4 + 5Fe2+ + 8H+ Mn2++ 5Fe3+ + 4H2O
occurs is 0.59 V at 25. The equilibrium constant for the given reaction is approximately.
[3]
50
5
(A) 50
(B) 10
(C*) 10
(D) 10

Q.4

Based upon the technique of reverse osmosis the approximate pressure required to desalinate sea water
containing 2.5% (mass/volume) NaCl at 27C will be:
[3]
(A) 10.5 atm
(B*) 21 atm
(C) 2.1 atm
(D) 1.05 atm

Comprehension
Question No. 5 to 7 (3 questions)
Freezing point of a liquid is defined as that temperature at which it is in equilibrium with its solid state.

Q.5

Phase diagram for a pure solvent and solution for depression in freezing point.
Freezing point of the following system is:
liquid solvent l solid solvent
(A)

H G
T.S

(B*)

H
S

(C)

G
S

(D)

S
H

[3]

Q.6

Freezing point of an ideal solution containing a non-volatile solute is smaller than that point of a solvent.
It is due to:
[3]
(A) H of solution and solvent is almost identical since intermolecular forces between solvent molecules
are involved
(B*) S of solution is larger than that for the solvent
(C) S of the solution is smaller than that of the solvent
(D) H of the solution is much higher than of solvent but S of solution is smaller than that of the solvent

Q.7

60 g of urea is dissolved in 1100 g solution. To keep T/ Kf as 1 mol/kg, water separated in the form of ice is:
(A*) 40 g
(B) 60 g
(C) 100 g
(D) 200 g
[3]

More than one may be correct


Q.8 In a AB unit crystal of NaCl types assuming Na+ forming FCC
[4]
+

+
(A*) The nearest neighbour of A is 6 B ion (B*) The nearest neighbour of B is 6 A ion
3
(C*) The second neighbour of A+ is 12 A+
(D) The packing fraction of AB crystal is
8
Q.9 Which of the following statement(s) is/ are correct?
[4]
(A*) The coordination number of each type of ion in CsCl crystal is 8.
(B) A metal that crystallizes in bcc structure has a coordination no. of 12.
(C*) A unit cell of an ionic crystal shares some of its ions with other unit cells.
(D*) The length of the unit cell in NaCl is 552 pm ( rNa 95pm,rCl 181pm )
Subjective
Q.10 A solution of a nonvolatile solute in water freezes at 0.30C. The vapor pressure of pure water at
298K is 23.51mmHg and Kf for water is 1.86 degree/molal. Calculate the vapor pressure of this solution
at 298K.
[4]
[Ans. 23.44 mm Hg]

ASSIGNMENT-29

Single correct
Q.1 How many nearest neighbours Cs+ are present in CsCl structure
(A) 6
(B*) 8
(C) 12

[3]
(D) 4

Q.2

If the positions of Na+ and Cl are interchanged in NaCl, the crystal lattice with respect to Na+ and Cl is :
[3]
(A*) unchanged
(B) changes to 8:8 coordination from 6:6
(C) additivity of ionic radii for a is lost
(D) none

Q.3

In the closest packing of atoms,


[3]
(A) the size of tetrahedral void is greater than that of octahedral void
(B*) the size of tetrahedral void is smaller than that of octahedral void
(C) the size of tetrahedral void is equal to that of octahedral void
(D) the size of tetrahedral void may be greater or smaller or equal to that of octahedral void depending
upon the size of atoms.

Q.4

A mineral having the formula AB2 crystallises in the cubic close-packed lattice, with the A atoms occupying
the lattice points. The co-ordination number of the A atoms, that of B atoms and the fraction of the
tetrahedral sites occupied by B atoms are
[3]
(A*) 8, 4, 100%
(B) 2, 6, 75%
(C) 3, 1, 25%
(D) 6, 6, 50%

Question Number 5 to 9 (5 questions)


In haxagonal close packing second row of spheres are placed in depressions of first row whereas third
row is vertically aligned with first, 2nd with fourth forming a pattern AB.AB...... The number of nearest
atoms is called the coordination number.
Q.5 What is the coordination number of a central sphere in hcp
[3]
(A) 8
(B*) 12
(C) 6
(D) 4
Q.6

Q.7
Q.8
Q.9

r
lies between 0.225 to 0.414, cation occupies
r
(A*) tetrahedral void (B) octahedral void
(C) trigonal void
If

The coordination number of octahedral void is


(A) 8
(B) 4
(C*) 6

[3]
(D) cubic void
[3]
(D) 3

Which of the following packing are most efficient


(A) HCP
(B) FCC
(C*) A & B both

(D) BCC

ABC.ABC....... is called
(A) HCP
(B*) FCC

(D) simple cubic

[3]
[3]

(C) BCC

Subjective
Q.10 10 gm of NH4Cl (mol. weight = 53.5) when dissolved in 1000 gm of water lowered the freezing point by
0.637C. Calculate the degree of cationic hydrolysis of the salt if degree of dissociation of salt is 0.75.
The molal depression constant of water is 1.86 kg mol1 K.
[5]
[Ans. h = 0.109]

ASSIGNMENT-30

Single correct
Q.1

Graph depicting correct behaviour of ideal gas & H2 gas will be (neglect a)

(A*)

Q.2

(B)

(C)

(D)

For a real gas, behaving ideally the pressure may be :


(A) abVmolar

(B)

Vmolar
ab

a
(C*) V
molar b

[3]

[3]
b
(D) aV
molar

More than one may be correct


Q.3 Following represents the Maxwell distribution curve for an ideal gas at two temperatures T1 & T2.
Which of the following option(s) are true?
[4]

(A*) Total area under the two curves is independent of moles of gas
(B*) If dU1= f Umps1 & dU2 = f Umps2 then A1 = A2
(C) T1 > T2 and hence higher the temperature, sharper the curve.
(D*) The fraction of molecules having speed = Umps decreases as temperature increases.
Q.4

Select the correct option(s).


[4]
(A*) Fraction of molecule in the range Uavg f Uavg is same for SO2 and O2 at same Temperature
(0 < f < 1).
(B) Fraction of molecule in the range Umps 100 (m/sec) is same for SO2 and O2 at same Temperature.
(C*) Fraction of molecule in the range Uavg f Uavg is same for O2 at 300 K and at 200 K (0 < f < 1).
(D) None of these

Subjective
Q.5 The gas having higher value of Vander Waal's constant "a" will be __________ compressible than the
one having lower value of "a", provided Vander Waal's constant "b" is same for both the gases. [2]
[Ans. more]
Q.6

At what temperature in C, the Urms of SO2 is equal to the average velocity of O2 at 27C. [4]
[Ans. 236.3C]

Q.7

Calculate the fraction of N2 molecules at 101.325 kPa and 300 K whose speeds are in the range of
ump 0.005 ump to ump + 0.005 ump.
[4]
[Ans. 8.303 103]

Q.8

The density of mercury is 13.6 g/cm3. Estimate the b value.

Q.9

The molar volume of He at 10.1325 MPa and 273 K is 0.011075 of its molar volume at 101.325 KPa
at 273 K.Calculate the radius of helium atom. The gas is assumed to show real gas nature. Neglect the
value of a for He.
[4]
[Ans. r = 1.33 108]

[4]

[Ans. 58.997 cm3]

Q.10 N2 molecule is spherical of radius 100 pm.


[4]
(a)
What is the volume of molecules is one mole of a gas?
(b)
What is the value of vander waal's constant b?
[Ans. (a) 2.52 103 l mol1, (b) 10.08 103 dm3 mol1]

ASSIGNMENT-31
Single correct
Q.1

Correct option regarding a container containing 1 mol of a gas in 22.4 litre container at 273 K is [3]
(A) If compressibility factor (z) > 1 then 'P' will be less than 1 atm.
(B*) If compressibility factor (z) > 1 then 'P' will be greater than 1 atm.
(C) If 'b' dominates, pressure will be less than 1 atm.
(D) If 'a' dominates, pressure will be greater than 1 atm.

Q.2

The compressibility of a gas is less than unity at STP. Therefore


(A) Vm > 22.4 L
(B*) Vm < 22.4 L
(C) Vm = 22.4 L

[3]
[JEE 2000]
(D) Vm = 44.8 L

More than one may be correct


Q.3 Which of the following statement(s) are true about Z vs P graph for a real gas at a given temperature.

[4]

(A) dZ dP 0 as P 0 for most real gases


(B*) dZ dP = ive as P 0 for most real gases
(C) dZ dP 0 at a pressure where repulsive and attractive forces are comparable.
(D*) dZ dP = +ive for real gases at extremely high pressure.
Q.4

Select the incorrect statement(s):


[4]
(A*) At Boyle's temperature a real gas behaves like an ideal gas irrespective of pressure.
(B*) At critical condition, a real gas behaves like an ideal gas.
(C) On increasing the temperature four times,collision frequency (Z11) becomes double at constant volume.
(D) At high pressure Vander Waal's constant 'b' dominates over 'a'.

Q.5

Match gases under specific conditions listed in Column I with their properties / laws in Column II.
Indicate your answer by darkening the appropriate bubbles of the 4 4 matrix given in the ORS. [8]
Column I
Column II
(A) Hydrogen gas (P = 200 atm, T = 273 K)
(P) Compressibility factor 1
(B) Hydrogen gas (P ~ 0, T = 273 K)
(Q) Attractive forces are dominant
(C) CO2 (P = 1 atm, T = 273 K)
(R) PV = nRT
(D) Real gas with very large molar volume
(S) P (V nb) = nRT
[JEE 2007]
[Ans. (A) P, S; (B) R; (C) P, Q; (D) R ]

Subjective
Q.6 Sign of initial slope of compressibility factor (z) versus P curves is ________ if a gas is below its Boyle's
temperature and ________ if it is above its Boyle's temperature.
[4]
[Ans. ive, +ive]
Q.7

At 273.15 K and under a pressure of 10.1325 MPa, the compressibility factor of O2 is 0.927. Calculate
the mass of O2 necessary to fill a gas cylinder of 100 dm3 capacity under the given conditions. [4]
[Ans. 15.40 kg]

Q.8

Using Vander Waals equation, calculate the constant "a" when 2 moles of a gas confined in a 4 litre flask
exerts a pressure of 11.0 atmp at a temperature of 300 k. The value of "b" is 0.05 litre mol 1. [4]
[JEE 1998]
[Ans. 6.46 atmp L2 mol2]

Q.9

The compression factor (compressibility factor) for one mole of a vander Waals gas at 0C and 100
atmosphere pressure is found to be 0.5. Assuming that the volume of a gas molecule is negligible, calculate
the vander waals constant 'a'.
[4]
[JEE 2001]
[Ans. 1.2544 atmp L2 mol2]

Q.10 1 mole of CCl4 vapours at 27C occupies a volume of 40 lit. If Vander Waals constant are 24.6 L2 atm
mol1 and 0.125 Lmol1. Calculate compressibility factor under
[6]
(a) Low pressure region
(b) High Pressure region
[Take R = 0.082 lit-atm/mol/K]
[Ans. (a) 0.975; (b) 1.003]

ASSIGNMENT-32
Single correct
Q.1 The term that corrects for the attractive forces present in a real gas in the Vander Waals equation is [3]
[JEE 2009]
(A) nb
Q.2

(B*)

an 2
V2

(C)

an 2
V2

Positive deviation from ideal behaviour takes place because of


(A) molecular attraction between atoms and

PV
>1
nRT

(B) molecular attraction between atoms and

PV
<1
nRT

(C*) finite size of atoms and


(D) finite size of atoms and

(D) nb
[3]

[JEE 2003]

PV
>1
nRT

PV
<1
nRT

More than one may be correct


Q.3

A gas described by van der Waals equation


[4]
[JEE 2008]
(A*) behaves similar to an ideal gas in the limit of large molar volumes
(B) behaves similar to an ideal gas in the limit of large pressures
(C*) is characterised by van der Waals coefficients that are dependent on the identity of the gas but are
independent of the temperature
(D*) has the pressure that is lower than the pressure exerted by the same gas behaving ideally

Subjective
Q.4 The density of water vapour at 327.6 atm and 776.4 K is 133.2 gm/dm3.
Determine the molar volume, Vm of water and the compression factor.
[4]
[Ans. Molar vol = 0.1353 L/mol; Z = 0.6957 ]
Q.5

The vander waals constant for O2 are a = 1.36 atm L2 mol2 and b = 0.0318 L mol1. Calculate the
temperature at which O2 gas behaves, ideally for longer range of pressure.
[4]
[Ans. 521 K]

Q.6

The vander Waals constants for gases A, B and C are as follows


[4]
6
2
3
1
Gas
a/dm kPa mol
b/dm mol
A
405.3
0.027
B
1215.9
0.030
C
607.95
0.032
Which gas has (i) the highest critical temperature, (ii) the largest molecular volume.
[Ans. (i) B, (ii) C]

Q.7

For a real gas obeying van der Waal's equation a graph is plotted between PVm (y-axis) and P(x-axis)
where Vm is molar volume. Find y-intercept of the graph.
[4]
[JEE 2004]
[Ans. RT]

Q.8

For a real gas (mol. mass = 30) if density at critical point is 0.40 g/cm3 and its Tc =
calculate Vander Waal's constant a (in atm L2mol2).

Q.9

[4]

2 105
K, then
821
[Ans. 1.6875 ]

A commercial cylinder contains 6.91 m3 of O2 at 15.18 M Pa and 21C. the critical constants for O2 are
TC = 118.4C , PC = 50.1 atmp. Determine the reduced pressure and reduced temperature for O2
under these conditions.
[4]
[Ans. PR = 2.99 , TR = 1.90]

Q.10 The density of the vapour of a substance at 1 atm pressure and 500 K is 0.36 Kg m3. The vapour
effuses through a small hole at a rate of 1.33 times faster than oxygen under the same condition. [6]
Determine
(i) mol. wt.; (ii) molar volume;
(iii) compression factor (z) of the vapour and
(iv) which forces among the gas molecules are dominating, the attractive or the repulsive
[JEE 2002]
1

[Ans. (i) 18.1 g/mol , (ii) 50.25 L mol , (iii) 1.224 , (iv) repulsive, ]

ASSIGNMENT-33

Single cor rect


Q.1
A particle A moving with a certain velocity has a de-Broglie wavelength of 1 . If particle B has mass
25% of that A and velocity 75% of that of A, the of B will be approximately
[3]
(A) 1
(B*) 5.3
(C) 0.2
(D) 3
Q.2

If the radius of first Bohr orbit is x, then de-Broglie wavelength of electron in 3rd orbit is nearly [3]
(A) 2 x
(B*) 6 x
(C) 9 x
(D) x / 3

Q.3

If E1, E2 and E3 represent respectively the kinetic energies of an electron, an alpha particle and a proton
respectively each having same deBroblie wavelength then
[3]
(A*) E1 > E3 > E2
(B) E2 > E3 > E1
(C) E1 > E2 > E3
(D) E1 = E2 = E3

Q.4

The ratio of difference in wavelengths of 1st and 2nd lines of Lyman series in Hlike atom to difference in
wavelength for 2nd and 3rd lines of same series is:
[3]
(A) 2.5 : 1
(B*) 3.5 : 1
(C) 4.5 : 1
(D) 5.5 : 1

Comprehension
Question No. 5 to 7 (3 questions)
The French physicist Louis de Broglie in 1924 postulated that matter, like radiation, should exhibit a dual
behaviour. He proposed the following relationship between the wavelength of a material particle, its
linear momentum p and planck constant h.

h
h
=
p
mv
The de Broglie relation implies that the wavelength of a particle should decreases as its velocity increases.
It also implies that for a given velocity heavier particles should have shorter wavelength than lighter
particles. The waves associated with particles in motion are called matter waves or de Broglie waves.
=

Q.5

The correct order of wavelength of Hydrogen (1H1), Deuterium (1H2) and Tritium (1H3) moving with
same kinetic energy is
[3]
(A*) H > D > T
(B) H = D = T
(C) H < D < T
(D) H < D > T

Q.6

The transition, so that the de-Broglie wavelength of electron becomes 3 times of its initial value in He+ ion
will be
[3]
(A) 2 5
(B) 3 2
(C*) 2 6
(D) 1 2

Q.7

If the uncertainty in velocity & position is same, then the uncertainty in momentum will be
(A*)

hm
4

(B) m

h
4

(C)

h
4m

(D)

1 h
m 4

[3]

Match the column


Q.8 Match the entries in Column I with the correctly related quantum number(s) in Column II. Indicate
your answer by darkening the appropriate bubbles of the 4 4 matrix given in the ORS.
[12]
[JEE 2008]
(A)

Column I
Orbital angular momentum of the
electron in a hydrogen-like atomic orbital

(P)

Column II
Principal quantum number

(B)

A hydrogen-like one-electron wave


function obeying Pauli principle

(Q)

Azimuthal quantum number

(C)

Shape, size and orientation of hydrogen


like atomic orbitals

(R)

Magnetic quantum number

(D)

Probability density of electron at the nucleus (S)


Electron spin quantum number
in hydrogen-like atom
[Ans. (A) Q,R (B) P, Q, R,S (C) P, Q, R (D) P, Q]

Subjective
Q.9 A proton is accelerated to one- tenth of the velocity of light. If its velocity can be measured with a
precision + 1%. What must be its uncertainity in position.
[5]
13
[Ans. 1.0510 m]
Q.10 What is de Broglie wavelength associated with an e accelerated through potential difference = 100 KV.
[5]
[Ans. 3.88 pm]
Q.11

Calculate the de-broglie wavelength associated with motion of earth (mass 6 1024 Kg) orbiting around
the sun at a speed of 3 106 m/s.
[5]
65
[Ans. 3.68 10 m]

ASSIGNMENT-34

Single cor rect


Q.1 Three energy levels P, Q, R of a certain atom are such that EP < EQ < ER. If 1, 2 and 3 are the wave
length of radiation corresponding to transition R Q ; Q P and R P respectively. The correct
relationship between 1, 2 and 3 is
[3]
(A) 1 + 2 = 3

1
1
1
(B*)
3
1
2

(C) 3 =

1 2

2
1
1
(D)
3
1
2

Q.2

Number of possible spectral lines which may be emitted in bracket series in H atom, if electrons present
in 9th excited level returns to ground level, are
[3]
(A) 21
(B*) 6
(C) 45
(D) 5

Q.3

The wavelength associated with a golf weighing 200g and moving at a speed of 5m/h is of the order [3]
(A) 1010m
(B) 1020m
(C*) 1030m
(D) 1040m

Q.4

What is uncertainity in location of a photon of wavelength 5000 if wavelength is known to an accuracy


of 1 pm?
[3]
14
8
(A) 7.96 10 m
(B*) 0.02 m
(C) 3.9 10 m
(D) none

Q.5

Assuming Heisenberg Uncertainity Principle to be true what could be the minimum uncertainty in de-broglie
wavelength of a moving electron accelerated by Potential Difference of 6 V whose uncertainty in position
7
n.m.
22
(A) 6.25

is

[3]
(B) 6

(C*) 0.625

(D) 0.3125

Assertion and Reason :


Q.6 It is a data sufficiency problem in which it is to be decided on the basis of given statements
whether the given question can be answered or not. No matter whether the answer is yes or no.
Question : Is the orbital of hydrogen atom 3px?
Statement-1: The radial function of the orbital is R(r) =

r
1
(4 ) e / 2 , =
3/ 2
2
9 6 a0

Statement-2: The orbital has 1 radial node & 0 angular node.


(A) Statement (1) alone is sufficient.
(B*) Statement (2) alone is sufficient
(C) Both together is sufficient.
(D) Neither is sufficient
More than one may be correct :
Q.7 Correct statement(s) regarding 3Py orbital is/are
(A) Angular part of wave function is independent of angles ( and )
(B*) No. of maxima when a curve is plotted between 4r2R2(r) vs r are '2'
(C*) 'xz' plane acts as nodal plane
(D) Magnetic quantum number must be '1'

[3]

[4]

Match the column


Q.8 Column I & column II contain data on Schrondinger WaveMechanical model, where symbols have
their usual meanings.Match the columns.
[12]
Column I
Column II (Type of orbital)

(A)

(P)

4s

(B)

(Q)

5px

(R)
(S)

3s
6dxy

(C)
(D)

(,) = K (independent of &)


atleast one angular node is present

[Ans. (A) P, (B) P,Q,S, (C) P, R (D) Q, S]


Subjective
Q.9 A base ball of mass 200 g is moving with velocity 30 102 cm/s. If we can locate the base ball with an
error equal in magnitude to the of the light used (5000 ), how will the uncertainty in momentum be
compared with the total momentum of base ball.
[5]
[Ans. 1.75 1029 ]
Q.10 An electron has a speed of 40 m/s, accurate up to 99.99%. What is the uncertainity in locating its
position.
[5]
[Ans. 0.0144 m]
Q.11

A cylindrical source of light which emits radiation radially (from curved surface) only, placed at the
centre of a hollow, metallic cylindrical surface, as shown in diagram.
The power of source is 90 watt and it emits light of wavelength 4000 only. The emitted photons strike
the metallic cylindrical surface which results in ejection of photoelectrons. All ejected photoelectrons
reaches to anode (light source). The magnit ude of photocurrent is
[Given : h = 6.4 1034 J/sec.]
[5]

[Ans.10 amp]

You might also like